Sei sulla pagina 1di 478

SOLUTION MANUAL

for
SEPARATI ON PROCESS ENGI NEERI NG.
I ncludes Mass Transfer Analysis
3
rd
Edition
(Formerly published as Equilibrium-Staged Separations)
by
Phillip C. Wankat
17
SPE 3
rd
Edition Solution Manual Chapter 1
New Problems and new solutions are listed as new immediately after the solution number. These new
problems in chapter 1 are: 1A3, 1A4, 1B2-1B4, 1D1.
A2. Answers are in the text.
A3. New problem for 3
rd
edition. Answer is d.
B1. Everything except some food products has undergone some separation operations. Even the
water in bottles has been purified (either by reverse osmosis or by distillation).
B2. New problem for 3
rd
edition. Many homes have a water softener (ion exchange), or a filter, or a
carbon water filter (actually adsorption), or a reverse osmosis system.
B3. New problem for 3
rd
edition. For example: the lungs are a gas permeation system, the intestines
and kidney are liquid permeation or dialysis systems.
B4. New problem for 3
rd
edition. You probably used some of the following: chromatography,
crystallization, distillation, extraction, filtration and ultrafiltration.
D1. New problem for 3
rd
edition. Basis 1kmol feed.
.4 kmole E .4 MW 46 18.4 kg

10.8 kg
.6 kmol Water .6 MW 18
total 29.2 kg

Weight fraction ethanol = 18.4/29.2 = 0.630


Flow rate = (1500 kmol/hr)[(29.2kg)/(1 kmol)] = 43,800 kg/hr.
18
Chapter 2
New Problems and new solutions are listed as new immediately after the solution number. These new
problems are: 2A6, 2A9 to 2A16, 2C4, 2C8, 2C9, 2D1.g, 2.D4, 2D10, 2D13, 2D24 to 2D30, 2E1, 2F4,
2G4 to 2G6, 2H1 to 2H3.
2.A1. Feed to flash drum is a liquid at high pressure. At this pressure its enthalpy can be calculated
as a liquid. eg.
high LIQ
F,P p F ref
h T c T T . When pressure is dropped the mixture is above
its bubble point and is a two-phase mixture (It flashes). In the flash mixture enthalpy is
unchanged but temperature changes. Feed location cannot be found from T
F
and z on the
graph because equilibrium data is at a lower pressure on the graph used for this calculation.
2.A2. Yes.
2.A4.
2.A6. New Problem. In a flash drum separating a multicomponent mixture, raising the pressure will:
i. Decrease the drum diameter and decrease the relative volatilities. Answer is i.
2.A8. a. K increases as T increases
b. K decreases as P increases
c. K stays same as mole fraction changes (T, p constant)
-Assumption is no concentration effect in DePriester charts
d. K decreases as molecular weight increases
2.A9. New Problem. The answer is 0.22
2.A10. New Problem. The answer is b.
2.A11. New Problem. The answer is c.
1.0
.5
0
1.0
.5
0
x
w

Flash
operating
line
y
w

Equilibrium
(pure water)
2.A4
z
w
= 0.965
19
2.A12. New Problem. The answer is b.
2.A13. New Problem. The answer is c.
2.A14. New Problem. The answer is a.
2.A15. New Problem. a. The answer is 3.5 to 3.6
b. The answer is 36C
2.A16. New Problem. The liquid is superheated when the pressure drops, and the energy comes from the
amount of superheat.
2.B1. Must be sure you dont violate Gibbs phase rule for intensive variables in equilibrium.
Examples:
drum drum
F, z, T , P
F
F, T , z, p
F
F, h , z, p
drum
F, z, y, P
F
F, T , z, y
F
F, h , z, y
drum
F, z, x, p
F
F, T , z, x etc.
drum
F, z, y, p
F drum drum
F, T , z, T , p
drum
F, z, x, T
F
F, T , y, p
Drum dimensions,
drum drum
z, F , p
F drum
F, T , y, T
Drum dimensions,
drum
z, y, p
F
F, T , x, p
etc.
F drum
F, T , x, T
F
F, T , y, x
2.B2. This is essentially the same problem (disguised) as problem 2-D1c and e but with an existing
(larger) drum and a higher flow rate.
With y = 0.58, x = 0.20, and V/F = 0.25 which corresponds to 2-D1c.
If
lb mole
F 1000 , D .98 and L 2.95 ft from Problem 2-D1e
hr
.
Since D V and for constant V/F, V F, we have D F .
With F = 25,000:
new old new old new new
F F = 5, D = 5 D = 4.90, and L = 3 D = 14.7.
Existing drum is too small.
Feed rate drum can handle: F D
2
.
2 2
existing
exist
F
D 4
1000 .98 .98
gives
existing
F 16,660 lbmol/h
Alternatives
a) Do drums in parallel. Add a second drum which can handle remaining 8340 lbmol/h.
b) Bypass with liquid mixing
20
Since x is not specified, use bypass. This produces less vapor.
c) Look at Eq. (2-62), which becomes
v
drum L v v
V MW
D
3K 3600
Bypass reduces V
c1) K
drum
is already 0.35. Perhaps small improvements can be made with a better demister
Talk to the manufacturers.
c2)
v
can be increased by increasing pressure. Thus operate at higher pressure. Note this
will change the equilibrium data and raise temperature. Thus a complete new
calculation needs to be done.
d) Try bypass with vapor mixing.
e) Other alternatives are possible.
2.C2.
A B
B A
z z V
F K 1 K 1
2.C5. a. Start with
i
i
i
Fz
x and let V F L
L VK
i i
i i
i
i
Fz z
x or x
L L L F L K
1 K
F F
Then
i i
i i i
i
K z
y K x
L L
1 K
F F
From
i i
i i
i
K 1 z
y x 0 we obtain 0
L L
1 K
F F

V = .25 (16660) = 4150
L
Total

x
y = .58,
8340
16,660
25,000
21
2.C7.
i
i
z V
1 f
V
F
1 K 1
F
From data in Example 2-2 obtain:
V/F 0 .1 .2 .3 .4 .5 .6 .7 .8 .9 1.0
f 0 -.09 -.1 -.09 -.06 -.007 .07 .16 .3 .49 .77
2.C8. New Problem.
drum
p
x
y
z
drum
T
F = L + V
z
F Lx Vy
Solve for L & V
Or use lever arm-rule
22
2.C9. New Problem. Derivation of Eqs. (2-62) and (2-63). Overall and component mass balances are,
1 2
F V L L
i 1 i,L1 2 i,L2 i
Fz L x L x Vy
Substituting in equilibrium Eqs. (2-60b) and 2-60c)
i 1 i,L1 L2 i,L2 2 i,L2 iV L2 i,L2
Fz L K x L x VK x
Solving,
i i
i,L2
1 i,L2 2 i,V L2 1 i,L1 L2 1 i,V L2
Fz Fz
x
L K L VK L K F V L VK
Dividing numerator and denominator by F and collecting terms.
i
i,liq2
1
i,L1 L2 i,V L2
z
x
L V
1 K 1 K 1
F F
Since
i i,V L2 i,L2
y K x ,
i,V L2 i
i
1
i,L1 L2 i,V L2
K z
y
L V
1 K 1 K 1
F F
Stoichiometric equations,
C C C C
i,L2 i i i,L2
i 1 i 1 i 1 i 1
x 1 , y 1 , thus, y x 0
which becomes
C
i ,V L2 i
i 1
1
i ,L1 L2 i ,V L2
K 1 z
0
L V
1 K 1 K 1
F F
(2-62)
Since
i,L1 L2 i
i,liq1 i,L1 L2 i,liq2 i,liq1
1
i,L1 L2 i,V L2
K z
x K x , we have x
L V
1 K 1 K 1
F F
In addition,
C
i ,L1 L2 i
i ,liq1 i ,liq 2
i 1
1
i ,L1 L2 i ,V L2
K 1 z
x x 0
L V
1 K 1 K 1
F F
(2-63)
2.D1. a. V 0.4 100 40 and L F V 60 kmol/h
Slope op. line L V 3 2, y x z 0.6
See graph. y 0.77 and x 0.48
b. V 0.4 1500 600 and L 900 . Rest same as part a.
c. Plot x 0.2 on equil. Diagram and
int ercept
y x z 0.3. y zF V 1.2
V F z 1.2 0.25 . From equil y 0.58 .
d. Plot x 0.45 on equilibrium curve.
23
L F V 1 V F .8
Slope 4
V V V F .2
Plot operating line, y x z at z 0.51. From mass balance F 37.5 kmol/h.
e. Find Liquid Density.

L
m m w w
MW x MW x MW .2 32.04 .8 18.01 20.82
Then,
w m
L m w
m w
MW MW 32.04 18.01
V x x .2 .8 22.51 ml/mol
.7914 1.00
L
L L
MW V 20.82 22.51 0.925 g/ml
Find Vapor Density.
v
v
P MW
RT
(Need temperature of the drum)
v
m w
m w
MW y MW y MW .58 32.04 .42 18.01 26.15 g/mol
Find Temperature of the Drum T:
From Table 3-3 find T corresponding to
y .58, x 20, T=81.7 C 354.7K
4
v
ml atm
1 atm 26.15 g/mol 82.0575 354.7 K 8.98 10 g/ml
mol K
Find Permissible velocity:
24


perm drum L v v
2 3 4
drum lv lv lv lv
u K
K exp A B nF C nF D nF E nF
2-60
Since
V
V F 0.25 1000 250 lbmol/h,
F

v
v
lb
W V MW 250 26.15 6537.5 lb / h
lbmol


L
L
L F V 1000 250 750 lbmol/h, and W L MW 750 20.82 15, 615 lb/h,

4
V L
lv lv
V L
W 15615 8.89 10
F 0.0744, and n F 2.598
W 6537.5 .925

Then
4
drum perm 4
.925 8.98 10
K .442, and u .442 14.19 ft/s
8.98 10


v
2
cs
4 3
perm v
V MW
250 26.15 454 g/lb
A 2.28 ft .
u 3600 14.19 3600 8.98 10 g/ml 28316.85 ml/ft

Thus,
cs
D 4A 1.705 ft. Use 2 ft diameter.
L ranges from 3 D 6 ft to 5 D=10 ft
Note that this design is conservative if a demister is used.
f. Plot T vs x from Table 3-3. When T 77 C, x 0.34, y 0.69. This problem is now
very similar to 3-D1c. Can calculate V/F from mass balance, Fz Lx Vy. This is
V z y 0.4 0.34
Fz F V x Vy or 0.17
F y x 0.69 0.34

g. Part g is a new problem. V = 16.18 mol/h, L = 33.82, y= 0.892, x = 0.756.

2-D2. Work backwards. Starting with x
2
, find y
2
= 0.62 from equilibrium. From equilibrium point
plot op. line of slope
2 2
2
V
L V 1 V F 3 7.
F
This gives
2 1
z 0.51 x (see Figure). Then from equilibrium,
1
y 0.78 .
For stage 1,
1 1
1 1
z x V 0.55 0.51
0.148
F y x 0.78 0.51
.

2.D3. a. z 0.4 V F 0.6 V 6.0 k mol h, L 4.0
Op. eq.
L
y x z V F
V
2
y x 2 3
3

See graph:
M
y 0.55 x 0.18 T ~ 82.8 C linear interpretation on Table 2-7 .

25
b. Product 78.0 C x 0.30, y 0.665,
Mass Bal: Fz Lx Vy F V x Vy
or 4.0 10 V 0.3 0.665V
V 2.985 and V F 0.2985
Can also calculate V/F from slope.
c.
V
F 10, 0.3 V 3 & L 7
F
L z 7 z
y x x
V V F z 0.3
If y 0.8, x 0.545 @ equil
Then
7
z 0.3 0.8 0.545 0.6215.
3

Can also draw line of slope
7
3
through equil point.
26
2.D4. New problem in 3
rd
edition. Highest temperature is dew point V F 0
Set
i i i i i
z y . K y x
Want
i i i
x y K 1.0

ref New ref Old i i
K T K T y K
If pick C4 as reference: First guess
bu tane
K 1.0, T 41 C:
C3 C6
K 3.1, K 0.125
i
i
y .2 .35 .45
4.0145
K 3.1 1.0 .125
T too low
Guess for reference
C4
K 4.014 T 118 C :
C3 C6
K 8.8, K .9
i
i
y .2 .35 .45
0.6099
K 8.8 4.0145 .9
C4,NEW
K 4.0145 .6099 2.45, T 85 :
C2 C6
K 6.0, K 0.44
i
i
y .2 .35 .45
1.20
K 6 2.45 .44
C4,NEW
K 2.45 1.2 2.94, T 96 C:
C3 C6
K 6.9, K 0.56
i
i
y .2 .35 .45
0.804 Gives 84 C
K 6.9 2.94 .56
27

Use 90.5 Avg last two T
C4 C3 C6
K 2.7, K 6.5, K 0.49

i i
.2 .35 .45
y K 1.079
6.5 2.7 .49

T ~ 87 88 C

Note: hexane probably better choice as reference.

2.D5. a)


b)
1 1
1 1
L F
y x z
V V
Plot 1
st
Op line.
y = x = z = 0.55
to x
1
= 0.3 on eq. curve (see graph)
Slope
L 0.55 0.80 .25
0.454545
V .55 0 .55


1
1 1 1
1
L
0.45454 & L V F 1000
V





c) Stage 2
2
V L 0.75F
0.25 , 3 , y x z 0.66.
F V 0.25F
Plot op line
At
2
0.66 F z 0.66
x 0, y z V F 2.64. At y 0, x z 0.88
0.25 L L F 0.75

From graph
2 2
y 0.82, x 0.63 .

2 2
2
V
V F 0.25 687.5
F
171.875 kmol/h

F
1
= 1000

z
1
= 0.55
p
1,2
= 1 atm
x
2


x
1
= 0.30

v
2

y
2

v
1
= F
2

y
1
= z
2

2
1
2
V
0.25
F

y
1
= 0.66 = z
2

V
1
= 687.5 kmol/h = F
2

1
V 687.5
0.6875
F 1000

28
2.D6.
RR eq.,
i i
i
K 1 z
0
1 K 1 V F
, First guess V/F = 0.6
1
1.4 .45 0.2 .35 0.7 .2
f 0.0215
1 1.4 .6 1 0.2 0.6 1 0.7 0.6
Use Newtonian Convergence
2
c
i i k
2
i 1
i
K 1 z df
d V F
1 K 1 V F
k
k 1 k
f V V
df
F F
d V F
T = 50C
P = 200 kPa
K
c4
= 2.4
z
c4
= 0.45
F = 1.0 kmol/min
z
c5
= 0.35
Z
c6
= 0.20
L
V
K
c5
= 0.80
K
c6
= 0.30
29
2 2
2
1
2 2 2
1.4 0.45 0.2 0.35 0.7 0.20
df
0.570
V
1 1.4 .6 1 0.2 0.6 1 0.7 0.6
d
F
2
V 0215
0.6 0.6377
F 0.570
2
1.4 .45 0.2 .35 0.7 0.2
f 0.00028
1 1.4 0.6377 1 0.2 0.6377 1 0.7 0.6377
Which is close enough.
i i i
i
c4
c4
i
y K x
z 0.45
x 0.2377,
V y 2.4 0.2377 0.5705 1 1.4 .6377
1 K 1
F
c5 c5
0.35
x 0.4012, y 0.8 0.4012 0.3210
1 0.2 0.6377

c6 8
i i
0.20 0.3613 0.1084
x , y 0.30 0.4012
1 0.7 0.6377 x 1.0002 y 0.9998
2.D7.
A B
B A
z z V
F K 1 K 1
M P
K 5.6 and K 0.21
V 0.3 0.7
0.2276
F 0.21 1 5.6 1
Eq. (2-38)
M
M
M
z 0.3
x 0.1466
V
1 4.6 0.2276
1 K 1
F
P M
x 1 x 0.8534 ,
M M M
y K x 5.6 0.1466 0.8208
P M
y 1 y 0.1792
2.D8. Use Rachford-Rice eqn:
i i
i
K 1 z
V
f 0
F 1 K 1 V / F
. Note that 2 atm = 203 kPa.
Find
i
K from DePriester Chart:
1 2 3
K 73, K 4.1 K .115
Converge on V F .076, V F V F 152 kmol/h, L F V 1848 kmol/h .
From
i
i
i
z
x
V
1 K 1
F
we obtain
1 2 3
x .0077, x .0809, x .9113
From
i i i 1 2 3
y K x , we obtain y .5621, y .3649, y .1048
2.D9. Need h
F
to plot on diagram. Since pressure is high, feed remains a liquid
L
F P F ref ref
h C T T , T 0 from chart
L EtOH w
P P EtOH P w
C C x C x
30
Where
EtOH w
x and x are mole fractions. Convert weight to mole fractions.
Basis: 100 kg mixture
30
30 kg EtOH 0.651 kmol
46.07

70 kg water 70 18.016 3.885
Total = 4.536 kmol
Avg.
100
MW 22.046
4.536
Mole fracs:
E w
0.6512
x 0.1435, x 0.8565
4.536
.
Use
L
EtOH
P P
C at 100 C as an average C value.
L
P
kcal
C 37.96 .1435 18.0 .8565 20.86
kmol C
Per kg this is
L
P
avg
C
20.86 kcal
0.946
MW 22.046 kg C
F
h 0.946 2000 189.2 kcal/kg
which can now be plotted on the enthalpy composition diagram.
Obtain
drum E E
T 88.2 C, x 0.146, and y 0.617 .
For F 1000 find L and V from F = L + V and Fz Lx Vy
which gives V = 326.9, and L = 673.1
Note: If use wt. fracs.
L L
P P avg
C 23.99 & C MW 1.088
F
and h 217.6 . All wrong.
31
2.D.10 New Problem. Solution 400 kPa, 70C
C4
z 35 Mole % n-butane
C6
x 0.7
From DePriester chart
C3 C4 C6
K 5, K 1.9, K 0.3
Know
i
i i i i i i i
i
z
y K x , x , x y 1 z
V
1 K 1
F
R.R.
i i
C3 C6 C4 C6
i
K 1 z
0 z 1 z z .65 z
V
1 K 1
F
For C6
C6 C6
C6
C6
z z V
0.7 z 0.7 1 0.7
V V
F
1 K 1 1 0.7
F F
C6
V
z 0.7 0.49
F
RR Eq:
C6 C6
4 .65 z 0.9 .35 0.7z
0
V V V
1 4 1 0.9 1 0.7
F F F
2 equations & 2 unknowns. Substitute in for
C6
z . Do in Spreadsheet.
Use Goal Seek to find V F.
V
0.594
F
when R.R. equation 0.000881.
C6
V
z 0.7 0.49 0.7 (0.49)(0.594) 0.40894
F
2.D11. L F 0.6 V F 0.4 & L V 1.5
Operating line: Slope 1.5, through y x z 0.4
32
2.D12. For problem 2.D1c, plot x = 0.2 on equilibrium diagram with feed composition of 0.3. The
resulting operating line has a y intercept z V/ F 1.2 . Thus V F 0.25 (see figure in
Solution to 2.D1) Vapor mole fraction is y = 0.58.
Find Liquid Density.
L
m m w w
MW x MW x MW .2 32.04 .8 18.01 20.82
Then,
w m
L m w
m w
MW MW 32.04 18.01
V x x .2 .8 22.51 ml/mol
.7914 1.00
L
L L
MW V 20.82 22.51 0.925 g/ml
Find Vapor Density.
v
v
p MW
RT
(Need temperature of the drum)
v
m w
m w
MW y MW y MW .58 32.04 .42 18.01 26.15 g/mol
Find Temperature of the Drum T:
From Table 2-7 find T corresponding to y .58, x 20, T=81.7 C 354.7K
33
4
v
ml atm
1 atm 26.15 g/mol 82.0575 354.7 K 8.98 10 g/ml
mol K
Find Permissible velocity:
perm drum L v v
2 3 4
drum,horizontal drum,vertical lv lv lv lv
u K
K 1.25 K exp A B nF C nF D nF E nF 1.25
Since V V F 0.25 1000 250 lbmol/h,
v
v
W V MW 250 26.15 lb lbmol 6537.5 lb / h
L
L
L F V 1000 250 750 lbmol/h, and W L MW 750 20.82 15, 615 lb/h,
4
V L
lv lv
V L
W 15615 8.98 10
F 0.0744, and n F 2.598
W 6537.5 .925
drum, vertical drum,horiz
4
perm 4
K 0.442, and K 0.5525
0.925 8.98 10
u 0.5525 17.74 ft/s
8.98 10
v
cs
4 3
perm v
V MW
250 26.15 454 g/lbm
A
u 3600 17.74 3600 8.98 10 g/ml 28316.85 ml/ft
2
Cs
A 1.824 ft ,
2
T Cs
A A 0.2 9.12 ft
With L/D = 4,
T
D 4A 3.41 ft and L 13.6 ft
2.D13. New Problem. The answer is y
cresol
= 0.19582
Since
p
c p p
p
x 1.76 .7
x 0.3, x 0.7, y 0.80418
1 1 x 1 .76 .7

c p
y 1 y 0.19582
Or
cp c
c
cp c
x
y
1 1 x
,
cp
pc
1
0.5682
2.D14. Raoults Law:
C4
C4
Tot
VP
K
P
4
10 C C4
10 C6 C6
log VP 4.04615 , VP 11121 mm Hg
log VP 3.2658 , VP 1844.36 mm Hg
i
i
i
z
x 1.0 1.0
1 K 1 V F

34
0.3 0.7
1
11121 1844.36
1 1 0.4 1 1 0.4
P P

Solve for P
drum
= 3260 mmHg
i
i
i
z
x
V
1 K 1
F
C4 C4 C4 C4
.3 11121
x 0.1527, y K x 0.1527 0.52091
11121 3260
1 1 .4
3260
C6 C4 C6
1844.36
x 1 x 0.84715, y 0.84715 0.47928
3260
Check 1.00019
2.D15. This is an unusual way of stating problem. However, if we count specified variables we see
that problem is not over or under specified. Usually V/F would be the variable, but here it
isnt. We can still write R-R eqn. Will have three variables: z
C2
, z
iC4
, z
nC4
. Need two other
eqns:
iC4 nC4 C2 iC4 nC4
z z constant, and z z z 1.0
Thus, solve three equations and three unknowns simultaneously.
Do It. Rachford-Rice equation is,
C2 C2 iC4 iC4 nC4 nC2
C2 iC4 nC4
K 1 z K 1 z K 1 z
0
V V V
1 K 1 1 K 1 1 K 1
F F F
Can solve for z
C2
= 1 z
iC4
and z
iC4
= (.8) z
nC4
. Thus z
C2
= 1 1.8 z
nC4

Substitute for z
iC4
and z
C2
into R-R eqn.
C2 iC4 nC4
nC4 nC4 nC4
C2 iC4 nC4
K 1 .8 K 1 K 1
1 1.8 z z z 0
V V V
1 K 1 1 K 1 1 K 1
F F F
Thus,
C2
C2
nC4
C2 iC4 nC4
C2 iC4 nC4
K 1
V
1 K 1
F
z
K 1 .8 K 1 K 1
1.8
V V V
1 K 1 1 K 1 1 K 1
F F F
Can now find K values and plug away. K
C2
= 2.92, K
iC4
= .375, K
nC4
= .26.
Solution is z
nC4
= 0.2957, z
iC4
= .8 (.2957) = 0.2366, and z
C2
= 0.4677
2.D16.
C1 C4 C5 C6 C1 C4 C5 C6
z 0.5, z 0.1, z 0.15, z 0.25, K 50, K .6, K .17, K 0.05
1
st
guess. Can assume all C
1
in vapor, ~ 1/3 C
4
in vapor, C
5
& C
6
in bottom
1
V/ F .5 .1 / 3 .53 This first guess is not critical.
35

R.R. eq.
i i
i
K 1 z
V
f 0
F 1 K 1 V F


49 .5 .4 .1 .83 .15 .95 .25
0.157
1 49 .53 1 .4 .53 1 .83 .53 1 .95 .53

Eq. 3.33
1
2
2 1
i i
2
i
f V F
V V
F F
z K 1
V
1 K 1
F

where
1 1
V/ F 0.53 and f V/ F 0.157.
calculate
2
V/ F .53 0.157 2.92 0.584
V .584 150 87.6 kmol/h and L 150 87.6 62.4

C1
C1
C1
z .5
x 0.016883
1 K 1 (V / F) 1 49 .584


C1 C1 C1
y K x 50 0.016883 0.844
Similar for other components.

2-D17. a. V 0.4F 400, L 600 Slope L F 1.5
Intercepts y = x = z = 0.70. Plot line and find x
A
= 0.65, y
A
= 0.77 (see graph)
b. V = 2000, L = 3000. Rest identical to part a.
c. Lowest x
A
is horizontal op line (L = 0). x
A
= 0.12
Highest y
A
is vertical op line (V = 0). y
A
= 0.52. See graph


36


d. V = 600, L = 400, -L/V = -0.667.
Find x
A
= 0.40 on equilibrium curve. Plot op line & find intersection point with
y = x line. z
A
= 0.52
2.D18. From
i
i
i
z
x
V
1 K 1
F
, we obtain
h
h
h
z
1
x V
F K 1

Guess
drum
T , calculate
h b p
K , K and K , and then determine V F .
Check:
1 i
1
K 1 z
0
1 K 1 V F
?
Initial guess: If
h
x .85 then
drum
T must be less than temperature to boil pure hexane

h
K 1.0, T 94 C . On this basis 85 to 90 would be reasonable. Try 85C.
h b p
K =0.8, K 4.8, K =11.7 .

0.6
1
V
0.85
1.471
F 0.8 1
. Not possible. Must have
h
0.6
K 0.706
0.85

Try T 73 C where
h
K 0.6 . Then
b p
K 3.8, K 9.9 .

0.6
1
V
.85
0.735
F .6 1

Check:

i i
i
K 1 z 8.9 .1 2.8 .3 .4 .6
0.05276
1 K 1 V F 1 8.9 .735 1 2.8 .735 1 .4 735

Converge on T ~ 65.6 C and V F ~ 0.57 .

2.D19. 90% recovery n-hexane means
C6 C6
0.9 Fz L x
Substitute in L F V to obtain
C6 C6
z .9 1 V F x

8
C balance:
C6 C6 C6 C6 C6 C6
z F Lx Vy F V x K Vx
or
C6 C6 C6 C6
z 1 V F x x K V F
Two equations and two unknowns. Remove
C6
x and solve

C6
C6 6
.9 z KV F
z .93C
1 V F

Solve for
C6
V .1
V F.
F .9K .1
. Trial and error scheme.
Pick T, Calc
C6
K , Calc V F, and Check f V F 0 ?
If not
new
ref old
ref
K T
K
1 d f T

37
Try
C4 C5 C6 ref
T 70 C. K 3.1, K .93, K .37 K
V .1
0.231
F .9 .37 .1
.
Rachford Rice equation
2.1 .4 .08 .25 .63 .35
f .28719
1 2.1 .231 1 .08 .231 1 .63 .231
ref new
.37
K T 0.28745 use .28
1 0.28719
Converge on
New C4 C8
T ~ 57 C. Then K 2.50, K .67, and V F 0.293.
2.D20. New Problem. The K values are:
E
K 8.7 ,
B
K 0.54,
P
K 0.14
Can use Eq. (2-40), (2-41) or (2-42). If we use (2-42) the R R eqn
i i
F B E B
i
K 1 z
V
f 0 Use z 1 z z .8 z
V
F
1 K 1
F
Then
B B
7.7 .2 .46 z .86 .8 z
RR eq =
1+7.7 25 1 .46 .25 1 0.86 .25
B B
0 0.5265 0.51977 z 0.8764 1.0955z
B B
0 .5757z 0.3499 z 0.6078
2.D21. a.)
C2 C5
K 4.8 K 0.153
Soln to Binary R.R. eq.
A B
B A
z z V
F K 1 K 1
V 0.55 0.45
0.5309
F .153 1 4.8 1

C2
C2 C2
C2
z 0.55
x 0.1823, y 0.8749
V
1 3.8 .5309
1 K 1
F
C5 C5
x 0.8177 , y 0.1251
Need to convert F to kmol.
Avg MW 0.55 30.07 0.45 72.15 49.17
kg kmol
F 100, 000 2033.7 kmol/h
hr 49.17 kg
V V F F 1079.7, L F V 954.0 kmol/h
b.)
L v
Perm drum
v
u K
To find L MW 0.1823 30.07 0.8177 72.15 64.48
38


V MW 0.8749 30.07 0.1251 72.15 35.33
For liquid assume ideal mixture:

C2 C5
1 C2 C2,liq C5 C5,liq C2 C5
C2,liq C5,liq
MW MW
V x V x V x x

L
30.07 72.15
V 0.1823 0.8177 103.797 ml/mol
0.54 0.63


L
L
L
MW 64.48
0.621 g/ml
V 103.797

For vapor: ideal gas:
v
v
MW
RT


v
atm g
700 kPa 35.33
101.3 kPa mol
0.009814 g / ml
ml atm
82.0575 303.16K
mol K


drum
K : Use Eq. (2-60) with
v L
lV
V L
W
F
W


V
kmol 64.48 kg
W 997.7 6, 4331.7 kg/h
h kmol


V
W 881.5 35.33 31,143.4 kg/h

lV
64331.7 0.009814
F 0.2597
31,143.3 0.621



2
drum
3 4
K exp 1.877478 0.81458 n .2597 0.18707 n 0.2597
0.0145229 n 0.2597 0.0010149 n 0.2597 0.3372


Perm
0.621 0.009814
u 0.3372 2.6612 ft/s
0.009814
ft 1.0 m
2.6612 0.8111 m/s
s 3.2808 ft


2 V
C
6 3
Perm v
3 3
kmol kg
1079.7 35.33
V MW h kmol
A 1.392 m
u 3600 m s g kg 10 cm
0.8111 3600 0.009814
s h cm 1000g m


C
D 4A 1.33 m
Arbitrarily
L D 4, L 5.32 m
39
2.D22.
i i iP iP NP NP
i
iP NP
K 1 z K 1 z K 1 z
V
f 0
V V
F 1 K 1 V F
1 K 1 1 K 1
F F
Solve for V/F.
NP iP iP iP NP NP
V V
1 K 1 K 1 z 1 K 1 K 1 z 0
F F
iP iP NP NP
NP iP
K 1 z K 1 z
V
F K 1 K 1
where
iP NP
z z 1.0
drum tot
p p 760 mm Hg, T 90 C
10
NP
1499.2
log VP 7.84767 2.75943
90 204.64

NP
VP 574.68 mm Hg ,
NP tot
K 574.68 p 0.75616
10 iP
1580.9
log VP 8.11778 3.011679
20 219.61
iP
VP 1027.256 mm Hg ,
iP
K 1027.256 760 1.35165
Note:
iP NP iP
MW MW . z 0.5 in both wt & mol frac., as does
NP
z .
0.35165 0.5 0.24384 0.5
V
0.629
F 0.24384 0.35165

iP
iP
iP
z
x 0.4095
V
1 K 1
F
NP iP ip iP iP NP
x 1 x 0.5905; y K x 0.55347 y 0.44653
2.D23. 5. 0C, 2500 kPa
Fig 2.12:
M ethylene Ethane C6
K 5.7, K 1.43, K 0.98, K 0.007
First, try
1
V
0.6
F
(equal split ethylene and ethane)
1
.47 .4 0.43 .05 .02 0.35 0.993 0.2
V
f 0.0108
F 1 4.7 .6 1 .43 .6 1 .02 .6 1 .993 6
Eq. (2-46)
1
2
2 1
i i
2
i
V
f
V V F
0.6059
F F
z K 1
V
1 K 1
F
Then Eq. (2-38),
i
i M ethylene
i
z
x . x 0.104, x 0.040
1 K 1 V F

ethane C6
x 0.354, x 0.502, 1.0001 OK
Find
i i i
y K x
40
2.D24. New Problem. p = 300 kPa
At any T.
C3 C3 C3
K y x
Ks are known.
C6 C6 C6 C3 C3
K y x 1 y 1 x
Substitute 1
st
equation into 2
nd

C6 C3 C3 C3
K 1 K x 1 x
Solve for x
C3,

C3 C6 C3 C3
1 x K 1 K x
C3 C3 C6 C6
x K K 1 K
C3 C6 C6
C3 C3
C3 C6 C3 C6
K 1 K 1 K
x & y
K K K K
At 300 kPa pure propane
C3
K 1.0 boils at -14C (Fig. 2-11)
At 300 kPa pure n-hexane
C6
K 1.0 boils at 110C
Check: at -14C
C6
C3
C6
1 K
x 1,
1 K

C6
C3
C6
1 1 K
y 1.0
1 K
at 110C
C3
C3
0
x 0,
K
C3
C3
C3
K 0
y 0
K
Pick intermediate temperatures, find
C3
K &
C6
K , calculate
C3
x &
C3
y .
T
0C 1.45 0.027
0.9915
10C 2.1 0.044 0.465 0.976 See
20C 2.6 0.069 0.368 0.956 Graph
30C 3.3 0.105 0.280 0.924
40C 3.9 0.15 0.227 0.884
50C 4.7 0.21 0.176 0.827
60C 5.5 0.29 0.136 0.75
70C 6.4 0.38 0.103 0.659
C6
K
C3
K
C3
x
1- 0.027
= 0.684
1.45- 0.027
C3 C3 C3
y K x
41
42
b.
C3
x 0.3 , V F 0.4, L V 0.6 0.4 1.5
Operating line intersects y x 0.3, Slope 1.5
L F
y x z
V V
at
F 0.3
x 0, y z 0.75
V 0.4
Find y
c3
= 0.63 and x
C3
= 0.062
Check with operating line: 0.63 1.5 .062 0.75 0.657 OK within accuracy of the graph.
c. Drum T:
C3 C3 C3
K y x 0.63 0.062 10.2, DePriester Chart T = 109C
d. y .8, x ~ .16
L y .8 .6
Slope 0.45
V x .16 .6
1 f
.45
f

V 1
f 0.69
F 1.45
2.D25. New Problem. 20% Methane and 80% n-butane
drum
T .50 C ,
V
0.40
F
, Find
drum
p
A A B B
B A
K 1 z K 1 z
V
0 f
V V F
1 K 1 1 K 1
F F
Pick
drum C4 nC4
p 1500 kPa: K 13 K 0.4
(Any pressure with
C1 C4
K 1 and K 1.0 is OK)
Trial 1
1
12 .2 .6 .8
f 0.2178
1 12 .4 1 .6 .4
Need lower
drum
p
C4 old
C4 new
old
K P 0.4
K P 0.511
1 d f P 1 .2138
1.0
new C1
P 1160 K 16.5
2
15.5 .2 .489 .8
f 0.4305 .4863 0.055769
1 15.5 .4 1 .489 .4
C4 new
0.511
K P 0.541
1 0.055769

new C1
P 1100 K 17.4
3
16.4 .2 .459 .8
f 0.0159
1 16.4 .4 1 .459 .4
, OK. Drum pressure = 1100 kPa
43

b.)
i
i C1
i
z 0.2
x , x 0.02645
V
1 16.4 .4
1 K 1
F


C1 C1 C1
y K x 17.4 0.02645 0.4603

2.D26. New Problem. a) Can solve for L and V from M.B. 100 = F = V + L
45 Fz 0.8V 0.2162L
Find: L = 59.95 and V = 40.05
b) Stage is equil.
C3
C3
C3
y 0.8
K 3.700
x 0.2162


C5
0.2
K .2552
0.7838

These K values are at same T, P. Find these 2 K values on DePriester chart.
Draw straight line between them. Extend to
drum drum
T , p . Find 10C, 160 kPa.
2.D27. New Problem. a.)
C5 10
1064.8
VP : log VP 6.853 2.2832
0 233.01
,
VP 191.97 mmHg
b.) VP 3 760 2280 mmHg ,

10
log VP 6.853 1064.8 / T 233.01
Solve for T = 71.65C
c.)
tot
P 191.97 mm Hg [at boiling for pure component
tot
P VP ]
d.) C5:
10
1064.8
log VP 6.853 2.8045
30 233.01

VP 637.51 mm Hg

C5 C5 tot
K VP P 637.51 500 1.2750

C6:
10 C6
1171.17
log VP 6.876 2.2725
30 224.41


C6
VP 187.29 mm Hg

C6
K 187.29 500 0.3746
e.)
A A A B B B A A
K y x K y x (1 y ) / (1 x )
If
A B
K &K are known, two eqns. with 2 unknowns
A A
K &y Solve.

C6
C5
C5 C6
1 K 1 0.3746
x 0.6946
K K 1.2750 0.3746


C5 C5 C5
y K x 1.2750 0.6946 0.8856
f.) Overall, M.B., F = L + V or 1 = L + V

F
C5: Fx Lx Vy .75 0.6946 L + 0.8856 V
Solve for L & V: L = 0.7099 & V = 0.2901 mol
g.) Same as part f, except units are mol/min.


44

2.D28. New Problem.











From example 2-4,
H drum H H
x 0.19, T 378K, V F 0.51, y 0.6, z 0.40
With
v
perm v
V MW
h D C, D
u 3600 C


C=4, MW
v
= 97.39 lbm/lbmole (Example 2-4)

3
v 3 3
1 28316.85ml lbm
3.14 10 g mol 0.198
454g lbm ft ft

Example 2.4


L v
perm drum horiz vertical
V
u K , K 1.25 K
From Example 2-4,
vertical
K 0.4433 ,
horiz
K 1.25 0.4433 0.5541

1 2
perm
0.6960 0.00314
u 0.5541 8.231 ft s
0.00314
[densities from Example 2-4]

V lbmol
V F 0.51 3000 1530 lbmol hr
F hr


3
lbmol lbm
1530 97.39
h lbmol
D 5.067 ft
ft s lbm
8.231 3600 0.1958
s h ft

h 4D 20.27 ft
Use
1
5 20 or 5 22
2
ft drum.

2.D29. New Problem. The stream tables in Aspen Plus include a line stating the fraction vapor in a given
stream. Change the feed pressure until the feed stream is all liquid (fraction vapor = 0). For the Peng-
Robinson correlation the appropriate pressure is 74 atm.
The feed mole fractions are: methane = 0.4569, propane = 0.3087, n-butane = 0.1441, i-butane = 0.0661,
and n-pentane = 0.0242.
b. At 74 atm, the Aspen Plus results are; L = 10169.84 kg/h = 201.636 kmol/h, V = 4830.16 kg/h =
228.098 kmol/h, and T
drum
= -40.22
o
C.
h
F
D
L
V
45

The vapor mole fractions are: methane = 0.8296, propane = 0.1458, n-butane = 0.0143, i-butane = 0.0097,
and n-pentane = 0.0006.
The liquid mole fractions are: methane = 0.0353, propane = 0.4930, n-butane = 0.2910, i-butane =
0.1298, and n-pentane = 0.0509.
c. Aspen Plus gives the liquid density = 0.60786 g/cc, liquid avg MW = 50.4367, vapor density =
0.004578 g/cc = 4.578 kg/m
3
, and vapor avg MW = 21.17579 g/mol = kg/kmol.

Since the flow area for vapor = LD and L = 4D, the area for flow = 4D
2
. Then the equation for the drum
diameter is

D = {[(MW
V
) V]/[
V
u
perm
(L/D)]}
0.5
= {[(21.17579 kg/kmol)(228.098 kmol/h)]/[(4.578 kg/m
3
)(u
perm

ft/s)(1 m/3.281 ft)(3600 s/h)(4)]
0.5


where the unit conversions are used to give D in meters. The value of u
perm
(in ft/s) can be determined by
combining Eqs. (2-59) and (2-60) for vertical drums with Eq. (2-64a).

F
lv
= (W
L
/W
V
)[
V
/
L
]
0.5
= (10169.84/4830.16)[0.004578/0.60786]
0.5
= 0.18272

Resulting K
vertical
= 0.378887 , K
horizontal
= 0.473608, and u
perm
= 5.436779 ft/s, and D = 0.4896 m and L =
1.9585 m. Appropriate standard size would be used.

2.D30. New Problem. a. From the equilibrium data if y
A
= .40 mole fraction water, then x
A
= 0.09 mole
fraction water.
Can find L
A
and V
A
by solving the two mass balances for stage A simultaneously.
L
A
+ V
A
= F
A
= 100 and L
A
(.09) + V
A
(.40) = (100) (.20). The results are V
A
= 35.48 and L
A
= 64.52.

b. In chamber B, since 40 % of the vapor is condensed, (V/F)
B
= 0.6. The operating line for this flash
chamber is,
y = -(L/V)x + F
B
/V) z
B
where z
B
= y
A
= 0.4 and L/V + .4F
B
/.6F
B
= 2/3. This operating line goes through
the point y = x = z
B
= 0.4 with a slope of -2/3. This is shown on the graph. Obtain x
B
= 0.18 & y
B
= 0.54.
L
B
= (fraction condensed)(feed to B) = 0.4(35.48) = 14.19 kmol/h and V
B
= F
B
L
B
= 21.29.

c. From the equilibrium if x
B
= 0.20, y
B
= 0.57. Then solving the mass balances in the same way as for
part a with F
B
= 35.48 and z
B
= 0.4, L
B
= 16.30 and V
B
= 19.18. Because x
B
= z
A
, recycling L
B
does not
change y
B
= 0.57 or x
A
= 0.09, but it changes the flow rates V
B,new
and L
A,new
. With recycle these can be
found from the overall mass balances: F = V
B,new
+ L
A,new
and Fz
A
= V
B,new
y
B
+ L
A,new
x
A
. Then V
B,new
=
22.92 and L
A,new
= 77.08.

46

Graph for problem 2.D30.


47


2.E1. New Problem. From Aspen Plus run with 1000 kmol/h at 1 bar, L = V = 500 kmol/h, W
L
= 9212.78
kg/h, W
V

= 13010.57 kg/h, liquid density = 916.14 kg/m
3
, liquid avg MW = 18.43, vapor density = 0.85
kg/m
3
, and vapor avg MW = 26.02, T
drum
= 94.1
o
C, and Q = 6240.85 kW.

The diameter of the vertical drum in meters (with u
perm
in ft/s) is

D = {[4(MW
V
) V]/[3600
V
u
perm
(1 m/3.281 ft)]}
0.5
=
{[4(26.02)(500)]/[3600(3.14159)(0.85)(1/3.281)u
perm
]}
0.5


F
lv
= (W
L
/W
V
)[
V
/
L
]
0.5
= (9212.78/13010.57)[0.85/916.14]
0.5
= 0.02157

Resulting K
vertical
= 0.404299, and u
perm
= 13.2699 ft/s, and D = 1.16 m. Appropriate standard size would
be used. Mole fractions isopropanol: liquid = 0.00975, vapor = 0.1903

b. Ran with feed at 9 bar and p
drum
at 8.9 bar with V/F = 0.5. Obtain W
L
= 9155.07 kg/h, W
V
= 13068.27,
density liquid = 836.89, density vapor = 6.37 kg/m
3

D = {[4(MW
V
) V]/[3600
V
u
perm
(1 m/3.281 ft)]}
0.5
=
{[4(26.14)(500)]/[3600(3.14159)(6.37)(1/3.281)u
perm
]}
0.5


F
lv
= (W
L
/W
V
)[
V
/
L
]
0.5
= (9155.07/13068.27)[6.37/836.89]
0.5
= 0.06112

Resulting K
vertical
= .446199, u
perm
= 5.094885 ft/s, and D = 0.684 m. Thus, the method is feasible.

c. Finding a pressure to match the diameter of the existing drum is trial and error.

If we do a linear interpolation between the two simulations to find a pressure that will give us D = 1.0 m
(if linear), we find p = 3.66. Running this simulation we obtain, W
L
= 9173.91 kg/h, W
V
= 13049.43,
density liquid = 874.58, density vapor = 2.83 kg/m
3
, MW
v
= 26.10

D = {[4(MW
V
) V]/[3600
V
u
perm
(1 m/3.281 ft)]}
0.5
=
{[4(26.10)(500)]/[3600(3.14159)(2.83)(1/3.281)u
perm
]}
0.5


F
lv
= (W
L
/W
V
)[
V
/
L
]
0.5
= (9173.91/13049.43)[2.83/874.58]
0.5
= 0.0400

Resulting K
vertical
= .441162, u
perm
= 7.742851 ft/s, and D = 0.831 m.

Plotting the curve of D versus p
drum
and setting D = 1.0, we interpolate p
drum
= 2.1 bar

At p
drum
= 2.1 bar simulation gives, W
L
= 9188.82 kg/h, W
V
= 13034.53, density liquid = 893.99 , density
vapor = 1.69 kg/m
3
, MW
v
= 26.07.

D = {[4(MW
V
) V]/[3600
V
u
perm
(1 m/3.281 ft)]}
0.5
=
{[4(26.07)(500)]/[3600(3.14159)(1.69)(1/3.281)u
perm
]}
0.5


F
lv
= (W
L
/W
V
)[
V
/
L
]
0.5
= (9188.82/13034.53)[1.69/893.99]
0.5
= 0.0307

Resulting K
vertical
= .42933, u
perm
= 9.865175ft/s, and D = 0.953 m.

This is reasonably close and will work OK. T
drum
= 115.42
o
C, Q = 6630.39 kW,
48

Mole fractions isopropanol: liquid = 0.00861, vapor = 0.1914

In this case there is an advantage operating at a somewhat elevated pressure.



2.E2. This problem was 2.D13 in the 2
nd
edition of SPE.
a. Will show graphical solution as a binary flash distillation. Can also use R-R equation. To
generate equil. data can use

C6 C8 C6 C8 C6 C6 C8 C8
x x 1.0, and y y K x K x 1.0
Substitute for x
C6

C8
C6
C6 C8
1 K
x
K K

Pick T, find K
C6
and K
C8
(e.g. from DePriester charts), solve for x
C6
. Then y
C6
= K
C6
x
C6


TC K
C6
K
C8
x
C6
y
C6
= K
C6
x
C6
125 4 1.0 0 0
120 3.7 .90 .0357 .321
110 3.0 .68 .1379 .141
100 2.37 .52 .2595 .615
90 1.8 .37 .4406 .793
80 1.4 .26 .650 .909
66.5 1.0 .17 1.0 1.0

Op Line Slope
L 1 V F .6
1.5
V V F .4
, Intersection y = x = z = 0.65.
See Figure.
y
C6
= 0.85 and x
C6
= 0.52. Thus K
C6
= .85/.52 = 1.63.
This corresponds to T = 86C = 359K


b. Follows Example 2-4.
49


L
C8 C8
C6 C8
MW x MW x MW .52 86.17 .48 114.22 99.63

C6 C8
L C6 C8
C6 C8
MW MW
86.17 114.22
V x x .52 .48 145.98 ml/mol
.659 .703


3
L
L 3
L
MW 99.63 28316 ml/ft lbm
.682 g/ml 42.57
V 145.98 454 g/lbm ft


v
C6 C6 C8 C8
MW y MW y MW .85 86.17 .15 114.22 90.38

v
3
v
1.0 90.38 g/mol
pMW
0.00307 g/ml 0.19135 lbm/ft
ml atm
RT
82.0575 359K
mol K

Now we can determine flow rates

V
V F .4 10, 000 4000 lbmol/h
F

v
v
W V MW 4000 90.38 361, 520 lb/h
L
L
L F V 6000 lbmol/h, W L MW 6000 99.63 597, 780 lb/h

v L
lv lv
v L
W 597, 780 0.19135
F 0.111, nF 2.1995
W 361, 520 42.57


2
drum
3 4
K exp 1.87748 .81458 2.1995 .18707 2.1995
0.01452 2.1995 0.00101 2.1995 0.423


Perm drum L v v
u K 0.423 42.57 19135 .19135 6.30 ft/s

v
2
Cs
Perm v
V MW
4000 90.38
A 83.33 ft
u 3600 6.3 3600 0.19135


Cs
D 4A 4 83.33 10.3 ft. Use 10.5 ft.
L ranges from 3 10.5 = 31.5 ft to 5 10.5 = 52.5 ft.

Note: This u
Perm
is at 85% of flood. If we want to operate at lower % flood (say 75%)
have

75% 85%
Perm Perm
u 0.75 0.85 u 0.75 0.85 .63 5.56
Then at 75% of flood, A
Cs
= 94.44 which is D = 10.96 or 11.0 ft.

2.F1

x
B
0 .1 .2 .3 .4 .5 .6 .7 .8 .9 1
y
B
0 .22 .38 .52 .62 .71 .79 .85 .91 .96 1

Benzene-toluene equilibrium is plotted in Figure 13-8 of Perrys Chemical Engineers
Handbook, 6
th
ed.

2.F2. See Graph. Data is from Perrys Chemical Engineers Handbook, 6
th
ed., p. 13-12.

50




Stage 1)
1
F
2 3
z .4 f 1 3 Slope 2,
1 3

Intercept
1 1 2
.4
1.2 y .872 x .164 z
1 3

Stage 2)
2
F
1 3
z .164 f 2 3 Slope 1 2
2 3

Intercept
2 2 3
.164
.246 x .01 y .240 z
2 3

Stage 3)
3
F
z .240 f 1 2 Slope 1
Intercept
3 3
.240
.480 x .022 y .461
1 2


2.F3. Bubble Pt. At P = 250 kPa. Want
1 1
K z 1
Guess
1 2 3
T 18 C, K 1, K .043, K .00095, .52
Converge to T 0 C
Dew Pt. Calc. Want
1
1
z
1.0
K

Try
1 2 3
T 0 C, K 1.93, K 0.11, K 0.0033, 120.26
51

Converge to T 124 C. This is a wide boiling feed.
T
drum
must be lower than 95C since that is feed temperature.
First Trial: Guess
d,1 1 2 3
T 70 C: K 7.8, K 1.07, K .083
Guess V F 0.5 . Rachford Rice Eq.

7.8 1 .517 .07 .091 .083 1 .392
f V F .14
1 6.8 .5 1 .07 .5 1 .083 1 .5

V F .6 gives f .6 .101
By linear interpolation V F .56. f 0.56 .0016 which is close enough for first
trial.
V V F F 56, L 44

i
i i i i
i
z
x and y K x
1 K 1 V F


1 2 3
x .1075 x .088 x .806 x 1.001

1 2 3
y .839 y .094 y .067 y .9999
Data: Pick
ref
T 25 C. (Perrys 6
th
ed; p. 3-127), and (Perrys 6
th
ed; p. 3-138)

1
81.76 cal/g 44 3597.44 kcal/kmol

2
87.54 cal/g 72 6302.88 kcal/kmol

3
86.80 cal/g 114 9895.2 kcal/kmol
at
pL1
T 0 C, C 0.576 cal / (g C) 44 25.34 kcal/(kmol C) .
For
pL3
T 20 to 123 C, C 65.89 kcal/(kmol C)
at
pL2
T 75 C, C 39.66 kcal/(kmol C) . (Himmelblau/Appendix E-7)

2
pv
C a bT cT
propane a = 16.26 b = 5.398 10
-2
c = -3.134 10
-5

n-pentane a = 27.45 b = 8.148 10
-2
c = -4.538 10
-5


**n-octane a = 8.163 b = 140.217 10
-3
c = -44.127 10
-6

** Smith & Van Ness p. 106
Energy Balance: E(T
d
) = VH
v
+ Lh
L
Fh
F
= 0
F
Fh 100 .577 25.34 .091 39.66 .392 65.89 95.25 297, 773 kcal/h

L
Lh 44 .1075 25.34 .088 39.66 .806 65.89 70.25 117, 450

2
v
2
3
VH 56 .839 3597.4 16.26 5.398 10 45
0.94 6302.88 27.45 8.148 10 45
0.67 9895.3 8.163 140.217 10 45 240, 423


drum
E T 60,101 Thus, T
drum
is too high.
Converge on
drum 1 2 3
T 57.2 C: K 6.4, K .8, K .054
For V F 0.513, f 0.513 0.0027. V 51.3, L 48.7
52


1 2 3 1
x .137, x .101, x .762, x 1.0000

1 2 3 1
y .878, y .081, y .041, y 1.0000

F L v drum
Fh 297, 773; Lh 90, 459; VH 209, 999; E T 2685
Thus T
drum
must be very close to 57.3C.

1 2 3
x .136, x .101, x .762

1 2 3
y .328, y .081, y .041
V 51.3 kmol/h, L 48.7 kmol/h
Note: With different data T
drum
may vary significantly.

2.F4. New Problem. yV Lx Fz or
L F
y x z
V V


L
V F 0.4, V 4kmol h , L 6, 1.5 slope
V


F
x 0, y z 2.5 .25 0.625
V

Find: V = 4 kmol/h, L = 6 kmol/h.

From the graph, x = 0.19 y = 0.34

Equilibrium is from NRTL on Aspen Plus.

53



FIGURE 2.F.4.



2.G1. Used Peng-Robinson for hydrocarbons.

Find
drum
T 33.13 C, L 34.82 and V 65.18 kmol/h
In order ethylene, ethane, propane, propylene, n-butane, x
i
(y
i
) are:
0.0122 0.0748 , 0.0866 0.3005 , 0.3318 0.3781 , 0.0306 0.0404 , 0.5388 0.2062.


2.G2. Used Peng-Robinson. Find
drum
T 30.11 C, L 31.348, V 68.66 kmol/h.
In same order as 2.G1,
i i
x y are: 0.0189 0.1123 , 0.0906 0.3023 , 0.3255 0.3495 , 0.0402 0.0501 , 0.5248 0.1858 .

54

2.G3. Used NRTL-2.
drum
T 79.97 C,
M M
x 0.2475, y 0.6287 . Compares to graph with
M M
x 0.18 and y 0.55 . Different equilibrium data.

2.G4. New Problem.

COMP x(I) y(I)
METHANE 0.12053E-01 0.84824
BUTANE 0.12978 0.78744E-01
PENTANE 0.29304 0.47918E-01
HEXANE 0.56513 0.25101E-01
V/F = 0.58354

2.G5. New Problem. Used NRTL. T = 368.07, Q = 14889 kW, 1
st
liquid/total liquid = 0.4221,
Comp Liquid 1, x
1
Liquid 2, x
2
Vapor, y
Furfural 0.630 0.0226 0.0815
Water 0.346 0.965 0.820
Ethanol 0.0241 0.0125 0.0989

2.G6. New Problem. Used Peng Robinson. Feed pressure = 10.6216 atm, Feed temperature = 81.14
o
C,
V/F = 0.40001, Q
drum
=0. Note there are very small differences in feed temperature with
different versions of Aspen Plus.


COMP x(I) y(I)
METHANE 0.000273 0.04959
BUTANE 0.18015 0.47976
PENTANE 0.51681 0.39979
HEXANE 0.30276 0.07086
V/F = 0.40001


2.H1. New Problem. The spreadsheet with equations for problem 2.D16 is shown in Appendix B of
Chapter 2. The spreadsheet with numbers for i-butane replacing n-butane is below.


MC flash, HW 2.G.b., MC flash with i-
butane
K const. aT1 aT2 aT6 ap1 ap2 ap3
M -292860 0 8.2445 -0.8951 59.8465 0
iB -1166846 0 7.72668
-
0.92213 0 0
nPentane -1524891 0 7.33129
-
0.89143 0 0
nHex -1778901 0 6.96783
-
0.84634 0 0
T deg R 509.688
p
psia 36.258 F 150
zM 0.5 z iB 0.1 z np 0.15 znhex
V/F 0.602698586

0.25
55

guess
KM 51.86751896
KiB 0.926804057
KnPen 0.175621816
KnHex 0.05400053


Use goal seek for cell B24 to = 1.0 change B9
xM 0.015793905
xib 0.104615105
xnPen 0.29812276
xnHex 0.581601672
Sum 1.000133443
RR M 0.803396766
RR nB -0.007657401
RRnP -0.2457659
RRnHex -0.550194874
sum RR -0.000221409


2.H3. New Problem. Use the same spreadsheet as for problem 2H1, but with methane feed mole fraction
= 0.
Answer: V/F = 0.8625,
xib 0.08596648
xnPen 0.203540261
xnHex 0.710481125
KiB 3.886544834
KnPen 1.264637936
KnHex 0.574940847
yib = xib Kib = .33411 and so forth

56
Chapter 3
New Problems and new solutions are listed as new immediately after the solution number. These new
problems are: 3A7, 3A10, 3A11, 3C3, 3C4, 3D4, 3D8, 3G2.
3.A7. Simultaneous solution is likely when one of the key variables can be found only from the
energy balances. For example, if only 1 of
D B
x , x , D, B,
A dist
FR are given energy balances
will be required. This is case for most of the simulation problems and for a few design
problems. In some simulation problems the internal equations have to be solved also.
3.B1. a.
D B
x , x , opt feed,
Reb
Q
D B
x , x , opt feed,
C
Q
D B
x , x , opt feed, S (open steam), satd vapor steam
All of above with fractional recoveries set instead of
D B
x , x
D,
B
x , opt feed, L/D
b. N,
F
N , col diameter, frac. recoveries both comp.
N,
F
N , col diameter,
A o
FR dist, L D
N,
F
N , col diameter,
A R
FR dist, Q
N,
F
N , col diameter,
A C
FR dist, Q
N,
F
N , col diameter, ( )
D C B
x , Q or x
N,
F
N , col diameter, S (satd steam), satd vapor steam, ( )
D B
x or x
Many other situations are possible [e.g., 2 feeds, side streams, intermediate condensers or
reboilers etc.]
3.C1. See solution to problem 3-D2.
3.C2. See solution to problem 3-D3.
3.C3. New Problem in 3
rd
Edition.
mix 1 2
F F F D B = + = +
mix mix 1 1 2 2 D B
F z Fz F z Dx Bx = + = + (Mole frac. MVC)
1 1 2 2
mix
mix
Fz F z
z
F
+
=
Now solve like 1 Feed Column ( )
mix mix
F & z . From Eq. (3-3),
mix B
mix
D B
z x
D F
x x

kmol/h.
mix
B F D = kmol/h.
3.C4. New Problem in 3
rd
Edition. See solution to 3D4, Part b.
57

3.D1.

Mass balance calculation is valid for parts a & b for problem 3G1.
a)
( ) ( ) ( )
o
C 0 D 1
L
3, Eq 3-14 Q 1 L / D D h H
D
= = +

D
h is a saturated liquid at
D
x 0.85 = wt. frac. From Fig. 2-4,
D
h ~ 45 = kcal/kg

1
H is saturated vapor at
D 1 1
x y 0.85, H ~ 310 = = kcal/kg
( )( )( )
C
Q 1 3 764.62 45 310 810, 497 = + = kcal/hour
EB around column.
1 2
1 F 2 F col C R D B
Fh F h Q Q Q Dh Bh + + + + = +

( ) ( )
1 2
F F
h 81 C, 60 wt% ethanol ~190kcal / kg; h 20 C, 10 wt% ethanol ~10kcal / kg

B
h (satd liquid leaves equil contact, ~ 0 wt% ethanol) ~ 100 kcal/kg, Q
col
= 0 (adiabatic)
( )( ) ( )( ) ( ) ( ) ( )
R
Q 764.62 45 735.38 100 1000 190 500 10 810, 497 657, 259 = + = kcal/kg
(b) V B 2.5 = mass.
1
F
2
F

D
D, x
L

V
R
Q

B
B, x
o
L
1
V
1

C
Q
( )
1 2 B
F F B D x 0.0001 0.01% + = + =
1 1 2 2 B D D
Fz F z Bx D x x 0.85 + = + =
avg B
total
D B
z x
Solve D F
x x
| |
=
|

\ .

total 1 2
F F F 1500 kg/h = + =
( )
0.43333 0.0001
D 1500 764.62 kg/h
0.85 0.0001

= =


total
kg
B F D 1500 764.62 735.38
h
= = =
( )( ) ( )
1 1 2 2
avg
total
1000 .60 500 0.10
Fz F z
z 0.43333
F 1500
+
+
= = =
58

Approximately
B V L
x ~ y ~ x . Thus
B L
h h 100 = = .
V
H 640kcal kg ~
( )( ) ( )( ) ( )( )
R
Q 1838.45 640 735.38 100 2573.83 100 992, 763 kcal / h = + =

EB.
1 2
C D B col 1 F 2 F R
Q Dh Bh Q Fh F h Q = +

C
Q 34407.9 7353.8 190, 000 5000 992, 763 1,146, 001 kcal/h = + =

3.D2. Column: mass bal: F + S = D + B (1)
MVC: Fz +
S D B
Sy Dx Bx = + (2)
Note:
S
y 0 =
energy bal:
f s C D B
Fh SH Q Dh Bh + + = + (3)
Condenser: mass bal. :
1 o
V L D = + (4)
energy bal.: ( )
1 1 C o D
VH Q L D h + = + (5)
Solve Eqs. (1) and (2) to get:

( ) ( ) ( )
B B
D B
100 .3 100 .05 100 .05
Fz Fx Sx
D 36.4
x x .6 .05


= = =


Note: Not Eq. (3-3). Solve Eqs. (4) and (5) to get:
( )( )
C D 1
Q D 1 L D h H = +
Substitute
C
Q into Eq. (3):

( )
( )
D B F
D 1
Dh F S D h Fh SH
L
1
D D h H
+ +
=

S

From Figure 2-4:
F D B 1
h 8, h 65, h 92, H 638, H 608 kcal/kg. = = = = =
S


( ) ( ) ( ) ( )
( )
36.4 65 163.6 92 100 8 100 638
L D 1 2.77
36.4 65 408
+
= =



3.D3. External balances: F + C = B + D (1)

C B D
Fz Cx Bx Dy + = + (2)

R F C B D
Q Fh Ch Bh DH + + = + (3)
R B L V
Lh Q VH Bh + = +
L

Reboiler
Satd liqd
B,
B
x =0.0001
R
Q
(Satd vapor)
V
Satd liqd
L V B L 1838.45 735.38 2573.83 = + = + =

V B 2.5 or V 2.5B 1838.45 = = =
59

F = 2000, C = 1000, z = .4,
C B D
x 1.0, x .05, y .80, = = =
( )
F
h 20 C 30.7, =
( )
C
h sat ' d liquid 50, = ( ) ( )
B D
h sat ' d liquid 92, H sat ' d vapor 327 kcal/kg = =
Around reboiler: L V B = +

N Reb B
Lx Vy Bx = +

N R reb B
Lh Q VH Bh + = +
For a total reboiler:
N B N N B N B
x x , y x x , h h 92 = = = = =
M.B.:
( )
N R reb B
V B h Q VH Bh + + = +
or
R
B N
reb N
Q
V since h h
H h
= =



Reb
H 617 = (saturated vapor at
N 1
y 0.05
+
= )
Solve Eqs. (1) and (2) for B:
C D D
B D
Fz Cx Fy Cy
B
x y
+
=


Thus
( )
800 1000 1600 800
B 800 and D 2200
.05 .8
+
= = =


From Eq. (3),
R B D F C
Q Bh DH Fh Ch = +

( )( ) ( )( ) ( )( ) ( )( )
R
Q 800 92 2200 327 2000 30.75 1000 50 804, 500 cal/h = + =

R
reb N
Q 804500
V 1532.4 kg/h
H h 617 92
= = =



3.D4. New Problem in 3
rd
Edition. F B D = +
MVC
B D
Fz Bx Dy = +

But given recoveries. Thus, use:

F,M D,M
Fz (Frac Rec Methanol in distillate) Dy =

and
F,W B,W
Fz (Frac Rec water in bottom) Bx =

F,M F,W F,M
z 0.3, z 1 z 0.7 = = =

D,M B,W
y unknown, x unknown.


( )( )
D,M
Methanol 29.7 100 0.3 .99 Dy = =

If 99% methanol recovered in distillate, 1% is in bottoms
( )( )
B,M
0.3 100 0.3 0.01 Bx = =

Water ( )( )
B,W
68.6 100 0.7 0.98 Bx 2% = = water in distillate
( )( )
D,W
1.4 100 0.7 0.02 Dy = =
Since
i
x 1 = and
( ) ( )
i D,i B,i
y 1, Dy D, and Bx B = = =
Thus,
D,M D,W
D Dy Dy 29.7 1.4 31.1 kmol h = + = + =

B,M B,W
B Bx Bx 0.3 68.6 68.9 kmol h = + = + =
Check: B D 100 F + = =
OK

60


D,M
d,M
Dy
y 29.7 31.1 0.955
D
= = =

B,M B,M
x Bx B 0.3 68.9 0.00435 = = =
a)
0
D 31.1 & L D 2. = = Thus
0
L 62.2 =
Reflux liquid is in equilibrium with vapor
D,M
y 0.955 =
From equilibrium data (Table 2-7)
M,0
x ~ 0.893 (linear interpolation)
b) E.B. Partial condenser:
1 1 c 0 0 0
VH Q DH L h + = +
1 0
V D L 93.3 = + =
( )
1 1 0 0 0 1 0 0 0 1
Vy Dy L x y Dy L x V = + = +
( )( ) ( )
1,M
y 29.7 62.2 0.893 93.3 0.914; = + =
1,W 1,M
y 1 y 0.086 = =

M
35, 270J mol @64.5 C 35, 270 kJ / kmol = = choose MeOH reference 64.5C.
40, 656J mol@100 C 40, 656 kJ/kmol = = choose water reference 100C.
The condenser is at 66.1C (linear interpolation Table 2-7).
( ) ( )
1 M 1,M W 1,W 1,M P,V,M 1 1,W P,V,W 1
H y y y C T 64.5 y C T 100 = + + +

1
V is at
1
T in equilibrium with
1,M
y 0.914. = From Table 2-7
1
T ~ 67.6 C
Assuming only constant & linear T term are important in
P,V
C eqs.,
( )
P,V P avg
C C T = . For
methanol
avg
67.6 64.5
T 66.65 C
2
+
= = . For water,
avg
67.6 100
T 83.8 C
2
+
= = .
( )
( )
PV,M
J 1000 mol kJ kJ
C 42.93 0.08301 66.05 48.41 48.41
mol kmol 1000J kmol C
= + = =

( )
P,V,W o
J kJ
C 33.46 0.00688 83.8 34.04 34.04
mol C kmol C
= + = =

M
T 67.6 64.5 3.1 A = =
;

W
T 67.6 100 32.4 A = =
Then
( )( ) ( )( ) ( )( )( ) ( )( )
1
H 35270 0.914 40656 0.086 48.41 3.1 0.914 34.04 32.4 0.086 = + + +

1
H 35775.5 kJ kmol =

Note terms dominate.


D
H is at
D
y 0.955@66.1 C =
( ) ( )
D M D,M W D,W D,M P,V,M D D,W P,V,W D
H y y y C T 64.5 y C T 100 = + + +

( )
P,V,M P,V,M M,avg
C C T = .
avg,M
66.1 64.5
T 65.3
2
+
= =
( )
P,V,M
C 42.93 0.08301 65.3 48.35 = + =

( )
P,V,W P,V,W W,avg
C C T . =
W,avg
100 66.1
T 83.05
2
+
= =
( )
P,V,W
C 33.46 0.00688 83.05 34.03 = + =

W
T 66.1 100 33.9 A = =
61

( ) ( ) ( )( ) ( )( )( )
D
H 35270 .955 40656 0.045 48.35 0.955 1.6 34.03 0.045 33.9 = + + +

D
H 33682.85 1829.5 73.88 51.91 35534.3 = + + =
Reflux liquid at 66.1C and
M,0 W,0
x 0.893, x 0.107 = =
Reference MeOH 64.5C, water reference 100 C

0 PL,M M,0 M PL,W W,0 W
h C x T C x T = A + A

( ) ( )
PL,M PL,M avg
kJ
C C T 75.86 0.1683 65.3 86.85
kmol C
= = + =
( )( )( ) ( )( )( )
0
h 86.85 0.893 1.6 75.4 0.107 33.9 149 kJ kmol = + =

C D 0 0 1 1
Q DH L h VH = + ( ) ( )( ) ( )( ) 31.1 35534.3 62.2 149 93.3 35775.5 2, 242, 030 kJ h = + =

Overall EB
F C R D B
Fh Q Q DH Bh + + = +

or
R D B C F
Q DH Bh Q Fh = +
B
h is saturated liquid with
B,M
x 0.00435 = and
B,W
x 0.99565 =
Interpolating in Table 2.7
Bot
T 99.2 C ~
( )( ) ( )( )
B PL,M PL,W
h C 0.00435 99.2 64.5 C 0.99565 99.2 100 = +

avg,M
99.2 64.5
T 81.86
2
+
= =

and ( )
PL,M
C 75.86 0.1683 81.86 89.64 = + =
( )
B
h 13.53 60.06 46.5 kJ kmol = + =

Feed is saturated liquid at
M W
z 0.3, z 0.7. = =
From Table 2-7,
F
T 78 C =
( )( ) ( )( )
F P,LM PL,W
h C 0.03 78 64.5 C 0.7 78 100 = +
( )
avg,M
T 78 64.5 2 71.25 = + = and ( )
P,L,M
C 75.86 0.1683 71.25 87.85 = + =
F
h 805.4 kJ kmol =

Then
( )( ) ( )( ) ( ) ( )( )
R
Q 31.1 35534.3 68.9 46.5 2, 242, 030 100 805.4
1,105,116 3204 2, 242, 030 80, 536 3, 424, 479 kJ h
= + =
+ + =


3.D5. Mass Balances: F = D + S + B,
D S B
Fz Dx Sx Bx = + +
Solving simultaneously, B = 76.4 kg/min, D = 13.6 kg/min.
Condenser: ( )
C 1 0 1
Q V h H =
( ) ( )
1 0
V L D L D 1 D 4 13.6 54.4 = + = + = = kg/min
From Figure 2-4,
0
h 7.7 = kcal/kg (x = .9, T = 20C),

1
H 290 = kcal/kg (y = .9, satd vapor).
Thus,
C
Q = 54.4 (7.7 290) = -15,357 kcal/min
Overall Energy Balance:
F R C D S B
Fh Q Q Dh Sh Bh + + = + +

R D S B F C
Q Dh Sh Bh Fh Q = + +
From Figure 2-4,
( ) ( )
S S F
h 61 x .7, sat'd Liq'd ; h 200 z .2, 93 C , = = = =
62

( )
B B D o
h 99 x .01, sat'd Liq'd , h h 7.7 = = = =
Thus,
( )( ) ( ) ( )( ) ( )( ) ( )
R
Q 13.6 7.7 10 61 76.4 99 100 200 15357 3635.3 = + + = kcal/min

3.D6. From Eq. (3-3), D = F
B
D B
z x .4 .002
2500 998
x x .999 .002
| | | |
= =
| |

\ .
\ .
lbmol/h.
Then B = F = 1502.
Condenser: ( ) ( )
o o
V L D L D D D = + = +
( ) ( )
C D V o
Q h H D L D 1 = +
With 99.9%
5
nC have essentially pure
5
nC . Thus, it is at its boiling point.
( )
D V C5
h H 11, 369 = = Btu/lbmol.
( )( )( )
C
Q 11, 369 998 4 45, 385, 048 = = Btu/h
Overall:
R D B F C
Q Dh Bh Fh Q = +
Distillate is at boiling point of pure (
5 C5
nC K 1.0 = on DePriester Chart) = 35C. Bottoms
is at boiling point of ( )
6 C6
nC K 1.0 = =67C.
Converting to F: 35C = 95F, 67C = 152.6F, 30C = 86F.
Note feed is obviously a subcooled liquid. Arbitrarily, pick a liquid at 0F as reference.
(This will not affect the result and other values can be used.)

F LC5
P C5 P C6 PLC6
C x C z C = +
( )( ) ( )( )
F
P
C .4 39.7 .6 51.7 46.9 = + = Btu/lbmol F
( ) ( )( )
F
F P F
h C T 0 46.9 86 4033.4 = = = Btu/lbmol
Distillate is almost pure
5
nC . Liquid at 95F
( ) ( )( )
LC5
D P Dist
h C T 0 39.7 95 3771.5 = = = Btu/lbmol
Bottoms is almost pure liquid
6
nC at 152.6F.
( ) ( )( )
pLC6
bot
hC T 0 51.7 152.6 7889.4 = = Btu/lbmol
( )( ) ( )( ) ( ) ( )
R
Q 998 3771.5 1502 7889.4 2500 4033.4 45, 385, 048 50,861, 491 = + = Btu/h
3.D7. Eq. (3-3),
B
D B
z x 0.7 0.001
D F 1000 700.4
x x 0.999 0.001
| | | |
= = =
| |

\ .
\ .
kmol/h
B F D 299.6 = = kmol/h
Condenser: ( ) ( )( )
o o
L L D D 2.8 700.4 1961.1 = = = kmol/h
Only this reflux is condensed since product is a vapor.
( )
C o
Q L = where is for essentially pure n-pentane.

C
kmol Btu 2.20462 lbmol
Q 1966.1 11, 369
h lbmol 1 kmol
| || || |
=
| | |
\ .\ .\ .


10
C -4
Btu 1 J J
Q 49,154, 204.85 5.18176 10
h 9.486 10 Btu h
= =


63

From overall balance
R D B F C
Q DH Bh Fh Q = +
Distillate is vapor at b.p. of pure n-pentane (35C from DePriester chart,
C5
K 1.0 = )
Bottoms is boiling n-hexane (67C)
Conversions: 35C = 95F - distillate & Feed and 67C = 152.6F - bottoms

As reference, arbitrarily choose liquid at 0F.
Feed is subcooled liquid.
( )( ) ( )( )
o
PF C5 PLC5 C6 PLC6
C z C z C 0.7 39.7 0.3 51.7 43.3Btu lbmol F = + = + =
( ) ( )( )
F PF F
h C T 0 43.3 95 0 4113.5Btu lbmol = = =
Distillate ( )
D C5 PLC5 dist
H C T 0 = +
( )( )
D
H 11, 369 39.7 95 0 15,140.5 Btu lbmol = + =
Bottoms is pure
6
C @152.6 F
( ) ( )
B PLC6 bot
h C T 0 51.7 152.6 0 7889.4 Btu lbmol = = =

R
kmol Btu kmol Btu
Q 700.4 15,140.5 299.6 7889.4
h lbmol h lbmol
kmol Btu 2.20462 lbmol Btu
1000 4113.5 49,154, 204.85
h lbmol kmol h
| || | | || |
= +
| | | |
\ .\ . \ .\ .
( | || | ( | |

| | | ( (
\ .\ . \ .


10
R -4
Btu 1 J
Q 68, 675,167.9 7.240 10 J h
h 9.486 10 Btu
= =


3.D8.
New Problem in 3
rd
Edition.

E w
F 300, z .3, z .7 = = =
98% rec. E in distillate, 81% rec water in bot.
( ) ( )( )( ) D Dist. .98 90 1 .81 300 .7 128.1 kmol/h = = + =

( )( )
DE
.98 90
y 0.6885
128.1
= =
( )( ) ( )( ) B Bottoms .02 90 .81 210 171.9kmol h = = + =



b. Partial Condenser.






( )
0
0
L
2, L 2D 2 128.1 256.2 kmol h.
D
= = = =

0
x

in equilibrium with
0
y , thus from equation data
0
x 0.575. =
Entering vapor
1
y (from graph) 0.61 =
D D
y , D, H
c
Q
0 0 0
x , L , h

1 1 1
H y V
Vapor
64


1 0
V L D 256.2 128.1 384.3 kmol h. = + = + =
c. E.B. on PC.
1 1 c dist 0 0
VH Q DH L h . + = + Can use Figure 2-4 by converting mole fracs to mass
fracs. Basis 1 kmole.
Distillate ( ) .6885 mol E MW 46 31.671 kgE = =
( )
5.607 kgW
.3115 mole W MW 18
37.28 kg total
= =
Mass frac. E = 31.671 37.28 0.8496 =
Vapor
1
V ( ) 0.61 mole E 46 28.06 kgE =
( )
7.02 kgW
.39 mole W 18
35.08 kg total
=
Mass frac E = 28.06 35.08 0.7999 =
Liquid reflux
0
L ( ) 0.575 mole E 46 26.45 kgE =

( )
7.65 kgE
0.425 mole W 18
34.1 total
=

Mass frac E = 26.45 34.1 0.7757 =
From Figure 2-4,
dist 1 0
H ~ 310 kcal kg, H ~ 330kcal kg, h ~ 65 kcal kg

c dist 0 0 1 1
kmol 37.28 kg kcal
Q DH L h VH 128.1 310 256.2 34.1 65
hr kg kg
35.08 kg
364.3 330 2, 400, 517kcal hr
kmol
= + = +
=

Overall EB.
F R c dist B
Fh Q Q DH Bh + + = +
Know
c
Q 2, 400, 517 kcal h =
and
dist
DH 1, 480, 426 kcal h. =
To find
F B
Fh and Bh , need to convert mole frac to wt frac.
Basis 1 kmol
Feed ( ) 30 mole % E: .3 mole 46 13.8 =
( )
12.6
70% W: .7 18
total 26.4 kg kmol
=
Mass frac E 13.8 26.4 0.5227 = =
Bottoms ( ) 0.01047 mole 46 .48162 =
( )
17.811
0.98953 mole 18
total 18.293 kg kmol
=
Mass frac E 0.48162 18.28 0.0263 = =
From Figure 2-4 ( )
F
h satd liqd 70kcal kg ~
( )
B
h satd liqd 97 ~

65

Then
R dist B F c
Q DH Bh Fh Q = +
R
kmol 18.29316 kg 97 kcal 26.4 kg kcal
Q 1, 480, 426 171.9 300 70
h kmol kg kmol kg
= +
( ) ( ) 2, 400, 517 1, 480, 426 305, 525 554, 408 2, 400, 517 = + 3, 632, 069kcal h =

3D9. New Problem 3
rd
Edition. B = (x
D
z)/(x
D
x
B
)F = [(0.9999 - 0.76)/(0.9999 0.00002)](500) = 120


R B
Q Lh VH Bh + = + and L V B = +
Assume
B
h h . =
( ) ( )
R
Q L B h Vh V H h V = + = =

( ) ( )( ) V V B B 1.5 120 180 kmol h. = = =
Bottoms is almost pure water.
w
9.72 kcal mol 9720 kcal kmol = =
( )( )
6
R
Q 180 kmol/h 9720 1.750 10 kcal h = =

3.D10. 2 atm 101.3 kPa/atm = 202.6 kPa. Pentane Recovery:
P D
0.995 Fz Dx =

( )( )( )
( )
0.995 1000 0.55
D 547.6333
0.9993
= = kmol/h
B = 1000 547.6333 = 452.3667
Since ( )
B p
Bx Pentane Recovery Bot F z , =

( )( )( )
B
1 .995 1000 0.55
x 0.006079
452.3667

= = mol frac pentane


Distillate is essentially pure Pentane. Bottoms Pure in Hexane. From DePriester Chart

P dist
K 1@p 202.6 kPa when T 59.5 C = = =

n H bot
K 1@p 202.6 kPa when T 94 C

= = =
For Total Condenser, Eq. (3-14) ( )
o
C D 1
L
Q 1 D h H
D
| |
= +
|
\ .

( ) ( )
D PLC5 dist ref ref
h pure pentane C T T choose T 25 C = =

( )
D
kcal kcal
h 39.7 59.5 25 1369.65
kmol C kmol
= =

1 D
H h = + assuming is independent of temperature

1
Btu 1 lbmol 0.252 kcal kcal
H 1369.65 11369 7680.196
lbmol 0.454 kmol Btu kmol
( (
= + =
( (


Eq. (3-14) is
( )( ) ( )
C
kmol kcal
Q 1 2.8 547.6333 6310.5 13,132, 288
h h
= + =

( ) ( ) ( )
B PLC6 bot ref
kcal kcal
h pure hexane C T T 51.7 94 25 3567.3
kmol C kmol
= = =
Feed is a liquid at 65C
| |( )
F PC5 C5 PC6 C6 F ref
h C z C z T T = +
( ) ( ) ( )
F
h 39.7 0.55 51.7 0.45 65.25 1804kcal / kmol = + = (


66


R D B F C
Q Dh Bh Fh Q = +
( )( ) ( )( ) ( )( ) ( )
R
Q 547.6333 1369.65 452.3667 3567.3 1000 1804 13,132, 288 = +

R
Q 13, 692, 081 = kcal/h.
Note that
C R
Q and Q are relatively close.

3.E1. Was 3.D8 in 2
nd
Edition.


















Pick as basis liquid at 0C,
W M
h 0 & h 0 = = (essentially steam table choice)
Assume ideal mixtures.
( )
avg avg W,L M,L
F P P ref P P P
h C T T where C 0.4 C 0.6 C = = +
Felder & Rouseau p. 637
PW
C 0.0754 = kJ/mol

5
PM
C 0.07586 16.83 10 T

= + kJ/mol

( ) ( ) ( ) ( )
PM PM avg PM PM
C C T C 0 40 2 C 20 C = = + =
( )
5
PM
C 0.07586 16.83 10 20 0.079226

= + =
( ) ( )
avg
P
C 0.4 0.0754 0.6 0.079226 0.077696 = + =
( )
F
h 0.077696 40 0 3.1078 = = kJ/mol feed = 3107.8 kJ/kmol
Can also use steam table for water

S
H is satd vapor steam 1 atm,
S
kJ 18.0 kg
H 2676.0
kg kmol
=
Steam Table F&R, p. 645
S
H =48,168 kJ/kmol

D
h is satd liquid at
D
x 0.99 = . From Table 2-7, T = 64.6C
( )
avg avg
D P P PW PM
h C 64.6 0 where C 0.01 C 0.99 C = = +

( ) ( ) ( )
5
PM PM avg PM
C C T C 32.3 C 0.07586 16.83 10 32.3 0.081296

= = = + =
( ) ( )
avg
P
C 0.01 0.0754 0.99 0.081296 0.08124 = + =
B
x 0.02 =
B
C
Q
p = 1.0 atm
M
z 0.6 =
100 kmol h

F
D
D, x 0.990 =
M.B. F + S = D + B
M S,M D B
Fz Sy Dx Bx + = +
Since steam is pure,
S,M
y 0 =
Know
M D B
F, z , x , x
Unknowns S, D, B, Need E.B.
F C S D B
Fh Q SH Dh Bh + + = +
S
67

( )
D
H 0.08124 64.6 5.2479 kJ mol 5247.9 kJ kmol = = =

B
h : Since leaving an equilibrium stage, satd liqd. 2% MeOH
Table 2-7, T = 96.4C
( )
avg avg
B P P PW PM
h C 96.4 0 where C 0.98 C 0.02 C = = +
( ) ( ) ( )
PM PM PM
C C 96.4 0 2 C 48.2 C = + =
( )
5
PM
C 0.07586 16.83 10 48.2 0.08397

= + =
( ) ( )
avg
P
C 0.98 0.0754 0.02 0.08397 0.07557 kJ mol = + =
( )
B
h 0.07557 96.4 7.28509 kJ mol 7285.09 kJ mol = = =


C
Q = do EB around condenser
( )
o
C dist 1 dist o dist
L
Q V L D 1 D
D
| |
= = + = +
|
\ .


dist MeOH W
0.99 0.01 ~ +
Felder & Rousseau:
M W
35.27 kJ mol & 40.656 kJ mol = =

( )( ) ( )( )
dist
kJ
0.99 35.27 0.01 40.656 35.324 35, 323,86 kJ kmol
mol
= + = =
( )( )
C
Q 35, 323.86 2.3 1 D 116, 568.7D kJ h = + =

Plug
C
Q & numbers into E.B.
( ) 100 3107.8 116, 568.7D 48,168S 5247.9D 7285.09B + = +
or 310,780 + 48,168S = 121,816.6D + 7285.09B

Solve simultaneously with 2 MB. 100 + S = D + B
60 + 0 = 0.99D + 0.02B
One can use algebra or various computer packages.

Obtain: D = 56.33 kmol/h, B = 211.71 kmol/h

C
S 168.04 kmol/h, Q 6, 566, 000 kJ/h. = =

E2. Was 3.D9 in 2
nd
Edition.
kmol mol
F 500 500, 000
h h
= =
F + S = D + B

S D B
Fz Sy Dx Bx + = +
2 eq. 3 unknowns
Condenser: ( ) ( )
C 0 o 1
Q 1 L / D D h H = +
Note Eq (3-14) not valid.

For enthalpy pick reference pure liquid water 0C and pure liquid methanol 0C. Felder &
Rouseau:
MeOH
P
C 75.86 0.01683T = + at
MeOH
avg P
64.5 0
T 3225, C 76.4 J mol C.
2
+
= = =
68

Assuming distillate pure methanol, boils at 64.5C
( ) ( )( )
Meoh,liq
D P ref
h C T T 76.4 J/mol 64.5 0 4928.0 J mol = = =

( )
1 D 1
H h at 64.5 F 4928 35270 J/mol 40,198 J/mol = + = + =
( )( )
C
Q 4 4928 40198 D 141, 080D = = J/h where D is mol/h
Overall Energy balance:
F S C D B
F h SH Q Dh Bh + + = +
Bottoms is essentially pure
2
H Oat 100C
( ) ( )
W,liq
B P ref
J J
h C T T 75.4 100 0 7540
mol C mol
| |
= = =
|
\ .


( )
S B W
H h at 100 C 7540 40656 J/mol 48196 = + = + =
For feed. 60 mole % Methanol boils at 71.2C (Table 2-7).
( ) ( )( )( ) ( )( )( )
i
F P i ref
h C z T T 0.6 76.459 71.2 0.4 75.4 71.2 5413.7 = = + =

J/mol
Now, Eqs are
(1) F + S = D + B or 500,000 + S = D + B
(2)
D B
Fz Dx Bx = + or (500,000) (.6) = 0.998D + 0.0013B
(3) ( )
o
C D 1 C
L
Q 1 D h H or Q 141, 080D
D
| |
= + =
|
\ .

(4)
F S C D B
Fh Sh Q Dh Bh + + = + or (500,000) (54137) + S (48196)
+
C
Q = D(4928) + B (7540)
Solve simultaneously: D = 298.98, B = 1245.5, S = 1044.2 kmol/h

C
Q = - 4.218 10
+7
kJ/h

3.F1. An enthalpy composition diagram is available on p. 272 of Perrys Chemical Engineers
Handbook, 3
rd
ed., 1950.
Eq. (3-3) ( )
B
D B
z x 0.79 0.004
D F 25, 000 19, 788.5
x x 0.997 0.004
| |
= = =
|

\ .
kmol/h
Note that
2
N mole fractions were used since
2
N is more volatile. B = F D = 5211.5
From enthalpy comp. diag.
D 1
h 0, H 1350 = = kcal/kmol,
B F
h 160, h 1575 = = . Then,
( ) ( ) ( )( )( )
C o D 1
Q 1 L D D h H 5 19788.5 0 1350 133, 572, 000 = + = = kcal/h

R D B F C
Q Dh Bh Fh Q = +
( )( ) ( )( )
R C
Q 0 5211.5 160 25, 000 1575 Q 95, 030, 000 = + = kcal/h

3.F2. We will use the enthalpy composition diagram on p. 3-171 of Perrys 6
th
edition or p. 3-158 of
Perrys 5
th
ed.Do for 1 kmol of feed:
Conversion of feed from kg to moles. Basis 100 kg
30 kg
3
NH = 1.765 kmol
70 kg
2
H O =3.888
Total 5.653 kmol

Thus 1 kmol is 100/5.653 = 17.69 kg
69

Will work problem in weight fractions since data is presented that way.
95% recovery: (0.95) Fz =
D
Dx or, D = (.95)
D
Fz / x = (.95) (17.69) (.3)/(.98) = 5.15 kg.
B = F D = 12.54 kg
( ) ( )( ) ( )( ) ( )
B D
x Fz Dx B 17.69 .3 5.15 98 12.54 0.021 = = = (


From diagram:
D 1 B F
h 55, H 415, h 150, h 5 kcal/kg = = = =
Eq. (3-14), ( ) ( ) ( )( )( )
C o D 1
Q 1 L D D h H 3 5.15 55 415 5562 = + = = kcal/kmol feed
and
R D B F C
Q Dh Bh Fh Q = +
( )( ) ( )( ) ( )( )
R C
Q 5.15 55 12.54 150 17.69 5 Q 7815 = + = kcal/kmol of feed

G1. a.) Using NRTL.
C R
Q 778,863 kcal/h, Q 709, 520 kcal/h = =
b.)
C R
Q 1, 064,820 kcal/h, Q 995, 478 kcal/h = =

G2. New Problem in 3
rd
Edition.
ASPENPlus. D = 988, L/D = 3, Peng Robinson,
F
N 40, N 20 = = (arbitrary values in Radfrac)

7
D DC6
x 1.000 x 1.211 10

= =

B
x 0.0013316 =

7
C
7
R
Q 4.4426 10 Btu h,
Q 4.9852 10 Btu h
=
=





70
SPE 3
rd
Ed. Solution Manual Chapter 4
New Problems and new solutions are listed as new immediately after the solution number. These new
problems are: 4A6, 4A13, 4C10, 4C16, 4D6, 4D9, 4D13, 4D15, 4D18, 4E4, 4E5, 4H1 to 4H3.
4A1. Point A: streams leaving stage 2 (L
2
, V
2
)
Point B: vapor stream leaving stage 5 (V
5
)
liquid stream leaving stage 4 (L
4
)
Temp. of stage 2: know
2 2
K y / x , can get T from temperature-composition graph or
DePriester chart of K = f(T,p).
Temp. in reboiler: same as above (reboiler is an equilibrium stage.)
4A2. a. Feed tray = .6, z = 0.51 (draw y = x line), y
F
=0.52, x
F
= 0.29.
b. Two-phase feed.
c. Higher
4A6. New Problem in 3
rd
Edition. Answer is a.
4A7. See Table 11-3 and 11-4 for a partial list.
4A13. New Problem in 3
rd
Edition.
A. Answer is b
B. Answer is a
C. Answer is a
D. Answer is a
E. Answer is b
F. Answer is a
G. Answer is b
4A14. If feed stage is non-optimum, the feed conditions can be changed to have an optimum
feed location.
4B2. a. Use columns in parallel. Lower F to each column allows for higher L/D and may be sufficient
for product specifications.
b. Add a reboiler instead of steam injection. Slightly less stages required and adds 1 stage.
c. Make the condenser a partial instead of a total condenser. Adds a stage.
d. Stop removing side stream. Fewer stages are now required for the same separation.
e. Remove the intermediate reboiler or condenser and use it at bottom (or top) of column. Fewer
stages, but all energy at highest T (reboilers) or lowest T (condenser) for same separation.
Many other ideas will be useful in certain cases.
4C7. Easiest proof is for a saturated liquid feed. Show point
D
z, y satisfies operating equation.
Solution: Op. Eq.
B
y L V x L V 1 x
Substitute in
D
y y , x z
D B
y V Lz L V x
But q 1.0, V D, L F, L V B
D B
y D Fz Bx
Which is external mass balance. QED.
71

Can do similar for enriching column for a saturated vapor feed.

4.C10. New Problem in 3
rd
Edition. If we consider , the latent heat per mole to be a positive quantity,
then
R
Q V . With CMO and a saturated liquid feed (1 / ) V V L D D , and then
/ (1 / )
R
Q D L D .

4.C16. New Problem in 3
rd
Edition. Define a fictitious total feed
T T T
F , z , h

T 1 2
F F F ,
1 1 2 2
T
T
Fz F z
z
F
,
1 2
1 F 2 F
T
T
Fh F h
h
F

Intersection of top & bottom operating lines must occur at feed line for fictitious feed F
T
.
(Draw a column with a single mixed feed to prove this.)
This feed line goes through
T
y x z

b.) Does
1 1 2 2
T
T
q F q F
q
F

x
B
x
B
T
z

y
0
z
A
2
z
1
z
Given p, L/D, saturated liquid reflux,
D B
x , x
opt feed locations,
1 2 1 2 F1 F2
z , z , F, F , h , h



Plot top op line. Plot all 3 feed lines. Draw
line from point A to y = x =
B
x to obtain
bot. op. line. Connect pts B & C to get
middle op. line.

T
F

with slope
T T
q q 1
where
mix T
T
mix mix
H h
q
H h
and
mix, mix
H h are saturated
vapor and liquid enthalpies at feed stage of column with
mixed feed.


72

check
1 2
1 2
1 F 2 F
mix
1 2 mix 1 F 2 F
T
mix T
T
mix mix mix mix mix mix T
Fh F h
H
F F H FH F h
F
H h
q
H h H h H h F

where
mix mix
H & h are vapor and liquid enthalpies on feed stage of mixed column


1
2
1 mix F
mix F
2
mix mix mix mix
T
T
F H h
H h
F
H h H h
q
F

Usual CMO assumption is >> latent heat effects in either vapor or liquid.
Then
1 2
mix F mix F
1 2
mix mix mix mix
H h H h
q and q
H h H h

Thus
1 1 2 2
T
T
Fq F q
q
F
if CMO is valid.
4D1. a. Top op line:
D
L L
y x 1 x
V V
and
L L D 1.25
0.5555
V 1 L D 2.25

Intersects
D
y x x 0.9
When
D
L
x 0, y 1 x 0.4
V
Plot See diagram
b. Bottom op line:
B
L L
y x 1 x
V V
, and
L V B V B 1 3
V V V B 2

Intersects y = x = x
B
= 0.05

1 0.5 / 2
@y 1 x 0.683 this is convenient point to plot
3 2

c. See diagram for stages. Optimum feed stage is #2 above partial reboiler.
5 equilibrium stages + PR is more than sufficient.
73



d. Feed line goes from y = x = z = 0.55 to intersection of two operating lines.

q
Slope 1.0 or q 0.5
q 1
.
This is a 2 phase feed which is liquid & vapor.

4D2. New Problem in 3
rd
Edition. Part a.

E
L F V 1 V / F .63
y x z .6 Slope 1.703
V V V F .37

b. From Table 2-1, at 84.1 C y .5089
c. liquid at 20C
F
H h
q
H h
and 40 mole % ethanol.
The pressure in Figure 2-4 is very close to 1.0 atm, thus it can be used, but must convert to wt
frac.
74

Basis 1kmol feed.
.4 kmole E .4 MW 46 18.4 kg 0.63 wt frac.

10.8 kg
.6 kmol Water .6 MW 18
total 29.2 kg

From Figure 2-4
F
H 398 kcal kg, h 75, h 20 C 10

398 10
q 1.20
398 75


q 1.2
Slope 6
q 1 .2

Alternate Solution: 40 mole % ethanol boils at 84.1C (Table 2-1).
Then if pick reference as saturated liquid at 40 mole %

F p,40%liq
h C 20 84.1

40%E
h 0, H
d.
vapor
F p
kcal
40 mole %E 63 wt%, H 398 kcal kg, h 65, h 398 C 120 84.1
kg


vapor Evapor w,vapor
P E P w P
C y C y C
Assume only 1
st
and 2
nd
terms in
P
C equations are significant.
From Problem 2.D9

vapor
P
C .4 14.66 0.03758T .6 7.88 .0032T
kcal/kmol T is C

which simplifies to
vapor
P
C 10.592 0.16952T
For linear
120
p
84.1
C dT is equal to
vapor
P avg
C @T

avg
T 84.1 120 2 102.05 . Then
v,avg
P
kcal
C 10.592 0.16952 102.05 12.32
kmol


F
kcal kcal 1 kmol
h 398 12.32 120 84.1
kg kmol 27.2 kg


F
h 398 15.149 413.15kcal kg

398 413.15 15.147
q 0.045.
398 65 333

e.
F 13
q L L f , L L F L F
12 12


13 12
q 13 12, slope q q 1 13
1 12


f. Flash
V L 1 V F .3 3
.7,
F V V F .7 7

See graph for feed lines.
75


Graph for 4.D2
76

4.D3*. a. Basis 1 mole feed.
0.4 moles EtOH 46 = 18.4 kg EtOH
0.6 moles H
2
O 18 = 10.8 kg H
2
O
Total = 29.2
wt frac 18.4 / 29.2 0.63 wt frac EtOH
Calculate all enthalpies at 0.63 wt frac. H
v
= 395, H
L
= 65 (from Figure 2-4). h
F
is liquid at
200C. Assume C
p,liq
is not a function of T. Estimate,

L
P,liq
h 60 C h 20
46.1 23 h kcal
C .63 wt frac ~ 0.864
T 60 20 80 kg C

Then
L
F L P L
h h 200 C 200 60 h 60 C .864 200 60 46.1 167.1

v F
v L
H h 395 167.1 q 0.691
q 0.691, 2.24
H h 395 65 q 1 0.309

b. From Figure 2-4 at 50 wt% ethanol H
v
= 446 and h
L
= 70. Since CMO is valid obtaining both
enthalpies at 50% wt is OK. The feed is a liquid

F P,liq F ref P,liq
h C T T C 250 0

w
P,liq P,EtOH EtOH P
w
C C z C z in Mole fractions
Basis 100 kg solution
50 kg EtOH 46.07=1.085 kg/kgmole
50 kg W 18.016 2.775 kg moles
Total 3.860 kg moles
Avg M.W. 100 3.86 25.91 kg/kgmole
Thus, z
W
= 0.719 and z
E
= 0.281

P,liq
C 37.96 .281 18.0 .719 23.61

P
P,liq
AVG
C 23.61
C in kcal kg C 0.911
MW 25.91
. Then,

F
kcal
h 0.911 250 C 228
kg C


v F
v L
H h 446 228
q 0.58
H h 446 70

4.D4*. a.
F Pv
h H C 350 50 H 25 300

F
25 300
H h
q 1.5
H h

Slope q q 1 0.6. y x z 0.6 is intersection.


b. q L L F where L L 0.6F. Then q L 0.6F L / F 0.6, and
slope q q 1 1.5
c. q L L F where L L F 5. q L F 5 L F 1 5 , slope q q 1 1 6
z
y = x
feed
line
.5 .6
.7
4.D4a
77

4.D5*.
liq reflux
L
vap liq
h h
3100 1500
f 0.1111
H h 17500 3100


0 0
1 0
L L D 1.1
0.524
V L D 1 2.1


c 0 1 1
2 c 0 1
1 f L V 1.1111 .524 L
0.55
V 1 f L V 1 .111 .524

Alternate Solution
For subcooled reflux,
0 1
0 1
H h L
q
L H h
17500 1500
1.111
17500 3100

Then,
1 0 0
L qL 1.1111 L

1 1 1
2 1 1
L L L D
V L D L D 1
,
0 1
1.111 L L
1.111 1.1 1.2222
D D


1
2
L 1.222
0.55
V 2.222

4D6. New Problem in 3
rd
Edition. a)
1 2
175 F F B D
85 75 .6 100 0.4 0.1 B 0.9D
Solve simultaneously.
D 84.375 and B 90.625 kmol hr
b) Feed 1.
1
q 1, vertical at
1
y x z 0.6
Feed 2. 60% vapor = 40% liquid
2
q 0.4
Slope feed line
2
2
q 0.4
2 3
q 1 .06
through
2
y x z 0.4
Bottom Op. Line
B
y L V x L V 1 x . Through
B
y x x

V B 1
Slope L V 3 2
V B

Middle
2
L F B V

2 2 B
2 2 B
F z Bx L
L x F z Bx V y y x
V V

When
2 2 B
F z Bx
x 0, y , Slope L V
V

Also intersects bot. op. line and Feed line 2.
Do External Balances and Find D & B. Then V V/ B B 2B 181.25
L V B 271.875
At feed 2, L .4F L or L L 0.4F 271.875 40 231.875
V V 0.6F 181.25 60 241.25
L V 0.961

40 9.625
x 0, y 0.126
241.25
Plot Middle Op Line.
78


D
L L
y x 1 x
V V

Know that
D
y x x and gives through interaction Middle and Feed line 1.
Also,
1
L L F 231.875 75 156.875 and V V 241.25 ; thus,
L V 156.875 241.25 0.65
c) See graph.

Graph for 4D6.

79


4.D7*. a. Plot top op. line: slope
D
L
.8 , x y x .9.
V
Step off stages as shown on Figure.
b. Plot bottom op. line: slope
B
L V
1 1 2 , x y x 0.13.
B V
Step off stages
(reboiler is an equil stage). Find y
2
= 0.515.
c. Total # stages = 8 + reboiler
Optimum feed plate = 7 or 8 from top. Plot feed line. Goes through x = y = z = .3, and
intersection of two operating lines.
slope
9 q
4 q 1
gives q = 0.692.


4.D8*. The equilibrium data is plotted and shown in the figure. From the Solution to 4.D7c,
q 0.692 and q q 1 9 4
a. total reflux. Need 5 2/3 stages (from large graph) 5.9 from small diagram shown.
b.
min
.9 .462
L V 0.660
.9 .236
(see figure)

min
min
min
L V
L D 1.941
1 L V

c. In 4.D7,
act
L V .8
L D 4
1 L V .2


act
min
L D Multiplier L D
Multiplier = 4/1.941 = 2.06
80

d. Operating lines are same as in Problem 4.D7. Start at bottom of column. Reboiler is an
equilibrium contact. Then use
MV
E AB AC 0.75 (illustrated for the first real stage)
Stage 1 is the optimum feed stage. 11 real stages plus a partial reboiler are sufficient.



4D9. New Problem in 3
rd
Edition. a)

1 2 1 1
F F D B 100 F 80 B F B 20

1 1 2 2 B B 1
Fz F z Dx Bx F .42 18 .66 80 0.04 B
Solve simultaneously,
1
B 113.68, F 93.68
b)
1 L L D 1 2 1
L D ,
2 V 1 L D 3 2 3


L
L D 40, V L D 120
D

Saturated Liquid Feed V V 120

1
L L F 40 93.68 133.68, L V 1.114
c) Top Op. Line Normal:
D
y L V x 1 L V x
Through
D
2
y x x , Slope 1 3, y intercept .66 .44
3

Bottom Normal:
B B
y L V x L V 1 x , through y x x
Also through intersection,
2
F feed line and middle op. line.
2
2
F
2
F
L
.7
Feed line F slope
V .3

81

Middle
1 1
D
Fz D
y L V x x
V V
(or do around bottom)
Slope L V . Through intersection feed line
1
F and top op. line.
Also,
D 1 1
80 .66 93.68 .42
Dx Fz
x 0, y 0.11212
V 120

d)Opt. Feed
2
F stage 1 from bottom, Opt feed
1
F , Stage 2. 4 stages + PR more than sufficient.

Graph for 4D9.
82

4.D10*. Operating Line
D
L L L D 4
y L V x 1 x , where .8
V V 1 L D 5

Thus, operating line is y = .8x + .192
a. Equilibrium is
1
1
1
y y
x or x
1 y 1.79 .76y

Start with y
1
= .96 =
D
x
Equilibrium:
1
1
1
y .96
x 0.9317
1.76 .76y 1.76 .76 .96

Operating:
2
y .8x .192 .8 .9317 .192 0.93736
Equilibrium:
2
2
2
y .93736
x 0.89476
1.76 .76y 1.76 .76 .93736

Operating:
3 2
y .8x .192 .8 .89476 .192 0.9078
b. Generate equilibrium data from:
1.76x
y
1 .76x


x 1.0 .9 .8 .7 .6 .5 .4
y 1.0 .9406 .8756 .8042 .7253 .6377 .5399


Plot equilibrium curve and operating line. (See Figure). Slope = L/V = .8, y intercept (x = 0)
= 0.192, y = x =
D
x = 0.96. Find
6
x = 0.660.


4.D11. a) Same as 4.D2 part g. q = 1.0668, slope feed line = 15.97.
b) Top
D
y L V x 1 L V x goes through y = x =
D
x = 0.99

L D
L V 0.6969
1 L D
@ x = 0 y = (1-L/V)
D
x = (1-0.6969) 0.99 = 0.300
Feed line: Slope q q 1 , y x z 0.6
83

Bottom Op line:

y= 0, x =
B
x . Also goes through intersection of feed line and top op.line.
Stages: Accuracy at top is not real high. (Expand diagram for more occupancy).
As drawn opt. Feed = #6. Total = 9 is sufficient,

c.
min
0.99 0.57
L V Slope 0.4242
0.99 0


min
min
L L V 0.4242
0.73684
D 1 L V 1 0.4242

Actual L/D is 3.12 this value.



V L

B
S
M,S B
yV Lx Sy Bx
But
M,S
y 0 (Pure steam)
With CMO B L
B
L L
y x x
V V

84

4.D12.
L D
L V 3 4
1 L D
slope. Top op line goes throug
D
y x x 0.998

D
L L
y x 1 x @ x 0, y .25 .998 0.2495
V V

Bottom slope
From Soln to 3.D9 or
B 1245167
L V 1.19
from graph. 1.169 S 1044168

Feed line is vertical at z = 0.6. Can also plot top and feed lines, and then find bottom from 2
points
B
y 0, x x & intersect top & feed .
For accuracy Use expanded portions near distillate & near bottoms.
From Table 2-7 from (x = .95, y = .979)
Draw straight line to (x = 1.0, y = 1.0)
From (x = 0, y = 0) draw straight line to (x = 0.02, y = 0.134)
or (x = 0.01, y = 0.067)

Opt feed = # 9 from top. Need 13 equilibrium stages.




85







86

4.D13. New Problem in 3
rd
Edition. a.) See Figure

b.) See figure.
MIN
L 0.665 0.95
0.4385
V 0.30 0.95


MIN
L L L V 0.4385
0.7808
D V L 1 L V 0.5615

c.)
MIN
L
2.0 L D 1.5616
D
,
L L D 1.5616
0.6096
V 1 L D 2.5616

y intersect
D
L
1 y 0.3709
V
. Top operating line
D
L L
y x 1 y
V V

Goes through
D
y x y 0.95
Bottom
B
L L
y x 1 x
V V

Goes through
B
y x x & intersection top operating line & feed line.
Feed Line: Vertical (saturated liquid, q = 1). Through y x z 0.3
Plot & Step off stages. Optimal feed = 5 below PC. 6 + PC + PR more than sufficient.
d.) Slope bottom: See figure for parts c & d.
0.85 0.025
L V 1.941
0.45 0.025


1 1
V B V L V 1.0625
L V 1 0.941
.
87



Graph for problem 4.D13.
88



4.D14a.
89



4.D14b. Two approaches to answer. Common sense is all methanol leaks out and
MA
x 0 .
McCabe-Thiele diagram: This is enriching column with
s
z y 0 . Intersection top op. line and
horizontal feed line is at
M,b
x x 0 , which is also a pinch point. Thus
M,d
x 0 also.
4.D15. New Problem in 3
rd
Edition. Saturated liquid.
q
q 1, ,
q 1
feed line vertical @ z .3.
Top operating line
D
L L
y x 1 y
V V
,
L L D 2
Slope
V 1 L D 3



D
y intercept 1 L V y 1 3 0.6885 0.2295 and
D
y x y
Bottom operating line
B
y L V x L V 1 x goes through
B
y x x
And goes through interaction feed line and top operating line. See graph.
Optimum feed is stage 2 below partial condenser. Partial condenser + Partial reboiler + 3
equilibrium stages are more than enough to obtain separation.



S
B
External M.B. S = B
Sy
s
= Bx
B
. Since y
S
= 0 (pure water)
x
B
= 0

New S.S.
90



Graph for problem 4.D15.
91


4.D16*.
v F
v L
H h L-L
q
F H h

Using 32F = 0C as reference T,
F
h 4033.4 Btu/lbmole.

v L
H h . Approx. at feed conditions.
.4 11369 .6 13572 12691 Btu/lbmole
For approx. temperature of feed stage, do bubble pt. calc.

1 1 1 1 1
y 1 K x K z
Pick T = 48C (~ 40% of way between boiling pts.)

C5 C6 1 1
K 1.5, K .54, K x 1.5 .4 .54 .6 .92

C5 new new
.54
K T = =.594, T 50 C
.92


C5 1 1
K 50 C 1.6, K x 1.6 .4 .584 .6 .99 Close enough.

v L feed
H h 50 C , 50 C 122 F

feed,Liq
v P
H C 122 46.9 122 5721.8

P feed,liq
Note : C 46.9 is from Prob. 3-D6.

v
H 5721.8 12691 18412.8 Btu/lbmole

V F
V L
H h 18412.8 4033.4
q 1.133
H h 12691

Note:
F
h is from Prob. 3.D6.
4.D17. Top Op. Line:
D D
L L
y x 1 x , goes through y x x 0.9
V V


L L D 7 2
7 9
V 1 L D 9 2


D
L 2
x 0, y 1 x .9 =0.2
V 9

Plot Top. Step off 2 stages. Find
S
x ~ 0.81
The vertical line at
S
x x 0.81 is the withdrawal line.
Bot. Op. Line intersects Top at
S
x x .
Also know it intersects feed line at
B
x x (unknown)
External Balances F D B S Dont know
B
D, B, or x .

D B S
Fz Dx Bx Sx
Feed enters as saturated vapor. Thus q 0 & V F
Bottoms leaves an equilibrium contact, it is saturated liquid L B
Do flow balances
V F 100
V V 100 since S is removed as saturated liquid.
92


L
L V 7 9 100 77.777. D V L 100 77.777 22.222
V

L L S 77.7777 15 62.7777. L V 62.7777 100 0.6278
B L 62.777

D S
B
60 22.222 0.9 15 0.81 Fz Dx Sx
x 0.444
B 62.7777

Plot. Op. line
Step off stages. 9 is more than sufficient.


4.D18. New Problem in 3
rd
Edition. Feed
1 1
F : z 0.6, saturated liquid, q 1, q / (q 1)
93

Feed
2 2
F : z 0.4, 80% vapor hence 20% liquid
F
q L / F 0.2F/ F .2

q .2
1 4
q 1 .8

Part a.) Bottom operating line goes through point,
B
y x x 0.04
Max L Vto point intersection feed F
2
line and equilibrium curve.
Slope
max
L 1.0 .04
2.2326
V .47 .04


min
V V 1 1
0.8113
B L V 1.2326 L V 1


Part b.
1
L F 100

2
F
L L L 100 .2 80 116
L V B and V B 1.5

116
116 L 1.5B B B 46.4
2.5


116
V L B 116 46.4 69.6 L V 1.66667
69.6


2
F
D V V V 69.6 .8 80 133.6

L 100
0.7485
V 133.6

Check overall balance
1 2
F F D B 180 133.6 46.4 180.0 OK
To find
D
y use MVC mass balance

1 1 2 2 D B
Fz F z Dy Bx
or
1 1 2 2 B
D
100 .6 80 .4 46.4 0.4
Fz F z Bx
y 0.675
D 133.6

Actual bottom op. line:
B
L L
y x 1 x
V V


L V B V B 1 2.5 5
V V V B 1.5 3

Goes through
B
y x x 0.04 , Slope 5 3
2
nd
point y = 1, x = 0.616 (this was arbitrarily found by setting y = 1.)
Plot bottom op. line
Top Op. line:
1 1 D
yV Fz Lx Dy .
D 1 1
Dy Fz L
y x
V V

Goes through intersection feed line for F
2
and bot. op. line. Does NOT go through
D
y x y .
Since D & F, passing streams, Point
1 D
z , y is on op. line.

94


Figure for 4D18

4.D19*. B = 0. Then from external balance F = D + B must have D = F = 1000. Acetone balance
becomes
D D
Fz Dx or x z 0.75 .
To predict
B
x need operating lines. Top:
D
L L
y x 1 x
V V


D
L L V 2
and y x x .75
V 1 L D 3

95

Bottom: L V 1.0 . Thus y = x is operating line. From Figure
B
x 0.01 to 0.02
Feed line can be calculated but is not needed.



4.D20*. To use enthalpy composition diagram change to wt. fractions. Basis = 1 kg mole

Distillate: 0.8 ETOH = (.8)(46.07) = 36,856
0.2 Water = (.2)(18.016) = 3.6032
Total = 40.459
Weight Fractions: EtOH = .911, Water = .089

Feed: 0.32 (EtOH) = (.32)(46.07) = 14.7424
0.68 (W) = (.68)(18.016) = 12.25088
Total = 26.993
Weight Fractions: EtOH = .546, W = .454

Bottoms: 0.04 EtOH = (.04)(46.07) = 1.8428
0.96W = (.96)(18.016) = 17.295
Total = 19.1378
Weight Fractions: EtOH = .0963, W = .9037

Condenser Energy Balance is
1 1 c o o D
VH Q L h Dh which can be solved for
o
L D.

o c
o 1
L Q
1
D D h H

From chart:
D 1
h 54 Kcal/kg and H 285 Kcal/kg
Need D in weight units. Convert feed to weight units.
Ethanol:
100 kgmoles
.32 46.07 1474.24 kg/hr
hr

Water: (100)(.68)(18.016) = 1225 kg/hr
Total: F = 2699.328 kg/hr
96

Then,
B
D B
F z x 2699.328 .0963
D 1489.93
x x .911 .0963
kg/hr
Then,
o o 1
2, 065,113
L D Q D h H 1 1 5.0
1489.98 54 285

Now do usual McCabe-Thiele analysis using molar units. Note
o
L D is the same in mass
and molar units.
Top Operating Line:
D
L L L L D
y x 1 x and
V V V 1 L D


D D
L 5 L 5
; x y x .8, y int ercept x 0 1 x 1 .8 .133
V 6 V 6

Feed Line: Goes through y = x = z = .32
Weight fraction of feed = .546. Then,
1
f v 1k
h 15 kcal/kg, H 430.7, and h 69.

V F
V L
430.7 15 H h
q 1.149
H h 430.7 69
Slope
q 1.149
7.711
q 1 1.149 1

Bottom Operating Line:
B
L L
y x 1 x
V V
. Goes through
B
x y x and
intersection top operating line and feed line.

From Figure need about 8 equilibrium contacts including a reboiler. Stage 1 above reboiler is the
optimum feed stage location.


97

4.D21. Feed 1:
1
q 0 , slope feed line = 0
Feed 2:
2
q 0.9 , slope
2 2
q q 1 0.9 0.1 9
Top:
L L D 1.375
0.579
V 1 L D 2.375
,
D
L L
y x 1 x
V V
.
Goes through
D
y x x , When x = 0,
D
L
y 1 x 0.40
V

Bottom: Since
1
F is saturated vapor,
1
V F 100 kmoles/hr

L L V V 0.579 108 62.532
D V L 108 45.46

1 2
B F F D 100 80 45.46 134.532

But B is saturated liquid. L B 134.532
Check
2
L L .9F 62.532 0.9 80 134.532, OK
Draw top op line. Intersects with
2
F feed line. Then draw bottom op line with slope
L V 1.3453.
Intersection bottom op & q. line gives
B
x 0.09.
Check
1 1 2 2 D
B B
Fz F z Dx 20 36 43.187
x or x 0.095
B 134.532
, OK
Check External MB

1 2
180 F F D B 45.46 134.53 179.99, OK

1 1 2 2 D B
Fz F z Dx Bx
56 20.0 36.0 45.46 0.95 134.53 0.095 55.97 OK
See McCabe-Thiele diagram: Optimum feed = 5, 7 equilibrium stages (6.65) more than
sufficient. fraction ab/ac 0.65

2
.1 F

L
V
2
.9 F
At feed
2
F
2
V V 0.1F 100 8 108

2
L L 0.9 F

98


4.D22*. Around top of column mass balances are: L D V Cand
D W
Lx Dy Vy Cx
Solving,
D w
L D C
y x y x
V V V

For pure entering water,
W
x 1.0 . With saturated liquid entering, L = C. Then from overall
balance, V = D. Thus L/V = C/D = and D/V = 1.0. Operating equation becomes
y 0.75x .92 .75 0.75x .17
Slope
D D
0.75, y x 0 0.17, y x 1 0.92 y , y x 0.68 y Not

99

4.D23*. Note L/V C/D since C is subcooled. Let c = amount condensed. The energy required to
heat stream W to the boiling point must come from this condensation. That is,

W
H h c h h C

p
W
C T 18 212 100 h h
c C C 0.1154C
H h 17465.4

L C c 1.1154C
V D c D 0.1154C

In addition, C/D = or D/C = 4/3.

L 1.1154C 1.1154 1.1154
0.77
V D .1154C 4 3 .1154 4 3 .1154

This compares to L/V = 0.75 if entering water is a saturated liquid. Very little effect since
is very large.

L L D 3.25
0.7647
V 1 L D 4.25

Goes through
D
y x y 0.85
When
D
x 0, y 1 L V y 0.20
Bottom Op Line:
B
L L
y x 1 x
V V

Through
D
y x x 0.05 and intersects top op line @ feed line
Opt. Feed is #4 below partial condenser see diagram. Need 6 equil stages + P.C. (an equil.
contact)

Note Commercial columns usually operate much closer to minimum reflux ratio and have
many more stages.

b.)
min
0.85 0.376
L V 0.558
0.85 0
,
Min
min
Min
L V
L
1.26
D 1 L V


c.) Total reflux 5 stages + PC sufficient or
4 3 4 equil contacts + PC = 5 3 4 eq. contacts
where fraction
ab 7.6mm
0.74
ac 10.3mm
or .75

100


101





4.D25. a. 99.9% methanol is essentially pure. Pure MeOH boils 64.5C.
Eq. (4-66)
c o 1
2 c o
1 f L D L
V 1 1 f L D
where
c PL BP reflux
f C T T
For pure MeOH,
5
PL
C 0.07586 16.83 10 T , average (40 + 64.5)/2 = 52.25C

PL
C 0.084654kJ gmole ,
MeOH BP
35.27kJ mole, T 64.5

c
f 0.084654 24.5 35.27 0.058804,
1
2
1.058804 1.2 L
0.5596
V 1 1.058804 1.2

102

b.
L L D 1.2
0.5454
V 1 L D 2.2
or 2.59% more reflux with 24.5C cooling! Usually
subcooling not important.
4.D26. a) 50% feed: q L L F, L L amt vaporized L F 20

1 F
q L L F
20 20
, Slope =
q 1 20 0.05
0.0476
q 1 1 20 1 1.05

35% feed: Saturated liquid vertical feed line. Plot both feed lines. The one with lowest
intersection point with equilibrium curve will normally control
min
V B
Find
max min
max
V 1
L V . Then V B
L V L V 1


max
1 0.1
L V slope 2.497
0.46 0.1
,
min
V 1
0.6681
B 2.497 1.0

b)
min
L V 3 V B 2.0043

V B V B 1 3.0043
L V 1.49892
V V B 2.0043
slope
Bot.
B
L L
y x 1 x
V V
. Goes through
B
y x x with total reboiler

B B
L
y 1, x 1 L V x or x 1 L V 1 x / L V
V


1
x y 1 1 0.49892 0.1 0.6338
1.49892

Intersects feed line with 50% feed first.
Middle operating line: Do mass balance around bottom of column. Mass balance intersects
streams L & V (in column), F
50%
and B.

50% 50% B
yV L x F z Bx

50 50 B
F z Bx L
y x
V V

Intersects bottom operating line & 50% feed line.
@
50 50 B
x 0, y (F z Bx ) / V , Slope L V
For 50% feed,
50 50
q 0.05 (L L ) / F .
External balances:
1 2
250 F F D Band
1 50 2 35 D B
Fz F z Dy Bx
Find D = 103.333 and B = 146.666 moles/min
Since V B 2.0034, V 293.96 and L V B 440.63
Then from
50% 50 50
q : L q F L 0.05 100 440.63 445.63

50
V L F B 445.63 100 146.666 398.964 , Slope L V 1.11698
y intercept (x = 0), y [ 100 .5 146.666 0.1 ] / 398.964 0.0885
Top Intersects feed line for 35% feed and middle op. line and goes through
D
y x y 0.85
103



Figure for 4D26

actual
EQ
op
eq. change
Stages
ML
actual change liquid
0.75 = E =
change at equilibrium

104

4.D27*. Top Op. Eqn:
D
L L
y x 1 x
V V


D D
L L D 1 L
, y intercept 1 x .46, y x x .92
V 1 L D 2 V

Feed:
1
L F F L
L L q 5 4 4
q 5 4, slope 5
F F q 1 1 4
, y x z .48
Middle: V B L S ,
B
B s s
Bx L
V y Bx L x Sy y 0 or y x
V V

Does not intersect at
B
y x x or at
B
y 0, x x . Does intersect top op. line at feed line.
Need another point.
Bottom:
B
L L
y x 1 x
V V
,
B
L V B V B 1 1.5
3, y x x 0.08
V B V B .5

The steam is another feed to the column: Satd vapor q = 0, q/(q-1) = 0, y = x = z = y
s
= 0.
Middle Op. Line intersects this steam at bottom op. line (see figure).

This problem is a two-feed column with the lower feed (steam) input at a non-optimum feed
stage. Otimum feed is 3
rd
above partial reboiler. Need 5.6 equilibrium stages plus PR.




4.D28. This problem was 4.D35 in 2
nd
edition.
Stripping Section:
B B
L L
y x 1 x , y x x 0.02
V V
. Feed line is vertical
105


max
min
max
1.0 0.02 V V 1 1
L V 1.24, 4.167
0.81 0.02 B L V 0.24 L V 1


min
V V V B V B 1 6.25 1
1.5 6.25, L V 1.16
B B V V B 6.25

Op line
B
L
1 1 x
1 .16 .02 V
y 1, x= 0.865
L V 1.16
. Intersection Op & feed lines is
D
y
Overall Balance: 10,000 = F = D + B

D B
6000 Fz Dy Bx D .695 B .02 6000 .695D .02D 200
D 5800 / 0.675 8592.6 kgmoles/day, B 1407.4 . Need a use for impure distillate.

Figure for 4.D28
106


4.D29.
D
L L 1
L D 3, 3 4, 1 x 0.9 0.225
V V 4

Step off 3 stages on top op line. Find
S
x 0.76 . Point on top op line at
S
x 0.76 is also
on middle op. line



In top: L = 3D = 150.375 and V = L + D = 200.500

Since saturated liquid withdrawn, V = V = 200.500
and L = L S = 150.375 15 = 135.375

Middle op. line slope L V 135.375 200.5 0.6752

Middle Op line:
S D
yV L x Sx Dx

S D
Sx Dx L 11.4 45.1125
y x , when x 0, y 0.2819
V V 200.5


Feed z = 0.6, 20% vapor = 80% liquid q = 0.8.
Feed line slope q q 1 0.8 0.4 4

x
D
10
x
s
, S

= 15
V
z = .6
3
6
F

= 100
L
x
6
= .1
F = D + B + S
D B S
Fz Dx Bx Sx
Solve simultaneously D = 50.125, B = 34.875
107


4.D30*. a. Subcooled Reflux:
o o
1 o 2 1
1 o
L L D 3
, L L c and V V c
V 1 L D 4


o
1
c L 500
3
,
o 1
o 1
o
2
1 o
1
4
L V
4 L 3 L 3
1 L 1
V
V L 1
3 3 V

Substituting in values, we have
1
2
L 1.0 1 4
L 1 1 4 5 4 5
, Top op. line
D
y x x 0.92
Step off two stages.
2 S S
x 0.62 x y

108

Mixed feed to column:
M
F S F 1500 ,
S M M
Fz Sy F z
Solving for mixed feed,
M
z 0.52667
Energy balance for mixed feed,
F S M FM
Fh SH F H
Since
s F
H h (satd vapor),
FM S F
h H h (satd vapor), and
FM
q 0 (horizontal).
External Balances: F = D + B,
D B
Fz Dx Bx , Solving simultaneously D = 500 & B = 500

o o
4
L 3D 1500, and L L 2000
3
(subcooled reflux), V L D 2500
Middle: L L S 2000 500 1500, V V 2500 , Slope
L 1500 3
.6
V 2500 5

Intersects Top Op. line at
S
x
Plot Bot. from
B
y x x to intersection feed line and middle
Step off stages (see figure). Need 4 8/9 equilibrium stages.

b. Mass balances for mixed feed injection:
M
V F V

M
V V F 2500 1500 1000, V B 1000 500 2


4.D31*. Was problem 4.D36 in 2
nd
edition. Solution is trial & error. Need to pick L/V. Final answer
shown in figure.

L .63 .385 0.245
.389
V .63 0.63


L V L 0.389
.636
1 L V D 0.611

Note feed stage is not optimum.
109




Figure for problem 4D31

4.D32*. External Balance: F = B = 50, and
B
Fz Bx . Thus
B
x z 0.4 .
4.D33*. a.
min
.75 .452
L V 0.397
.75
(tangent pinch)

min
min
L L V
0.659
D 1 L V

b.
act
L D
L D 1.318. L V 0.569
1 L D

Top operating Line through
D
y x y 0.75
Bottom through
B
y 0, x y 0.1 and intersection feed and top operating lines.
Feed: L L F .25F, q 5 4, slope q q 1 5
Optimum feed is 3
rd
from bottom. Need 9 real stages plus partial condenser (see figure).
c. From figure slope of bottom operating line L V 2.025
Since saturated steam and CMO valid, B S L V
Also have mass balances, S + F = B + D

S B D s
Sy Fz Bx Dy y 0
Solve 3 eqs. simultaneously. S = 760 lbmoles/hr = 13,680 lb steam/hr.

110


4.D34*. Trial and Error, Feed: L L F F 2, q 3 2, Slope 3.

Figure for 4.D34
4-E1. Find (L/V)
min
(see diagram)

min
0.95 0.613
L V 0.3547
0.95 0
,
min
min min
L V L
0.5497
D 1 L V

111


act min
L L
2 1.0994
D D
,
act
L L D
0.5237
V 1 L D

External M.B. F = D + B,
D B
Fz Dx Bx
or Eq. (4.3)
B
D B
z x 0.6 0.025 kgmol
D F 100 62.162
x x 0.95 0.025 h
, B = F D = 37.838

Top op line
D
L
y x 1 L F x normal
V
. At x = 0, y = 0.4525
1
st
middle
B
L L
y x 1 x
V V
looks like usual bottom!
Goes through
B
y x x , and intersection top & feed line.
Slope
L 168.030
1.2906
V 130.192

At pump-around return, V V 130.192
L L P 208.030, L V 208.030 130.192 1.5979
At pump-around removal, V V 130.192 , L L P L 168.030
Check at bottom L V B or 130.192 168.030 37.838, OK
Bottom Op line
B
L L
y x 1 x
V V
, Same as first middle!!!
Step off P.R. stage 1 & 2 above.
P
x is liquid from stage 2,
P
x 0.335 . Vertical line at
P
x is
withdrawal line for pump-around and it is feed-line for return of pump-around. 2
nd
middle op line
slope L V intersects
P
x withdrawal & feed line where bottom & 1
st
middle intersect.
L L D D 68.030
V = L + D = 130.192
L L F 168.030
V
L
L
F
V

L
L
L

V
V
P =
P
x

B
D
At feed V V (satd liquid)
112

Using MB:
P B
yV Px Bx L x ,
B
P
Bx L P
y x x
V V V

When
B
P
32838 0.025
Bx P 40
y 0, x x .335 0.0690
L L 208.030 208.030

Draw, 2
nd
middle Step off stage 2 & start 3. 3 is op loc. for feed and where pump-around is
returned. Need PR + 6 equil stages. (Actually 5 and a large fraction)

















4-E2*. Feed:
L L q
q , L L 1.5F, q 3 2, slope 3
F q 1

Bottom op. line: Since steam is saturated vapor S V and B L
Thus, 1 S B L V 1.2 . Operating line goes through
B
y 0, x x 0.015
Middle op. line: V B Side L S or V L S B Side

B side S
Vy Bx Side x L x Sy
Since
B side
S
Bx Side x L
y 0 this is, y x
V V

Side stream is removed as a saturated liquid so q = 1.
Step off two stages (see figure) and find
side
x 0.0975
113

Find slope: V S, B L Side

L Side B Side B 1 0.4 1
1.68 slope
V S S B 0.833

Draw in the middle operating line. Step off 4 stages. Trial and error to find
D
x .85 (see figure
for final result).



4-E3*. This column has 4 sections. The exact shape is not known ahead of time. Plot top operating line

D
L L D 1.86 L L
0.650, y x 1 x
V 1 L D 2.86 V V


D D
L
1 x .35 .8 .28, y x x 0.8
V

Step off 8 stages and find
S S
x 0.495 y . Feed line for this vapor is horizontal. Feed line for
feed to column is vertical at z = 0.32. From figure the feed is injected below the liquid
withdrawal and above vapor stream from intermediate reboiler. Can now calculate flows in each
section of columns.
Overall Balance:
D B
Fz Dx F D x
114


B
D B
F z x .3 1000
D 385
x x .78

Flows:
L
L D 716.1, V L D 1101.1 V V , V V S 643.8
D

L L S 258.8, L V 0.235
L L L F 1258.8, L V 1.143
L V 1.955 (this is a check)
To plot: From stage 8 draw line of slope L/V. From intersection of first intermediate operating
line and feed line draw line of slope L/V. Draw line from intersection of second intermediate
operating line with line
s
y y to
B
y x x 0.02 . Check if slope L V 1.955 . Optimum
feed is 10
th
below condenser while vapor from intermediate reboiler is returned on 11
th
stage.
Need 12 stages.
Note: Small differences in stepping off stages may change column geometry.



















4.E.4. New Problem in 3
rd
Edition. External Balance.
a)
D B
F D B, Fz Dx Bx

B
D B
z x .25 .025
D F 100 25.7 kmol h
x x .9 .025
, B 74.3kmol h
b) V B 1.0, V B 74.3
L V B 148.6, L V 2.0
At feed, amount condensed = C = F/10 = 10

1
L L F C L F F 148.6 110 38.6
10

V V C 74.3 10 64.3
115

At stage 2
R
L L L and V V 64.3

0
L L 21.4

L 21.4
0.333 1 3
V 64.3


R
L L L 38.6 21.43 17.17 kgmoles hour
c) Top Op. line:
R
D D
L D
L L L
y x x x 1 x
V V V V




Plot
Middle:
D D
L 38.6
y x 1 L V x . Slope 0.600 y x x 0.9
V 64.3

Bottom:
B
y L V x L V 1 x

B
y x x

V B 1
Slope L V 2
V B

Now have somewhat redundant information.
Can plot bottom.
Intersection bottom and feed line should also be on middle. Or use this pt to find middle or
bottom op. line.

From graph: Opt. Feed = #4.
Need 6 stages + P.R.









D
Slope L V 1 3, y x x 0.9
R
L

V
L
D
0
L
Envelope for top
R
D D R
L D
y x, y x x sin ce V L L D
V L

D
L
x 0 , y 1 x 0.6
V

116

Graph for 4.E.4.


4.E.5. New Problem in 3
rd
Edition.
L V B V 1 2.25 9 4 9
V V V B 1.25 5 4 5

Bot. Op. Line:
B B
L L
y x 1 x , y x x , Slope L V
V V

External M.B.:
D B
F D B and Fz Dx Bx ,
B
D B
z x D .3 0.10 .2 1
F x x 0.90 0.10 .8 4

D 250 kmol day, B 750 kmol day , V V B B 1.25 750 937.5 kmol day
L V B 937.50 750 1687.5 kmol day
At feed stage: V V 937.5, L L F or L L F 1687.5 1000 687.5

L 687.5
0.733333
V 937.5

117

MB:
D
L x Dy V y, D V L ,
D
L L
y x 1 y
V V

Goes through
D
y x y , and intersects feed and bot. op. lines.
At side withdrawal: 687.5 L L , V S V or V 937.5 200 737.5
Op line by intermediate condenser: L D V S

D S
D S
Dy Sx L
L x Dy V y Sx or y x
V V

Find from intersection L V op line @
S
y y plus
687.5
slope 0.932
737.5
or intersection
top op line and
S
x x
At side stream feed point: L L S 487.5, V V 737.5. Thus,
L 487.5
0.661
V 737.5

Top op. line:
D
L L
y x 1 y
V V
Can draw,
int ercept
y 1 .661 .9 0.305
118


Graph for 4.E5.
4-F1.

S
V
B
L

970.33
Since bottoms are very pure
B water
h h @ 212F

S
H 1381.4
Equil
H 1192 (in column)
Extra heat 189.4 Btu/lb
Since y x, MW are same
v
119

Must vaporize material in column.

extra heat MW
189.4
v S S 0.1952S
MW 970.33

B L v, V S v

L B v B S v S 2 0.1952
1.837
V S v 1 v S 1 0.1952

If super heat not included L V B S 2 , which is incorrect.

4-F3*. An approximate check is to compare molar latent heats of vaporization. Data is available in
Perrys and in Himmelblau.
a. See Example 4-4.
b. isopropanol = 159.35 cal/g. MW = 60.09, = 9.575 kcal/mole. Water = 9.72 kcal/mole.
CMO is OK.
c. CMO is not valid.
AA W
5.83 kcal / gmole, 9.72
d. nC4. = 5.331 kcal/mole, MW = 58.12. = 0.0917 kcal/g
nC5. = 6.16 kcal/mole, MW = 72.15. = 0.0854 kcal/g
Constant mass overflow is closer than constant molar overflow.
e. benzene. = 7.353 kcal/mole, MW = 78.11, = 0.0941 kcal/g
toluene. = 8.00 kcal/mole, MW = 92.13, = 0.0868 kcal/g
CMO is within about 6%.

4G1. a*. Answer should be close, but not identical, to result obtained in Example 4-4.

4G2. Was 4.G4 in 2
nd
edition. Used Peng-Robinson.
C
Q 44, 437, 300 Btu/hr,

R
Q 49, 859, 400 Btu/hr.
Optimum
F
N 17 & N 27 (Total condenser is #1)

D B
x 0.9992 and x 0.00187.
4G3*. See answers to selected problems in back of book.

4.H.1. New Problem in 3
rd
Edition. Use VBA program in Appendix B of Chapter 4.
xd 0.995 xb 0.011 F 250 z 0.4
L/D 3 q 0 feed stg 4
partial reboiler total condenser Ethanol-water Prob. 4H1.
VLE 6th 5th 4th 3rd 2nd 1st constant
-24.75 85.897 -118.03 82.079 -30.803 6.6048 0
yeqatxint 0.584177 yint 0.4 xint 0.2016667
stage x y
1 0.011 0.069033
2 0.039445 0.217358
3 0.112146 0.451872
4 0.227091 0.607193
5 0.477924 0.769849
6 0.694798 0.867005
7 0.824341 0.919132
8 0.893842 0.954863
9 0.941484 0.979257
120

10 0.974009 0.992216
11 0.991288 0.996557

4.H.2. New Problem in 3
rd
Edition. The VBA program is in Appendix B of Chapter 4. With L/D = 6.94
the result is:
xd 0.7 xb 0.0001 F 1000 z 0.1
L/D 6.94 q 0 feed stg 28
partial reboiler total condenser Ethanol-water Prob. 4H3.
VLE 6th 5th 4th 3rd 2nd 1st constant
-47.949 161.42 -212.43 138.68 -46.65 7.9322 0
yeqatxint 0.099219 yint 0.1 xint 0.0135447
stage x y
Reflux rate too low Reflux rate too low

With L/D = 6.95 the result is given below. With feed stages below 85 the feed stage was too low.
xd 0.7 xb 0.0001 F 1000 z 0.1
L/D 6.95 q 0 feed stg 85
partial reboiler total condenser Ethanol-water Prob. 4H3.
VLE 6th 5th 4th 3rd 2nd 1st constant
-47.949 161.42 -212.43 138.68 -46.65 7.9322 0
yeqatxint 0.100057 yint 0.1 xint 0.0136691
stage x y
1 0.0001 0.000793
2 0.000194 0.001538
3 0.000295 0.002338
4 0.000404 0.003197
5 0.000521 0.004117
6 0.000646 0.005102
7 0.000779 0.006154
8 0.000922 0.007277
9 0.001075 0.008472
10 0.001237 0.009742
11 0.00141 0.011089
12 0.001593 0.012515
13 0.001786 0.014021
14 0.001991 0.015608
15 0.002206 0.017276
16 0.002433 0.019025
17 0.00267 0.020853
18 0.002919 0.022758
19 0.003178 0.024739
20 0.003447 0.026791
21 0.003725 0.02891
22 0.004013 0.03109
23 0.004309 0.033327
24 0.004613 0.035613
25 0.004924 0.037941
26 0.00524 0.040302
27 0.005561 0.042689
28 0.005885 0.045091
121

29 0.006211 0.0475
30 0.006538 0.049907
31 0.006865 0.052301
32 0.00719 0.054674
33 0.007513 0.057017
34 0.007831 0.059321
35 0.008144 0.061579
36 0.00845 0.063782
37 0.00875 0.065925
38 0.009041 0.068002
39 0.009323 0.070008
40 0.009595 0.071938
41 0.009858 0.07379
42 0.010109 0.075561
43 0.01035 0.077251
44 0.010579 0.078857
45 0.010797 0.08038
46 0.011004 0.08182
47 0.0112 0.083179
48 0.011384 0.084459
49 0.011558 0.085661
50 0.011721 0.086787
51 0.011874 0.087841
52 0.012018 0.088826
53 0.012151 0.089743
54 0.012276 0.090598
55 0.012392 0.091392
56 0.0125 0.092129
57 0.0126 0.092812
58 0.012693 0.093445
59 0.012779 0.09403
60 0.012858 0.094571
61 0.012932 0.09507
62 0.012999 0.095531
63 0.013062 0.095955
64 0.01312 0.096346
65 0.013173 0.096705
66 0.013222 0.097036
67 0.013267 0.09734
68 0.013308 0.09762
69 0.013346 0.097876
70 0.013381 0.098112
71 0.013413 0.098328
72 0.013442 0.098526
73 0.013469 0.098708
74 0.013494 0.098874
75 0.013516 0.099027
76 0.013537 0.099167
77 0.013556 0.099295
78 0.013573 0.099412
122

79 0.013589 0.099519
80 0.013604 0.099618
81 0.013617 0.099707
82 0.013629 0.09979
83 0.013641 0.099865
84 0.013651 0.099934
85 0.01366 0.099997
86 0.013665 0.100032
87 0.013706 0.100305
88 0.014018 0.102401
89 0.016416 0.11824
90 0.034534 0.223718
91 0.155188 0.516574
92 0.490181 0.652848
93 0.646064 0.724878

4.H.3. New Problem in 3
rd
Edition. The Spreadsheet is:
xd 0.7 xb 0.0001 F 1000 z 0.17
Multiplier 1.05 q 0.5 feed stg 17
partial reboiler total condenser Ethanol-water Problem 4.H4.
VLE 6th 5th 4th 3rd 2nd 1st constant
-47.949 161.42 -212.43 138.68 -46.65 7.9322 0
L/Dmin 1.687377 L/Vmin 0.62789 L/D 1.771746 L/V 0.639217
stage x y
1 0.0001 0.000793
2 0.000229 0.001812
3 0.000418 0.003309
4 0.000696 0.0055
5 0.001104 0.008697
6 0.001698 0.013331
7 0.002559 0.019994
8 0.003797 0.029452
9 0.005555 0.042644
10 0.008006 0.060585
11 0.01134 0.08415
12 0.015719 0.113686
13 0.021208 0.148522
14 0.027681 0.186647
15 0.034766 0.224911
16 0.041877 0.259918
17 0.048382 0.28916
18 0.057276 0.325158
19 0.113592 0.469947
20 0.340102 0.575494
21 0.505222 0.659654
22 0.636883 0.719124
Because the multiplier is close to 1.0, this answer is very sensitive to the data fit used.
One solution to the coding is the following VBA program:

Option Explicit
123

Sub McCabeThiele()
' Find minimum reflux ratio assuming it occurs at feed plate. Then
' L/D actual = L/D min times Multiplier. Steps off stages from the bottom up.
' Assumes that the feed stage is specified.
Sheets("Sheet2").Select
Range("A8", "G108").Clear
Dim i, feedstage As Integer
Dim D, B, xd, xb, F, z, q, LoverD, LoverV, x, y, xint, yint, yeq As Single
Dim a6, a5, a4, a3, a2, a1, a0, L, V, LbaroverVbar, LoverDmin As Single
Dim LoverVmin, LoverVdelta, Multiplier As Single
' Input values from spread sheet
xd = Cells(1, 2).Value
xb = Cells(1, 4).Value
F = Cells(1, 6).Value
z = Cells(1, 8).Value
Multiplier = Cells(2, 2).Value
q = Cells(2, 4).Value
feedstage = Cells(2, 8).Value
' Fit of equilibrium data to 6th order polynomial to find y. a6 is multiplied
' by x to the 6th power.
a6 = Cells(5, 1).Value
a5 = Cells(5, 2).Value
a4 = Cells(5, 3).Value
a3 = Cells(5, 4).Value
a2 = Cells(5, 5).Value
a1 = Cells(5, 6).Value
a0 = Cells(5, 7).Value
' Calculate intersection point of two operating lines and use this to find
' minimum L/D and L/V. Initialize
LoverV = 1
LoverVdelta = 0.00001
Do
LoverV = LoverV - LoverVdelta
xint = ((-(q - 1) * (1 - LoverV) * xd) - z) / (((q - 1) * LoverV) - q)
x = xint
yint = LoverV * xint + (1 - LoverV) * xd
' Equilibrium y at value of x intersection. When yint=yeq, have minimum L/V and L/D.
yeq = a6 * x ^ 6 + a5 * x ^ 5 + a4 * x ^ 4 + a3 * x ^ 3 + a2 * x ^ 2 + a1 * x + a0
Loop While yint < yeq
'Print intersection and equilibrium values.
LoverVmin = LoverV + LoverVdelta
LoverDmin = LoverVmin / (1 - LoverVmin)
LoverD = Multiplier * LoverDmin
LoverV = LoverD / (1 + LoverD)
Cells(6, 2).Value = LoverDmin
Cells(6, 4).Value = LoverVmin
Cells(6, 6).Value = LoverD
Cells(6, 8).Value = LoverV
' Calculate flow rates and ratios.
D = ((z - xb) / (xd - xb)) * F
L = LoverD * D
124

V = L + D
LbaroverVbar = (LoverV + (q * F / V)) / (1 - ((1 - q) * F / V))
' Step off stages from bottom up. First stage is partial reboiler. Initialize
x = xb
i = 1
' Loop in stipping section stepping off stages with equilibrium and operating eqs.
Do While i < feedstage
y = a6 * x ^ 6 + a5 * x ^ 5 + a4 * x ^ 4 + a3 * x ^ 3 + a2 * x ^ 2 + a1 * x + a0
Cells(i + 7, 1).Value = i
Cells(i + 7, 2).Value = x
Cells(i + 7, 3).Value = y
i = i + 1
x = (y / LbaroverVbar) + (LbaroverVbar - 1) * xb / LbaroverVbar
Loop
' Calculations in enriching section continues to Loop While y < xd.
Do While y < xd
y = a6 * x ^ 6 + a5 * x ^ 5 + a4 * x ^ 4 + a3 * x ^ 3 + a2 * x ^ 2 + a1 * x + a0
Cells(i + 7, 1).Value = i
Cells(i + 7, 2).Value = x
Cells(i + 7, 3).Value = y
i = i + 1
x = (y / LoverV) - (1 - LoverV) * xd / LoverV
If x < 0 Then
Cells(i + 7, 4).Value = "Feed stage too low"
Exit Do
End If
If i > 100 Then
Cells(i + 7, 6).Value = "Too many stages"
Exit Do
End If
Loop
End Sub

125
SPE 3
rd
Ed. Solution Manual Chapter 5
New Problems and new solutions are listed as new immediately after the solution number. These new
problems are: 5A15, 5C1, 5D1, 5D2, 5D9, 5D10, 5H1 to 5H5. Problems and solutions from the first
edition that were not in the second edition are: 5D6, 5D8, 5D11-5D13, 5E1.
5.A6.
Ethane is less volatile than methane so it decreases toward distillate. At bottoms it is more
volatile than propane and butane, so must decrease towards bottoms. Thus ethane
concentrates within column.
5.A7. 1. c; 2. c; 3. a (saturated liquid feed); 4. c; 5. B
5.A9. Pure LK cannot be withdrawn because LNK is present. Pure LNK can be removed at
distillate if all LK is removed in side stream. However, recovery of LNK will be < 100%.
5.A13. If
HNK
z F (frac. rec. HK in bot)
HK
z F, then there is more HNK in bottoms and curves
cross.
5A15. New Problem in 3
rd
Edition. a. a, b. d, c. b, d. c, e. b.
5.C1. New Problem in 3
rd
Edition. Start with equilibrium equation, y
i,j
= K
i,j
x
i,j
and multiply right hand
side by K
ref,j
/K
ref,j
. One obtains y
i,j
=
i-ref,j
K
ref,j
x
i,j
. Then 1 = y = (
i-ref,j
K
ref,j
x
i,j
) . Since K
ref,j
is
constant, bring it outside the summation and solve for K
ref,j
= 1/ (
i-ref,j
x
i,j
). This is Eq. (5-29).
126

5.D1. New Problem in 3
rd
Edition.

wt frac. in distillate: M = 0.38399, E = 0.61273, P = 0.00327, B = 0.0.

M,B
B,B
E,B
P,B
i,b
Bx 0 0
Bx 1.0 12, 000 0.23 2760.0
Bx 1 0.978 12, 000 0.31 81.84
Bx 0.994 12, 000 0.27 3220.56
B x B 6062.4 kg
i,b
wt. frac
x
0
0.4553
0.0135
0.5312
hr

Check B+D=F, OK.

5.D2. New Problem in 3
rd
Edition.
4 5 6
y 0.30, y 0.5, y 0.20, P 760 mmHg
Raoults Law
i i i
y P VP x
Antoines Eqn.
10
B
log VP A
T C

Dew Point condition is
i
x 1

i
i
i
y P
x
VP
(from Raoults Law)
M,d
B,d
E,d
P,d
Dx 1.00 2, 000 0.19 2280
Dx 0 0
Dx 0.978 2, 000 0.31 3638.16
Dx 1 0.994 12, 000 0.27 19.44

i ,d
D x D 5937.6 kg/h

a)
b) assume NK (Methanol and n-butanol) do not distribute (all
methanol in top and all butanol in bottom).
127


i
B
VP 10 A
T C
(from Antoines Eqn)
Combine with Dew Point condition

4 5 6
4 5 6
4 5 6
5 6 4
B B B
A A A
T C T C T C
y P y P y P
1
10 10 10



935.86 1064.8 1171.17
6.809 6.853 6.876
T 238.73 T 233.01 T 224.41
0.30 760 0.50 760 0.20 760
1
10 10 10


Using Goal Seek in Excel, T = 41.3C

5.D3. Assume that ethanol is HK. Then assume that HNKs are totally in the bottoms.

M,dist E,dist
x .99, x .01
2195.6 = (.998) (0.22) (10,000) =
MD
Dx

MD
2195.6 2195.6
D 2217.78 and B F D 7782.22
x .99

Bottoms: MeOH: .0021 (.22) (10,000) = 4.4

Mbot
4.4
x 0.0006
7782.22


n propanol,bot
1.0 .18 10, 000
x 0.2313
7782.22


n butanol,bot
1.0 .13 10, 000
x .1670
7782.22


EtOH,bot MeOH,bot n p,bot nbut ,bot
x 1 x x x 0.6011

5.D4. a. .99 F
C5 C5,dist C5 C5,bot
z D x (1), and .01 Fz =B x (2)
.98 F
C6,bot C6,bot C6 C6,dist
z Bx (3), and .02 F z D z (4)
Assume all heptane in bottoms

C7 bot ,C6 dist ,C7
Fz Bx (5), x 0 (6)
Take Eqs. 1, 4 & 6: .99 (1000) (.4) =
C5d
Dx
.02 (1000) (.3) = D
C6d
x
0 = D
C7,dist
x

l,dist
Dx D 402 kg moles/hr
B = F D = 1000 402 = 598
b.
C7,dist
x 0

C5,d
.99 1000 .4
x 0.9851
402


C6,d
x 1 0.9851 0.0149
128


C7,bot
1000 .3 1.0
x 0.5017
598


C6,bot
.98 1000 .3
x 0.4916
598


C5,bot
x 1 .5017 .4916 0.0067
c. L = (L/D)D = (2.5) (402) = 1005
V = L + D = 1005 + 402 = 1407
At feed stage: L = L + .6F = 1005 + 600 = 1605
V = V - .4F = 1407 400 = 1007

5.D5. Assume all methanol and ethanol in distillate.

MeOH,dist
Dx 0.55 100 0.01 150 56.5

EtOH,dist
Dx 0.21 100 0.03 150 25.5


prop,dist
Dx 0.993 0.23 100 0.26 150 61.57

butanol,dis
Dx 1 0.995 0.01 100 0.70 150 0.53

D 144.1

1 2
B F F D 105.9
Check:
Pr op,bot
Bx 1 0.993 .23 100 .26 150 0.434

but ,dist
Bx 0.995 .01 100 0.7 150 105.47

Check = 105.90
Mole fractions:

M,bot E,bot prop,bot
0.434
x 0 , x 0, x 0.0041
105.90


but ,bot prop,bot
x 1 x 0.9959


MeOH,dist
M,dist
Dx
56.5
x 0.392
D 144.1


E,dist
25.5
x 0.1767
144.1


p,dist
61.566
x 0.427
144.1


But ,dist
x 0.53 144.1 0.0037

Check = 1.000 OK

5.D6. This is 8.D1. in 1
st
ed.

129


130



5.D7. Assume 100% recovery
2
C , & propylene in distillate.
Assume 100% recovery pentane & hexane in bottoms.

Comp. Distillate kg/h
D
x
C
2
0.3 (1000) + 0.02 (1500) = 330 0.2824
Propylene: 0.006 (1000) + 0.001 (1500) = 7.5 0.0064
n-C
3
(0.991) [1000 (0.45) + 1500 (0.249)] = 816.0885 0.06983
n-C
4
(0.02) [1000 (0.154) + 1500 (0.40)] = 15.08 0.0129
C
5
& C
6
0 = 15.08 0.0

D = = 1168.7


Bottoms flow rate =
1 2
F F D 2500 1168.7 1331.3 kg/h

2 3
B,C B,propylene B,C
.009 1000 0.45 1500 0.249
x 0, x 0, x 0.0056
1331.3


4
B,C
.98 1000 .154 1500 .40
x 0.5550
1331.3


5 6
B,C B,C
1000 0.09 1500 0.18 1500 .15
x 0.2704, x 0.1690
1331.33 1331.31


5.D8. 8.D.6. in 1
st
edition. Assume all benzene is in the distillate.
131


132





5.D.9. New Problem in 3
rd
Edition.
133


5.D10. New Problem in 3
rd
Edition. At the bubble point
i
y 1.0

C5 C6
x .40 , x .60 , p 760 mmHg ,
C5 C6 5 C5 6 C6
y y K x K x 1.0
134

or
C5 C6
C5 C6
tot tot
VP VP
x x 1.0
P P
,
C5 C5 C6 C6 tot
VP x VP x P

1064.8 1171.17
6.853 6.876
T 233.01 T 224.41
0.40 10 0.60 10 760

TC (C5 term) x (.4) C6 term x (.6) SUM
20 441.03 .4 = 176.4 + 121.387 .6 = 249.23
Final result is:
51 1270.095 .4 = 508.04 + 420.28 .6 = 252.17 760.21

5.D11. Was problem 6.D2 in 2
nd
edition SPE. Since
i
x known, want
i i BP i
y 1 K p x
Find new pressure from,
ref old
ref new
i i
K p
K p
K x

Use ethane, or n-pentane as reference.
First guess: Try
5
C
K 1.0, p 115 kPa

7 2
NC C
K 0.13 K 29

i i
K x 29 0.1 1.0 .35 0.13 .55 3.32 p too low.

rep new new
1.0
K p ~ 0.3 p 440 kPa
3.32
,
7 2
NC C
K 0.042 K 8.0

i i
K x 8.0 0.1 0.3 .35 0.042 0.55 0.927

5
C new new
0.3
K p .032 p 400
0.927
kPa,
7 2
nC C
K 0.045 K 8.7

i i
K x 8.7 0.1 0.32 0.35 0.045 0.55 1.004
Answer (within accuracy DePriester Chart) = 400 kPa

5.D12. Was problem 6.D3 in 2
nd
edition SPE. a. Highest B.P. Temp. is pure n-octane.
C8
K 1.0, T 174 C
b. Lowest B.P. Temp. is pure n-hexane.
C6
K 1.0, T 110 C

5.D13. Was problem 6.D6 in 2
nd
edition SPE. Let 1 = n-butane, 2 = n-pentane and 3 = n-hexane. p = 101.3 kPa.
Bubble Point: First guess.
1 2 3
K 1 at T 1 , K 1 at T 36 , K 1 at T 68 .

first 1 1
T z T .2 1 .5 36 .3 68 38

1 2 3 1 1
K 3.6, K 1.08, K 0.36. K x .2 3.6 .5 1.08 .3 .36 1.368
Choose 2 as ref. Eq. (6-14) is:
2 new
K T 1.8 1.368 0.789

new 1 5 i i
T 29 C. K 2.7, K 0.26, and K x 1.013
Eq. (6-14) is:
2 new
K T 0.789 1.013 0.779

new 1 8 BP
T 28.8 K 2.7, K 0.255, and K x 1.006. OK. T 28.8 C.
i i


5. E1. This is 8.E4. in 1
st
edition.
135


136


137


5.H1. New Problem in 3
rd
Edition. Same program as 5.H5 except do not list y values as distillate.
Different input in spreadsheet see below.
Ternary Distillation: Constant relative volatility. Step off stages from bottom up. Use whole stages.
System has A = LK, B = HK and C = HNK

C5H9
alpha
A-B 3.58 alpha B-B 1.86
Alpha C-
B 1 feedstage 8
z A 0.35 z B 0.4 z C 0.25 epsilon (values for 0.00000001
F 200 q 1 L/D 6
N loop(
convergence 100
frac rec B in dist 0.996 guess frac rec C bot 1
df(HNK frac
recovery) 0.9
frac rec A in dist

0.961
D 67.59011 B 132.4099 L/V 0.857143 Lbar/Vbar 1.279858988
xAdist 0.995264 xBdist 0.004734 xCdist 1.56E-06
Mass balance
values

xAbot 0.020618 xBbot 0.601768 xCbot 0.377614
Mass balance
values

138

Stage by stage calculations
i xA yA xB yB xC yC Note x1 =
1 0.020618 0.046992 0.601768 0.7125981 0.377614 0.240409437 xbot
2 0.041225 0.0869 0.688364 0.7538807 0.270411 0.159219617
3 0.072406 0.143487 0.720619 0.7419438 0.206975 0.114569647
4 0.11662 0.218289 0.711292 0.6917342 0.172088 0.089976459
5 0.175066 0.308791 0.672062 0.6158891 0.152873 0.075319919
6 0.245778 0.407144 0.612801 0.5274175 0.141421 0.0654388
7 0.322624 0.502187 0.543675 0.4396808 0.1337 0.058132319
8 0.396885 0.584094 0.475123 0.3632905 0.127992 0.05261578
9 0.515565 0.685129 0.42305 0.2920855 0.061385 0.02278591
10 0.633439 0.774848 0.339977 0.2160685 0.026583 0.009083172
11 0.738112 0.846817 0.251291 0.1497869 0.010597 0.003395923
12 0.822076 0.899855 0.173962 0.098934 0.003962 0.001211305
13 0.883953 0.936484 0.114634 0.0630978 0.001413 0.000418127
14 0.926687 0.960636 0.072825 0.0392226 0.000488 0.000141178
15 0.954865 0.97607 0.044971 0.0238835 0.000164 4.69552E-05
16 0.97287 0.985732 0.027075 0.0142529 5.45E-05 1.54307E-05
17 0.984143 0.991702 0.015839 0.0082925 1.77E-05 4.9941E-06
18 0.991109 0.995362 0.008886 0.0046363 5.57E-06 1.56158E-06


Calc frac recovery C in bottoms

0.9999979 j 3
5.H2. New Problem in 3
rd
Edition. The spreadsheet is,
Ternary Distillation with Constant relative volatility. Step off stages from top down.
System has A = LNK, B = LK and C = HK
alpha A-B 2.25 alpha B-B 1 Alpha C-B 0.21 feedstage 6
z A 0.25 z B 0.35 z C 0.4 epsilon (values for 0.0001
F 100 q 1 L/D 0.3 N loop( convergence 10
frac rec B in dist 0.9 frac rec C in bot 0.97 df(LNK frac recovery) 0.9
Guess: frac rec A in dist 1
D 57.66689 B 42.35298 L/V 0.230769 Lbar/Vbar 1.565105
xAdist 0.43295 xBdist 0.546241 xCdist 0.020809 Mass balance values
xAbot 0.001251 xBbot 0.082639 xCbot 0.91611 Mass balance values
Stage by stage calculations
i xA yA xB yB xC yC Note y1 =
1 0.229688 0.43295 0.65203 0.546241 0.118282 0.020809168 xdist
2 0.180904 0.386044 0.601681 0.570654 0.217416 0.043302907
3 0.16005 0.374786 0.537147 0.559034 0.302804 0.066179941
4 0.147137 0.369973 0.486906 0.544142 0.365957 0.085884852
5 0.13893 0.366993 0.453607 0.532548 0.407464 0.100458716
6 0.133981 0.365099 0.433372 0.524864 0.432647 0.1100371
7 0.062603 0.208987 0.425676 0.631573 0.511721 0.159440004
8 0.021494 0.097273 0.308019 0.619528 0.670486 0.283199293
9 0.004909 0.032934 0.146011 0.435383 0.84908 0.531682943
10 0.000766 0.006976 0.044919 0.181823 0.954315 0.811200627


Calc frac recovery A in distillate 0.998702 j 2
139

5.H.3. New Problem in 3
rd
Edition. (L/D)
min
= 0.26761 by trial and error using spreadsheet in Table 5.A-
1..

5.H.4. New Problem in 3
rd
Edition.
Ternary Distillation with Constant relative volatility. Step off stages from top down.
System has A = LK, B = sandwich and C = HK 5.G.e.
alpha A-B 1.4 alpha B-B 1 Alpha C-B 0.7 feedstage 19
z A 0.25 z B 0.35 z C 0.4 epsilon (values for 1E-10
F 100 q 1 L/D 5 N loop( convergence 100
frac rec B in dist 0.583 frac rec C in bot 0.995 df(LNK frac recovery) 0.8
Guess: frac rec A in dist 0.95
D 44.10818 B 55.89182 L/V 0.833333 Lbar/Vbar 1.21119
xAdist 0.532853 xBdist 0.462613 xCdist 0.004534 Mass balance values
xAbot 0.026781 xBbot 0.261129 xCbot 0.71209 Mass balance values
Stage by stage calculations
i xA yA xB yB xC yC Note y1 =
1 0.447934 0.532853 0.544443 0.462613 0.007623 0.004534307 xdist
2 0.378937 0.462087 0.609404 0.530804 0.011659 0.007108533
3 0.325116 0.40459 0.658055 0.584939 0.016829 0.010471366
4 0.284385 0.359739 0.692247 0.625481 0.023368 0.01477984
5 0.254167 0.325796 0.71427 0.653975 0.031563 0.020228923
6 0.231958 0.300615 0.726286 0.672327 0.041757 0.027058086
7 0.215598 0.282107 0.730061 0.68234 0.054342 0.035552935
8 0.203353 0.268473 0.726901 0.685486 0.069746 0.046040694
9 0.193893 0.258269 0.717703 0.682853 0.088403 0.058877558
10 0.18623 0.250387 0.703059 0.675188 0.110711 0.074425299
11 0.179649 0.244001 0.683384 0.662985 0.136967 0.093014635
12 0.173652 0.238516 0.65905 0.646589 0.167298 0.114894592
13 0.167915 0.233519 0.630501 0.62631 0.201584 0.140170911
14 0.162254 0.228738 0.598352 0.602519 0.239393 0.168742359
15 0.156599 0.224021 0.563437 0.575729 0.279964 0.200250121
16 0.150964 0.219308 0.526798 0.546633 0.322237 0.234059083
17 0.145428 0.214612 0.489617 0.516101 0.364955 0.269286831
18 0.140099 0.209999 0.453098 0.485116 0.406803 0.304884695
19 0.135091 0.205558 0.418339 0.454684 0.446571 0.339758041
20 0.100543 0.157965 0.402362 0.45154 0.497094 0.39049513
21 0.071479 0.116121 0.372448 0.43219 0.556074 0.45168881
22 0.048123 0.080918 0.329669 0.395957 0.622209 0.523124245
23 0.030232 0.05263 0.276757 0.344144 0.693012 0.603226557
24 0.017191 0.03096 0.2177 0.280057 0.765109 0.688982619
Mass balance: fraction stage, A, B, C calculated at bottom, % error B
0.264616 0.026781 0.261129 0.71209 6.65242E-09
Calc frac recovery A in distillate 0.940127 j 4


Part d. Fractional recovery of B in distillate. is 0.725. Note that B goes through a maximum of close to
1% on stages 7 and 8.
Ternary Distillation with Constant relative volatility. Step off stages from top down.
System has A = LK, B = trace sandwich and C = HK 5.G.e. Part d.
alpha A-B 1.4 alpha B-B 1 Alpha C-B 0.7 feedstage 19
140

z A 0.38 z B 0.02 z C 0.6 epsilon (values for 1E-10
F 100 q 1 L/D 4 N loop( convergence 100
frac rec B in dist 0.725 frac rec C in bot 0.995 df(LNK frac recovery) 0.8
Guess: frac rec A in dist 0.99
D 39.37044 B 60.62956 L/V 0.8 Lbar/Vbar 1.308
xAdist 0.95555 xBdist 0.03683 xCdist 0.00762 Mass balance values
xAbot 0.00626 xBbot 0.009071 xCbot 0.984668 Mass balance values
Stage by stage calculations
i xA yA xB yB xC yC Note y1 =
1 0.934659 0.95555 0.050434 0.03683 0.014907 0.00761993 xdist
2 0.909255 0.938837 0.064694 0.047713 0.026051 0.013449322
3 0.878109 0.918514 0.079128 0.059121 0.042762 0.02236485
4 0.839847 0.893598 0.092985 0.070669 0.067168 0.035733649
5 0.793216 0.862987 0.105202 0.081754 0.101582 0.055258739
6 0.737611 0.825683 0.114471 0.091527 0.147918 0.082789801
7 0.673775 0.781199 0.119471 0.098942 0.206753 0.119858439
8 0.604361 0.73013 0.119295 0.102943 0.276345 0.166926476
9 0.53382 0.674599 0.113888 0.102802 0.352293 0.222599678
10 0.467334 0.618166 0.104227 0.098476 0.428438 0.283358088
11 0.409234 0.564978 0.092025 0.090748 0.498741 0.344274648
12 0.361849 0.518497 0.079126 0.080986 0.559026 0.400517103
13 0.325379 0.480589 0.066982 0.070666 0.607639 0.448744495
14 0.298551 0.451414 0.056436 0.060951 0.645013 0.487634893
15 0.279454 0.429951 0.047786 0.052515 0.67276 0.517534464
16 0.266162 0.414673 0.040972 0.045595 0.692866 0.539732259
17 0.257043 0.40404 0.035754 0.040143 0.707203 0.555816779
18 0.250839 0.396745 0.031838 0.035969 0.717324 0.567286133
19 0.246633 0.391781 0.028939 0.032836 0.724428 0.575383054
20 0.193371 0.320667 0.029598 0.035058 0.777031 0.644274899
21 0.145301 0.251 0.029111 0.03592 0.825588 0.713080242
22 0.105056 0.188125 0.027585 0.035283 0.867359 0.776592117
23 0.073452 0.135485 0.025264 0.033287 0.901284 0.831228269
24 0.049874 0.094147 0.022436 0.030252 0.92769 0.875601733
25 0.032959 0.063306 0.019353 0.026552 0.947688 0.910141552
26 0.02117 0.041182 0.016207 0.02252 0.962623 0.93629802
27 0.013123 0.025762 0.013125 0.018405 0.973752 0.955833007
28 0.00771 0.015236 0.010183 0.014373 0.982107 0.970390508
29 0.004108 0.008156 0.007421 0.010525 0.988471 0.981318291
Mass balance: fraction stage, A, B, C calculated at bottom, % error B
0.402462 0.00626 0.009071 0.984668 5.53216E-07
Calc frac recovery A in distillate 0.990012 j 16

5.H.5. New Problem in 3
rd
Edition.
Ternary Distillation with Bubble point Calc. Step off stages from bottom
up.
System has A = LK, B = HK and C = HNK
feedstage 5
z A 0.3 z B 0.3 z C 0.4 epsilon 1E-08
F 100 q 1 L/D 8 N loop 100
frac rec B in bot 0.997 guess frac rec C bot 1 df 0.9
141

frac rec A in dist 0.995 Trebguess 500 p, psia 14.7
K const. aT1 aT2 aT6 ap1 ap2 ap3
nB=A -1280557 0 7.94986 -0.96455 0 0
nPen=B -1524891 0 7.33129 -0.89143 0 0
nHex=C -1778901 0 6.96783 -0.84634 0 0

D 29.940006 B 70.05999 L/V 0.88889 Lbar/Vbar 1.26
xAdist 0.9969938 xBdist 0.003006 xCdist 2.2E-07
xAbot 0.002141 xBbot 0.42692 xCbot 0.57094

stage xA yA xB yB xC yC T KB
1 0.002141 0.0103991 0.42692 0.6614042 0.57094 0.3282 582.283 1.549247
2 0.0086951 0.0361347 0.613018 0.7887656 0.37829 0.1751 570.622 1.286692
3 0.0291201 0.1069354 0.714099 0.7929732 0.25678 0.10009 561.854 1.110453
4 0.0853111 0.270011 0.717438 0.6674458 0.19725 0.06254 551.835 0.930318
5 0.214736 0.5324156 0.617813 0.4297616 0.16745 0.03782 536.481 0.695617
6 0.4743433 0.786406 0.483106 0.2080991 0.04255 0.00549 513.748 0.430752
7 0.7600825 0.9294091 0.233736 0.0700679 0.00618 0.00052 498.351 0.299774
8 0.920961 0.980028 0.078451 0.0199309 0.00059 4.1E-05 491.769 0.254057
9 0.9779073 0.9946891 0.022047 0.0053079 4.6E-05 3E-06 489.686 0.240759
10 0.994401 0.9986729 0.005596 0.0013269 3.4E-06 2.2E-07 489.103 0.237137

Distillate mole fracs = y values 0.9986729 0.00133 2.2E-07
Calc frac recovery C in bottoms 0.9999998 j 3

Option Explicit
Sub Ternary_bottom_up_BP()
' Ternary distillation with constant alpha. Frac recoveries of LK and HK given.
' There is a HNK present and its frac rec in bottoms is guessed.
Sheets("Sheet1").Select
Range("A18", "I150").Clear
Dim i, j, k, feedstage, N As Integer
Dim AaT1, AaT6, Aap1, BaT1, BaT6, Bap1, CaT1, CaT6, Cap1 As Double
Dim F, fracBbot, fracCbot, q, LoverD, LoverV As Double
Dim LbaroverVbar, D, B, L, V, Lbar, Vbar, Eqsum, fracAdist As Double
Dim xA, xB, xC, yA, yB, yC, zA, zB, zC, xAbot, xBbot, xCbot As Double
Dim DxA, DxB, DxC, BxA, BxB, BxC, xAdist, xBdist, xCdist As Double
Dim fracCbotcalc, difference, epsilon, df As Double
Dim T, p, Tinit, KA, KB, KC, sum As Double
'Input data
AaT1 = Cells(9, 2).Value
AaT6 = Cells(9, 4).Value
Aap1 = Cells(9, 5).Value
BaT1 = Cells(10, 2).Value
BaT6 = Cells(10, 4).Value
Bap1 = Cells(10, 5).Value
CaT1 = Cells(11, 2).Value
CaT6 = Cells(11, 4).Value
Cap1 = Cells(11, 5).Value
feedstage = Cells(3, 8).Value
F = Cells(5, 2).Value
142

q = Cells(5, 4).Value
LoverD = Cells(5, 6).Value
zA = Cells(4, 2).Value
zB = Cells(4, 4).Value
zC = Cells(4, 6).Value
fracBbot = Cells(6, 3).Value
fracCbot = Cells(6, 6).Value
fracAdist = Cells(7, 4).Value
epsilon = Cells(4, 8).Value
N = Cells(5, 8).Value
df = Cells(6, 8).Value
p = Cells(7, 8).Value
Tinit = Cells(7, 6).Value
' The For loop (remainder of program) is to obtain convergence of guess of
' fractional recovery of A in distillate.
For j = 1 To N
' Calculate compositions and flow rates.
DxA = F * zA * fracAdist
DxB = F * zB * (1 - fracBbot)
DxC = F * zC * (1 - fracCbot)
BxA = F * zA * (1 - fracAdist)
BxB = F * zB * fracBbot
BxC = F * zC * fracCbot
D = DxA + DxB + DxC
B = BxA + BxB + BxC
xAdist = DxA / D
xBdist = DxB / D
xCdist = DxC / D
xAbot = BxA / B
xBbot = BxB / B
xCbot = BxC / B
L = LoverD * D
V = L + D
LoverV = L / V
Lbar = L + q * F
Vbar = Lbar - B
LbaroverVbar = Lbar / Vbar
' Print values of flowrates and mole fractions
Cells(13, 2) = D
Cells(13, 4) = B
Cells(13, 6) = LoverV
Cells(13, 8) = LbaroverVbar
Cells(14, 2) = xAdist
Cells(14, 4) = xBdist
Cells(14, 6) = xCdist
Cells(15, 2) = xAbot
Cells(15, 4) = xBbot
Cells(15, 6) = xCbot
' initialize (reboiler =1) and start loops
i = 1
xA = xAbot
143

xB = xBbot
xC = xCbot
T = Tinit
' Calculations in stripping section: equilibrium then operating.
Do While i < feedstage
' Bubble point calculaton
For k = 1 To 10
KB = 1
KA = Exp((AaT1 / (T * T)) + AaT6 + (Aap1 * Log(p)))
KB = Exp((BaT1 / (T * T)) + BaT6 + (Bap1 * Log(p)))
KC = Exp((CaT1 / (T * T)) + CaT6 + (Cap1 * Log(p)))
yA = KA * xA
yB = KB * xB
yC = KC * xC
sum = yA + yB + yC
KB = KB / sum
T = Sqr(BaT1 / (Log(KB) - BaT6 - (Bap1 * Log(p))))
Next k
' Print values
Cells(i + 17, 1).Value = i
Cells(i + 17, 2).Value = xA
Cells(i + 17, 3).Value = yA
Cells(i + 17, 4).Value = xB
Cells(i + 17, 5).Value = yB
Cells(i + 17, 6).Value = xC
Cells(i + 17, 7).Value = yC
Cells(i + 17, 8).Value = T
Cells(i + 17, 9).Value = KB
' Bottom operating line
i = i + 1
xA = yA / LbaroverVbar + (1 - (1 / LbaroverVbar)) * xAbot
xB = yB / LbaroverVbar + (1 - (1 / LbaroverVbar)) * xBbot
xC = yC / LbaroverVbar + (1 - (1 / LbaroverVbar)) * xCbot
Loop
' Calculations in enriching section
Do
For k = 1 To 10
' Bubble point calculation
KA = Exp((AaT1 / (T * T)) + AaT6 + (Aap1 * Log(p)))
KB = Exp((BaT1 / (T * T)) + BaT6 + (Bap1 * Log(p)))
KC = Exp((CaT1 / (T * T)) + CaT6 + (Cap1 * Log(p)))
yA = KA * xA
yB = KB * xB
yC = KC * xC
sum = yA + yB + yC
KB = KB / sum
T = Sqr(BaT1 / (Log(KB) - BaT6 - (Bap1 * Log(p))))
Next k
' Print values
Cells(i + 17, 1).Value = i
Cells(i + 17, 2).Value = xA
144

Cells(i + 17, 3).Value = yA
Cells(i + 17, 4).Value = xB
Cells(i + 17, 5).Value = yB
Cells(i + 17, 6).Value = xC
Cells(i + 17, 7).Value = yC
Cells(i + 17, 8).Value = T
Cells(i + 17, 9).Value = KB
' Test for feed stage too low
If xA < 0 Or xB < 0 Or xC < 0 Then
Cells(i + 18, 3) = "Feed stage too low"
Exit For
End If
i = i + 1
' Test for too many stages, which may mean reflux rate is too low.
If i > 100 Then
Cells(i + 18, 2).Value = "Too many stages"
Exit For
End If
' Top operating line
xA = yA / LoverV - ((1 / LoverV) - 1) * xAdist
xB = yB / LoverV - ((1 / LoverV) - 1) * xBdist
xC = yC / LoverV - ((1 / LoverV) - 1) * xCdist
' Test for calculations being done.
Loop While yA < xAdist
' Fractional recovery of C based on stage-by-stage calculation.
fracCbotcalc = 1 - (yC * D) / (F * zC)
difference = fracCbot - fracCbotcalc
If Abs(difference) < epsilon Then Exit For
fracCbot = fracCbot + df * (fracCbotcalc - fracCbot)
' Test if have convergence of fractional recovery of C.
Next j
Cells(i + 19, 1).Value = "Calc frac recovery C in bottoms"
Cells(i + 19, 5).Value = fracCbot
Cells(i + 19, 6).Value = "j"
Cells(i + 19, 7).Value = j
Cells(i + 18, 1).Value = "Distillate mole fracs = y values"
Cells(i + 18, 5).Value = yA
Cells(i + 18, 6).Value = yB
Cells(i + 18, 7).Value = yC
End Sub

145
Chapter 6
New Problems and new solutions are listed as new immediately after the solution number. These new
problems are: 6A1, 6A5, 6D3, 6D4, 6G4-6G7.
6.A1. New Problem in 3
rd
Edition. Trial and error. Try a feed stage and determine the distillate and
bottoms mole fractions of the key components. Repeat for additional feed stages. The feed stage
that produces the best separation is the optimum feed stage for this value of N.
6.A5. New Problem in 3
rd
Edition. Trial and error. Pick an N that you think is close (a systematic
method to do this is described in Chapter 7). Find the optimum feed stage. If you need more
separation to meet specifications, increase N, and if you exceed specifications, try decreasing N.
For an initial estimate of the optimum feed location for the new N, assume that the ratio
(Optimum feed stage)/(total number of stages) is constant. Continue process until specifications
are met or slightly exceeded.
6.C1. a. With a side stream, mass balance is,
j j j j S j 1 j 1 j 1 j 1 j j
Vy L x Sx V , y L x Fz
where
j
L is the flow into the stage below,
j j S j
L L S, x x . Since
j j S j j
L x Sx L x , Eqs.
(6-4) to (6-6) are unchanged. Note that the
j 1
L input into the matrix will be changed.
b. Now
j j j j j j
L h L h S h in the E.B. and
j j 1 k k
j j
L V D F S
k 1 k 1
,
j 1 j k k
j 1 j 1
L V D F S
k 1 k 1
Substituting into EB we find
j j
E j F j j 1 j K j K j 1
K j 1 K j
j j 1
D F h Q D h h F h F h
k 1 k 1
j 1 j
S h S h
k 1 k 1
6.C2. Partial condenser mass balances is:
1 1 1 2 2 1 1
Dy L x V y Fz (6-7)
This becomes,
1 2
1 1 2 2 1 1
1 2
DK V
K Fz
L L
Thus,
1 2 2
1 1 1 1 1
1 2
DK K V
B 1 , C , and D Fz
L L
(6-8)
Note that only
1
B differs.
6.D1. For n-pentane from Example 6-1, T = 60C,
C5 1 2
K 1.05, L L 825 kmole/hr,
3 4 2 3 4
L 1825, L 450 B, V V V 1375.
Matrix: j = 1 (total condenser),
146


2 2
1
2
1.05 1375
K V
C 1.75
L 825
,
1 1
1
D 550
B 1 1 1.67, D 0
L 825


3 3
2
3
1.05 1375 K V
j 2, C 0.791
L 1825


2 2
2 2 2
2
V K
B 1 2.75, A 1, D 0
L


4 4
3
4
1.05 1375
K V
j 3, C 3.208
L 450


1
3 3
3 3 3 C5
3
V K
B 1 1.791, A 1, D Fz 1000 0.35 350
L


4 4
4 4 4
4
1 V K
j 4 Reboiler , B 4.208, A 1, D 0
L

1,C5
2,C5
3,C5
4,C5
1.67 1.75 0 0 0
1 2.75 0.791 0 0
0 1 1.791 3.208 350
0 0 1 4.208 0


Values for
1,C5
to
4,C5
are in Example 6-1.

6.D2. p = 5 atm:
C2 C3 C4 C5
z 0.08, z 0.33, z 0.49, z 0.10
As satd liquid & for bp calculate
i i
z x . Want
i
y 1.0.
Pick
3
C as ref. 5 atm 101.3 kPa atm 506.5 kPa
1
st
Guess: Want
C3 C4
K 1, K 1. DePriester Chart.
Try T = 20C.
C2 C3 C4 C5
K 5.4, K 1.7, K 0.47, K 0.14

i
y 5.4 0.08 1.7 0.33 0.47 0.49 0.14 0.10 1.237
Need lower T.
C3
C3 new
i i
K 20
1.7
K T 1.37
K x 1.237


new C2 C4 C5
T 12 , K 4.6, K 0.35, K 0.10

i
y 0.368 0.4521 0.1715 0.0 1.0016 OK
Propane Matrix Analysis:
C3 6
K 1.37, B F D 1000 410 590 L

3 c3 1 2 4 5 6
D Fz 330, D D D D D 0

2 3 4 5 6 1
V V V V V L D 1435

1
1 2 3
L
L D 1025 L L
D


3 2 4 5
L L F 2025 L L
Total Condenser (1):
147


1 1 2 2 2 1
B 1 D L 1.40, C K V L 1.37 1435 1025 1.918
Stage 2.
3 3 2 2
2 2 2
2 3
1435 1.37 V K V K
A 1, B 1 =2.918, C 0.9708
L L 2025

Stage 3.
3 3 4 4
3 3 3
3 4
1435 1.37 V K V K
A 1, B 1 =1.9708, C 0.9708
L L 2025

Stage 4:
5 5 4 4
4 4 4
4 5
V K V K
A 1, B 1 =1.9708, C 0.9708
L L

Stage 5:
5 5 6 6
5 5 5
5 6
1435 1.37 V K V K
A 1, B 1 =1.9708, C 3.32
L L 590

Reboiler (Stage 6).
6 6
6 6
6
V K
A 1, B 1 =4.32
L

Mass balance matrix.

1.40 -1.918 0 0 0 0
-1 2.918 -0.9708 0 0 0

0 1 1.9708 0.9708 0 0
0 0 1 1.9708 0.9708 0
0 0 0 -1 1.9708 -3.32

0 0 0 0 -1 4.32



6.D3. New Problem in 3
rd
Edition. p = 5 atm = 506.5 kPa
C2 C3 C4 C5
z 0.08, z 0.33, z 0.49, z 0.10
As satd liquid & for bp calculation at
i i
z x . Calculation is same as in 5.D11 to obtain T.
Result is:
bp C2 C3 C4 C5
T 12 , K 4.6, K =1.37, K 0.35, K 0.10
n-butane Matrix Analysis:
C4 6
K 0.35, B F D 1000 410 590 L

3 c3 1 2 4 5 6
D Fz 490, D D D D D 0

2 3 4 5 6 1
V V V V V L D 1435

1
1 2
L
L D 1025 L
D
,
3 2 4 5
L L F 2025 L L
Total Condenser (1):

1 1 2 2 2 1
B 1 D L 1.40, C K V L 0.35 1435 1025 0.49
Stage 2.
3 3 2 2
2 2 2
2 3
1435 0.35 V K V K
A 1, B 1 =1.49, C 0.2480
L L 2025

Stage3.
3 3 4 4
3 3 3
3 4
1435 0.35 V K V K
A 1, B 1 =1.2480, C 0.2480
L L 2025

Stage 4.
5 5 4 4
4 4 4
4 5
V K V K
A 1, B 1 =1.2480, C 0.2480
L L

148

Stage5:
5 5 6 6
5 5 5
5 6
1435 0.35 V K V K
A 1, B 1 =1.2480, C 0.8513
L L 590

Reboiler (Stage 6).
6 6
6 6
6
V K
A 1, B 1 =1.8513
L

Mass balance matrix.

1.40 -0.49 0 0 0 0
-1 1.49 -0.248 0 0 0

0 1 1.248 0.248 0 0
0 0 1 1.248 0.248 0
0 0 0 -1 1.248 -0.851

0 0 0 0 -1 1.851


6.D4. New Problem in 3
rd
Edition. L L D D 5 60 300
V L D 360
Saturated liquid feed: V V 360; L L F 400





1 2 3 4
V 0, V V, V V, V V



Bubble Pt. Set
i i i i i
z x y K x



i
y 1.0 or
i i
K x 1.0

M E NP NB
3.58, 2.17, 1, 0.412





b. Eq. (5-30),
i i
i
i i
x
y
x


i i
z .3 3.58 .25 2.17 .35 1.0 0.1 .412 2.0077
Then
NP NP
nP
i i
0.35 1.0 z
y 0.1743
z 2.0077

and
nP
nP
0.1743
K y x 0.4981
0.35

1 2 3 4
L L, L L, L L, L B F D 40
1
2
L
3
L
4
V
3
V
1
V
F
4
2
3
149


M M NP NP E n-B nB NP NP
K K 1.7832; K 1.0809, K K 0.2052


c. Matrix for n-butanol
nB nB
K 0.2052, z 0.1
Stage 1.
1 1 1
A , B 1 D L 1 0.2 1.2,


2 2
1 1
2
360 0.2052
V K
C 0.2463, D 0
L 300


Stage 2.
2 2
2 2
2
V K
A 1, B 1 1.2463
L



3 3
2 2
3
360 .2052 V K
C 0.1847, D 0
L 400


Stage 3.
3 3
3 3
3
V K
A 1, B 1 1.1847
L



4 4
3
4
360 0.2052
V K
C 1.8468
L 40



3 3 nBut
D F z 100 .1 10

Stage 4.
4 4
4 4 4 4
V K
A 1 B 1 2.8468, C , D 0
B


1
2
3
4
1.2 0.2463 0 0 0
1 1.2463 0.1847 0 0
0 1 1.1847 1.8468 10
0 0 1 2.8468 0

d. y 1
1 1 1 1
.2463
V1 B 1.2, V2 0, V3 .20525
1.2

j 2
2 2 2
1
V1 B A V3 1.2463 1 .20525 1.04105

2 2 2 2
1
V2 D A V2 V1 0 1 0 0

2 2 1
0.1847
V3 C V1 0.1539
1.2

j=3
3 3 3
2
V1 B A V3 1.1847 1 0.1539 1.0308

3 3 3 2 3
V2 D A V2 V1 10 1 0 1.0308 9.7014

3 3 2
V3 C V1 1.8468 1.04105 1.7740

150

j=4
4 4 4
3
V1 B A V3 2.8468 1 1.7740 1.0728

4 4 4 3 4
V2 D A V2 V1 0 1 9.7014 1.0728 9.0428

4 4 3
V3 C V1 not needed.

4,NB N 4
V2 9.0428 (bottoms flow rate)

3NB 3 3 4
V2 V3 9.7014 1.7740 9.0428 25.743

2NB 2 3 3
V2 V3 0 0.1539 25.743 3.9619

1NB 2 1 2
V2 V3 0 .20525 3.9619 0.8132
e. Raoults Law

nP
VP K 760 0.4981 760 378.5 mmHg
Need to interpolate VP data. We know n VP 1 T
n 200 5.2983, n 378.5 5.9362, n 400 5.99146
1 66.8 273.16 2.9415E 3 1 82.0+273.16 2.81563E 3
Linear Interpolation:


bP
T 273.16 351.74 or
bP
T 78.6 C.


6.F1. Plots of vapor pressure are available in Maxwell (see Table 2-2 for reference) while tabulated
values are in Perrys
i tot
K VP p . Dew point calculation on feed gives 245.7F.
Overall Mass Balances: D = 30, L = 5D = 150
V = L + D = 180, V V F 180 100 80 , L L 150, B = F D = 70
First Trial Values
Stage T L V K
B
K
T
K
x

4 245.7 70 = B 80 2.307 1.042 0.534
3 245.7 150 180 2.307 1.042 0.534
2 245.7 150 180 2.307 1.042 0.534
1 245.7 150 30 = D 2.307 1.042 0.534

Benzene
Stage C B A D


4 - 3.705 -1 0 7.9886
3 -2.705 3.8404 -1 35 29.5978
2 -2.8404 3.8404 -1 0 57.058
1 -2.8404 1.2 - 0 135.6569

Toluene
Stage C B A D


4 - 2.191 -1 0 27.1651
3 -1.191 2.2504 -1 40 59.5188
2 -1.2504 2.2504 -1 0 61.5875
1 -1.2504 1.2 - 0 64.1742
in mmHg
nP
nP
tot
VP
VP
K
P 760
2.9415E 3 2.8163E 3
5.9362 5.2983 2.9415E 3 0.00284
5.2983 5.99146
151

Xylenes
Stage C B A D


4 - 1.6103 -1 0 22.7361
3 -.6103 1.6408 -1 25 36.6120
2 -.6408 1.6408 - 0 21.1971
1 -.6408 1.2 - 0 11.3193

6.G1. Using Peng-Robinson. Aspen-Plus solution:


Stage
T
1
C
L kmol/h V
C
4
C
5
C
8
1 38.31 825 0 x
1
0.360 0.6013 0.0386
y
1
0.6568 0.3424 0.00083
2 69.16 557.3 1375 x
2
0.0993 0.4499 0.4508
y
2
0.3601 0.6013 0.0386
3 107.02 1533.9 1107.3 x
3
0.03355 0.1731 0.7934
y
3
0.2288 0.5251 0.2461
4 140.92 450 1083.9 x
4
0.00436 0.0429 0.9528
y
4
0.04568 0.2271 0.7272


6.G2. 1. What VLE package did you use? Peng- Robinson.

2. Report the following values:
Temperature of condenser = - 2.77
o
C
Temperature of reboiler = 79.97
o
C
Distillate product mole fractions
2 3 4
C 0.3636, C 0.6360, C 0.0004
Bottoms product mole fractions
2 3 4
C 1.2 E 13, C 0.000492, C 0.9995
3. Was the specified feed stage the optimum feed stage? Yes No
If no, the feed stage should be: a. closer to the condenser, b. closer to the reboiler.

4. Which tray gives the largest column diameter (in meters) with sieve trays when one uses the
originally specified feed stage? Tray # 28 Diameter = 0.792 m.

5. Which components in the original problem are the key components? LK = Propane, HK =
butane

6. Change one specification in the operating conditions (keep original number of stages, feed
location, feed flow, feed composition, feed pressure, feed temperature/fraction vaporized
constant) to make ethane the light key and propane the heavy key.
What operating parameter did you change, and what is its new value? D = 20
Temperature of condenser = - 31.54
o
C
Temperature of reboiler = 50.87
o
C
Distillate product mole fractions
2 3 4
C 0.9955, C 0.00448, C 1.32 E 07
Bottoms product mole fractions
2 3 4
C 0.00112, C 0.4364, C 0.5625


152

6.G3. For column 1 report the following:
a. Final value of L/D 1.8
b. Split fractions of ethanol (distillate) 0.9999 and n-propanol (bottoms) 0.9913
c. Mole fractions in bottoms 1.70 E-5, 0.00871, 0.9913
d. Mole fractions in distillate 0.4545, 0.5383, 0.00714
For column 2:
a. Optimum feed location in the column. 18
b. Mole fractions in bottoms 0.00689, 0.9800, 0.0131
c. Mole fractions in distillate 0.9917, 0.0083, 0.0

6.G4. New Problem in 3
rd
Edition.
1. Temperature of condenser = 389.9_ K. Temperature of reboiler = __547.4 K
Q
condenser
= _-772260____cal/sec, Q
reboiler
= _____912459__cal/sec
Distillate product mole fractions: B= 0.23529, T= 0.76471, BiP = 0.12E-08_________
Bottoms product mole fractions:_B = 0.5 E-10, T = 0.67 E-08, BiP= 1.0000_________

2. Was the specified feed stage the optimum feed stage? Yes No x
If no, the feed stage should be: a. closer to the condenser, b. closer to the reboiler. a__
(Note: Do minimum number of simulations to answer these questions. Do not optimize.)

3. Which tray gives the largest column diameter with sieve trays when one uses the originally
specified feed stage? Aspen Tray #__16______Column Diameter =______2.28____meters
[Use the default values for number of passes (1), tray spacing (0.6096 m), minimum downcomer
area (0.10), foaming factor (1), and over-design factor (1). Set the fractional approach to flooding
at 0.65. Use the Fair design method for flooding.]

4. Which components in the original problem are the key components (label light and heavy
keys) _____LK = toluene, HK = biphenyl_____________________________________________

5. Change one specification in the operating conditions (keep N, feed location, feed flow, feed
composition, feed pressure, feed temperature or fraction vaporized constant at original
conditions) to make benzene the light key and toluene the heavy key. Also increase the reflux
ratio to 4.0.
What operating parameter did you change (not including the reflux ratio), and what is its new
value? D = 40________
Temperature of condenser = _368.9____ K, Temperature of reboiler = 407.7____ K
Distillate product mole fractions: _B = 0.9283, T = 0.07173, BiP = 0.8 E-19________
Bottoms product mole fractions: _B = 0.01793, T = 0.79457, BiP = 0.1875_________

6.G.5. New Problem in 3
rd
Edition.
1. Temperature of condenser = _121.07___ K. Temperature of reboiler = _166.23___ K
Q
condenser
= ____-757506.6____cal/sec, Q
reboiler
= ______1058466.75____cal/sec
Distillate product mole fractions:__B = 0.9779, T = 0.22070, pxy = 0.6004 E-05__
Bottoms product mole fractions:___B = 0.0055189, T = 0.55698, pxy = 0.43750___

2. Was the specified feed stage the optimum feed stage? Yes No x
If no, the feed stage should be: a. closer to the condenser, b. closer to the reboiler. a
(Note: Do minimum number of simulations to answer these questions. Do not optimize.)

3. Which tray gives the largest column diameter with sieve trays when one uses the originally
specified feed stage? Aspen Tray #____24______Column Diameter =______2.28___meters
153

[Use the default values for number of passes (1), tray spacing (0.6096 m), minimum downcomer
area (0.10), foaming factor (1), and over-design factor (1). Set the fractional approach to flooding
at 0.7. Use the Fair design method for flooding.]

4. Which components in the original problem are the key components (label light and heavy
keys) ________benzene = LK, toluene = HK______________________

5. Change one specification in the operating conditions (keep N, feed location, feed flow, feed
composition, feed pressure, feed temperature or fraction vaporized constant) to make toluene the
light key and p-xylene the heavy key.
What operating parameter did you change, and what is its new value?__D=260______
Temperature of condenser = _142.2____ K, Temperature of reboiler = _183.98__ K
Distillate product mole fractions: _B = 0.30769, T = 0.68850, Pxy = 0.003805________
Bottoms product mole fractions: __B= 0.3177 E-06, T = 0.007066, Pxy = 0.99293_____

6.G.6. New Problem in 3
rd
Edition. Part a. L D 27 . b. L D 60 . c. D = 147, S = 453 (liquid)
distillate mole fracs: E = 0.99007, B = 0.00993, P = 0.5 E-9
side stream mole fracs: E = 0.0009845, B = 0.98930, P = 0.000854
bottoms mole fracs: E = 0.7 E-14, B = 0.00043, P = 0.99957
d. distillate : E = 0.89146, B = 0.10854, P = 0.556 E-8
side: E = 0.041845, B = 0.95794, P = 0.000218
bottoms: E = 0.1 E-14, B = 0.0001095, P = 0.99989
Since vapor mole fraction ethane > liquid mole fraction (ethane is LK), have more ethane in
vapor side stream.
e. The separation of n-pentane and n-butane is much more difficult than between ethane and n-
butane. Thus side stream purity is less. Also feed has lot more pentane than ethane, which makes
side stream below feed less pure.

6.G.7. New Problem in 3
rd
edition.
1. Report the following values:
Temperature of condenser = _373.28____ K. Temperature of reboiler = ___411.75___ K
Q
condenser
= _-829828_____cal/sec, Q
reboiler
= ____1012650_____cal/sec
Distillate product mole fractions: M = 0.59998, E = 0.36184, NP = 0.038177, NB = 0.3087E -05
Bottoms product mole fractions: M= 0.2042E-04, E = 0.03816, NP = 0.46182, NB = 0.50000_

2. Was the specified feed stage the optimum feed stage? Yes No X
If no, the feed stage should be: a. closer to the condenser, b. closer to the reboiler. Answer a
(Note: Do minimum number of simulations to answer these questions. Do not optimize.)

3. Which tray gives the largest column diameter with sieve trays when one uses the originally specified
feed stage? Aspen Tray #_____18_____Column Diameter =___1.77____meters
[Use the default values for number of passes (1), tray spacing (0.6096 m), minimum downcomer area
(0.10), foaming factor (1), and over-design factor (1). Set the fractional approach to flooding at 0.7. Use
the Fair design method for flooding.]

4. Which components in the original problem are the key components (label light and heavy keys)
______________LK = ethanol, HK = n-propanol_______________________

154

5. Change one specification in the operating conditions (keep N, feed location, feed flow, feed
composition, feed pressure, feed temperature or fraction vaporized constant) to make methanol the light
key and ethanol the heavy key.
What operating parameter did you change, and what is its new value?_____D = 60____
Temperature of condenser = __368.66__ K, Temperature of reboiler = _404.23___ K
Distillate product mole fractions: M = 0.97858, E = 0.021417, NP = 0.155 E-07, NB = 0.1 E-10_
Bottoms product mole fractions: M = 0.0091787, E = 0.27654, NP = 0.35714, NB = 0.35714__



155
Chapter 7
New Problems and new solutions are listed as new immediately after the solution number. These new
problems are: 7.A1, 7.A4, 7.D2, 7.D10, 7.D11, 7.D14, 7.D21, 7.G1.
7.A1. New problem in 3
rd
edition. f. none of the above.
7.A.4. New problem in 3
rd
edition. a. estimate fractional recoveries nonkeys at total reflux.
7.C4. Use
i, j 1 i i, j 1
y K x . Then substituting into Eq. (7-20), we have
min i i, j 1 min ij i,dist
V K x L x Dx
which is
min i i, j 1 min ij 1 i,dist
V K x L x Dx where
i i HK
K K . Rearranging,
min i HK
min i , j 1 i ,dist
min
V K K 1
L x Kx
L
, or
i,dist i
min i, j 1
min HK
min i
Dx
L x
V K 1
L
Total flow rate
min
L is
i,dist i
min min ij 1
min HK
min i
c c
Dx
L L x
V K 1
i 1 i 1
L
(A)
By a similar analysis obtain,
i , bot i
min
min HK
min i
Bx
L
V K 1
L
(B)
Let
min 1 min
L L
min min
V K V K
L L
(C)
Add Eqs. (A) and (B), and use external mass balance,
c
i i
feed min min
i 1
i L
Fz
qF L L L
1/
(7-33 analogue)
From Eqs. (A) and (C) we have
c
i,dist i
min
i 1
i L
Dx
L
1/

(7-29 analogue)
7.C5. For saturated vapor feed have
F
V F . For binary system Eq. (7-33) is,
1 1 2 2
1 2
z z
1
Clearing fractions we obtain
1 2 2 1 1 2 2 1
z z
After some algebra this
2 1 1 2 2 1 2
z z 0
Solutions are,
1 2 1 1 2 2
0 or z z
For satd liqd
F
V 0 . Clear fractions and equation is linear.
156

7.D1. a. Eq. (7-16),
dist
bot
min
AB
x 1 x
.992 .008
n
n
x 1 x
.014 .986
N 10.36
n n 2.4

This includes the partial reboiler. Eq. (7-40a) gives,

dist
feed
f ,min
AB
x 1 x
.992 .008
n
n
z 1 z
.4 .6
N 5.97
n n 2.4

b. Saturated liquid:
f
V 0 . Eq. (7-33) becomes
B B T T
B T
z z
0
After clearing fractions and solving for ,

T B
B B T T
1.0 2.4 z
1.53846
z z 2.4 .4 1.0 .6

Which does lie between the s of the keys. To use Eq. (7-29) we need D. From mass
balances (Eq. (3-3)).

bot
dist bot
z x .4 .014
D F 10 3.9468
x x .992 .014
kg moles/hr
Eq (7-29) is:
B Bdist T Tdist
min
B T
Dx Dx
V .

min
2.4 3.9468 .992 1.0 3.9468 .008
V 10.848
2.4 1.53846 1 1.53846


min min
L V D 6.9013 .
min
L D 1.75

c.
min
L D 1.1 L D 1.9234 ,
min
L D L D
x 0.0598
L D 1

Using Eq. 7-42b,
min
N N
0.5563
N 1

Solving for N,
min
.5563 N
N 24.6
1 .5563
(includes reboiler)
From Eq. (7-40b),
F,min
F
min
N
5.97
N N 24.6 14.2
N 10.36

Try stage 14 from top for feed stage.

7.D.2. New problem in 3
rd
edition.
C2 C3 C4 C5
p 5 atm, z 0.08, z 0.33, z 0.49, z 0.10
Saturated liquid and for bp. Calc.,
i i
z x . Want
i
y 1.0 .
Pick
3
C as reference (this is arbitrary).
kPa
5 atm 101.3 506.5 kPa
atm

1
st
Guess: Want
C3
K 1 (light key),
C4
K 1 (heavy key). Use DePriester Chart.
157

Try T 20 C,
C2
K 5.4 ,
C3
K 1.7 ,
C4
K 0.47 ,
C5
K 0.14

i
y 5.4 0.8 1.7 0.33 0.47 0.49 0.14 0.10
0.432 0.561 0.230 0.014 1.237

Need lower T.
C3
C3 new
i i
K 20
1.7
K T 1.37
1.237
K x


new C2 C4 C5
T 12 , K 4.6, K 0.35, K 0.10

i
y 0.368 0.4521 0.175 0.0 1.0016
For remainder, let
4
C HK be reference.
3 4
C C
1.37 0.35 3.914
a)
MIN
0.997 0.998
n
0.003 0.002
12.01874
N 8.808
n 3.914 1.36456
(includes PR).
b)
C2 C4 C4 C4 C5 C4
4.6 0.35 13.143, 1.0, 0.1 0.35 0.286
Satd liquid feed
MIN MIN
V V . In Eq (7-33) divide through by F.

i i
i
13.14 0.08 3.914 0.33 1.0 0.49 0.10 0.286 z
0
13.14 3.914 1.0 0.286

Solve for . Find = 1.74 (Note
LK HK HK HK
3.914 )
Eq (7-29)
i i ,dist
MIN
i
Dx
V . Assume all
2
C in distillate & all
5
C in bottoms

i,C2 i,C5
Dx 80, Dx 0

i
C3
Dx 0.997 1000 0.33 329.01

i
C4
Dx 1 0.998 1000 0.49 0.98

i,d
D Dx 409.99

MIN
13.14 80 329.01 3.914 0.98 1.0
V 0 683.23kmole h
13.14 1.74 3.914 1.74 1.0 1.74


MIN MIN
MIN
L V D 273.24; L D 0.6664
c)
MIN
MIN
L 1.15 L D 0.7664
Ordinate Gilliland
0.7664 0.6664
0.05662
1.7664

Eq (7-42b)
MIN
N N 0.002743
0.545827 0.591422 0.05662 0.56079
N 1 0.05662


0.5608 8.808
N 21.33
1 0.5608
(include PR)
158


LK HK
dist
LK HK
F,MIN
LK HK
Dx Dx
329.01 0.98
n
n
z z
0.33 0.49
N 4.552
n n3.914

Eq. (7-40b)
F,min
F
F
min
N
N .552
N 21.33 11
N N 8.808
(approximate)
7.D3. At total reflux use Fenske Eq. (7-11).
A A
B B
d bot
min
AB
x x
log
x x
N
log

Rearrange, log
A A
AB
B B min
d bot
1 x x
log
x x N


min
1 N
1 3
A A
AB
B B
d bot
x x .545 .36
1.287
x x .455 .64

7.D4. a.)
dist
bot
min
AB
x 1 x
.9915 .0085
n
n
x 1 x
.01773 0.98227
N 10.02
n n 2.4

b.) Feed is saturated liquid, feed line is vertical.




c.) Abscissa =
min
L D L D
2.2286 1.144
0.336
L D 1 3.2286

From Eq. (7-42b)

min
N N 0.002743
0.545827 0.591422 0.336 0.3553 ordinate
N 1 0.336


min
N ordinate 10.022 0.3553
N 16.1
1 ordinate 1 0.3553

From fitted curve ordinate = 0.3474,
10.022 0.3474
N 16.2
1 .3474

Error = 25-16 25 36%low. Aspen Plus equilibrium data is not = 2.4. Note that =
2.24 may be a better fit.
(L/V)
min
=
*
0.9915 y
0.9915 0.6

*
x z 0.6
x
y 0.7826
1 1 x

(L/V)
min
= 0.534,
min
min
L L V
1.144
D 1 L V

y*
* z = .6
y
159

7.D5. Fenske Eq. is:
D B
N
D B
min
D B
min
D B
x x
log
1 x 1 x
x x
N or
1 x 1 x log

Solving for
B
x , we obtain:
D D
B N
min
D D
x 1 x
x
x 1 x

Since
min D B
N 30, 1.30 and x 0.984, this is x 0.229

7.D6. Fenske Eq.:
D B
D B
min
x x
.993 .01
log
log
1 x 1 x
.007 .99
N 10.82
log log 2.4

Determine y in equilibrium with feed z x 0.4 .

*
2.4 .4
x
y .616
1 1 x 1 1.4 .4


D
min D
x y* L .993 .616
0.636
V x z .993 .4
,
min
min
L L V
1.746
D 1 L V
.
Then,
act
L
1.15 1.746 2.01
D

Gilliland Correlation: Abscissa
min
L D L D
2.01 1.746 .264
.0878
L D 1 3.01 3.01

Ordinate
min
N N N 10.82
0.557
N 1 N 1

Clear fractions, and find N = 25.3 (including partial reboiler).

7.D7. p 5 atm. From the solution to problem 6.D9:
bp
T 12 C

C2 C3 C4 C5
K 4.6, K 1.37, K 0.35, K 0.10
Let
4
C HK be reference.
C3 C4
1.37
3.914
0.35

a) Eq. (7-15), Including PR
min
0.98 0.992
n
0.02 0.008
8.7121
N 6.38
n 3.914 1.36456

b)
4 C2-C4 C4 C4
C HK reference. 4.6 0.35 13.143, 1.0

C5 C4
0.10 0.35 0.286
Satd/liquid feed,
min min
V V
Eq. 7-33,
i i
i
13.14 0.08 3.914 0.33 1.0 .49 0.10 0.286
z
0
13.14 3.914 1.0 0.286

Want
LK HK HK HK
3.914 1.0 . Converge to = 1.74.
160


i i i
C2 C3
Dx Fz 80, Dx 0.98 1000 0.33 323.4

i i
C4 C5
Dx .008 1000 0.49 3.92, Dx 0
Eq. (7-29),
min
13.14 80 323.4 3.914 3.92 1.0
V 0 671.05
13.14 1.74 3.914 1.74 1.0 1.74


i,d min min
min
D Dx 407.32, L V D 263.73, L D 0.647
c)
min
L D 1.2 L D 0.777 . Ordinate Gilliland 0.777 0.647 1.777 0.073
Eq. (7-42b),
min
N N 0.002743
0.545827 0.591422 0.073 0.5402
N 1 0.073


0.5402 6.38
N 15.05,
1 0.5402
incl. PR. N
feed
~ 9
7.D8. a. Can do this graphically, or can calculate slope of a line from
D
y x x .992 to
intersection of feed line and equilibrium, or use Underwood. Easiest to calculate slope. Feed
line
F
y z .4 .
Equilibrium:
y .4
x .3755
y 1 y .4 1.11 .6


min
L .992 .4
.958
V .992 .3755
,
min
L L V
22.83
D 1 L V

b.
D B
D B
min
x x
.992 .005
n
1 x 1 x
.008 .995
N 96.9
n n 1.11

This is 95.9 stages plus partial reboiler.
c. L D 1.2 22.83 27.4 , Abscissa
min
L D L D
27.4 22.83
.161
L D 1 27.4 1

Ordinate
min
N N
.47 or N 181.9
N 1
which includes partial reboiler.
This separation would probably not be done by distillation.
7.D9. Feed 80% liquid,
F F
L .8F, V .2F. Feed line:
F
F F
L Fz
y x
V V
, Slope
8
4
2

a. See Graph. Min top op line is tangent.
min
L .8 0.386
slope 0.5175
V 0.8 0


min
min
min
L V
L 0.5175
1.0725
D 1 L V 1 .5175

b.
min
3
N 6
4
eq. contacts. See graph.
c.
actual
L
1.05 1.0725 1.1262
D
. Abscissa, Gilliland Correlation is
161


min
L D L D
1.1262 1.0725 0.053666
0.02524
L D 1 2.1262 2.1262

Ordinate ~ 0.63 from graph.
From eq. (7-42b), Ordinate
0.002743
0.545827 0.591422 0.02524 0.6396
0.02524

(agrees with graph).
Then
min
N ordinate 6.75 0.6396
N 20.5
1 ordinate 1 0.6396

Need 20 eqs. contacts + P.R.

F,min
N from graph = 6.
F,min
F
min
N
6
N N 20.5 18
N 6.75
(7-40b)

7.D10. a) New problem in 3
rd
edition. Eq. (7-15)

E,dist B,bot
E,dist B,bot
MIN
EB
FR FR
n
1 FR 1 FR
N
n

162


EB
13.14
MIN
0.989 0.998
n
0.011 0.002
10.7114
N 4.159
n 13.14 2.5756


b)
MIN PB
N 4.159 is known.. 3.91.
Eq. (7-17)
MIN
MIN
N
PB
P,dist
N
B,bot
PB
B,bot
FR
FR
1 FR
4.159
4.159
3.91
0.3677
0.998
3.91
0.002

c)
D i,dist i
i
Dx FR Fz
Ethane
DE
Dx 0.989 100 0.3 29.67
Propane
DP
Dx 0.3677 100 0.33 12.134
n-butane
DB
Dx 0.002 100 0.37 0.074

3
i,d
i 1
kmol
D Dx 41.878
h

Also accept D = 0 since total reflux.

7.D.11. New problem in 3
rd
edition.















Use Underwood Eqns. Case A Assume LNK (propane) is all in distillate.
p,dist p
Dx Fz 20
b)
feed
V F 1 q F since q 0
Eq. (7-33). F
i
1
F
i
i
z
where is between s of two keys (B and H)
1.0 > > 0.2. Equation is,

2.04 0.2 1.0 .35 0.20 0.45
1.0
2.04 1.0 0.20

Solving for obtain = 0.62185.
200
n
iP
NP
z 0.35
z 0.4
z 0.25

1
V

B

D

1

2

163

Then
i,dist
min
Dx
V . Find D from fractional recoveries.

B,dist
p,dist
H,dist
Dx 0.99 100 .35 34.65
Dx 20
Dx 1 0.98 100 .45 0.9
D 55.55



MIN
2.04 20 1.0 34.65 0.2 0.9
V 119.98
2.04 0.62188 1.0 0.62188 0.2 0.62188


min MIN
min
L V D 64.4314 and L D 1.1599

7.D12. A = benzene (LK),
AB A,dist
2.25, FR 0.98
B = toluene (HK),
BB B,bot
1.0, FR 0.99
C = cumene (HNK),
CB
0.210
a. Use Fenske eqn. at total reflux.
A,dist B,bot
A,dist B,bot
min
AB
FR 1 FR
n
1 FR FR
N
n


min
0.98 0.01
n
0.02 0.99
N 10.47
n 2.25


N
min
AC
AB
C,bot AC
N
min
A,dist
CB
AC
A,dist
2.25
FR where 10.71
FR
0.21
1 FR


10.47
C,bot
10.47
10.71
FR 1
0.98
10.71
0.02
. All cumene goes to bottoms.
(We can also substitute into Eq. (7-17)).

N 10.47
min
12 AC
C,dist
N 10.47
B,bot min
CB
B,bot
0.21
FR 8.1 10 0
FR .99
0.21
.01
1 FR

b. Underwood equations Case B analysis
Feed is sat. vapor.
feed
q 0, V F 1 q F 100 ,
C
i 1
feed
i 1
1
Fz
V

2.25 40 1.0 30 0.21 30
100
2.25 1.0 0.21

Find 1.6516.
164


C
i i ,dist
min i ,dist 1
i ,dist
i 1
1
Dx
V where Dx Fz FR

A,dist B,dist C,dist
Dx 100 0.4 0.98 39.2, Dx 100 0.3 0.01 0.4, Dx 0, D 39.6

min
2.25 39.2 1.0 39.2
V 0 146.78
2.25 1.6516 1 1.6516

From mass balance,
min min
L V D 146.78 39.6 107.2 ,
min
L 107.2
2.71
D 39.6

c.
min
L D 1.25 L D 1.25 2.71 3.39
abscissa for Gilliland correlation
min
L D L D
0.155
L D 1
. From Figure 7-3 the
ordinate = 0.46. With
min
N 10.47, we find N = 20.24.
To find
feed feed
min
N , we need N

A A
B B
dist feed
feed
min
AB
x z
n
x z
N
n


A,dist B
39.2
x 0.9899, x 1 .9899 0.0101
39.6


feed
min
0.9899 0.4
n
0.0101 0.3
N 5.30.
n 2.25


feed
feed min
feed
min
N
N
gives N 10.25.
N N
Use stage 10 or 11.

7.D13. Use Fenske eq.
T T
.99
FR 1 FR 99
.01
,
C C
.02
FR 1 FR 0.0204
.98


min
TC
log 99 0.0204 log 4851
N 5.438
log log 1 0.21

Underwood:
f
V V V 0 ,
2.5 .25 1.0 .30 0.21 45
0
2.5 1.0 21

1.526 or 0.3374. Use 0.3374 as it is between keys.
3
1 i,d
min
i 1
1
Dx
V

B,d B
Dx Fz 25 (assume all benzene in dist.)

T,d T c,d c
Dx .99 Fz 29.7, Dx 0.02 Fz 0.9, D 55.6

min
2.5 25 1.0 29.7 .21 9
V 72.24
2.5 0.3374 1.0 0.3374 0.21 0.3374


min
min
L V D 16.64 and L D 0.2993
165

Gilliland: Ordinate
min
N N 9 5.438
0.3562
N 1 10

Abscissa ~ .29 (original Gilliland) or .36 (Liddle)
If use 0.29 have,
min
L L L
0.29 0.29
D D D


L .29 0.2993
.83
D 1 .29

If use Abscissa = 0.36,
L .36 .2993
1.03
D 1 .36
which are quite different. Safer to use
higher value.
If
BT min
2.25, N is same. Underwood Eq. gives 1.4666 or 0.3367. Use 0.3367.

min
V 29.40 44.78 1.492 72.68,
min
min
L V D 17.084 and L D 0.3073
Which is 2.7% different than for
BT
2.5.

7.D14. New problem in 3
rd
edition. Use Gilliland correlation to find the value of the minimum reflux
ratio, (L/D)
min
= 1.4
7-D15. Fenske: Eq. (7-15),
B,dist C,bot
B,dist C,bot
min
BC
FR FR
.9992 .9999
n
n
1 FR 1 FR
.0008 .0001
N 6.89
2.25
n
n
.21

Eq. (7-17),
min
min
N
TC
T,dist TC
C,bot N
TC
C,bot
1
FR where
FR
.21
1 FR


6.89
tol,dist tol,bot 6.89
1
.21
FR 0.8238, and FR 0.1762
.9999 1
.0001 .21


dist
d, tol
d,benz
d,cum
x
Dx 0.8238 167 137.57
0.2568
Dx 0.9992 397 396.68 0.7418
.0436
Dx 0.0001 436 0.0008
D 534.294


LK LK
HK HK
dist
F,min
LK-HK
x z
.7418 .397
n
n
x z
.000815 .0436
N 1.94
n n 2.25 .21
.
Underwood: Use a Case C analysis since toluene is a sandwich key.
Eq. (7-33):
3
1 1
F
i 1
1
2.25 397 1.0 167 .21 436
Fz
0 V is, 0
2.25 1.00 .21

1.216 and 0.3373 which lie between s.
166

Eq. (7-29):

3
i i,dist tol
min min
i 1
i
Dx 2.25 396.88 1.0 Dx .21 0.436
V becomes V
2.25 1.0 .21

Write for 1.216 and for 0.3373. Obtain 2 eqns and 2 unknowns:
min tol,dist
V and Dx .
For 1.216,
min tol
V 863.525 4.629 Dx . For
min tol
.3373, V 466.15 1.509 Dx .
Solving simultaneously,
min tol
V 563.84, Dx 64.740

i,d
D Dx 396.88 64.74 .436 462.056

min min
min
L V D 101.79 and L D 0.2203
Gilliland Abscissa
min
L D L D 1.2 .2203
0.445
L D 1 2.2

Ord.
min
N N
.245
N 1
(Original Gilliland)
Obtain N = 9.45 (includes reboiler)
Estimate
F,min
F
min
N
2.91
N N 9.45 2.66
N 6.89
(Use stage 3)

7.D16. a. Fenske:
dist bot
min
x x
.99 .992
n
n
1 x 1 x
.01 .008
N 12.69
n n 2.1

b. Underwood:
p
EP E
feed
EP
1.0 F z
Fz
V
1.0
,
E P feed
z .6, z .4, V V V F

2.1 .6 1.0 4
1
2.1 1.0
, 0 or 1.44. Use 1.44 which is between 1.0 and 2.1
To find D:
B
D B
z x .6 .008
D F 1000 602.85
x x .99 .008
. Then

min
2.1 .99 1.0 .008
V
3.132
D 2.1 1.44 1.00 1.44


min min min
min
L
V 1888.12, L V D 1285.3, 2.13
D

c. Use Gilliland: Ordinate
min
N N .30 12.69
0.558
N 1 31

Abscissa ~
min
L D L D
0.8
L D 1
(Original Gilliland), L D 2.4

7.D17. Sounds harder than it is:
F i i
tol-xy xy xy
i
V z
0 f , 3.03, 1
-
F

167

Expand & Solve for ,
xy xy
tol tol
tol xy
z
z
0
Result is linear,
xy tol
tol
tol tol xy xy tol tol tol
z z z 1 z

xy
since 1
Then
i i,dist
min
i
Dx
V ,
B
D B
z x z .008
D F 100
x x 0.988


min min min
3.03 .996 1.0 .004
V D , L V D
3.03 1


tol
z

D
min
V
min
L
min
L
D

0.1 3.03
2.51870
3.03 .1 .9

9.3117
3.03 .996
.004
54.93685
3.03 2.51870 1 2.51870

45.625 4.8997
0.3 3.03
1.88316
3.03 .3 .7

29.5547
3.03 .996
.004
D 77.6383
3.03 1.88316 1 1.88316

48.0836 1.6269
0.5 3.03
1.503722
3.03 .5 .5

49.7976
3.03 .996
.004
D 98.0683
3.03 1.503722 1 1.50372

48.2707 0.96934
.7 3.03
1.25155
3.03 .7 .3

70.0405
3.03 .996
.004
D 117.739
3.03 1.25155 1 1.25155

47.6985 0.68101

.9 3.03
1.071806
3.03 .9 .1

90.2834
3.03 .996
.004
D 129.08
3.03 1.071808 1 1.071808

38.7990 0.429

Check for z = 0.5.




min
L L L V .4922
0.96934
D V L 1 L V 1 4922
Perfect
As z , although
min min c min
min
L D , V , thus Q V as expected.

R,min
Q also.
y*
y
.5 = z
x
D
min D
x y* L
V x .5
where
F
F
x
y* 0.75186
1 1 x


0.996 0.75186
0.4922
0.996 0.5

min
Slope L V
D
x
168

7.D18.a. Fenske: Eq. (7-15):
A,dist B,bot
A,dist B,bot
min
AB
FR FR
n
1 FR 1 FR
N
n

Where A = propane, B = butane,
AB
1/ .49 2.04 . (Note value .)

min
.9854 .8791
n
.0146 .1209
N 8.7
n 2.04

For
F,min
N assume no LNK is bot and no HNK in distillate
D = .229 + (.9854) (.368) + (.1209) (.322) = .631

prop C4
.9854 .368 .1209 .322
x 0.575, x 0.0617
.631 .631


C3 C3
C4 C4
dist F
F,min
C3-C4
x z
.575 .368
n
n
x z
.0617 .322
N 2.94
n 0.713

Underwood Eqns. (Case A.)
1 1
feed
1
Fz
0 V for 1.0 .49

9.92 .229 1.0 .368 .49 322 .081 0.10
0 f
9.92 1.0 .49 .10

Find 0.6213,
min
9.92 .229 1.0 .363 .49 .0389
V 1.057
9.92 .6213 1.0 .6213 .49 .6213


min min
min
L V D 1.057 .631 .426, L D 0.676
Gilliland Correlation (Fig.7-3): abscissa
min
L / D (L / D)
0.33
L / D 1

Ordinate
min
N N
.32
N 1
(Original Gilliland, ~.36 Liddle). Find N = 13.24 (14.13 Liddle)
b. With N = 20, ordinate to Gilliland correlation is,

min
N N 20 8.7
0.538
N 1 21

Abscissa = 0.1. Since
min
L D 0.676, solve for L/D = 0.862.
c.
min
min
N
C6 C3
C6
dist
C3 N
bot
C6C3
C3
bot
FR
FR
1 FR

where
bot dist
C6 C3 C3 C3 min
0.10, FR 1 FR 0.0156, and N 8.7.

8.7
C6
dist
8.7
0.10
FR 0.00000013
0.0156
0.10
1 0.0156
,
C6
bot
FR 0.99999987
169

For all practical purposes all C6 in bottoms at total reflux.

d.
dist
C3 B,bot
FR 0.999, L/ D 1.5, FR 0.8791
1)
dist bot
dist bot
C3 C4
C3 C4
min
C3C4
FR FR
.999 .8791
n
n
1 FR 1 FR
.001 .1209
N 12.47
n 0.713


2) For D assume all LNK in dist, No HNK in dist

D = 0.299 + (0.999) (0.368) + (0.1209) (0.322) = 0.6356

Now
9.92 .229 1.0 .368 .49 .332 .081 .1
f 0
9.92 1 .49 .1


Which is same [ depends only on feed & s]. Thus, same = 0.6213


min
9.92 0.229 1.0 0.368 0.49 4689??
V 1.0709
9.92 .6213 1 .6213 .49 .6213



min min
L 1.0709 0.6356 0.4345 L D 0.6848. Very little change.

7.D19. Use Figure 7-3. Ordinate
min
N N 25 11
0.5385
N 1 26

Then Abscissa
min
L D L D
0.08 with L D 2.2286
L D 1
.
min
L D 1.97
Abscissa approximated between original & fitted curves.

7.D20. a) Distillate
dist dist
B B T T
Dx Fz 5, Dx Fz 15,

dist
x 1.0 becomes
5 15
0.57895 0.07018 1.0
D D


Find D = 57.001 kmoles/hr, B = 100 D = 42.999
b) Can use Fenske eq. (7-11) or alternatives.
A B
dist
A B
bot
min
AB
x x
n
x x
A xylene
N
B cumene n


xy xy tol xy
AB xylene cumene
cum cum tol cum
K K K
0.330
1.57143
K K K 0.210

Xylene balance,
x,bot
Fz 35 57.00 0.57895 42.999 x

x,bot cum,bot
x 0.0465, x 1 .0465 0.9535
170


min
0.57895 0.07018
n
0.04650 0.9534
N 11.35
n 1.57143

This is # equil contacts at total reflux.
c) Alternative:
x,dist
xy,dist
x
Dx 57.001 0.57895
FR 0.94288
Fz 35


C,dist
C,bot
C
Dx 57.001 0.07018
1 FR 0.088896
Fz 45
,
cum,bot
FR 0.91110
Use Eq. 7-15.
dist
dist
A B,bot
A B,bot
min
AB
FR FR
n
1 FR 1 FR
A xylene
N ,
B cumene n


min
0.94288 0.91110
n
0.057122 0.088896
N 11.35
n 1.57143


7.D21. New problem in 3
rd
edition. Assume all ethanol in distillate and all n-butanol in bottoms.




E,dist E
i P,dist ip
n P,dist nP
n B,dist
Dx Fz 100 .3 30
Dx Fz Frac Rec iP dist 100 .25 .986 24.65
Dx Fz 1 Frac Rec nP dist 100 .35 .008 0.28
Dx 0 0






x
E,dist
= x
E,dist
/D = 0.5461, x
i-P,dist
= x
i-P,dist
/D = 0.4488, x
n-P,dist
= x
n-P,dist
/D = 0.0051


E,bot
i P,bot iP
n P,bot nP
Bx 0 0
Bx Fz 1-Frac Rec iP dist 100 .25 .014 0.35
Bx Fz Frac Rec nP bot 100 .35 0.992
n B,bot n B
34.72
Bx Fz 100 0.10 10.0




i,bot i,bot
x Bx B

x
i-P,bot
= Bx
i-P,bot
/B = 0.0108, x
n-P,bot
= Bx
n-P,bot
/B = 0.7704, x
n-B,bot
= Bx
n-B,bot
/B = 0.2188


B = 45.07

i,dist
D Dx 54.93
171

b. Fenske eq. (7-15)
ip,dist nP,bot
iP,dist nP,bot
Min
iP nP
FR FR
n
1 FR 1 FR
N
n


min
.986 .992
n
.014 0.008
n 8733 9.0748
N 14.62
n 1.86 0.62058 0.62058

This includes PR

F,MIN
N Eq. 7 40a ,
iP iP
nP nP
dist
F, MIN
iP-NP
x z
0.4488 0.25
n
n
x z
0.0051 0.35
N 7.76
n 0.62058


c. Underwood Equation: Assume NKs do NOT distribute: Case A.
Eq. (7-33)
i i
feed
i
Fz
V
For saturated vapor
feed
V F divide (7-33) by F.
i i
i
z
1 , which becomes

nP NP NP NB NP NB E nP E iP nP iP
ENP iP NP NP NP NB NP
z z z z
1

3.58 0.3 1.86 0.25 1 0.35 0.412 0.10
1
3.58 1.86 1 0.412

Solve for between values of keys. LK = i-propanol, HK=n-propanol. Thus, want
1.0 1.86 . From Goal Seek on spreadsheet 1.48648
Then from Eq. (7-29)
i i ,dist
min
i
Dx
V where
i,dist
Dx values from part a.

MIN
3.58 30 1.86 24.65 1.0 0.28
V 0 173.47
3.58 1.48648 1.86 1.48648 1.0 1.48648


MIN MIN
L V D 173.47 54.93 118.54 ,
MIN
L D 118.54 54.93 2.16

d.
Min
L
L D 1.1
D

Gilliland abscissa,
MIN
MIN
MIN
L D
1
1
1
L D L D
L D
1.1
x
1 1
L D 1
1 1
L D 1.1 L D

or
MIN
MIN
MIN
L D
1
L D
1.1 1
x 0.0683
L D 1 1
1.1
L D L D 2.16


172

From Eq. 7-42b,
MIN
N N
0.5456
N 1


MIN
N 1 0.5456 0.5456 N 33.4 Nincludes PR

Assuming
F,MIN
F
F
MIN
N
N 7.76
N 33.4 17.7
N N 14.62
or Stage 18 below total condenser.

7.F2 Equilibrium data is available in a variety of sources such as Perrys Handbook. Data used
here is from Perrys (3
rd
ed.), p. 574.
a) Need to obtain avg. from equilibrium data.

N2 N2
N2
N2
y 1 x 0.1397 0.9615
x 0.0385, y 0.1397, 4.055
1 y x 0.8603 0.0385


N2
x 0.4783, y 0.7893 (needed for part b)

N2 N2
.9770 .0810
x .9190, y .9770, = 3.744
.0230 .9191


top&bot
1/ 2
avg
4.01 3.744 3.875
Fenske
dist
bot
min
AB
x 1 x
.998 .002
n
n
x 1 x
.001 .999
N 9.685
n n 3.875

b)
D
min
D
x z
L V
x x *
where x* is in equilibrium with feed y z 0.79
From equilibrium data x* ~ 0.48.

min
.998 .79
L V 0.40
.998 .48
,
min
min
min
L V
L 0.40
0.66667
D 1 L V 1 0.40

c)
min
L / D 1.1 L / D 0.7333
Gilliland Correlation: abscissa
min
L D L D
0.06666
0.0385
1 L D 1.7333


Original correlation, ordinate
min
N N
0.6
N 1
N = 25.7 including PR. Need 25 equil. Stages
7.G.1. New problem in 3
rd
edition. a. At total reflux
MIN
N 9
b.
MIN
L D 0.92
173
Chapter 8
New Problems and new solutions are listed as new immediately after the solution number. These new
problems are: 8.A1, 8.A2, 8A7, 8A12, 8.D1, 8D6, 8D12, 8.D13, 8D15, 8D17, 8D20, 8.D22, 8D23 to
8.D25, 8.E1, 8G1-8G5, 8.H3.
8.A1. New Problem in 3
rd
edition. a. 2-pressure distillation
8.A2. New Problem in 3
rd
edition. b. extractive distillation
8.A7. New Problem in 3
rd
edition. If there are volatile and non-volatile organics, a single equilibrium
contact gives an organic layer that contains no non-volatiles. Extra stages do not increase the
separation. If there is entrainment, a second stage may be useful.
8A.12. New Problem in 3
rd
edition. Steam distillation is normally operated with 2 liquid phases in the
still pot and in the settler after the condenser. There is usually no reflux. Azeotropic
distillation is normally operated with one liquid phase in the column and in the reboiler, but
with 2 liquid phases in the condenser and settler. One of the liquid phases is refluxed to the
azeotropic column.
8.C2.
org org org org
org w in water in w in w
w w w w
y x p x
y x p x
org org org org org org in org
w w org in w
w w w
VP
H x x H x
VP x x VP VP x
At solubility pt.
w in w org in w org in org
x .975 and x .025, x .573
Vapor pressures (Perry & Green, 1984). N-butanol:
T = 70.1C 84.3C l00.8C 117.5C
VP = 100 mm Hg 200 400 760 mm Hg
Water: T = 100.8C, VP = 782 mm Hg, T = 84.3, VP ~ 421.8 mm Hg
at 84.3C:
org
w
VP
200
0.474
VP 421.8
, at 100.8:
org
w
VP
400
0.5115
VP 782
For
org w
use average between 92 and 100C. Can linearly interpolate at T = 96C,
org w
VP / VP 0.501,
org w in w
0.501 .573 .025 11.483
w org in w org w in w
1 1 11.483 0.0871
From
w o w w
w
w o w w
x 0.0871x
y
1 1 x 1 0.9129x
generate equilibrium curve,
w
w
x 1.0 .995 .990 .985 .980 .975
y 1.0 .9495 0.8961 0.8512 0.8102 0.7726
At constant
w
x , the calculated
w w,experi min tal
y y . Difference at
B
x 0.975 is
approximately .7726 .752 .752 100 2.74%.
These equations work better for mixtures which are more completely immiscible.
174

8.D1. New Problem in 3
rd
edition. Top Op. Eq.,
D D
y L V x 1 L V x , x .975

L L D 4
.8
V 1 L D 5
,
D
L
y intercept x 0 1 x .2 .975 .195
V


B
x 0.11 (from diagram). Need 2 equil. Stages.

Graph for problem 8.D1.
8.D2. The columns are sketched in the Solution to Problem 8-C2.
1
B is butanol phase and
2
B is water
product. Two equilibrium diagrams are shown.
a.
1 2
F B B ,
1 2
1 B 2 B
Fz B x B x
175


2
1 2
B
1
B B
z x
.28 .995
B 3743.45
x x .04 .995
,
2 1
B F B 1256.55
b. Col. 1. Bottom Op:
1
B
1
L L
y x 1 x
V V
, Intersects
1
B
y x x 0.04 .
Feed: 70% Liquid, q = .7,
q .7 7
q 1 .3 3
. Intersects y = x = z = .28.
Top
2
B
L L
y x 1 x
V V

From Figure 8-D2a:
min
L 0.995 .436
0.562
V 0.995 0
,
L
1.23 .562 0.69
V

y intercept
2
B
L
x 0 1 x 0.307
V


Note that reflux is
0
x 0.573 . Optimum feed stage = 3. Need about 3 stages + partial reboiler.
Stripper (Column 2):
2
2 B
2
L L
y x 1 x
V V


2
2
2
V B 1
L V B 1.132
8.57
V V .132 V B

Construction is shown in Figure 8-D2b. Need 1 2/3 eq. contacts or P.R. + 2/3 equil. stage.


176

8.D3.





















a) y = 0.4, x = 0.09 from graph. B) V/F = 0.3

20 = Fz = Vy + Lx
L F V 1 V/ F .7 7
V V V/ F .3 3

100 = F = V + L
L F 7 .99
y x z x
V V 3 0.3

20 = 0.4 V + 0.09 (100 V) See Enlarged figure [Be careful with scales]
V 11 .31 35.48 kg moles / h
w w
y 0.969, x 0.999
L = 64.52 kg moles/h
w w
w w
F z x 100 0.99 0.999
kg moles/hr
V = 30
y x 0.969 0.999 hr

Use Table 8-2 to find V,
drum w w
T @y 0.40, x 0.09, L = 70 and
drum
T ~108 C

177


8.D4. Compositions x 0.573, x 0.975 [Shown on Figure 8-D3a]
100 F L L and 88 = Fz = L x L x

F z x 100 .88 0.573
L
x x .975 0.573
= 76.37 kmoles/hr, L 23.63 kmoles/hr
8.D5. a) Water conc. W is 0.975.
200 = F = W + B W = 200 B
Water balance: (200) (.8) = Fz = W(0.975) + B (0.04)
Solve: B = 37.433 kmol/hr, W = 162.567 kmol/hr
b)
V B V B 1 5
L V
V V B 4
, Bot. Op. Eq.
B
L L
y x 1 x
V V
.
Goes through
B
y x x
178

Plot operating line: If y = 1,
1 0.25 .04
x 0.808
1.25

PR + 2 stages mores than sufficient. (see graph)
c)
min
0.75 0.04
L V Slope 1.332
0.573 0.04


min
V 1 1
L V 3.012
L V L V 1 0.332


179


8.D6. New Problem in 3
rd
Edition.

8.D.6 Part a) F D B
F .65 .975D .02B

B
D B
z x .65 .02
D F 100 65.969
x x .975 .02

b) V V B B 136.124, Reflux L L V V 170.155.
Or V F D L 136.124 100 65.969 170.155
c)
L V B V B 1 4 1
1.25
V V V B 4


B
L L
y x 1 x
V V

Goes through
B
y x x

Calculate arbitrary point at x .6
y 1.25 .6 .25 .02 .745
See Figure: Need 2 stages + PR

d) What is
MIN
V B ?
MIN
MAX
V V 1
3.0894
B L V L V 1
On graph.

F

V

L

bot
B, x

dist
D, x

Reflux
180



181

8.D7.
A W W W in organic
W A
in organic
A in organic W in organic
A A in organic
y 0.0364, y 1 0.0364 0.9636 y x

x 0.628, x =1-0.628=0.372
y x



W A
in organic
0.9636 0.372
44.69
0.0364 0.628


8.D8. Convert wt frac to mole frac.

8 14
MW C H O 72 14 16 102 and MW water 18

Basis 1000 kg
0.994 wt frac. ether: 6 kg W = .333 kg moles
994 kg E = 9.745
Total = 10.078

W in organic
x 0.033

0.012 wt frac. ether: 988 kg W = 54.889 kg moles
12 kg E = 0.118 kg moles

W in organic
x .998

y = 0.959 ether: 41 kg W = 2.278 kg moles
959 kg E = 9.402 kg moles

W
y 0.155

W W in org
W - E in org
E E
y x
.195 .805
7.026
.033 .967 y x



z(wt) = 0.004 water: 4 kg W = 0.222 kg moles
996 kg E = 9.765 kg moles

W
z 0.022


in equil w feed
7.026 0.022
x
y 0.138
1 1 x 1 6.026 0.022



D equil w feed
min D
x y
L .998 .138
0.882
V x z .998 .022



min
min act act
min
L V
L L L
7.467; 11.20; 0.918
D 1 L V D V


Generate following equilibrium data using
w E in org
7.026 :

x
W
0 0.01 .022 .033
y
W
0 0.066 0.137 0.195

182

Top Op. line:
D
L L
y x 1 x
V V

Where L/V = 0.918,
D
x .998, and y = intercept = (1 - .918) (.998) = 0.082

Bottom: From
B
y x x to intersection of feed line and top operating line.

B
x 0.0004 wt frac.: 0.4 kg W = 0.022 kg moles
999.6 kg E = 9.800 kg mole
Obtain
B
x 0.0023
See plot in Figure: Optimum feed is top stage. Need
3
4
5
equil. Contacts.


























8.D9. Convert to Mole fractions:

6 14 water
MW C H O 72 14 16 102; MW 18

Basis for all conversions is1000 kg soln.
Top Layer Separator = 0.994 wt frac. ether
6 kg W = 0.333 kmole
994 kg ether = 9.745 kmol
Total = 10.078 kmol
Mole frac.
w in org
0.333
x 0.033
10.078


183



W W in org
W E
in organ phase
E E in org
y x
0.195 0.033
7.026
y x 1 .195 1 0.033

Feed is
W
in org
y 0.02 mole frac water = z. In equil. With feed:

*
f
y 0.02
x 0.002896
1 y 7.026 6.026 0.02


D
* min
D f
x z 0.998 0.02
L V 0.9828
x x 0.988 0.002896
,
min
L V
L
57.18
D 1 L V
,

min
act
L D 2 L D 114.36,
act
L D
L V 0.9913
1 L D

Top Op.
D,W
y L V x 1 L/ V x through
D,W
y x x 0.998.
When x = 0, y = 0.00868
Eq. Data.
7.026x
y
1 6.026x
. Generate curve,


x 0 0.01 0.022 0.033
y 0 0.066 0.137 0.195

Plot on graph, and plot top op. line:
x 0, y 0.00868. x 0.04, y 0.9913 .04 0.00868 0.04833
x 0.03, y .9913 0.03 0.00868

W,bot
x is at intersection y = z = 0.0208 top op. line,
W,bot
x 0.0123
Step off stages from top down. 1 equil stage is sufficient.
But with this very high reflux rate consider alternatives.
D
x 0.998

0.033
z = 0.02
Bottom Layer separator is
= 0.012 wt frac. ether
Mole frac.
W in org D
x 0.998 x
Vapor into Condenser is
y
azeotrope
= 0.959 wt frac. ether
988 kg W = 54.889 kmol
12 kg E = 0.118 kmol
41 kg W = 2.278 kmol
959 kg E = 9.402 kmol
Mole frac.
W
y 0.195
184


8.D10. a.
C10 W
C10 W
VP x VP x 760
Assume the water layer is pure,
W
x 1.0. Try 95.5C,
C10 W
VP 60, VP 645.7.
(.99) (60) = 645.7 = 705.1. Too low. Try higher temperature. The attached plot of VP
vs. T allows estimation of vapor pressure. (Note: a plot of log (VP) vs 1/T will be easier
to interpolate and extrapolate.)
97.0C:
C10 W
VP 63, VP 682.07 , (.99) (63) + 682.07 = 744.4
97.5C:
C10 W
VP 65, VP 694.57 , (.99) (65) + 694.57 = 758.9
T = 97.6 gives
W
VP 697.1 which will be too high. Thus T = 97.5C is close enough.
185

b.
tot org org
w
org org org
p VP x
760 65 .99 n
10.81
n 65 .99 VP x

This is significantly less than in Example 8-2 where 296.8/4.12 = 72.04 mol decane are
used. Difference is due to higher n-decane concentration in liquid.

8-D11.
F,org
x 0.9, 95% recovery 5% left. Octanol left = 0.05 (.9) (1.0) = 0.045 kmol/h
Nonvolatiles in bottoms = 0.10 kmol/h

water
steam
octanol
F
x
oc tan ol
in org
0.045
x 0.3103
1.10 0.045

W
186

a) Water VP can be fit to
10
B
log VP A
273.16 T
over short ranges T. T in C, VP is mmHg.
T = 95.5
0
B
log 645.67 A
273.16 95.5
2.8100 A B 368.66 (1)
T = 100
10
B
log 760 A
273.16 100
2.8808 A B 373.16 (2)

To solve for A and B, subtract 1 from 2

B B
0.07080 0.00003271B
368.66 373.16
B = 2164.42

B
A 2.8808 8.68105
373.16

Now find T for which
tot W W O O W O
p VP x VP x 760 mm Hg where x 1.0, x 0.3103
On Spread Sheet find T = 99.782C
b)
O
O
O
tot
VP x
19.075
y 0.025098 0.3103 0.007788
p 760


W
754.072
y x 1.0 0.99220
760

a) Moles octanol =
O
F z .95 1.0 0.90 0.95 0.855 kmol/h
b) Moles water
W
W org
org
0.9922 y
n n 0.855 108.93
y 0.007788

Check Eq. (8-18):
W
0.855
n 760 19.075 0.3103 108.93
19.075 0.3103


8.D12. New Problem in 3
rd
edition. All cases
1 2
B 60, B 40 ,
P,B2 P,dist1
2 2
P,dist1 P,dist 2
x x
D B
x x

a)
2
1.0 .65
D 40 140
.65 .55


1
D 140 40 180. Total feed Col 1 = 246
b)
1
.35
D 280 1400
.01


1
D 1440.

Total feed Col 1 = 1500


187

8.D13. New Problem in 3
rd
edition.


















Part a.
1 2
F F D B 100 80 D B
Water:

1 1 2 2 dist bot
Fz F z Dx Bx 100 .84 80 .20 D 0.975 B 0.04
Solve simultaneously, D = 99.25 and B = 80.75
b)
V
V B 121.125, L V B 201.875
B

Since feed 2 is saturated liquid V V 121.125

2
L L F 121.875

1 1 dist bot 2 2
dist 1 1 2 2 bot
c) Doing Mass balance around top Doing Mass balance around bottom
V y Fz L x Dx V y Bx L x F z
Dx Fz F z Bx L L
y y x
V V V V


These two equations are equivalent.
d) Bot. op. line:
B
L L
y x 1 x
V V
. Goes through
B
y x x .
Slope
V B 1
L V B
5 3
V V V B
. Plot Bot Op. line.
Op. line above feed 2: Slope
L 121.875
1.0062
V 121.125

At intersection
2
F feed line and bot op. line (at x .2, y 0.306667 ) with
slope 1.0062
2 stages + PR is more than sufficient (See graph).
1
F

2
F
V
V L

L

bot
B, x

dist
D, x

188


Graph for 8.D13.

8.D14. Figure is on next page.

Part b.
1 2
F B B ,
B1
B2
1 E 2 E
Fz B x B x

B2
1
B1 B2
z x 0.85 0.006
B F 100 85.60
x x 0.992 0.006
,
2 1
B F B 14.40 kmol/h
c)
b1 b2 d1
2
b2 b1 d1 d2
z x x x
D F
x x x x
0.85 0.992 0.006 0.449
100 21.196
0.006 0.992 0.449 0.75


1 2 2
D D B 35.596 kmol/h

189


8.D.15. Part a) New Problem in 3
rd
Edition.
org octanol octanol total octanol
p VP x =P where x is mole fraction octanol in organic phase.
At 0.05 atm and boiling T,
org
p 0.05 atm. 38 mmHg
From Antoine equation,

10
oc tan ol
1310.62
log VP 6.8379
T 136.05

At
octanol
T 129.8C, VP 80.905 mmHg
Since
org
org octanol,mole
benzene
p
38
p 38 mmHg, x 0.470
VP 80.905

Average mole wt solids and non volatile organics can be calculated. Basis 100 kg

mol octanol 15 130.23
0.470
15 85 mol octanol mol non-volatiles
130.23 MW


15 85 15
0.470 MW 654.04
130.23 MW 130.23

b) 95% recovery is true on both mass and mole basis.
Distillate octanol flow rate 0.95 100 0.15 14.25 kg h.
Since
octanol
MW 130.23, this is 14.25 130.23 0.109 kmol h.
In waste there are 0.05 15 0.75 kg hr octanol and 85 kg h (organics + solids), or
85.75 kg h total. Wt frac octanol 0.75 85.75 0.00875.
Mole frac. octanol in waste
0.75 130.23
0.0424
0.75 130.23 85 654.04

Water
99.4 mole % water
2
B
2
D

E
x 0.75
1
B

101.3 kPa
F
E
z 0.85
Ethanol
99.2 mole %
0.449
ethanol
1333
kPa
1
D
190

c) For equilibrium in still pot
oct in org w tot
oct
VP x VP 1.0 p 760 mmHg .
The still pot is perfectly mixed; thus
oct in org oct in waste
x x 0.0424 mole frac.
Since water boils at 100C when P 760 mmHg, T < 100C.
Eq. (8-15) becomes
oct w
VP 0.0424 VP 1.0 760
Substituting in the Antoine equations for octanol and water and solving with a spread sheet, T =
99.97C.

oct w
VP 19.27 mmHg and VP 759.18 mmHg.
d) From Eq. (8-18),
oct oct oct
w tot oct oct
n VP x
n p VP x

From spread sheet
oct w
n n 0.001076
Since
oct
n 0.109 kmol h,
w oct
n n 0.001076 101.27 kmol h

kmol 18.016 kg
water 101.27 1824.5 kg h
kmol kmol
water in distillate.
This is a lot of steam!

8.D16. Distillate 1: 0.997 EtOH, 0.0002 solvent. Calculate
1,W
d
x 1 .9972 0.0028
Distillate 2: 0.999 water, 0.00035 solvent. Calculate
2E
D
x 1 0.99935 0.00065
F = 100,
F,E F,solv
x 0.81, sat'd liq'd, x 0
Find
1 2
D , D , M where Makeup is pure solvent.
0
Water:
W W 1 D1W 2 D2W
x M Fz D x D x
0
Ethanol:
E,M E 1 d1,E 2 d2,E
Mx Fz D x D x

Ethylene Glycol:
solv
E solv 1 d1,solv d2,solv
Mx 1.0 Fz D x Dx

Solving water & ethanol balances obtain:
2
D 18.7913 and
1
D 81.2316 kmol/h.
From Ethylene Glycol balance,
M 81.2316 0.0002 18.7913 0.00035 0.02282 kmol/h
Can also use overall balance instead of EG bal. Then

1 2
M D D F 18.7913 81.2316 100 0.2290 , OK

8.D17. New Problem in 3
rd
edition. Since everything now exits the bottoms, B = S + F, and x
A
=
Fz
A
/(S+F), x
B
= Fz
B
/(S+F), x
solvent
= S/(S+F).

8.D18. Ethanol Product: 0.997E, 0.0002 solvent, 0.0028 water

Water Product: 0.9990W, 0.00035 solvent, 0.00065 ethanol


E W
F 100, z 0.20 z 0.80

191

Water bal:
WM W E EP,W W WP,W
x M Fz P x P x (A)

E bal:
EM E E EP,E W WP,E
x M Fz P x P x (B)
0
EG
M,solvent solv E EP,solv W WP,solv
x M Fz P x P x where
M,solvent
x 1.0

Solve A & B
E W
for P & P :
W
P 80.0240 kmol/h,
E
P 20.0074 kmol/h

M 80.0240 0.00035 20.0074 0.0002 0.03201 kmoles/h

8.D19. M.B. around System. Since everything in wt. units do M.B. in wts.

Overall:
1 2
F H B B where H = makeup hexane.

Ethanol:
E 1 E,bot 2 2 E,bot 2
Fz 0 B x B x
0
Hexane:
H 1 H,bot1 2
Fz H 1.0 B x B 0

Solving simultaneously,
1 2
B 8000.04, B 2000.04 and H 0.08 kg/h.

8.D20. New Problem in 3
rd
edition. a. 1000 F E W


E,Ethanol prod E,wprod
1000 0.8094 Ex Wx
809.4 0.998E 0.0001W

808.3
E 811.0 kmol h
0.9979

W F E 1000 811 189.0

b)
F
W W,Ethanol prod
w,1
Fx Ex 1000 0.1906 811.0 0.002
V V 629.93
y 0.300

boilup ratio
629.93
V E Pr od 0.777
811.0


L V EPr od. 629.93 811.0 1440.93
L L F sat ' d liquid feed 1440.93 1000 440.93
If CMO strictly valid then, L reflux 440.93

Can also estimate
from_settler 1
L Pentane flow rate in V

Ethanol flow rate in
1
V
Ethanol lost in Water Product.

Pentane flow rate
P,1 1
y V 629.93 0.6455 406.62
192

Ethanol flow rate in
1
V
E,1 1
y V 629.93 0.0555 34.96

1 lost
E,reflux
from settler to Col1
E in V E
x
L



prod
prod E in W
Ethanol lost in water product W x 189.0 .0001 0.0189

from_settler _calculation
L 441.6

Match not perfect because: 1. CMO not totally valid
2. There is some water in reflux
3.
dE
K value may be incorrect.

c. Ethanol returned to distillation column

1 E1 Pr od E,WPr od
Vy W x 34.96 0.0189 34.19 kmol h.
Using average estimate for
0
440.93 441.6
L 441.3
2

Then
E in pentane
34.91 441.3 x

E,pentane E,Reflux,pantane_layer
x 0.0792 x

Then since assume
d E,Water layer
K 1, x 0.0792



193


d.
















8.D21.

E,water
x 0.0792
1
V
V 0.5 V 0.5W 94.5 kmol h.
W
L W V 283.5 kmol h.

W 189.0

L 283.5
3
V 94.5

1
E
L 0.0792 W .0001
y
V

1
E
283.5 .0792 189.0 .0001
y 0.237
94.5

E, WPr od
x 0.0001
1
194

Bottom:
B
L V B 1 1.5 L L
3, y x 1 x
V V B 0.5 V V
.Goes through
B
y x x with slope = 3
Feed line = Horizontal (q = 0). Through y = x = z = 0.4
Top. MB:
D
yV Lx Dx and V L D

D D
L L
y x 1 x goes through y x x 0.975
V V

Intersects Feed line where bottom op line does.
Opt. Feed #1 above reboiler. 3 equilibrium stages + PR is sufficient.

8.D22. New Problem in 3
rd
edition.


Part a.
dist bot
F D B & Fz Dx Bx

bot
dist bot
z x 0.20 0.08
D F 100 13.41 kmol/h
x x 0.975 0.08
and B 86.59

Part b.
dist
yV Lx Dx

dist
L D
y x x
V V
Substitute in D V L to obtain
dist
L L
y x 1 x
V V

Points on operating line:
dist
y x x 0.975 and
dist
L
x 0, y intercept 1 x
V

Alternative point is at feed line (y = z = 0.2) &
bot
x x 0.08
V F, L B, thus slope L V B F 86.59 100 0.8659
y intercept 1 0.8659 0.975 0.1307
Part c. Need 2 Stages. See graph.
Part d. Pinch at feed line intersection with equilibrium is at x 0.02 .



V

L

B
dist
D, x

Reflux
F
195


Figure for problem 8D22.

196


8.D23. New Problem in 3
rd
edition.



















Part a. External balances
Pr od Pr od
F NM W

Pr od Pr od
NM: F .25 NM .98 W .01

Pr od Prod
.25 .01
NM 100 24.74, W 75.26
.98 .01


a. W Column: z = .25, horizontal feed line
Top
NM NM NM,bot ,col NM
L L
y x 1 x
V V
Mass balance through top of W column
and around col. NM. Can easily show that
NM
W F
x
w
= 0.312
Nitro Methane
Product
Water
Product
X
NM
= 0.086
Water phase
N.M. Phase
197

NM NM NM,bot ,col NM
y x x
But do not know L/V so cannot plot yet.
Bottom operating line looks familiar:
NM NM,bot col w
L L
y x 1 x
V V


NM NM NM,bot ,col w
y x x 0.01

col w
L V B 1 5 4
5
V V B 1 4

Can plot bottom operating line. Arbitrary point: x 0.2, y 5 0.2 .01 .96
Now can plot top operating line from intersection of bottom operating line and feed line to point

NM NM NM,bot ,col w
y x x 0.01
See graph. Need PR + ~ 1 2 stage. Build PR + 1 stage.
b. NM Column is a stripping column:
w w w,bot ,col NM
y L V x L V 1 x

col NM
V
1
4
B
L V
V 3
B


w w w,bot ,NM_col
y x x 0.02
To plot,
w w w
4 1
x .3, y .3 .02 0.3933 x 0.3 is arbitrary point
3 3

Need PR + ~
1
1
3
stages.
c. W col. Want V.
Prod
V 1 1
V 100 V, V= B W 75.26 18.81
B 4 4

V to cond. from
col
W 118.81 kmol hr
NM col want
Pr od
V
V. V= B 3 NM 3 24.74 74.22 kmol hr
B
. To condenser.

198


Graph for 8.D23.



199

8.D24. New Problem in 3
rd
edition. From Equilibrium,
bu tanol
y 0.092 at
bu tanol
x 0.004
Overall Mass Balance: 100 F V B
Butanol MB: 100 0.025 V .092 B .004
2.5 .092 V F V .004
2.1 .088V V 23.864, B 76.136 kmol hr

This problem can also be solved graphically, but using basic mass balances is easier.

8.D25. New Problem in 3
rd
edition. Part a)
org benzene benzene
p VP x where
benzene
x is
mole fraction benzene in organic. At boiling T,
org
p 1.0 atm.
From Antoine equation,
10
benzene
1211.033
log VP 6.90565
T 220.790

At T 93 C,
benzene
VP 1112.44 mmHg . Since

org
org ben,mole
benzene
p
760
p 760, x 0.683
VP 1112.44

Average mole wt solids and non volatile organics can be calculated. Basis 100 kg


Moles benzene 20 78.11
0.683
20 80
Moles benzene + Moles non-volatiles
78.11 MW


80 .683
20 80 20 20
0.683 1 .683 MW 673.2
78.11 MW 78.11 MW 78.11


b) 90% recovery is true on both mass and mole basis.
Distillate benzene flow rate 0.9 100 0.2 18.0 kg h.
Since
benzene
MW 78.11, this is 18 78.11 0.230 kmol h
In waste there are 2.0 kg/h benzene and 80 kg/h (organics + solids), or 82 kg/h total.
Wt frac benzene 2 82 0.0244

2 78.11
Mole frac. benzene 0.1773
2 78.11 80 673.2

c) For equilibrium in still pot
b in org w tot
b
VP x VP 1.0 p 760 mmHg
The still pot is perfectly mixed; thus,
b in org b in waste
x x 0.1773 mole frac.
Since water boils at 100C when P 760 mmHg, T 100 C. Benzene is more volatile and
boils at 80.1C, but mole fracs low.
Antoine equation for water:
10 w
2164.42
log VP 8.68105
273.16 T

Eq (8-15) becomes
b w
VP 0.1773 VP 1.0 760
Substituting in the Antoine equations for benzene and water and solving with a spread sheet,
T 92.0411 C
. ben w
VP 1082.5 mmHg and VP 568.1 mmHg.

200

d) From Eq. (8-18),
ben ben ben
w tot ben ben
n VP x
n p VP x

From spread sheet
ben w
n n 0.337876
Since
ben
n 0.230 kmol hr,
w ben
n n 0.337876 0.6807 kmol hr


kmol 18.016 kg
water 0.6807 12.264 kg h
kmol kmol

water in distillate.
e. To vaporize benzene condense moles water
ben ben
w
n
.
This occurs at 92.0411 C 365.1911K; From Perrys table 2-237,


T = 360 886.7 498.7 388
T = 370 898.6 518.1
380.5
f
h h
g
H h
H h kJ kg


Linear interpolate
5.1911 kJ
x 380.5 388 388 384.1 30, 002 kJ kmol.
10 kg

Note: 8
th
edition, Table 2-193 is very slightly different after unit conversion.

Water Table 2-352.


w
0.0911
2265.67 2278.3 2278.3 2277.8 kJ kg 41,037 kJ kmol
5

Moles water condensed
0.230 30, 002
0.1682 kmol h
41037
water (in waste)
kg h water in waste 0.1682 18.016 3.029 kg h





T = 360 2663 384.7 2278.3
T = 370 2671 405.88 2265.67
201


8.E.1. New Problem in 3
rd
edition.


























Part a.
1 2 NM W
250 F F P P
NM balance
NM NM
1 1 2 2 MN NM Prod w wPr od
NM mol frac NM mol frac
Fz F z P x P x

NM w
135.5 8 127.5 P .98 P 0.01

NM NM NM
135.5 P 250 P 0.01 .97P 2.5

NM
P 133 .97 137.11 kmol hr

w
P 250 137.11 112.89
b. Column W Use
NM NM
y vs. x (water phase) plot.
Top operating line
2
2 NM
water
NM NM NM
wcol
col
NMPr od
wcol
F z
y L V x 1 L V x
V

Bottom Operating Line
water water NM NM NM
col col Water Pr od
y L V x L V 1 x


col w
col w 1
V
V B 37.63 V V B V B 1 4 3
L V 4. .
B
V V B 1 3
L V B 150.52 L F

NM
W
x
w
= 0.312
Nitro Methane
Product
Water
Product
X
NM
= 0.086
Water phase
2
F
1
F

N.M. Phase
202

Top IS NOT from
NM
NM Pr od
y x x 0.98 to intersection feed and bottom operating line.
Instead from intersection of feed and bottom operating line with slope

1
L V L F V 50.52 37.63 1.32 25.
Optimum feed is top stage. Need PR + 1 stage.

c. Column NM. Top
1 1, NM
NM
NM w w
col
col W,W Prod.
Fz
y L V x 1 L V x
V

Bottom
w w w
NMcol NMcol
in WM Prod.
y L V x L V 1 x

w w w
NM Pr od
y x x 0.02.
L V B 1
2.00
V V B

Draw bottom operating line. Top is through intersection bottom operating line and feed line F
2
.
Slope L V 0.906 (see item d). Need 2 stages + PR. Optimum feed is stage above PR.

d. Column W:
W col w
P B 112.89

col w
V V B B 1 3 112.89 37.63

col w
L V B 150.52. L L F 50.52.
Saturated liquid feed:
kmol
V V 37.63
hr.

Column NM:
NM col NM
V B 1.0, P B 137.11

V B 1
L V 2.0
V B

V V B B 137.11 kmol hr, L 2.0 137.11 274.22
Saturated liquid feed V V 137.11, L L 150 124.22 , L V 0.906.
e. Minimum boilup rate
column
NM gives combination bottom & top operating lines to go through
reflux point:
W w
y 0.5, x 0.312 .
From bottom operating line intersection with feed line
2
x z 0.15 is

INTER 2 W, NM Prod
L L
y z 1 x
V V

Slope of top operating line to reflux point is

INTER 2
0.5 y L F L
V 0.312 0.15 V



V 1
V B .
L V L V 1
V V B B 137.11 V B .

L V B

2
int er
L F
Guess V B Calc V&L&L V Calc y Calc L V
V

Check is two calculated values L V are same.
203

Spreadsheet.
MIN
V B 0.6105, L V 0.846


Graph for Problem 8.E1.
204

8.E2. Balances at mixing point for F & R.

To Butanol Column:
Overall:
T
F F R 100 R
Water:
T T,W W W,reflux T T,W
F z Fz Rx F z 30 0.573R

T,W
R .573 30
z
100 R

External balances: 100 = W + B, water: 30 = 0.995W + 0.02 B
Solve simultaneously: W = 28.72 & B = 71.28 kmol/h

Butanol Col: V B 1.90, V 1.90B 135.432

T
F L V B 206.712, L V 1.5263

W,butonal
y x x 0.02

T
R F F 206.712 100 106.712

T
106.712 0.573 30
z 0.4409
206.712

Vertical feed line at
T
z intersects bot. operating Line at y = 0.67 (see graph)

Water Col.
V B V B 1
V B 0.1143, L V 9.748
V V B



B,water,watercol,
y L V x L V 1 x
B, W
watercol
y x x 0.995
See graph, y leaving column = 0.8


205







206




8.E3. Basis: 1000 kg sea water (1 h):

F,W
965 kg water kmol 18.016 kg 53.5635 kmol, x 0.98894

F,salt
0.5988
35 kg NaCl kmol 58.45 kg kmol, x 0.011056
Total 54.1623

Water Condensate = (0.60) (53.5635) = 32.1381 kmol/h =
W
n
Water Remaining 53.5635 32.1381 = 21.4254
Waste Water is
21.4254 kg moles W
0.9728
21.4254+0.5988 salt
mole frac. water

a. In still, organic phase is pure decane.

C10 W W tot W
VP VP x p 760 mmHg where x 0.9728.
Try T = 99C.
C10 W
VP ~ 68, VP 733.2 mm Hg
68 + (0.9728) (433.2) = 781.27 mm Hg, which is too high.
Converge to T ~ 98.2C.
b. Distillate:
W W W W W
org org org org org
707.27 0.9728 n y p VP x
10.4225
n y p VP x 66 1.0

(This calculation is at 98C, not 98.2, but will be close.)
207



C10 10
32.1381 kmol water/h kmol
n 3.0819 C
10.425 mol water/mol organic h
in distillate

8.F1. V.P. Data n-nonane (p. 3-59 Perry & Green, 1984)

VP = 20 40 60 100 200 400 760 mm Hg
T = 51.2 66 75.5 88.1 107.5 128.2 150.8C


See Solution problem 8.D10 for plot.
nC9
MW 128.25 . From p. 3-268 (Perry & Green,
1984), nonane enthalpies are,
T h
liquid
H
gas
360 K 671.3 KJ/kg 998.2
380 K 722.5 1036.5

Water VP is given in Problem 8-D10 and on p. 3-45 of Perry and Green (1984).

a. Try 95.0C.
C9
VP 127 mm Hg,
W
VP 633.9 . Assume water is pure.
Pressure: (.99) (127) + (1.0) 633.9 = 759.6. Close enough and lucky!

b.
tot org org
W
org org org
p VP x
760 127 .99 n
5.045
n 127 .99 VP x
mol water/mol nonane

c. Need to calculate the energy required to vaporize the nonane. T = 273 + 95 = 368 K.
By linear interpolation for pure nonane:
1 gas nonane
h ~ 691.78, H ~1013.52. 321.74 KJ/kg

Table 3-302 of Perry and Green (1984):
W
liq,W vap W
h 397.36, H 2667.8, 2270.44 KJ/kg

C9
9 W
321.74 KJ/kg 128.25 kg/kmol
mol water condensed
1.009
mol C vaporized 2270.44 KJ/kg 18.016 kg/kmol

b. Now,
C9 W
VP .020 VP 760. Temperature will be higher.
Try 99C:
C9 W
VP 149 mm Hg, VP 733.24
(149) (.020) + 733.24 = 736.22 which is too low.

Try 99.9C:
C9 W
VP 154, VP 757.29
154 (.020) + 7570.29 = 760.37. Close enough.

The low nonane conc. reduces nonane partial pressure and operation is much closer to 100C.

tot org org
W
org org org
p VP x
760 154 .02 n
245.75
n 154 .02 VP x


Need lot more steam!
208

8.F2.


Basis: 1 hour
a)
All junk in feed (0.50 kmol) is in bottoms
Organic Bottoms is 0.95 n-C9 0.5 junk 1.45 (see part C)


C9,bot ,org
.95
x 0.65517
1.45

b) Still T.
W org tot
p p p 102.633 kPa 770 mm Hg

W W W W
p VP (T)x where x 1

org C9 c9,org,bot C9 C9
p K T x K T 0.65517 770 mm Hg 504.481 K T
Procedure: Guess T, determine
W C8
VP &K check if pressure eq. is valid

C9
W
VP 504.481K 770 mm Hg?
Try T = 96C
W C9
VP 657.62, K 0.16 (DePriester Chart)
652.62 80.717 738.34 Need higher T
T = 97C,
W C9
VP 682.07 and K 0.17
682.07 + 85.762 = 767.83 slightly low, but close enough

c) 9.50 kmol n-nonane .90 = 8.55 kmol n-nonane in distillate. 9.5 8.55 = 0.95 kmol n-C9 in
bottoms
d) Eq. (8-18)
org org org
W
W W
n p n
85.762 8.55
0.12567, n 68.031
n p 682.07 0.12567 0.12567
kmol water

e) EB simplies to
W W,condensed nonane org,dist.
n n where s are at 97C = 370 K.

C9 C9
W,condensed org,dist.
W W
n n 8.55 kmol

water
F = 1000
steam
n-nonane
water
organic
waste
95% n-nonane
209

From Perrys 6
th
ed. Table 3-268 or 7
th
ed. Table 2-292, nonane s are:

g f
h h , @ 360 K, 998.2 671.3 326.9 kJ/kg
380 K, 1036.5 722.5 314.0 kJ/kg

@ 370 K,
326.9 314
320.45
2
kJ/kg
MW
9
C 128.258 . Then at 370 K,

C9
kJ 128.258 kg
320.45 41,100.3 kJ/kmol
kg kmol

Water 370 K: (Perrys 6
th
Ed., Table 3-302).

W
kJ 18.016 kg kJ
2671 405.8 2265.2 40, 809.84
kg kmol kg


W,cond
41,100.3
n 8.55 8.611
40, 809.84
kmol water

8.G.1. New Problem in 3
rd
Edition.
1. Final makeup solvent flow rate _________0.02__________ kmol/h.
2. Final value solvent recycle rate (B
2
) __1400___kmol/h and L/D in col 1 _0.100_.
3. Final values of flow rates D
1
_140.0_, B
1
_1460.02_, and D
2
__60.02___ kmol/h.
4. Mole fractions in stream D
1
_Pyr=0.0084259, W=0.99157, Bisphenol=.49E-10_
5. Mole fractions in stream D
2
_Pyr = 0.98001, W = 0.019654, Bisphen = .000333___
6. Mole fractions in stream B
1
_Pyr=0.040287, W= 0.0008079, Bisphen = 0.95890_
7. Mole fractions in stream B
2
(solvent recycle stream) Pyr = .526E-8, W = .2E-12, Bisphenol =
1.0000__
8. Heat load in cooler on solvent recycle line__-0.15216E8___ cal/s.

8.G2. New Problem in 3
rd
edition. Aspen Plus Residue Plot 4.0 atm using NRTL
Pressure can have major effect on VLE for non-ideal systems. Compare T-xy diagrams for
acetone MEK at 1.0 and 4.0 atm.
Also compare residue curves for acetone-MEK-MIBK at 1.0 & 4.0 atm.

210




211



212

8.G3. New Problem in 3
rd
edition.
a. Final reflux ratio column 1___0.01_ and final reflux ratio column 2 _0.01_______. If these values are
not 0.01 you are not finished with Part B.
b. Flow rates furfural product ___166.0___ kmol/h and water product ___34.0__ kmol/h.
c. Boilup rate in column 2 _____8.0________ kmol/h.
d. Mole fraction furfural in furfural product ____0.99816___& mole fraction water in water product
___0.99102____.
e. Flow rate of distillate from column 1 _____42.10_____ kmol/h.
f. Column 1 condenser temperature __370.3___K, & column 1 reboiler temp. __433.59__ K.
g. Outlet temperature of decanter ____375.2______ K.
h. Molar ratio of water phase/total liquid in decanter _____0.8393_____

8.G.4. New Problem in 3
rd
edition.
Column 1: a. Bottoms product mole fraction acetonitrile______0.99915______
b. Distillate flow rate ___240____ kmol/h and bottoms flow rate ___170____kmol/h.
c. Distillate mole fraction acetonitrile ____0.67910___________ .

Column 2: a. Distillate flow rate ___210_______ kmol/h, and reflux ratio ___1.2____.
b. Bottoms product mole fraction water______0.99517______________ .
c. Distillate mole fraction acetonitrile _____.77542__________ .

8G5. New Problem in 3
rd
edition.
Results are residue curves and profiles of mole fraction vs plate location. For an equal molar feed, N = 10
does not give the desired purity even if L/D = 10. N = 50 does work with L/D = 2, but not for L/D = 1.0.

8.H1. Part b. Was 8.D12 in 2
nd
edition of SPE. Use Eq. (8-25b) with
AB BB BC
2.4, 1.0, 0.21.
A = benzene, B = toluene, C = cumene. Results from Spreadsheet:
Stage: Reboiler:
1
2
3
4
5
6
7
8
9
10
11
12
A
x 0.0003
0.003298
0.03137
0.18019
0.46126
0.70274
0.85421
0.93403
0.97145
0.98791
0.99493
0.99788
0.99912
B
x 0.0097
0.04443
0.176085
0.42145
0.44952
0.28536
0.14453
0.06585
0.028535
0.012091
0.005074
0.00212
0.000885
C
x 0.990
0.95227
0.79255
0.39836
0.08923
0.01189
0.001265
0.000121
1.102 E-5
9.802 E-7
8.637 E-8
7.580 E-9
6.641 E-10

8.H2. Was 8.D13 in 2
nd
edition of SPE. Use a spreadsheet with Eq. (8-30) as recursion equation.
Result is shown in Figure. The VBA program was given in Example 8-3. The results obtained
for the starting conditions given are:
k x
A
x
B
x
C

1 0.990 0.001 0.009
100 0.9763 0.0017 0.0220
200 0.9431 0.0029 0.0534
300 0.8630 0.0049 0.1331
213

400 0.6740 0.0077 0.3183
450 0.5044 0.0089 0.4867
475 0.3946 0.0092 0.5962
500 0.2696 0.0089 0.7214
600 0.00042 0.00095 0.9986

Results for other starting conditions are shown in the figure.

214

Figure for problem 8H2.


8H.3. New Problem in 3
rd
edition. The spread sheet including the first 10 time steps and time steps 600
to 610, and the VBA program are listed.
Part a



215

Simple distillation calc (residue curves) with BP calcs.
aT1 aT6 ap1
iB -1166846 7.72668 -0.92213
nB -1280557 7.94986 -0.96455
iP -1481583 7.58071 -0.93159
nP -1524891 7.33129 -0.89143
nhex -1778901 6.96783 -0.84634

Residue curve calc. h 0.01 N 1000 epsilon 1E-09
x1iB 0.98 x1nB 0.01 x1iP 0 x1nP 0.01
sumx 1 T1guess,R 500 p,psia 14.7 x1nHex 0
time step xiB xnB xiP xnP x nHex T R
1 0.98 0.01 0 0.01 0 472.0604
2 0.979883 0.010032 0 0.010085 0 472.0642
3 0.979764 0.010065 0 0.010171 0 472.068
4 0.979645 0.010097 0 0.010258 0 472.0718
5 0.979525 0.01013 0 0.010345 0 472.0757
6 0.979405 0.010162 0 0.010433 0 472.0797
7 0.979283 0.010195 0 0.010522 0 472.0836
8 0.979161 0.010228 0 0.010612 0 472.0876
9 0.979037 0.010261 0 0.010702 0 472.0916
10 0.978913 0.010294 0 0.010793 0 472.0956

600 0.071054 0.009754 0 0.919192 0 542.3988
601 0.069204 0.009586 0 0.92121 0 542.71
602 0.067391 0.009418 0 0.923191 0 543.0165
603 0.065615 0.009253 0 0.925133 0 543.3183
604 0.063874 0.009089 0 0.927037 0 543.6155
605 0.06217 0.008927 0 0.928903 0 543.9078
606 0.060502 0.008766 0 0.930732 0 544.1955
607 0.058869 0.008608 0 0.932524 0 544.4783
608 0.057271 0.008451 0 0.934279 0 544.7563
609 0.055708 0.008296 0 0.935997 0 545.0295
610 0.054179 0.008142 0 0.937678 0 545.2978

Option Explicit
Sub Residue_Curve_BPcalc()
' K value data for nbutane,ibutane, ipentane,npentane and nhexane included.
' Only want 3 for residue curve. Thus, set x values = 0 for 2 components.
' The reference component is nbutane.
Dim i, N, j As Integer
Dim h, epsilon, xiB, xnB, xiP, xnP, xnHex As Double
Dim T, p, aT1iB, aT6iB, ap1iB, aT1nB, aT6nB, Ap1nB As Double
Dim aT1iP, aT6iP, ap1iP, aT1nP, aT6nP, ap1nP, aT1nHex, aT6nHex, ap1nHex As Double
Dim KiB, KnB, KiP, KnP, KnHex, Ksum, chksum, inside As Double
Dim yiB, ynB, yiP, ynP, ynHex As Double

Sheets("Sheet1").Select
216

Range("A15", "G1045").Clear
aT1iB = Cells(5, 2).Value
aT6iB = Cells(5, 3).Value
ap1iB = Cells(5, 4).Value
aT1nB = Cells(6, 2).Value
aT6nB = Cells(6, 3).Value
Ap1nB = Cells(6, 4).Value
aT1iP = Cells(7, 2).Value
aT6iP = Cells(7, 3).Value
ap1iP = Cells(7, 4).Value
aT1nP = Cells(8, 2).Value
aT6nP = Cells(8, 3).Value
ap1nP = Cells(8, 4).Value
aT1nHex = Cells(9, 2).Value
aT6nHex = Cells(9, 3).Value
ap1nHex = Cells(9, 4).Value
h = Cells(11, 4).Value
N = Cells(11, 6).Value
epsilon = Cells(11, 8).Value
xiB = Cells(12, 2).Value
xnB = Cells(12, 4).Value
xiP = Cells(12, 6).Value
xnP = Cells(12, 8).Value
xnHex = Cells(13, 8).Value
T = Cells(13, 4).Value
p = Cells(13, 6).Value
For i = 1 To N
j = i + 1
Do
KiB = Exp((aT1iB / (T * T)) + aT6iB + (ap1iB * Log(p)))
KnB = Exp((aT1nB / (T * T)) + aT6nB + (Ap1nB * Log(p)))
KiP = Exp((aT1iP / (T * T)) + aT6iP + (ap1iP * Log(p)))
KnP = Exp((aT1nP / (T * T)) + aT6nP + (ap1nP * Log(p)))
KnHex = Exp((aT1nHex / (T * T)) + aT6nHex + (ap1nHex * Log(p)))
Ksum = KiB * xiB + KnB * xnB + KiP * xiP + KnP * xnP + KnHex * xnHex
KnB = KnB / Ksum
inside = aT1nB / (Log(KnB) - aT6nB - (Ap1nB * Log(p)))
T = Sqr(inside)
chksum = Ksum - 1
Loop While Abs(chksum) > epsilon
Cells(13 + i + 1, 1).Value = i
Cells(13 + i + 1, 2).Value = xiB
Cells(13 + i + 1, 3).Value = xnB
Cells(13 + i + 1, 4).Value = xiP
Cells(13 + i + 1, 5).Value = xnP
Cells(13 + i + 1, 6).Value = xnHex
Cells(13 + i + 1, 7).Value = T
yiB = xiB * KiB
217

ynB = xnB * KnB
yiP = xiP * KiP
ynP = xnP * KnP
ynHex = xnHex * KnHex
xiB = xiB + (h * (xiB - yiB))
xnB = xnB + (h * (xnB - ynB))
xiP = xiP + (h * (xiP - yiP))
xnP = xnP + (h * (xnP - ynP))
xnHex = xnHex + (h * (xnHex - ynHex))
Next i
End Sub


218
Chapter 9
New Problems and new solutions are listed as new immediately after the solution number. These new
problems are: 9.A4, 9.A5, 9C1, 9D1, 9.D5, 9.D8, 9D11, 9D13, 9D18, 9D19, 9D21, 9D22, 9D25, 9E2, 9.H1.
9.A4. New Problem in 3
rd
edition. Answer is g.
9.A5. New Problem in 3
rd
edition. Answer is c.
9.A6. b. The same.
9.B1.
Multi Stage Single Stage
F Davg reflux
F, x , x , N, L/ D, T , P
F Davg
F, x , x , P
F Wf r
F, x , x , N, L/ D, T , P
F Dtot
F, x , x , P
F final r
F, x , W , N, L/ D, T , P
F wf
F, x , x , P
F tot r
F, x , D , N, L/ D, T , P
f
W final
F, x , W , P
f
F Davg W r
F, x , x , x , N, T , P
F Davg tot
x , x , D , P
f
F Davg W r
F, x , x , x , L/ D, T , P
f
F W final
x , x , W , P
F Davg tot r
x , x , D , N, L/ D, T , P
f
F W tot
x , x , D , P
F Davg tot Wf r
x , x , D , x , N, T , P
F Davg W
F, x , x , x
F Davg tot Wf r
x , x , D , x , L/ D, T , P
etc.
F Davg final r
x , x , W , N, L/ D, T , P
final
F W final r
x , x , W , N, L D, T , P
etc.
9.B2. a. Replace the column with one containing more trays or more packing.
b. Retray or repack existing column.
c. Run a batch in several steps. For example, take the feed and operate so that the desired bottoms
concentration is met. Collect all the distillate and use this as the feed for a second batch. Operate
so that the distillate for this run meets specifications. The bottoms from this run can be used as
feed for a 3
rd
run, or it can be mixed with the next feed batch.
An alternate is to first collect distillate of desired purity. Then collect distillate which does not
meet purity requirements while bottoms is reduced to the desired purity. The material not
meeting requirements is then mixed with fresh feed for next batch.
Other operating variations are possible.
d. Hook up two batch stills in series Either to run 1 batch or to run separate batches (second still
takes distillate from first as the charge).
e. See if product specifications can be relaxed.
f. Reducing the pressure increases the relative volatility and may help. However, one must watch
for earlier flooding.
9.C.1. Rayleigh eqn
F
W,fin
x
W
final
D W x
d x
W F exp -
x x
219
Because
D
x constant, can integrate analytically.

F
F
w,final
w,final
x
x
W D F
D W
D W D W,final x
x
d x x x
n x x n
x x x x


D F D F
final
D W,final D W,final
x x x x
W F exp n F
x x x x

Mass balances are
final
F W D


F final W,final D
Fx W x D x

Solve for
D F
final
D W,final
x x
W F
x x

Thus results are identical.


9.C2. External balances over entire cycle

final
D wAvg
Fz Dx Wx and F = D + W
a. Ignoring holdup on stages and in reboiler
- Out = Accum in accumulator is -
w D
x dD d Dx
which becomes,
w D D
x dD Ddx x dD
Rearrange,
D W D
dD
dx x x
D
and integrate,

final Dfinal
D F
D x
D
D F x x
w D
dx dD
D x x
which is,
x
Dfinal
F
final D
x
D w
D dx
n
F x x


b. Assume CMO and draw mass balance envelop around bottom of the column.

L V B
w
Lx Vy Bx

w
L B
y x x
V V


w
L L
y x 1 x
V V

220
9.D.1. New Problem in 3
rd
Edition. Eq. (9-9)
F
W,final
x
final
x
dx
W F exp -
y-x

From Simpsons rule
0.00346 x 0.005173 0.1
0.1 0.00346
1 4 1
Area
6 y x y x y x

From equilibrium curve (Table 2-1).
x y y-x
1
y-x

0.00346 0.03096 0.027498 36.366
0.05173 .34 .28827 3.469
0.1 0.4416 .3416 2.9273

The y values are found by linear interpolation of data in Table 2-1. For example, at x = 0.00346,
linearly interpolated first 2 data pts Table 2-1.

0.170
y x for x 0.19.
0.019

For x = 0.00346,
0.170
y 0.00346 0.03096
0.019

For y at x = 0.1,
y = 0.4375 +[ (.4704 - .4375)/(.1238 - .0966)](.10 - .0966) = 0.4416

[Alternatively, could fit equilibrium data to constant .]

0.1 0.00346
Area 36.366 4 3.469 2.9273 0.8555
6


final
W 0.5 exp 0.8555 0.2125 kmol


total
D 0.5 .2125 0.2875 kmol


Avg
F final W,final
D
avg
F x W x .5 .1 .2125 .00346
x 0.1714
D 0.2875

9.D2. Rayleigh equation is
.75
final
.55
dx
W F exp -
y-x

Most of values of 1/(y x) are listed in Example 9-1. From Table 2-7 can easily generate
values for x = .55: y = 0.805, y x = .255. (y x)
-1
= 3.92. The mid-point for Simpsons
rule is at x = .65. Then from Eq. (9-12) and values in Example 9-1,

.75
.55
dx .2
3.92 4 5.13 6.89 1.044
y x 6


1.044
final total final
W 100e 35.20, D F W 64.8

F final final
Davg
total
75 35.2 0.55 Fx W x
x 0.859
D 64.8

A graphical integration counting squares gives
Davg
x 0.861.
9.D3. Operating equation is
0
D
0
L D
y L V x 1 L V x where L V 0.65
1 L D

221
This is y = 0.65 x + 0.35
D
x
We have two equil. stages (stillpot and one in the column). From McCabe-Thiele diagram
we can get the values of
D
x which are related to
W
x . Pick
D
x and get
W
x from figure.
From this we can generate the following table (only two values are shown in Figure).










Simpsons rule (Eq. (9-12)). [Midpoint
w
x 0.36]

f
w,final
x
w
x
D W
.57 .15 dx
3.077 4 2.096 1.754 0.925
x x 6

Raleigh Equation:
F
wfinal
x
final
x
D w
W dx
F x x
,
0.925
final
W Fe 39.7 kmol

total final
D F W 100 39.7 60.3 kmol

F final wf
Davg
total
100 0.57 39.7 0.15 Fx W x
x 0.847
D 60.3


x
D
x
W
1/(x
D
-x
W
)
0.90 0.588 3.205
0.895 0.570 3.077
0.85 0.394 2.193
0.837 0.360 2.096
0.720 0.150 1.754
0.70 0.128 1.748
222
9.D4.
final F wf
W 2.0 kg moles, x 0.8, x 0.4 . Find F,
DAvg total
x , D




Can use Simpsons Rule (Eq. 9-12) or evaluate numerically.

F
W
fin
x
x
dx 0.4
4.76 4 7.143 16.666 3.3333
y x 6

Rayleigh eqn
F
W
fin
final
x
x
W 2.0
F 56.063 kmol
exp 3.3333
dx
exp
y x


total fin
D F W 54.063 kmol


fin
AVG
F f W
D
total
Fx W x 56.068 0.8 2.0 .4
x 0.815
D 54.063


9.D.5. New Problem for 3
rd
Edition a)
F
w,fin
x
final
x
dx
W F exp
y x

Can use Simpsons rule, eq. (9-12) with equilibrium values from plot.

w,final
0.028
1 1
f x 2.7397
y x 0.645 0.28


w,fin F
0.40
x x
1 1
f 3.2787
2 y x 0.705 0.52


F
0.52
1 1
f x 4.65116
y x 0.735 0.52


x y(equil) 1
y x

0.80 0.86 16.666
0.70 0.80 10
0.60 0.74 7.143
0.50 0.67 5.882
0.40 0.61 4.76
16.66
.8
7.143
.4
.6
4.76
x
1
y x


223



F
fin
avg
w,fin
x
F w
w,fin w F
x
x x
dx
f x 4f x f x
y x 6


0.52 0.28
2.7397 4 3.2787 4.65116
6
0.24
20.5056 0.8202
6


fin
W 3exp 0.8202 3 0.44033 1.321 ,
V,tot fin
D F W 1.6790

fin
fin w
D,avg
V,tot
Fz W x
y 0.7088
D

b. Settler:
V,tot 1 2 1 2
D D D 1.6790 D D

V,tot D,avt 1 2 2 B 1 2
D y D x D x 1.6790 0.7088 0.573 D 0.975 D
Solving simultaneously,
2 1
D 0.567 and D 1.112
9.D6. Use eq. (9-13)
W,F F
F f
W,f F W,f
x 1 x
W 1 x 1
n n n
F 1 1 x x 1 x

224
a) F = 1.3,
F Wf
x .6, x .3, 2.4

F
.3 4 W 1 .4
n n n
1 3 1.4 .6 7 .7
0.8948 0.5596 1.4544

F F
W 1.3 0.2335 W 0.3036 kmol

f
AVG
F fin W
D
fin
Fx W x 1.3 0.6 0.3036 0.3
x 0.6914
F W 1.3 0.3036

b) Now
f f
W 3.5 0.2335 W 0.81725

AVG
D
3.5 0.6 0.81725 0.3
x 0.6914
3.5 0.8125

c)
AVG
F D
F 2.0, x 0.6, x 0.75
Since
AVG
AVG f
AVG
D F
fin
fin D F fin W
D W,f
x x
W
F W x Fx W x ,
F
x x
.
Then Eq. (9-13) becomes

AVG f
f AVG f f
f
D f W
W D W W
W
x x x .4
0.75 .6 1 4
n n n n
0.75 x x x 1.4 1 x
.6 1 x

Solution is
fin
w
x 0.5025 .
AVG
AVG f
D f
final
D W
F x x
2.0 .75 0.60
W 1.212
x x .75 0.5025
kmol.
9.D7.
L D 1
L D 1/ 2, L V slope
1 L D 3
. Pick series
D
x values. Plot enriching section op
line. Step off two stages. Find
W
x . Calculate
F
W
f
x
x
D W D W
1 dx
, determine
x x x x

D
x


0.56

0.49
W
x


0.11
0.10
0.09
D W
1
f
x x


1/.45 = 2.2222
Interpolate 2.361
1/.40 = 2.50

0.44
0.38
0.26

0.06
0.05
0.02

1 .38 2.6316
1 .33 3.0303
1/.24 = 4.1666
Simpsons Rule

F
f
F
W
fin
x
F W
W W F
x
D W
x x
dx
f x 0.02 4f x 0.06 f x 0.10
x x 6


0.10 0.02
4.1666 4 2.6316 2.361 0.22739
6

225

Final
W 4.0 exp 0.22739 4 0.7966 3.1864


f
Avg
F f W
D
f
Fx W x 4 .1 3.1864 0.02
x 0.4133
F W 4 3.1864





9.D8. New Problem 3
rd
Edition. a) Op. Eqn.
d
L L
y x 1 x
v V


d d
4 L L
L V 1 x , x .8@y x
D D 5
,
Intercept D
L
y 1 x .2 .8 .16
V

From McCabe-Thiele plot
final
w
x ~ 0.075

b)
final tot final tot
10 F W D W 10 D

final
F final w tot D final tot
10 .4 F x W x D x 0.075 W .8 D
Substitute in for
final
W ,
tot tot
4.0 0.075 10 D .8 D

tot
4.0 10 .075
D 4.483
.8 .075


final
W 5.517


226
9.D8. Figure




9.D8. Part C.
Trial & Error to 3 stages ending at
F
x 0.4 (See figure)

INtercept D
L L
y 0.645 1 x 1 .8
V V

227

L .645
1 .80625
V .8


Initial
L L L L V .15375
.19375, 0.24
V D V L 1 L V 1 .19375



9.D9. a. Mass balances:
total total
F D W or 20 D W

f
F total D W total
F x D x W x or 8 .975 D .28 W
Solving simultaneously:
total
D 3.453 and W 16.547
Can also use Rayleigh equation to obtain same result. (Use of the Rayleigh equation for this
type problem is illustrated in the Solution to Problem 9-D14.)

b. Vapor in equilibrium with
f
W
x must be within the two phase region. x y x
Minimum is when y x . This is
f ,min
W
x 0.21. See graph.
228


9.D10. Op. Eq.,
D
L L
y x 1 x
V V
, Slope, L V L D 1 L D 1 5
Plot on McCabe-Thiele graph for series of
D
x values.
Simpsons rule,
D W
1
x x
at
fin fin
W F W W W f
x x .52, x x .20, and (x x ) / 2 .36

F
F W,fin
W,fin
F W,fin
x
W
x x
x
d W D W D W D W
x x
2
dx 0.52 0.20 1 4 1
x x 6 x x x x x x


0.32
5.95 4 3.70 2.67 1.2491
6

Rayleigh eq.
F
W,fin
x
final W
final
x
d W
W dx
n W Fe
F x x

or
1.2491
final
W 10 e 10 0.28677 2.8677 and
total final
D F W 7.132

final
F final W
D,avg
total
Fx W x 10 0.52 2.8677 0.2
x 0.648
D 7.132


229










x
D
x
W
x
D
-x
W
1/(x
D
-x
W
)
0.70 0.55 0.15 6.666
0.65 0.405 0.245 4.082
0.60 0.25 0.35 2.857
0.50 0.094 0.406 2.463
0.40 0.055 0.345 2.899
0.30 0.035 0.265 3.774
230
W
x
D W
1
x x

F
x = 0.52
5.95
fin
F W
x x
2
= 0.36
3.70
W,fin
x = 0.20
2.67

9D11. New Problem in 3
rd
edition. Eq. (9-17)
butanol Inital
butanol final
x
pot
x
dx
S
W y

Note W water
Equation is in terms of butanol.

1.0
but pot
but .6
dx
S
W y
Simpsons rule need y at
pot
x 1.0, .8, .6 butanol.

but
x
water
x
water
y
but
y
but
1
y

1.0 0 0 1.0 1.0
.8 .2 0.565 0.435 2.2999
.6 .4 0.70 0.30 3.333



.4
3.333 4 2.299 1.0 0.90191
6


S 0.9019 W 1.804 kmol.

More accurate if done in 2 steps. Thus add points below:



but
x
w
x
w
y
but
y
but
1
y

.9 .1 .42 .58 1.7241
.7 .3 0.66 .34 2.9412


1
0.8
.2
2.299 4 1.7241 1 0.33985
6


.8
.6
.2
3.3333 4 2.9412 2.299 0.5799
6


Total Area = .91975. S = 1.8395

231
9.D12.
F
W
final
x
final
x
dx
W F exp -
y x
F
final
W F
final
W W F final final
x
F W
x x
x x x
2
x x
dx 1 1 1
4
y x 6 y x y x y x


final
final
F W
F W avg
x x
x 0.48, x 0.16, x 0.32
2


x

y (from eq. data)

(y-x)
1
y x

0.16 0.36 0.20 5.0
0.32 0.545 0.225 4.444
0.48 0.66 0.18 5.555
F
W
fin
x
x
dx 0.32
5.0 4 4.444 5.555 1.51
y x 6

final
, W 3.0 exp 1.51 0.662 k/moles

tot final
D F W 2.338
,

fin
AVG
F fin W
D
tot
Fx W x 3.0 0.48 0.662 0.16
x 0.571
D 2.338

9.D.13. New Problem for 3
rd
Edition. a) Rayleigh equation:
f
w
final
x
final
x
W dx
n
F y x

x y
(from graph)
y-x
1
y x

0.4 0.7 0.3 3.333
0.24 0.61 0.37 2.7027
0.08 0.37 0.29 3.448


Simpsons Rule

F
w ,fin
x
x
0.4 0.08
3.333 4 2.7027 3.448 0.93826
6


final
W F exp 0.93826 3.1305
total final
D F W 4.8695

F final w,final
D,AVG
total
F x W x
x 0.6057
D

b) Two Liquids. x 0.573 and x 0.975

total total
L L D L D L


total DAVG total total
L x L x D x D L .573 L .975 D .6057


232

total
D .6057 .573
L 0.3963
0.975 0.573

total
L D .3963 4.4732

9.D14. a.
org w
p p 760 ,
org C10 C10
p VP x , and assuming water is pure,
W W
p VP .

C10 C10 W
VP x VP 760
Vapor pressure data for C10 was shown in solution to Problem 8.D10. Guess 99.5C.

W C10
VP 746.52, VP ~ 70.5 mm Hg
7 + 746.5 = 753.5 < 760
At 100C
C10 W
VP ~ 70.5 and VP 760 .
7.05 + 760 = 767.05 > 760
By linear interpolation:
6.5
T 99.5 .5 99.74
13.55
C
b. Use Mass balances. Initially 9 moles n-decane, 1 mole non-volatile
Final: a mol n-decane where
a
1
1 a
; thus,
.1
a .111 mol
.9


final
W 1.111 mol (Water free)

total final
D F W 10 1.111 8.889 kmol

Alternate Solution: Raleigh Eq. with

F
F
W,final
W,final
x
x
D final W
x
W x
dx
x 1: W F exp - F exp n 1-x
1 x


final
W F exp n .1 n .9 .111 F 1.111

total final
D F W 8.889. Same result as mass balance.
c.
tot org org in org
W org
org org in org
p VP x
n D
VP x

Should really calculate numerically from integral for most accuracy

D
tot org org in org
W org
0 org org in org
p VP x
n dn
VP x

However, estimate at final conditions with
ndecane
VP ~ 70.25 (from part a)

W
org
760 70.25 .1 n
107.185
n 70.25 .1

This should be a good estimate.

9.D15. Column is similar to figure in Solution to 9.D13, but with 1 stage in column.

a) For finding
fin tot
W &D dont actually need to step off stages. Just want to make sure
Wfinal
x
is obtainable I checked this at total reflux It works. Use Mass balances:

fin
F tot d fin W
F x D x W x
Substitute in
tot final
F D W
233
Then,
fin
D F
final
D W
F x x 100 0.975 0.48
W
0.975 0.08
x x

Solution is
final
W 55.307 kmol,
tot final
D F W 44.673
b) Need to draw operating lines until:
Feed
initial 2 stages gives x .
W,final
final 2 stages gives x . Then L/V = slope.
Initial There will be a pinch at point reflux is returned.

d int ercept
Initial
D
y x y
0.975 0.41
L V Slope 0.579
x 0 0.975 0

Final: A few trials resulted in final result.

d int ercept
final
d
y x y
0.975 0.17
L V Slope 0.826
x 0 0.975 0




234
9.D16. a. Need L/V so that 3 stages go from
F D
x .4 to x 0.84 . This is line a in Figure (trial-and-
error) was used to find this).

a
a a
a
L V
L .84 .57 L
.3214 and .4737
V .84 0 D 1 L V


b. Need L/V so that 3 stages go from
wfinal D
x 0.08 to x 0.84 (see line b in Figure). Again,
trial-and-error was used to find line.

b
b b
b
L V
L .84 .13 L
.8452 and 5.4615
V .84 0 D 1 L V

c.
total final
F D W
total final
10 D W

feed total D final wfinal
F x D x W x
total final
4 .84 D .08 W
Solving simultaneously,
final
W = 5.789 kmol and
total
D 4.211kmol.
The Rayleigh equation could be used, is not needed, but gives the same result.

9.D17.
init
x 1.0 . Start 1 kmol and keep 1 kmol. Add water as boil.
Eq. (9-17),
W,initial
W,final
x
tan k
x
dx S
W y
Note balance is on original solvent, methanol.
Use equilibrium data from Table 2-7. Generate table of methanol mole fractions:
x y 1/y
1.0 1.0 1.0
0.611 0.830 1.20489
0.222 .588 1.6722
0.11611 .450 2.222
0.01 0.07 14.9254


Use Simpsons rule in two steps. Step 1 (x from 1.0 0.222)
235

0.778
1.0 4 1.20489 1.6722 0.97143.
6

Step 2 (x from 0.222 0.01)

0.216
1.6722 4 2.222 14.9254 0.9005.
6

Total = 1.87194 = S/W with W = 1.
9.D.18. New Problem 3
rd
Edition Eq. (9-14)
D,final
F
y
final y
F dy
n
D y x



D,final
F
y
final y
F dy
exp
D y x


D,final
F
final
y
y
F
D
dy
exp
y x

Read x values from equilibrium diagram or interpolate from Table 2-1.
y x y-x
1
y x

0.1 0.008 0.092 10.87
0.3 0.045 .255 3.92
0.5 0.155 .345 2.90


0.5 0.1
Area 10.87 4 3.92 2.90 1.963
6


final
0.5
D exp 0.0702 kmol
1.963


total final
C F D 0.4298 kmol
Ethanol MB:

final D,final total C,avg F
D y C x F y


F final D,final
C,avg
total
F y D y 0.5 0.1 0.0702 0.5
x 0.00346 mole frac ethanol
C 0.4298



9.D19. New problem in 3
rd
edition.
w w oct oct tot
VP x VP x P , or in mm Hg, 526.123 1.0 10.964 .6 532.7

9.D20. Was 9.D18 in 2
nd
edition.

F,C5 W,final,C5 C5,AVG C8,AVG
x 0.35 & x 0.05: x 0.20, x 0.80, p 101.3 kPa.
236
B.P.
i i i
y K x 1.0 . For average mole fractions, the BP calculation converges to T =
84 with
C5
K 3.7 and
C8
K 0.30 from the DePriester charts. The
i
y 0.98 which is
close enough to estimate .

C5
C5 C8
C8
K 3.7
12.33
K 0.30

Eq. (9-13),
f
0.05 0.65 W 1 0.65
n n n 0.5847
F 11.33 0.35 0.95 0.95


fin
W F 0.5573,
fin
W 0.5573 1.5 0.8359 kmol

total fin
D F W 1.5 0.8359 0.6641 kmol

F fin W,fin
D,Avg
total
Fx W x 1.5 .35 0.8359 0.05
x 0.7276
D 0.6641

9.D.21. New Problem for 3
rd
Edition.
L D
L D 1.0 L V 1 2
1 L D
D D
L L
y x 1 x , but x varies.
V V
Thus, plot series operating lines of arbitrary
D
x & slope 1 2. With a total of 21 equilibrium contacts there will be a pinch where the operating line
intersects the equilibrium curve. This intersection is
W
x for this
D
x value.
D
x
W
x
D W
x x
D w
1
x x

.7 .067 .633 1.5798
.6 .05 .55 1.818
.5 .038 .462 2.1645
.4 .027 .373 2.6801
.3 .018 .282 3.5461
.2 .01 .19 5.26316

Want to integrate from
W,final
x 0.02 to
F
x 0.06 and want middle point at
W
x 0.04 .
Plot
D W
1
x x
vs
W
x and find values.

W,final W,Avg
D W D W
1 1
x 0.02, 3.23; x 0.04, 2.1
x x x x


F
D W
1
x 0.06, 1.65
x x


0.06 0.02
Area 3.23 4 2.1 1.65 0.08853
6

237

final
W F exp Area 2.5 exp 0.08853 2.288


total final
D F W 0.2118
F final W,final
D,Avg
total
F x W x
x 0.492
D

238
9.D.22. New Problem for 3
rd
Edition.
tot
OP
R
D
t
Q


t
operating
= __1.49 to 1.50 h
9.D23. Prelim. Calc. Feed; Avg MW
E water
0.1 MW 0.9 MW

avg
MW 0.1 46 0.9 18 20.80 kg/mol

1000 kg
48.0769
20.8 kg mol
kmol

L L D 2 3
2 5 0.4
V 1 L D 5 3

All op. lines have slope 0.4.
Can draw op. line to
W
x . Ten stages will go from
D W
x to x because have large number
of stages. Thus, do not need to step off stages.



239


From Graph can create table of 1/(x
d
1/x
w
) versus x
W
.











D
x
W
x
d w
x x
d w
1
x x

0.665 0.10 0.565 1.770
0.630 0.08 0.550 1.8182
0.499
0.052
Avg
x
0.447 2.237
0.440 0.040 0.400 2.500
0.278 0.020 0.258 3.876
0.140 0.010 0.130 7.692
0.057 0.004 0.053 18.868
240

Simpsons Rule:
F W,final W,Avg
x 0.1, x 0.004, x 0.104 2 0.052

F
fin
W
W AVG F fin
fin
x
F W
W
d W d W d W d W x
x x x
x x
dx 1 1 1
4
x x 6 x x x x x x


0.096
18.868 4 2.237 1.770 0.4734
6


F
W
fin
x
Final W
final
d W x
W dx
exp 0.62289; W 48.0769 0.62289 29.9466
F x x


total Final
D F W 48.0769 29.9466 18.1303 kmole

fin
AVG
F Final W
D
total
Fx W x 4.80769 29.9466 0.004
x 0.2586
D 18.1303



































241
9.D24. Was 9D22 in 2
nd
edition.
a)
fin
total fin F total D W
F D W and Fx =D x Wx

fin
fin fin
fin
F W
F total D W W total
D W
x x
Fx D x x +Fx D F
x x


total
0.62 0.45
D 3.0 1.275
0.85 0.45
kmol,
fin total
W F D 1.725 kmol
b) Want operating line where 2 equil. contacts gives
fin
w D
L L
x 0.45 . y x 1 x
V V
.
Surprisingly, with 2 contacts T & E not needed. Start stepping off stages from top & from
bottom simultaneously. The intersection point must be on op. line as is
D
y x x .

L 0.85 0.44 L L L V
Slope 0.482, 0.932
V 0.85 0 D V L 1 L V



Figure for 9.D24.


9.D25. New Problem in 3
rd
edition.

Mix together
1 2
F F F 2.5
242

1 2
F 1 F 2 F
x Fx F x 2.5 0.8 2.5 0.32

F
FIN
final
x
F w
FINAL
AVG
xw
x x
w dx
n x 0.21
F y x 2


F
FIN
w w,FIN w,AVG w,F Final
x
F
x x x x
x x
1 4 1
6 y x y x y x


x y
1
y x

0.1 0.4 3.333
0.21 .585 2.666
0.32 .675 2.817


.22
3.333 4 2.666 2.817 0.6166
6


area
final
W Fe 2.5 0.5398 1.349 kmol

FIN
F FIN w
final DAVG
Fx W x
D F W 1.151, x 0.578
D

Part b) First go from 0.4 to 0.2 with
1 1
F D

Then from 0.2 to 0.1 with
FIN1 2
F W F












Now

FIN1 2 F
F W F 0.865 1.0 1.865, x 0.2


0.1
3.3333 4 2.7777 2.666 0.285185
6

area
final2
W F e 1.865 0.7518 1.4022


2 final2
D F W 1.865 1.4022 0.46275
Total
total
D .635 .46275 1.09775
Total
fin
tot tot fin2 W
D,AVG
total
F x W x
x
D
2.5 .32 1.4022 .1
1.09775
0.6010
Higher distillate mole fraction.
x y
1
y x

0.2 0.575 2.666
0.3 0.67 2.703
0.4 0.73 3.03

x y
1
y x

0.1 0.4 3.3333
0.15 0.51 2.7777
0.2 0.575 2.6666

Values are from methanol-water
equilibrium data.
.2
2.666 4 2.703 3.030 0.55026
6

1
-area
FINAL 1
W = Fe = 1.5 .5769 = .865

1 1 Final,1 D,AVG,1
1.5 .4 .865 .2
D F W 0.635, x
0.635

243
Part c) Go from 0.4 to 0.1 for
1
F

0.3
3.3333 4 2.7027 3.030 0.8587
6

final
F1
W 1.5 exp .8587 0.6356

For
2
F go from 0.2 to 0.1. Same as 2
nd
part of Part b. 0.28518

area
final 2
F2
W F e 1.0 .75187 0.75187

final total final fin
F1 F2
W 1.3874 W W ,
total 1 2 final tot
D F F W 1.11255
Differs from b Numerical error!

tot
DAVG
F .32 1.3874 .1
x 0.59436
1.11255

Should be same as part b. There are numerical errors in use of Simpsons rule.

More accurate for .4 to .1 is .4 .2 (Area = .55026) + .2 .1 (Area = .285185)
Total Area = 0.835445,
1
area
final 1
W F e 1.5 .433681 0.65052
Then
fin total
W 1.40239 ,
total
D 2.5 1.40239 1.09761

AVG
D
2.5 .32 1.40239 .1
x 0.60109
1.09761

Same as for Part b.

9.D.26. New Problem for 3
rd
Edition. a&b)
L D 2 3
L D 2 3, L V 0.4
1 L D 5 3


final final
L D 4, L V 4 5 0.8
For Part b, draw op. lines with slope 0.4 for arbitrary
D
x 0.7 . Step off 2 stages.
For Part a, From
D
x 0.7 draw op. line slope = 0.8. Two stages gives

final
w
x 0.11 (See graph (labeled 9.D.c))
Also, draw a few lines with
D
x 0.7 and L Vbetween 0.4 and 0.8. Find
w
x values with
2 stages.
b) Generate Table
D w D w
x , x , 1 x x
Plot
w
D w
1
vs x
x x
. Note there is a break in curve at
D w
x 0.7 x 0.185 to 0.110
Find 2 areas
final
F w
w w
1. from x 0.6 to x 0.185
2. from x 0.185to x 0.110

Area 1. Simpsons rule.
avg
w
0.6 0.185
x 0.3925
2
.
x y
0.1 0.4 3.3333
0.25 0.62 2.7027
0.4 0.73 3.030

1
y x
244
From graph
w
avg
D w
x
1
2.37
x x
.
Area 1
0.6 0.185
3.56 4 2.37 1.942 1.0363
6

Area 2.
D w
1
x x
curve is straight line. Thus, Area = width x Avg height

1.6949 1.942
0.185 0.110 0.13638
2

Total area = 1.1726.
Rayleigh eqn.,
F
w
final
x
final w
x
D w
W dx
n 1.1726
F x x


1.1726
final
W 100 e 3.0955

total final
D F W 6.9045
c)
final
F final W
D,AVG
total
Fx W x 6.0 3.0955 0.11
x 0.8197
D 6.9045

245



L/D
D
x

w
x

D w
1
x x

2/3 0.9 0.65 4.0
2/3 .85 0.48 2.703
2/3 0.8 0.36 2.273
2/3 0.75 0.253 2.012
2/3 0.7 0.185 1.942
-- 0.7 0.145 1.8018
4.0 0.7 0.110 1.6949

w,final
x


246


9.D.25. New Problem for 3
rd
Edition. From the methanol-water equilibrium data, the following table can
be obtained.

247

M
x
M
y
M
1 y
1.0 1.0 1.0
.8 .92 1.08696
.6 .825 1.21212

pot ,Initial
pot ,final
x
MeOH
MeOH x
dx S
W y

Simpsons rule:
.4
1.0 4 1.08696 1.2121 0.4373
6

S 0.4373W 0.8747
kmol

9.E1.


10 W W
2164.42
log VP 8.68105 , T C, VP mmHg
273.16 T

a) Final conditions
oct , in org W, in W
x 0.3103, x 1.0, T 99.782 C from solution to
problem 8.D11. Initial conditions:
oct,in org W in W
x 0.90, x 1.0

i i
x VP 1.0 atm 760 mm Hg
From spread sheet find T = 99.377C
b)
fin
final,org
W
1 z 1.0 0.9
W F 1.0 0.1450
1 x 1 0.3103

c)
org final org
D F W 0.8550 n
d) Eq. (9-24)
org
D
tot oct
oct
W oct
oct 0
oct
p VP x
n dn
VP x

Estimate
oct
VP at average T (99.782 99.377) / 2 99.5795 ,
octanol
VP 18.87 mm Hg

org org org
D D D
tot oct tot oct
W org org
oct oct oct oct 0 0 0
p dn p dn
n dn D
VP x VP x

water
octanol
pot
steam
F
x 0.90, F=1.0 kmole
Final octanol in pot 1 0.95 .9 0.045 kmol
Nonvolatiles in pot 1.0 0.1 0.1
oct ,W,final
x 0.045 0.145 0.3103
248

org org feed
F
F 1.0, n F W, x 1 1 x 1 0.1 F W
W

Step-by-Step integration,
org
org
org org
dn
1
dn
x x

org
n
org
dn

W
org
x

org
Avg x
oct
org,avg
dn

x

0 1.0 .9
> 0.8944
0.1 .1 .9 .8888 0.11181
> 0.8819
0.2 .1 .8 0.875 0.11339
> 0.8661
0.3 .1 .7 0.8571 0.11546
> 0.8452
0.4 .1 .6 0.8333 0.1183
> 0.81666
0.5 .1 .5 0.8000 0.12245
> 0.775
0.6 .1 .4 0.750 0.12903
> 0.7083
0.7 .1 .3 0.6666 0.14118
> 0.58333
0.8 .1 .2 0.50 0.17143
0.855 0.055 0.145 0.3103 > 0.40517
0.1357
1.1588

P
org
org
tot
W org
org org O
dn
p
n D
VP x
760
1.15882 0.8550 45.826
18.8666
kmol

e. Continuous had 108.93 kmoles water/kmole organic fed. The continuous always operates at
lowest octanol mole fraction in liquid & thus
oct
y is always at lowest value. Thus, requires
more water to carry over octanol then the batch operation.

9.E2. New Problem in 3
rd
edition. Parts a & b. See solution to problem 8.D25.

benz
mol B 78.11
x
mol B 78.11 80 673.2

c. Find T from Eq. 8-15 With Spread Sheet:

SPE, Problem 9E5. Solution for temperature
T deg C 92.04234 Do step by step Antoine VP values
VPW A,B,C 8.68105 2164.42 273.16 2.754418 568.0906
VPben A,B,C 6.90565 1211.033 220.79 3.034461 1082.583
X ben 0.17727 xw 1 ptot 760
Eq8 -5E-05 Goal seek B6 to zero changing B2
massbeninit 20 massbenfin 2
249
dmorg-sum 18 massben-still 2 dmassorg 1
dnw/dnorg 9-23 2.960203 dnw 0.037898

d) Step-by-step integration.
W org
dn dn Eq. 9-23 Use Spread Sheet for each time
step. Did the addition of steps off-line in table below.

still init
still final
W W org benz
n dn for n 20 78.11 n 2 78.11
Set
org
dm 1 kg.

Values from spreadsheet.

org
dm kg
Mass benz still,
777 (kg)
W
dn divide all
values by 78.11
kmol
TC
from spreadsheet

Initial T 0 1 20 19 .664431 75.99
1 2 19 18 .677266 76.27
2 3 18 17 .691548 76.577
3 17 .707537 76.91
4 16 .725558 77.269
5 15 .746073 77.66
6 14 .769462 78.0996
7 13 .79657 78.58
8 12 .828276 79.12
9 11 .865852 79.715
10 10 .91108 80.39
11 9 .966543 81.156
12 8 1.036129 82.03398
13 7 1.12596 83.0487
14 6 1.246275 84.2358
15 5 1.415548 85.6435
16 4 1.670846 87.339
17 18 3 2 2.098857 89.422
final T 18 2 2.960203 92.04

d) For accuracy add time steps 1 to 17 + (time steps 0 & 18)/2. This is then identical to the use of
average values of dm for every step.
Steps_1 to 17 17.2794168


0.5 Steps _ 0 18 1.8122565


W
n 19.0916733 78.11 18.016 4.4034 kg water

Compared to continuous, n
w
= 12.3, batch requires less because benzene mole fraction is higher
in batch operation for most of batch. If we take final cut and used that value for entire operation,

W
18.016
dn 18
78.011
kg benzene overhead 12.3 same as continuous.
e) To vaporize
ben ben w
(n ) / varies since T is different.
For accuracy, do for each
org
dm and find values at these T.
9.E3. Simpsons Rule:
d,fin
F
d d D
x
d
d B d B d B d B x
x .63 x 0.565 x 0.50
dx 0.13 1 4 1
x x 6 x x x x x x

250
Generate table from graph

0.13
3.0303 4 2.15 2.004 0.2954
6


0.2954
final
D Fe 1 e 7.442 gmole
final
B F D 2.558 mole.

F D,fin
B,avg
F x Dx
5.0 4.6886
x 0.1218
B 2.558




9.E4.
init
W,fin
x
final
x
dx
W F exp
y x

T & E since dilution effects F &
init
x .
Dilute with 5 kg water Start with 6,
init
x 1 6 0.1667
Dilute with 4 kg water Start 5,
init
x 1 5 0.2
Dilute with 3.5 Start 4.5,
init
x 1 4.5 0.2222
Dilute with 2.75 Start with 3.75,
init
x 1 3.75 0.26667
Dilute with 1.75 Start with 2.75,
init
x 1 2.75 0.364
B
x
d
x
d B
1
x x

0.3 0.36 3.0303
0.2 0.597 2.5189
0.1 0.565 2.15
0.04 0.500 2.004
251
For each dilution want to integrate using Simpsons rule until find
final
W 1.0. Thus,
need values of 1 y x at
init avg final
x , x , and x 0.01 for each dilution. These values
are determined in the following table.

Table of Values for Integrations

Dilute x y y-x 1/(y x)
5 kg
Water
0.16667
0.08833
0.01
0.537671
0.38708
0.067
0.3710
0.29875
0.057
2.695412
3.34724
17.544
4 kg
Water
0.200
0.105
0.01
0.579
0.428
0.067
0.379
0.3230
0.057
2.63
3.095975
17.544
3.5 kg
Water
0.2222
0.11611
0.01
0.598
0.450
0.067
0.376
0.334
0.057
2.6596
2.994
17.544
1.75 kg
Water
0.364
0.1870
0.01
0.706
0.563
0.067
0.342
0.376
0.057
2.923977
2.6596
17.544
2.75 kg
Water
0.2666
0.138
0.01
0.636
0.493
0.067
0.369
0.355
0.057
2.7100
2.8169
17.544

Integrations: Use Simpsons rule for each addition.
Dilute 5 kg:
0.15667
2.695412 4 3.34724 17.544 0.87809
6


final
W 6 exp -0.87809 2.4934. Value is too high. Want 1.0 kg.
Dilute 4 kg:
0.19
32.5579 1.031
6


fin
W 5 exp -1.031 1.783 too high
Dilute 3.5:
0.2122
32.1796 1.138
6


fin
W 4.5 exp -1.138 1.442 too high
Dilute 1.75:
.354
31.106 1.833
6


fin
W 2.75 exp -1.835 0.4389 too low
Dilute 2.75:
.2566
31.52 1.348
6


fin
W 3.75 exp -1.348 0.9740
Close to desired 1.0 kg. Thus, 2.75 kg water. The final still pot is 99% water so have (.99)
(.974) = 0.964 moles water remaining. Moles of water distilled off is 2.75 0.964 = 1.786.
9.H.1. New Problem in 3
rd
edition. This problem is challenging for students because they must first
derive the forms of the equations they need to use.
252
A. Define. The system is the simple still pot shown in Figure 9-1. Find W
final
, D, x
A,Wfinal
, and x
A,dist,avg
.
B. Explore. At first it may appear that the problem in Part a is under specified since there are now five
unknowns. However, in specifying the problem based on the fractional recovery of benzene in
the distillate we have added the equation for the definition of fractional recovery of A in the
distillate. This equation is most conveniently written as,

Fz
A
(1 Frac. Rec. A in distillate) = W
final
x
A,Wfinal
(9-35a)

which becomes, W
final
= Fz
A
(1 Frac. Rec. A in distillate)/ x
A,Wfinal
(9-35b)

C. Plan. If we write Eq. (9-13) for A and substitute in Eq. (9-35b) we obtain Eq. (9-36),

A, W,final A,F A,F
A
AB A, W,final A,F A, W,final A, W,final
x 1 x 1 x
z 1 Frac. Rec. A.dist
1
0 n n n
1 x x 1 x 1 x
Part a. In a spreadsheet Eq. (9-36) is easily solved for x
A,Wfinal
using Goal Seek. Then W
final
can
be determined from Eq. (9-35b). Then D
Total
is determined from Eq. (9-11) and x
A,dist,avg
is
determined from Eq. (9-10) written for component A or from the fractional recovery.

Part b. Now solve Eq. (9-36) for frac. rec. of A in distillate using Goal Seek. For both parts a and b can
use fractional recovery values and D
Total
to find x
A,dist,avg
= Fz
A
(Frac Rec. A in distillate)/ D
Total


D. Do It. Because Eq. (9-36) for x
A,Wfinal
is nonlinear, it is easiest to solve this problem with a spreadsheet
and use Goal Seek to solve Eq. (9-36). The spreadsheets are shown below.
Part a
F 5 zA 0.37 The 0.37 is in cell D2
alpha AC 10.71
frac rec A in distillate 0.75
xA,Wfin 0.143185
9-36 term 1 -1.25687 term 2 -0.3075 term 3 0.436926

Eq 9-36 -1.7E-05 Use Goal seek
Wfinal from 9-35b 3.230095 D total 1.769905 xAdist,avg 0.78394
Part a. Use Goal Seek for cell B8, setting it equal to zero by varying cell B5 (xA,Wfin).
Part b. Use Goal Seek for cell B8, setting it equal to zero by varying cell C4.

Part b
F 5 zA 0.37 The 0.37 is in cell D2
alpha AC 10.71
frac rec A in distillate 0.930081
xA,Wfin 0.05
9-36 term 1 -2.41222 term 2 -0.41074 term 3 0.658942

Eq 9-36 -0.00023 Use Goal seek
Wfinal from 9-35b 2.586992 D total 2.413008 xAdist,avg 0.713073
Part a. Use Goal Seek for cell B8, setting it equal to zero by varying cell B5.
Part b. Use Goal Seek for cell B8, setting it equal to zero by varying cell C4 (frac rec A in distillate).

253
SPE 3
rd
Edition Solution Manual Chapter 10
New Problems and new solutions are listed as new immediately after the solution number. These new
problems are: 10A4, 10A5, 10A7, 10A12, 10A17, 10C4, 10C5, 10D13, 10D19, 10D21, 10G1-
10G4.
10.A9. A good packing will have: good contact between liquid and vapor, high surface area, low
pressure drop, inert, inexpensive, and self-wetting.
Marbles have low surface area, poor contact and relatively high p.
10.A13. Figure 10-25 shows that if viscosity increases the ordinate increases and p/foot increases.
10.A14. a.Want low F since this gives low p.
b. F decreases as size increases.
c. Ceramics have much thicker walls than metal or plastic. Ceramics are used in corrosive
environments.
10.A16. 1. a. fewer; 2.b. larger; 3. a. lower
10.A.17. Answer is c.
10.B1. The trick is to have maximum and minimum positions of the valve with a larger area for
vapor flow at the maximum position.
a. Use a cage to prevent excess movement.
b. Use feet.
c. Have a flap that moves up and down.
d. Use a spring to provide force and maximum position.
e. Sliding valve controlled by an external feedback mechanism.
f. Two flaps to make a roof.
Many other ideas can be generated.
10.B3. Some possible candidates:
Bottle caps SOS pads
Bent bottle caps Scotch brite pads
Natural sponge Steel wool
Synthetic sponge Cooper cleaning pads
Miscellaneous junk Plastic coated wire
Broken crockery or glassware Tin foil - crushed
Cut up tubing (Tygon) String - balls
Glass tubing (broken) lines tied together
Crushed beer cans lines twisted
Cut up - crushed beer cans lines stretched taught
Coal Rope
Egg shells frayed rope
Styrofoam packing material Rope tied into bow ties
Old seat cushions Porous rock pieces
254

Nuts/bolts/screws/nails Pumice
Metal filings Ash from Mt. St. Helens
Wood shavings Ashes from coal stove
Kindling Pieces of cement block
Left-over redwood Pieces of brick
Staples Pop-tops
Window screens, rolled up Old watch bands - twisted
Chicken or barbed wire Bent wire coat hangers
Christmas Tree Ornaments Cookie cutters
Corn cobs Combination of the above

10.C1. L/V constant since L/D const. Thus L/G const. In
v
F only
G
changes. From Perfect Gas
Law
G v v
n P
MW MW
v RT
. Thus,
v
F increases as P . However, curve is almost
flat in this range and
flood
s,b
C ~ constant.
Then
flood
.5
.5
sb
L G
flood L G 0.2
G
C
1
u since
P
20 /
.

D
D B
x z L
V L D 1 D and D F so V F
D x x

Thus,
V flood
F F
Dia
1
u
P
P

F

P

Then, F
2
Dia P , and exponent =

10.C2. See derivation in solution to Problem 10.C1.

D
D B
x z L L
V 1 D 1 F
D D x x

Then from Eq. (10-16)

1/ 2
D
D B
flood
x z L
4F 1
D x x
Dia
3600 p fraction u

Since
1/ 2
1/ 2
Dia F , Dia L D 1
10.C3. Plot points based on p and
1/ 2
G
L
L
G
on Figure 10-25. From ordinate values calculate F
for each point. Use an average value.

255

10.C4. New Problem in 3
rd
edition. Part of the operating lines become closer to the equilibrium curve.
Thus, for the same separation more stages are needed. Fortunately, this effect is often small.

10.C5. New Problem in 3
rd
edition. You can show this by proving that the minimum reflux ratio (Figure
10-18A) or the minimum boilup ratio (Figure 10-19A) must increase compared to the base
cases. These ratios increase because cooling the entire feed (Figure 10-18A) or heating the
entire feed (Figure 10-19A) automatically changes the feed line and moves the minimum
reflux (Figure 10-18A) or minimum boilup (Figure 10-19A) operating lines towards the y = x
line. This means larger minimum external reflux ratio or larger minimum boilup ratio.

10.D1.
C6 C7
K y / x and K 1 y 1 x
Solve simultaneously.
C7
C6
C6 C7
1 K
x , y K x
K K

Pick T and generate equilibrium curve.
C6 C7
Bounds are K 1.0 and K 1.0 .
T K
C6
K
C7
x y
149
o
C 1.0 - 1.0 1.0
169 1.3 .72 .483 .63
171 1.34 .75 .424 0.568
193 - 1.0 0 0

Average temperature is T = 171C = 444.1 K.

y x .568 .424
1.786
1 y 1 x .432 .576

Viscosity equation and terms are given in Example 10-1.

10 C6 C6
1 1
log 362.79 .935, 0.16
T 207.09
.

10 C7 C7
1 1
log 436.73 .895, 0.127
T 232.53
.

MIX MIX
n .5 n .116 .5 n .127 2.107, 0.122.
From Eq. (10-6) for
o
0.217, E 0.730 . The higher pressure results in higher
temperatures and lower viscosities. This increases the predicted column efficiency by 24%.

10.D2. T 98.4 273.1 371.5 K (almost pure n-heptane at bottom).

3
v
1 100.2
pMW
0.205 lb/ft
RT 1.314 371.5
,
3
L
0.684 62.4 42.68 lb/ft , 20
Need L V. First, find y at intersection of operating lines.
Top Operating Line Slope
L .999 y
.8
V .999 .5

y .999 .8 .499 0.5998
Then,
L 0.5998 0.001
1.20
V 0.5 0.001

256


L
v
MW L L L
1.2
G V MW V
. This is at bottom where
L v
MW ~ MW .

0.5
0.5
v
v
L
L .205
F 1.2 0.08317
G 42.68

From Figure 10-16,
0.2 0.2
sb sb
12.5
C 0.32, C 0.32 0.29
20 20


0.5
flood
42.68 0.205
u 0.29 4.19
0.205

Saturated liquid feed V V 2500. Use = .90

1/ 2
4 2500 1.314 371.5
D 12.35 ft
.90 3600 1 .75 4.19

Somewhat larger. Would design at bottom of column. Use a 12 ft. diameter column.

10.D3. New 12 dia col. First, redo entrainment calculation.

0.5
g
L 0.205
1.2, F 1.2 0.0832
V 42.68
, L 1.2 V 3000
For Fig. 10-17 need % flood. In problem 12.D2 designed for 75% flood.

1/ 2
const
D 11.78
75% flood
,
1/ 2
1/ 2
const 11.78 .75 10.20
Use 12 feet:
1/ 2
% flood 10.202/12.0. % flood 72.3%. Then Fig. 10-17, = 0.022.

0.022 3000
L
e 67.48
1 1 .022
, L e 3067.48
This is reasonable amount.

2
2
total
A 12.0 4 113.1 ft

2
d
A 1 0.9 113.1 11.3 ft
Table 10.2,
weir
0.9, Dia 0.726 ,
weir
0.726 12 ft 8.71 ft

2
active
A 113.1 1 .2 90.48 ft ,
2
hole
A 0.1 90.48 9.05 ft
14 gauge,
tray 0 tray
t 0.078 in, 3 16 holes, d t 2.4

v
0
v
VMW
v 37.51 ft s
3600 9.05
,
0
C 0.759 (unchanged from Ex. 10-3)

2
p,d ry 2
59.87 1 0.01
h 0.003 37.51 0.205 2.086 in
42.68
0.759


g
100.2 7.48
L 3067.48 897.8 gpm
42.68 60

257

Abscissa
g
2.5 2.5
weir
L
897.8
4.01
8.71

Parameter
w weir
Dia 0.726, F Fig.10 20 1.03
Eq. (10-26),
2/ 3
crest
897.8
h 0.092 1.03 2.083 in
8.71

With 1 gap,
2
du
A 8.71 12 0.726 ft
Eq. (12-27),
2
du
897.8
h 0.56 4.248 in
449 0.726


dc
h 2.086 2 2.083 0 4.248 10.417

dc,aereated
h 10.417 0.5 20.83 OK, but close to distance between trays.

res
11.3 10.417 3600 42.68
t =4.904
3067.48 100.2 12
This is OK.
Weeping, Eq. (10-32),
0.040 12.5
h 0.0625 in
42.68 3 16

Eq. (10-31), LHS
p,dry
h h 2.086 0.0625 2.148
x 2 2.083 0 4.083

2
RHS 0.10392 0.25119 4.083 0.021675 4.083 0.7682, OK
LHS > RHS
Operation is slightly marginal with high downcomer backup.
Increase apron gap to 1.5 inches:
2
du
1.5
A 8.71 ft 1.089 ft
12


1.5 gap
2
du
897.8
h 0.56 1.888 in
449 1.089


dc
h 2.086 2 2.083 1.888 8.05 in

dc,areated
h 16.11 inch, OK

10.D4.
1/ 2
v valve
o,bal
v v v
C W 2g
v
K A
(10-36)
Given
2
valve v,closed v v,open
W 0.08 lb, g 32.3 ft s , K 33C 1.25, K 5.5
Pressure drop in terms of inches of liquid of density
L
:

2 valve v v
p,valve v 0
v L L
W K
h C v
A 2g


2
v
1
A 0.02182 ft
12
,
3 3
v L
0.1917 lb ft , 41.12 lb ft
258


1/ 2
o,bal,closed
1.25 .08 2 32.2
v 6.83 ft sec
33 0.2182 .1917


1/ 2
o,bal,open
1.25 .08 2 32.2
v 16.73 ft sec
5.5 0.218 .1917

At balance point,
v valve
p, valve
3 3
L L
1.25 0.8 lb C W
h
A 0.2182 ft 41.14 lb ft

0.1115 ft 1.338 in liquid
closed:
2
p,valve 0 0
33 .1917
h v for v 6.83
2 32.2 41.12


3 2 2 2
0 0
2.39 10 v ft 2.87 10 v inches
open:
2
p,valve 0 0
5.5 .1917
h v for v 16.73
2 32.2 41.12


4 2 3 2
0 0
3.98 10 v ft 4.78 10 v inches

10.D5. Do calculation at total reflux. From a McCabe-Thiele diagram (not shown). Total #
Contacts = 4.2 N = Total 1 (P.R.) = 3.2

Length 1
HETP 0.31 m
N 3.2


10.D6. From Fenske eq. and definition of HETP

dist bot
AB
z
HETP
x x
n
1 x 1 x
n


dist bot
x x .987 .008
n n 9.150
1 x 1 x .013 .992


z = 3.5 meters. Obtain:
a. = 2.315, HETP = 0.321
b. = 2.61, HETP = 0.367
c.
1/ 2
AVG AVG
2.315 2.61 2.4581 and HETP 0.344
Can also use McCabe-Thiele diagrams although the solution shown is easier.

10.D7. Current:
v nf nf
F 0.090, 12 spacing, Ordinate 0.2 U const., const. 0.2 U 0.2 6.0
New:

new v,new v,old
old new old
L V 1.11 L V , then L G 1.11 L G , F 1.11F 0.0999
Trays Spacing 24, Ordinate ~ 0.32, Ordinate =
nf
U Const.
259


Nf
Ordinate 0.32
U 9.6 ft s
Const 0.2 6


10.D8. At
v sb,f 1
F 0.5, C 0.12

0.2 0.2
L G L G
f 1 sb 0.2
G G G
L
6.0 6.0 1
U C
20 0.12 20
0.12
20

Since
v
G v
MW p n
pV nRT, MW
V RT

New Condition:
0.5
0.5
G,new
v sb
G,old
1
F 0.5 0.5 0.25; C ~ 0.18
4


f 1,new sb,new G,old sb,new old
f 1,old sb,old G,new sb,old new
U C C
0.18 3
4 2 3
U C 0.12 2 C


fl,new
U 3 6 18 ft/s

10.D9. Mass Balance:
B
D B
z x D 0.6 0.01
0.59656
F x x 0.999 0.01

D 596.56 kmol/h, B 403.44
At top: L V 0.6, V L D V D 0.6V

D 596.56
V 1491.47 kmol h
0.4 0.4

L V D 491.41 kmol h
At top of col.
L V
W W L V since pure MeOH, same mol. wt. 0.6

kmol 32.04 kg MeOH 2.046 lbm lbm
V 1491.41 105, 346
h kmol kg h

Assume ideal gas. Top of column is essentially pure MeOH.

M
v M 3 3
lb
1 atm 32.04
n MW
p lbm lbmol
MW 0.07219
V RT ft ft atm
0.7302 607.79 R
lbmol R

where pure MeOH boils at
1.8 R
64.5 C 337.66K 607.79 R
K


3
3
L 3 3
1 kg 2.2046 lbm 28317 cm lbm
MeOH 0.7914 g cm 49.405
1000g kg ft ft

At top 24 0.0773 64.5 19.0
260


1/ 2
1/ 2
G L
v
G L
W 0.07219
F .6 0.02294
W 49.405

Fig 10-16 with 18 tray spacing:
sb
C 0.28

0.2 0.2
L V
flood sb
V
19 49.405 0.07219
U C 0.28 7.25 ft s
20 20 0.07219

Use Eq. 10-14 (Modified),
V flood
4V lbm h
D
frac u 3600


3
4 105, 346 lbm h
D
ft
0.90 0.07219 lbm ft 0.75 7.25 3600 s h
s

D = 10.27ft Use either 10 ft (slightly higher frac flood) or 11 ft (lower frac. flood).

10.D10. Ref. Bonilla (1993).
feed
x
3.0, y= , at z 0.4 x, y 2 3
1+ -1 x


min
.9 0.66667
L V 0.46667
.9 .4
,
min
L L V 0.46667
L D 0.875
V L 1 L V 1 .46667


act min
L D 2 L D 1.75 ,
act
L D 1.75
L V 0.636
1 L D 2.75


D
1 L V x 0.3273
At Minimum (Pinch Point),
low
min
L V L V or
low
min
L D L D 0.875

low
L 0.875D,
avg
L 1.75D,
low avg
L 0.5 L
Generalize
low avg
actual min
L L M where L D M L D



10.D11. a. Since Liquid & Vapor have the same mole fractions L G L V
pV = nRT,
G v v
n MW V p MW RT ,
3 -1 o 1
R 45.6 cm atm gmoles R
261


v
MW .8 46 .2 18 40.4
3 1 1 3
G
1 ATM 40.4 g mol 45.6 cm atm mol R 176 460 R .001393 g cm
V L D 3D 6750 lb day, L 2D 2 2250 4500 lb day
L V 4500 6750 .6667 L G

1/2
1/ 2
3
G L
L G .6667 1.393 10 .82 =0.0275

2
3 3
H O L
1 g cm .82 g cm 1 .82
F = 97 (Table 10-3)
Ordinate
2 .2
F G L c
G F g .197 , from Figure 10-25 (flooding line).
This is,

2
3
0.2
2
F 3
3 3 3
1 1 g cm
G 97 .52
.82 62.4 lb ft
.197
1.393 10 g cm .82 g cm 32.2


2 4 2
F
G 0.5216 lb (s ft ) 8.64 10 s day 45067 lb (day ft ) ,
F
G .75 G

FIND AREA AND DIAMETER FOR 75% OF FLOODING

2 2
AREA V .75 G 6750 lb Day 45067 lb Day ft .75 .19970 ft

2 2
D 4 .19970 ft , D .5042 ft 6.05 in

COLUMN DIAMETER
6.05 5 8 9.68
PACKING DIAMETER
which is probably OK.

b. From Fig. 10-25,
1/2
2 0.2
G L c G L
G F ( g ) .036 at L G .0275

0.2 2
2
3
1
G 97 .52 1 62.4
.82
0.036
1.393 10 .82 32.2


2 2 4 4
G 0.2230 lb (s ft ) 8.64 10 s day 1.927 10 lb (day ft )

2 2
AREA 6750 lb day 19267lb (day ft ) 0.3503 ft

2
AREA D 4 .3503, D .6679 ft 8.01 inches
c. L G will be the same; thus
1/ 2
G
L
L
G
will be the same, and G will be the same.
Area V .75 6 , V 3D as before 67, 500 lb day

4 2
area 67500 .75 .521 8.64 10 1.998 ft 10 earlier value

1/ 2
4 1.998
D 1.59 ft 19.14 inches

262

10.D12. a.
D
y L V x 1 L V x . When
D
x 0, y 1 L V x 0.1828 . See figure. Need
2 equilibrium stages. Stop where feed line and operating line intersect.

B
HETP 5 / 2 2.5 ft, x ~ 0.065
b. M is at
in 1
x y .43

L
L M 6.13
3.88
L 1.58 L M

Within accuracy of graph,
L L L
L
0.8
V L L L L 1

If try a shorter column with same feed wont work.

L
L
.8
L V
and must adjust column.

10.D.13. New Problem in 3
rd
edition. Saturated vapor feed in problem 10.D.9 has minimum L/V =
(0.999-0.6)/(0.999-0.22)=0.5122. This is (L/D)
min
= 1.05. The actual L/V = 0.6, which is an
L/D = 1.5. Thus, the multiplier M of the minimum was M = (L/D)/(L/D)
min
= 1.5/1.05 =
1.43.
For a saturated liquid feed (L/V)
min
= (0.999-0.825)/(0.999-0.6) = 0.4361, which corresponds to
(L/D)
min
= 0.7733. If we use the same multiplier, L/D =1.43(0.7733) =1.106 and L/V =
0.525.
Mass Balance:
B
D B
z x D 0.6 0.01
0.59656
F x x 0.999 0.01

D 596.56 kmol/h, B 403.44. These are same as in 10.D9.
At top: L V 0.525, V L D V 0.525V D
263


D 596.56
V 1255.9 kmol h
0.475 0.475

L V D 659.4 kmol h
At top of col.
L V
W W L V since pure MeOH, same mol. wt. 0.525

kmol 32.04 kg MeOH 2.046 lbm lbm
V 1255.9 82, 330
h kmol kg h

The density and surface tension calculations are the same as in 10.D9.
Assume ideal gas. Top of column is essentially pure MeOH.

M
v M 3 3
lb
1 atm 32.04
n MW
p lbm lbmol
MW 0.07219
V RT ft ft atm
0.7302 607.79 R
lbmol R

where pure MeOH boils at
1.8 R
64.5 C 337.66K 607.79 R
K


3
3
L 3 3
1 kg 2.2046 lbm 28317 cm lbm
MeOH 0.7914 g cm 49.405
1000g kg ft ft

At top 24 0.0773 64.5 19.0

1/ 2
1/ 2
G L
v
G L
W 0.07219
F .525 0.02007
W 49.405

Fig 10-16 with 18 tray spacing:
sb
C 0.28

0.2 0.2
L V
flood sb
V
19 49.405 0.07219
U C 0.28 7.25 ft s
20 20 0.07219

Use Eq. 10-14 (Modified),
V flood
4V lbm h
D
frac u 3600


3
4 82, 330 lbm h
D 9.08ft
ft
0.90 0.07219 lbm ft 0.75 7.25 3600 s h
s

Probably use 9 ft, which is a slightly higher fraction of flooding. This compares with 10.27 ft for the
saturated vapor feed. The smaller diameter column will be less expensive.
With a saturated liquid feed and CMO, the vapor flow rate in the bottom of the column is the same as in
the top, V = 1255.9 kmol/hr. For problem 10.D9 with a saturated vapor feed,
1491.47 1000 491.47 V V F . Since
R
Q V ,
, _ , _ , _
(1255.9 / 491.7) 2.55
R liquid feed R vapor feed R vapor feed
Q Q Q .

Thus, in this case there is a significant energy price for reducing the column diameter by this method.

10.D14.
B
D B
z x 0.4 0.0001
D F 1000 400.7415
x x 0.998 0.0001
, B 1000 D 599.258 kmol/day
264

V V L D 1 D 3 400.7415 1202.225 kmol day
or
kmol 1 day 1 h
V 1202.225 0.013915 kmol s
day 24 h 3600 s

At bottom, L V B 1202.225 599.258 1801.483 kmol day

L 1801.483
1.49846
V 1202.225

Bottom of column is essentially pure water. Also
boilup B
y x
Thus
2
L G L V 1.49846 G is lb (s ft )

lb kmol 18.016 kg 2.20462 lb
V in 0.013915 0.55268 lb s
s s kmol 1 kg


3
L 3 3 3
kg 2.20462 lb m lbm
density water at 100 C 958.365 59.83
m kg 35.31454 ft ft

Data from Perrys, 7
th
ed., p. 2-92.
W L
1.0
3
G w w 3 3
lbm
1 atm 18.016
n p lbm ft
pV nRT MW MW 0.0367
V RT ft ft atm
0.7302 671.688 R
lbmol R
where 100C
1.8 R
373.16K 671.688 R
K


1/ 2
1/ 2
G
v
L
L 0.0367
F 1.49846 0.03713
G 59.83

From Fig. 10.25, Ordinate at flooding = 0.18

1/ 2
G L c
flood c 0.2
0.18 g
G , g 32.2, F 110 Table 10 3
F


1/ 2
1/ 2
flood 0.2 2
0.18 0.0367 59.83 32.2
lbm
G 0.15147 0.3892
s ft
110 1.0 0.26

Where
w
100 0.26 cp from Perrys p. 2-323.

actual flood
G 0.80 G 0.80 0.3892 0.31136

2
2
actual
V in lb s 0.55268
Area 1.77505 ft
G lb s ft 0.8 0.3892


1/ 2 1/ 2
4 area 4 1.77505
Diameter 1.503 ft
b. F = 33,
plastic
1/ 4
1/ 4
int
Intalox Berl
Berl
F 33
Diameter Dia 1.503 1.112 ft
F 110

265

10.D15.
v L
v
v L
W
F 0.03713 from prob. 10.D14
W

From Fig. 10-16 with 12 tray spacing ,
sb,flood
C 0.21

0.2
sb
K C
20
where = surface tension water at 100C.
Perrys, 7
th
ed., p. 2-306 @ 373.15 K,
5
10 dynes 1 m dynes
0.0589 N m 58.9
1N 100 cm cm


0.2
K 0.21 58.9 20 0.21 1.2411 0.2606

L v
flood
v
59.83 0.0367
u K 0.206 10.52 ft s
0.0367
,
act
u .8 10.52 8.416 ft s

v act 3
lb
4V
s
Dia
lb ft
u
ft s
Eq. (10-14) modified for units. V is from Problem 10.D14.


4 0.55268
Dia 1.637 ft
0.85 0.0367 8.416
. This can be compared to 1.5 ft for packed.
Tray columns with this small a diameter are seldom used in industry.

10.D16. From Eq. (10-44). Diameter
1/4
F .
1 3
F 98 and F 22 Table 10-3

1/ 4
3
1
F
Diameter (3 ) Diameter (1 )
F

1/ 4
22
14.54 10.0 ft
98

Can also repeat entire calculation which is a lot more work.

10.D17. At the bottom of the column have essentially pure n-heptane. Then, following Example 10-4,
we have.

3
v
p MW 1 100.2
0.205 lb ft
RT 1.314 371.4

Need L V. Since
1 y
L V .8 where z .5, we have y 0.6
1 z
at intersection of
operating lines. Then
0.6 0
L V 1.2
.5 0
.

L
v
MW L L
1.2 1.0 1.2
G V MW


1/ 2 1/ 2
v
L
L 0.205
1.2 0.084
G 1.684 62.4

Figure 10-25, Ordinate = 0.05 at p = 0.5
266


.2
0.05 .205 0.684 62.4 32.2
G 0.375
98 .9595 .684 0.205

The value 0.9595 is
Water
at 98.4C, and
Water L
. Since feed is a saturated liquid
V V 0.6944 lbmol/s.

2 v
0.6944 100.2 VMW
Area 185.5 ft
G 0.375


1/ 2
D 4 Area 15.37 ft
This is somewhat larger than in Example 10-4. Therefore design at bottom.

10.D18.
G L c
flood 0.2
g ordinate
G
F

Assume changing p changes only
G
& ordinate. Then take ratios

flood,new G,new
flood,old G,old
G ordinate, new
G ordinate, old


G,new
new new
G,old old
old
p MW RT
p
4
p p MW RT
. Assumes small change in
T (in Kelvin). T set by boiling conditions (Vapor Press) not by ideal gas law.

0.5
G
v
L
L
F
G
,
0.5
0.5
v, new G, new
new
v,old L,old old
F
p
2.0
F p
,
v,new v,old
F 2F 0.4 .
New Ordinate Value ~ 0.5, old value ~ 0.09

new
flood,new flood,old
old
ordinate, new p 0.05
G G 4 0.5 0.75
p ordinate, old 0.09


10.D.19. New Problem in 3
rd
edition. Saturated vapor feed in problem 10.D.9 has minimum L/V =
(0.999-0.6)/(0.999-0.22)=0.5122.
This is (L/D)
min
= 1.05. The actual L/V = 0.6, which is an L/D = 1.5. Thus, the multiplier M of the
minimum was M = (L/D)/(L/D)
min
= 1.5/1.05 = 1.43. For a saturated liquid feed (L/V)
min
= (0.999-
0.825)/(0.999-0.6) = 0.4361, which corresponds to (L/D)
min
= 0.7733. If we use the same multiplier as in
10.D9, L/D =1.43(0.7733) =1.106 and L/V = 0.525. This is the slope we use in the top section for the 2-
enthalpy feed. In the middle section of the column at minimum reflux conditions the slope of the middle
operating line is / (0.825 0.6) / (0.6 0.22) 0.592 L V .

The external mass balances still gives D 596.56 kgmoles/hr, B 403.44 . At minimum reflux
min min
( / ) 0.7733(596.56) 461.3 L L D D and
min min
461.3 596.56 1057.86 V L D . At
the saturated liquid feed V V and
liquid
L L F . Thus,
min min
[( ) / ] ( / ) 0.592
liquid
L F V L V and
,min
0.592 165.06
liquid
F V L .
Since the total feed rate is 1000, the fraction liquefied is 0.16506. The same fraction can be liquefied at
the finite reflux ratios. Thus, 165.06
liq
F and 834.94
vap
F .
267

At top use saturated liquid reflux ratio L V 0.525, V L D V 0.525V D

D 596.56
V 1255.9 kmol h
0.475 0.475

This is the same as for problem 10.D13 and the remainder of the calculation of the diameter is identical to
that calculation. The result of the calculation at the top of the column is

3
4 82, 330 lbm h
D 9.08ft
ft
0.90 0.07219 lbm ft 0.75 7.25 3600 s h
s

We now need to calculate the vapor flow rate in the bottom. Assuming CMO, in the middle section
1255.9 V V . In the bottom section,
1255.9 834.94 420.96
vap
V V F .
Since
R
Q V ,
,2 _ , _ , _
(420.96 / 491.7) 0.86
R enthalpy feed R vapor feed R vapor feed
Q Q Q .
Thus, in this case the two-enthalpy feed design results in the same reduction in diameter as liquefying the
entire feed, and it has energy savings compared to the vapor feed. However, the two enthalpy feed
system will require more stages than the other systems. A complete economic analysis is required to
determine the most economical system.

10.D20. Use Fig. 10-16 to find
sb
C . Gas is
2
N . Liquid is ammonia.
Since system very dilute, treat as pure ammonia liquid & pure
2
N gas.

L L
G V
W MW L kg h L kmol h 17.03
27.36 16.642
G W kg h V kmol h MW 28.08


3
3
L 3 3
100 cm
g 1 kg
0.61 610 kg m
cm 1000 g m


v
G 3 3
175 atm 28.02g mol
pMW 1000L kg kg
236.0
L atm RT m 1000 g m
0.08206 253.2 K
mol K


1/ 2
v
236
F 16.642 10.35
610

Off chart. Extrapolate using Eq. (10-10e).

2
10 sb 10 10
log C 0.94506 0.70234 log 10.35 0.22618 log 10.35

10 sb sb
log C 1.891 C 0.01286
Assume
0.2
20 1.0. Then

flood
610 236
u 0.01286 0.0162 ft s
236
,
op flood
u 0.75 u 0.01215

V
v op
4 V MW
D
u 3600
,
v
V 100 kmol h, MW 28.02 kg kmol
3
v op
0.85, 236.0 kg m , u 0.01215 ft s , Need to watch units
268


4 100 28.02
D
0.85 236.0 0.01215 3600 1 3.2808 ft
=1.155 m 3.79 ft
Probably use 4 ft diameter column (standard size)
Using larger diameter helps take into account the uncertainty in extrapolating to find
sb
C .

10.D.21. New Problem in 3
rd
edition. The mass balance and flow rate calculations are the same as for
problem 10.D14.

B
D B
z x 0.4 0.0001
D F 1000 400.7415
x x 0.998 0.0001
, B 1000 D 599.258 kmol/day
V L D 1 D 3 400.7415 1202.225 kmol day
or
kmol 1 day 1 h
V 1202.225 0.013915 kmol s
day 24 h 3600 s

L/V = 2/3. Top of column is close to pure methanol
Thus
2
L G L V 0.66667 G is lb (s ft )

lb kmol 32.04 kg 2.20462 lb
V in 0.013915 0.98290 lb s
s s kmol 1 kg

Pure MeOH boils at
1.8 R
64.5 C 337.66K 607.79 R
K


3
3
L 3 3
1 kg 2.2046 lbm 28317 cm lbm
MeOH 0.7914 g cm 49.405
1000g kg ft ft


W L
59.83/ 49.405 1.211
Assume ideal gas. Top of column is essentially pure MeOH.

M
v M 3 3
lb
1 atm 32.04
n MW
p lbm lbmol
MW 0.07219
V RT ft ft atm
0.7302 607.79 R
lbmol R


1/ 2
1/ 2
G L
v
G L
W 0.07219
F .66667 0.02548
W 49.405

From Fig. 10-25, Ordinate at flooding = 0.20

1/ 2
G L c
flood c 0.2
0.20 g
G , g 32.2, F 110 Table 10 3
F


1/ 2
1/ 2
flood 0.2 2
0.20 0.07219 49.405 32.2
lbm
G 0.2146 0.4633
s ft
110 1.211 0.34

Where
methanol
64.5 C 0.34 cp from Perrys (8
th
ed.) p. 2-449.

actual flood
G 0.80 G 0.80 0.4633 0.3706
269


2
2
actual
V in lb s 0.98290
Area 2.6521 ft
G lb s ft 0.3706


1/ 2 1/ 2
4 area 4 2.6521
Diameter 1.838 ft
Note that this is larger than the calculation of 10.D14 at the bottom of the column. Thus, do calculations
at top of column.
b. F = 33,
plastic
1/ 4
1/ 4
int
Intalox Berl
Berl
F 33
Diameter Dia 1.838 1.360 ft
F 110



10.E1. op time
D kmol
where D 18.1303 kmol
D kmol hr
,
L 2
, L V 0.4
D 3

Then
op
D
D V L V 0.4V 0.6V and t
0.6V

Use Fig. 10-25 or Eq. (10-39a) to find flooding at the end of the operation at bottom of column.

2
liq
2
vapor
kg
MW
L lbm s ft L kmol h
kmol
kg
G lbm s ft V kmol h
MW
kmol

At end of operation at bottom of column x 0.004, y 0.036 (pinch)

L 18.128
0.4 0.381
G 19.023

avg,liq vap
MW 0.004 46 0.996 18.016 18.128 & MW 0.036 46 0.964 18.016 19.023
3
L
62.4 lbm ft (Essentially pure water). Boils at ~ 100C = 373 K

v
G 3 3
lbm
1.0 atm 19.023
p MW
lbm lbmol
0.038806
RT ft atm ft
1.314 373 K
K lbmol

Then
1/ 2
1/ 2
G
V
L
L 0.038806
F 0.381 0.009501
G 62.4

From Eq. 10-39a.

2
10 10 10 v
log ordinate 1.6678 1.085 log 0.009501 0.29655 log F

10
log ordinate 0.6864 & ordinate 0.2059 agrees with Fig. 10 25

2 G L c
0.2
0.2059 g
G
F

From Table 10-3,
c water p
F 70, 1.0, g 32.2, 100 C ~ 0.26 c (Perrys 5
th
ed., 3-213)
270


0.5
flood 0.2 2
0.2059 0.03698 62.4 32.2
lbm
G 0.53488
s ft
70 1.0 0.26

From Eq. (10-41),
2
2
vapor
Area ft
lbmol lbm
V G
lb
s s ft
MW
l bmol


2
2
act flood
1 2
Area 0.19635 ft and G 0.7G
4


0.19635 0.53488 0.7
lbmol
V 0.0038646
s 19.023


kmol lbmol 3600s 0.453593 kmol
V 0.0038646 6.310665 kmol h
hr s h 1.0 lbmol

Then
op
D 18.1303 kmol
t 4.7883 h 287.3 min.
0.6V 0.6 6.310665

10.F1. Need average
M M
M M
y 1 x
.
x 1 y
From Equil. data

0.134 0.98
Bot : 7.582
0.02 0.866
,
0.979 0.05
Top 2.454
0.95 0.021

Geometric avg 7.582 2.454 4.31
Column temperature varies from 64.5 to ~ 98.2 C.

64.5+98.2
Avg T= 81.35 C
2


liquids
from Perrys 7
th
ed., T = 81.35C, p. 2-323.
M W
0.28 cp, 0.35 cp
(Note 40% MeOH is probably wt%)
Estimate:
mix M M W W
n x n x n
Feed is 60% M 40% W

mix
n 0.60 n 0.28 0.40 n 0.35 1.1837 ,
mix
0.306
Then
mix
4.31 0.306 1.3195
From OConnells Correlation, Fig. 10-14,
o
E 45%
Eq. (10-6):
2
o 10 10
E 0.52782 0.27511 log 1.3195 0.044923 log 1.3195 49.5%
If conservative use 45%

10.F2. To use OConnells correlation (Fig. 10-14), need and viscosity of feed.

M M M M M
MW
W W W M M
y 1 x K y x
K y x x 1 y
. Used Table 2-7 for values.
Can estimate a geometric average at bottom, feed & top

M W,bot
0.134 0.98
7.582
0.02 1.866
,
MW,feed
0.729 0.6
x .4 4.035
0.4 0.271

271


MW,top
0.979 0.05
2.454
0.75 0.021
,
1/ 3
avg bot F T
4.2184
Averages can be calculated many other ways.
The feed is saturated liquid. From Table 2-7, T = 75.3C
Viscosities from Perrys, p. 2-323,
W MeOH
0.39cp & 0.30
Note: (MeOH, 40% probably refers to wt % - p. 2-322 Perrys)
Estimate
mix 1 1 2 2
n x n x n

mix
n .4 n 0.30 .6 n 0.39 1.0465,
mix
0.351
Then
Feed
4.2184 0.351 1.481
Overall Plate effic. = 43.7%

10.F3.
B
D B
z x F 0.30 0.01 100
D 36.71
x x 0.8 0.01
B F D 100 36.71 63.29 lbmol h
At top of column L = D(L/D) = 73.4 and V = L + D = 110.1
Stripping section
L F F 3 L
L L qF where q 4 3
F

L 206.7, V L B 143.3
Feed line has slope 4/3 and goes through y x z .3 . Top operating line has slope L/V =
0.667 and goes through
D
y x x 0.8 . Bottom operating line goes through
b
y x x 0.01and the intersection of top operating line and feed line. McCabe-Thiele
solution is shown in Figure. Optimum feed is 8
th
from top. Need 8 7/8 equilibrium stages
plus partial reboiler.


272

Overall Efficiency. For OConnell Correlation, need
AVG Feed col
and T

E E
E E
y 1 x
1 y x
. Using Table 2-1 we find and following mole fraction.

0.170 .981
x 0.019, y 0.170 : 10.575
.830 .019


.5826 .6727
x 0.3273, y .5826 : 2.87
.4174 .3273


.7815 .2528
x .7472, y .7815: 1.210
.2185 .7472


1/ 3
AVG 1 2 3
3.324
Can estimate from p. 99 Ethyl Alcohol Handbook at z .3 .523 wt.
frac., 0.55 cp. Thus = 1.83. From OConnell Correlation
o
E .42 .
N 8.875 .42 21.1. Thus, need 22 stages plus partial reboiler
Height 22 18 18 disengagement 48 (bottom sump) 38.5 ft

Diameter Calculation
V
v flood
4V MW
Dia
flooding fraction u 3600

Use average values of parameters in stripping and enriching sections.
v V L
eth W
eth W
MW MW y MW y for both MW and MW .
stripping section: 18.25 MW 21.5
L
L
W L MW 206.75 20 4135 lb h
V
V
W V MW 143.46 20 2869.2 lb h

3
L
0.96225 g ml 60.07 lb ft Bottom
Bottoms T =100C = 672R

3
V
V
P MW RT 1.0 20 0.7302 672 R 0.04076 lb ft

v V L sb L V
F W W 4135 2869.2 0.04076 60.07 0.0375, C 0.28 .
Enriching section: 26.4 MW 40.4
L
L
W L MW 73.42 35 2569.7
V
V
W V MW 110.13 35 3854.55 lb h

3
L
0.766 g ml 47.92 lb ft Distillate
Distillate T = 82C = T = 639.6R;

3
V
V
P MW RT 1 35 0.7302 639.6 R 0.05472 lb ft
273


v L V V L
F W W 2569.7 3854.55 0.05472 47.92 0.0225,
sb
C 0.28

0.2
sb
K C 20 , ft/sec. , surface tension in dyne/cm. 57
th
ed. Hdbk of Physics + Chemistry, F-45.
Bottoms, ~ 46 dyn/cm, Middle, ~ 25 dyn/cm, Top ~ 18.6 dyn/cm
stripping:
0.20
K 0.28 46 20 0.33075 ft s
enriching:
0.20
K 0.285 18.6 20 0.2809 ft s

flood L V V
u K , ft s
stripping:
flood
60.07 0.04076
u 0.33075 12.693 ft s
0.04076

enriching:
flood
47.90 0.05472
u 0.2809 8.31 ft s
0.05472


1/ 2
V flood
V
Dia. 4V MW 0.90 0.75 u 3600
stripping:
3
V
V
V MW 143.46 lbmol h 20 0.04076 lb ft 70392.5
enriching:
3
V
V
V MW 110.13 lbmol h 35 0.05472 lb ft 70441.37
Diameters: stripping section: Dia = 1.7 ft and enriching section: Dia = 2.1 ft
Probably use 2.5 ft diameter since there is little if any cost penalty.

10.F4. Numbers from solution of Problem 10.F3 are used.
Cross Sectional Area
V
2
V lbmol s MW lb lbmol
G lb (s ft )

Bottom:
L V
L G L V MW MW 1.44 ~1 1.44
Top:
L V
L G L V MW MW 0.667 ~1 0.667
1 metal Pall rings: F 48, 0.15, 0.15
= Density of water/density of liquid At Top: 61 47.92 1.27
At 81C ,
w E
0.35 cp, 0.45 cp ,
MIX 1 1 2 2
n x n x n
At top y=
d MIX MIX
x 0.8, n .8 n .45 .2 n .35 , and 0.43

v E w
MW y MW 1 y MW 0.8 46 .2 18 40.4

3
G L
0.05472 lb ft , 47.92

1/ 2 1/ 2
G L
L G 0.667 0.05472 47.92 0.02254

2 0.2
G L c
G F g 0.078 from Figure 10-25

1/ 2
2 G L c
0.2 0.2
0.078 0.05472 47.92 32.2 0.078 g
G 0.358 lbm (s ft )
F
48 1.27 0.43

V 110.13 mol h 0.0306 lbmol s
274


v
2
V MW
Area D 4 , or
G


1/ 2
1/ 2
v 4V MW
4 0.0306 40.4
D 2.096
G 0.358

Probably use 2 or 2.5 foot diameter columns.
The calculation at the bottom of the column gives a smaller diameter.
HETP N Height of Packing, or 1.2 ~10 12.0 ft

10.G.1. New Problem in 3
rd
edition. The result from Wankat (2007a) is listed in the following Table:
Results for distillation of vapor feed 5 mole % methanol, 95 mole % water. Distillate is 0.9543 mole
fraction methanol and bottoms is 0.9976 mole fraction water. Tray spacing = 0.4572 m. Base case
conditions are listed in Tables 1 and 2 in Wankat (2007a). When two trays are listed, they have the same
diameters. The decrease in volume and increase in Q
R
are compared to the base case.

F
L
N
F,V
N
F,L
N dia A Vol

tray Q
R
Q
c, total
decr Vol incr Q
R
0(base) 10 -- 20 2.84 6.33 55.0 2 1065 -12,330 -- --

Two-enthalpy feed:
500 11 6 20 2.08 3.38 29.4 2 1070 -12,340 46.5 % 0.5 %
500 12 6 22 2.07 3.38 32.5 2 1065 -12,330 40.9 % 0
600 12 6 20 1.89 2.80 24.3 2 1070 -12340 55.8 % 0.5 %
750 13 6 20 1.56 1.92 16.7 2 1090 -12,360 69.6 % 2.3 %
750 16 7 26 1.56 1.91 21.8 2 1065 -12,330 60.3 % 0
1000 (all liquid) 9 20 1.20 1.13 9.8 2 2415 -13,680 82.1% 127 %

Intermediate condenser:
F
Withdr
N
F,V
N
V,with
N
L,ret
N dia A Vol tray Q
R
Q
c, total
decr Vol incr Q
R
300 11 10 6 20 2.41 4.56 39.6 2 1067 -12,340 28.0 % 0.2 %
450 11 10 6 20 2.22 3.88 33.7 10/11 1067 -12,340 38.6 % 0.2 %

Two-enthalpy feed (F
L
= 600 kmol/hr) plus one intermediate condenser:
F
withdr
N
F,V
N
F,L
N
V,with
N
L,ret
N dia A Vol

tray Q
R,total
Q
c
decr Vol incr Q
R
100 12 6 5 5 20 1.69 2.23 19.4 2 1073 -12,340 64.7 % 0.8 %
80 12 6 13 6 20 1.72 2.33 20.2 2 1079 -12,345 63.2 % 1.3 %

Two-enthalpy feed (F
L
=600 kmol/hr, N
F,V
=12, N
F,L
=6, N=20) plus two intermediate condensers:
F
wthd1
N
Vwth1
N
Lret1
F
wthd2
N
Vwth2
N
Lret2
dia A Vol

tray Q
R,total
Q
c
decr Vol incr Q
R
100 5 4 80 13 8 1.50 1.77 15.4 2/6 1081 -12,350 72.1 % 1.6 %

Two-enthalpy feed (F
L
= 680 kmol/hr) plus one intermediate condenser:
F
withdr
N
F,V
N
F,L
N
V,with
N
L,ret
N dia A Vol

tray Q
R,total
Q
c
decr Vol incr Q
R
100 12 6 5 5 20 1.51 1.79 15.5 6 1081 -12,350 71.7 % 1.6 %

With constant Q
c
and Q
R
, the two-enthalpy feed with F
V
= 750 and N = 26 appears to be best.


10.G.2. New Problem in 3
rd
edition. Results are from Wankat, P. C., "Balancing Diameters of
Distillation Column with Vapor Feeds," Ind. Engr Chem. Research, 46, 8813-8826 (2007).
275


Table 1. Simulation conditions and results for base cases. F 1000 kmol / h, D = distillate flow rate
kmol/h, N = number of trays + condenser + reboiler, tray spacing = 18 inches = 0.4572 m,
operation at 80% of flooding, dia = maximum calculated diameter in m, tray = tray at which max
diameter occurred. A = max calculated column area in
2
m , Vol = column volume in
3
m = A(N
2 +1) (tray spacing) where N 2 is the number of trays and +1 is for disengagement space for
liquid and vapor,
R
Q and
c
Q = reboiler and condenser duties in kW,
cond
p condenser pressure
in atm, p pressure drop in psi/tray,
feed
N = optimum feed stage, and condenser is stage 1.
Note this solution has a p, each stage. Thus, solution slightly different than students solutions.

Feed
N
N Dia A Vol tray
R
Q
c
Q ,
1
L D
D
cond
p
p


Ethanol (10 mole %). Water (90 mole%) Vapor Feed Base Case:
23 26 2.61 5.35 85.6 2 902 -10,700 8.0 125 1.80 0.1


Table 2. Diameters calculated for standard distillation base cases listed in Table 1. Vapor flow
rate
j
V and liquid flow rate
j
L are in kmol/hr, diameter is in m, area is
2
m .
Tray V
j
L
j
Dia Area
2 1125 997 2.61 5.35
23 1082 949 1.99 3.11
24 74.3 950 0.71 0.396
35 79.4 956 0.71 0.396
Ethanol-water, vapor


Table 3. Simulation conditions and results for a distillation column separating a vapor 10 mole %
ethanol, 90 % water feed (see Table 1 for base conditions). Partial condenser is stage 1.
F,V
N and
F,L
N
are optimum feed locations for vapor and liquid portions of the feed, respectively. Decrease in column
volume Vol (equal to change in area when the number of stages is unchanged) and increases in
R
Q are
compared to the ethanol-water base case (Table 1). For both runs
D,E
y 0.7901

and
B,W
x 0.9986.
L
F
F,V
N
F,L
N
N Dia A Vol Tray
R
Q
C,col
Q
0(base) 23 -- 36 2.61 2.24 35.8 2 902 -10,700

L
F
F,V
N
F,L
N
N Dia A Vol Tray
R
Q
C,tot
Q
Decr Vol
Incr
R
Q
600 23 17 36 1.69 2.24 35.8 2 902 -10,700 58.2% 0
c,total c,col c,feed condenser
Q Q Q


10.G3. New Problem in 3
rd
edition. Part a. S Dia = 2.032 m. Distillate mole fractions (vapor) = 0.22222
Eth, 0.77765 Propane, 0.12383 E-03 B, and 0.28503 E-9 pentane. Bottoms mole fractions = 0.14843 E-
10 Eth, 0.10132 E-03 Propane, 0.81808 Butane and 0.18182 pentane. Other values are in Table for 10.G4.
Part b. Worst backup is 0.232 m on stages 30 and 31. Maximum weir loading is 0.0204 m
2
/s on plate 31.
Part c. Same mole fractions, same Q
c
and Q
R
. Max backup 0.1614 m in panel A on stages 29 to 32.
Maximum weir loading is 0.01183 m
2
/s on plate 31 of panel A which is acceptable.

10.G4. New Problem in 3
rd
edition.

276

V feed
Kmol/s
L feed
Kmol/s
Q
c
kW Q
R
kW Max Dia
m
Stage
max dia
y
D,C4
x
B,C3
0* 1 pass .1(N
F
=16) -1463 2827 2.032 31 .000124 .000101
0 part d .1(N
F
=15) -1463 2827 2.032 30 .000115 .0000942
0* 2 pass .1(N
F
= 16) -1463 2827 2.032 31 .000124 .000101
.01 .09 -1463 2600 1.956 31 .000145 .000119
.02 .08 -1463 2373 1.876 31 .000199 .000163
.03 .07 -1463 2147 1.792 31 .000300 .000246
.04 .06 -1464 1921 1.905 32 .000525 .000429
.05 .05 -1466 1696 1.650 18 .00112 .000915
.06 .04 -1472 1474 1.600 18 .003188 .002609
b. Change
N
N=41 N
F,liq
=18 N
Fvap
=21
.03 .07 -1463 2146 1.793 34 .0000817 .0000668
* Values from problem 10.G3.
Part c. Tray rating program with Dia = 1.793 m and defaults for tray spacing (0.6096m) & for DC
clearance (0.0373m) obtain 0.2207 m backup on tray 34, which is acceptable. Maximum weir loading is
0.01887 m
2
/s on tray 34 which is acceptable.
Part d. Shown above, plus maximum backup is 0.2320 on plate 31 (acceptable) and maximum
weir loading is 0.0204 m
2
/s on plate 31, which is marginal.
277
SPE 3
rd
Edition Solution Manual Chapter 11
New Problems and new solutions are listed as new immediately after the solution number. These new
problems are: 11.A19-11.A22,11.B6, 11.C3, 11.D5,11.D8, 11.G3-11.G4.
11.A3. As feed temperature increases
MIN
L D increases, hence L/V increases and
c
Q increases.
L V increases and
R
Q decreases. The number of stages probably decreases.
(See Figure 7-3. The abscissa increases as
min
L D and L/D increase. Thus ordinate drops.
Since
MIN
N is constant, N decreases).
11.A.6. New Problem for 3
rd
edition.
b. A liquid side-stream between the feed and the distillate. 1 point for b.
c. A vapor side-stream between the feed and the bottoms. 2 points for c.
11.A.9. If the feed rate is consistently one half the design capacity, the entire economy of scale will be
lost. In addition, distillation columns probably operate at lower than design efficiencies.
11.A11. Column 2 would have to be at a lower pressure than column 1.
11.A.12. New Problem for 3
rd
edition. The heuristics to not do would include items such as:
1. Remove dangerous, corrosive, and reactive components last.
2. Use distillation only for very difficult separations ( < 1.10). And so forth.
11.B4. Use heuristics. Hardest separation is xylene-cumene,
X,T
x,c
C,T
0.33
1.57
0.21
Toulene is most abundant. Use heuristics to:
Remove toluene early
Do hardest separation last
One-by-one in overhead
If use equil-molal splits:
Can also derive coupled systems.
B
X T
F
C
278



11.B5. Add in Heuristic 9 for sloppy separation.

11.C.1. New Problem for 3
rd
edition. Take the log of both sides of Eq. (11-2).

log (cost A/cost B) = (exponent) log (size A/size B)

log (cost A) log (cost B) = (exponent) [log (size A) log (size B)]

exponent = [log (cost A) log (cost B)]/ [log (size A) log (size B)]

11.D1. NOTE: This solution requires the solution to Problem 10-D1. Estimate at feed composition
x = z = 0.5
a&b. For binary,
c7
c6 c6 c6 c6
c6 c7
1 K
x , y K x .
K K
Use Fig. 2-11.
T = 168C:
c6 c7 c6 c6
K 1.29, K 0.71, x 0.50, y 0.645
(Guess was aided by solution to Problem 10-D1)

.645 .5
1.817
.355 .5


MIN
.999 .001
n
.001 .999
N 23.13 stages
n 1.817


D
min D
x y L 99 .645
0.7094
V x z .999 .5


X
T
F
C
B
X
F
C
B
OR
T
X
T
F
C
B
279


min
min
L L V
2.441
D 1 L V

c. Gilliland:
MIN
L D L D
4 2.441
x 0.3117
L D 1 5

Eq. (7-42b):
MIN
N N N 1 0.37027
N 0.3703 23.13 1 0.3703 37.3 contacts or 36.3 stages
d.
act eq o
N N E 36.3 0.73 49.7 or 50 stages
24 tray spacing 50 trays = 100 ft
+ 4 disengagement
H = 104 ft = 31.7 m

m
D 9 ft 2.743m
3.2808 ft

Tray area
2
2 3
D
5.91 m . Column Vol area H 187.3 m
4

Fig. 11-1.
3 3
P, tower 3
cos t $700
$700 m , C 187.3 m $131,110
vol m

Fig. 11-2.
2 2
P, tray
Tray cos t
$710 m , C 710 5.91 m $4196 tray
area

Eq. 11-8.
BM,tower
C 131,110 2.25 1.82 2.3 1.0 $844, 000

p
7 2.743
pD
700 kPa 7 bar, Eq (11-5), F .5 .5 2.30
10.71 0.00756p 10.71 0.00756 7

Fig.11-9.
BM.tray
$ 209, 800
C 4196 50 1 1
total $1,054,000


11.D2. 69C (bp) = 156.2F. F 1000, D 500, B 500, L V 0.8, L D 4
L 2000 lbmol h, V 2500
Energy Bal:
v c D c o v
VH Q Vh gives, Q V h H

D v
h is pure boiling hexane, H is vapor.
Thus,
D v c6
h H 13, 572 Btu/lbmol

7
c
Q 2500 13, 572 3.39 10 Btu h

R D B F C
Q Dh Bh Fh Q
Pick 25C as basis.

D PL,c6
1.8 F
h C 69 25 51.7 44 1.8 4094.6
C


B PL,c7
Btu
h C 98.4 25 1.8 50.8 73.4 1.8 6711.69
lbmol

Feed is a saturated liquid. From Example 10-1, T = 80C
280


F PL,c6 c6 PL,c7 c7
h C z C z 80 25 1.8

F
h 51.7 .5 50.8 .5 55 1.8 5073.75 Btu lbmol

7
R
Q 500 4094.64 500 6711.69 1000 5073.75 3.39 10

7
R
Q 3.423 10 Btu h
Reboiler:
7
2
2
Q 3.423 10 Btu h
A 32, 800 ft
U T Btu 1.8 F
50 110 98.4 C
h ft F C

where U is average from Table 11-2.
Condenser:
7
c 2
Avg
3.39 10
Q
A 2850 ft
U T 110 70
180 156.2
2

Note these areas are very approximate. For detailed design need a much better estimate of U.
Costs: Condenser
2
2 2
2
1 m
A 2850 ft 264.8 m
3.2808 ft

Fixed Tube Sheet S&T Fig. 11-3, Cost = $125/m
2

Reboiler
2
2 2
2
1 m
A 32, 800 ft 3047 m
3.2808 ft
large because of low U.
Extrapolate $70/m
2

1 atm,
p m
F 1.0, F 1.0 . Eq. (11-9)
BM p m Q p
C C 1.63 1.66 F F 3.29 C
Condenser
2
BM 2
$125
C 3.29 264.8 m $109, 000
m

Reboiler
2 2
BM
$702, 000
C 3.29 $70 m 304 m
$811, 000 Very sensitive to U.


11.D3. Note: This solution requires the solution to Problem 11-D2.
Steam rate,
7
R
Q lb 3.423 10 lb
35, 704.6
h 958.7 h

where
R
Q is from Solution to problem 11-D2.
Steam Cost
$ lb $20.00
35, 704.6 $714 h.
h h 1000 lb

Cooling water,
w
7
c
p w
Q
lb 3.39 10 lb
847, 500
h C T 1.0 40 h

where
R
Q is from Solution to problem 11-D2.
Water Cost
$ lb $3.00 1
847, 500 $306 h
h h 1000 gal 8.3 lb gal


281

11.D4. From Example 11-1 needed 21.09 eq stages + P.R.

pack
h 21.09 1.1 ft stage 23.2 ft 7.07 m

2
2 3
pack
D
Vol h 15 23.2 4099.6 ft
4 4
3
3 3
3
1 m
4099.6 ft 116.1 m
3.2808 ft

From Fig. 11-2
3
p
C ~ $250 m packing
Tower 23.2 ft + 2 ft between sections + 2 ft top + 2 ft top = 29.2 ft
h =
1 m
29.2 ft 8.9 m
3.2808 ft
, Vessel Vol.
3
146.1 m
Fig. 11.1
3
p
C $700 m for tower

p M BM p
F 1.0 1 atm , Tower F 1 carbon steel , C C 2.25 1.82 $700 146.1 4.07
$416, 312
Packing,
m
F 4.1.

BM p
C C 1.63 1.66 4.1 250 116.1 8.436 244, 855
Total $661,000

Does not include cost distributors, supports, hold down plates, etc.

11.D.5. New Problem for 3
rd
edition. n = [log (cost A) log (cost B)]/[ log (size A) log (size B)]
Let size A = 10 m
2
and size B = 1.0 m
2
. The cost A = $400/m
2
= ($400/m
2
)(10 m
2
) = 4000, and
cost B = $2100.
n = [log (4000) log (2100)]/[log (10) log (1)] = [3.602 3.322]/[1 - 0] = 0.28

11.D6. See residue curves in Figure. The recycle is pure MB. Mixing point is determined in same way
as in Fig. 11-11. Now mixing point splits into light (L) component methanol on
1
B . Thus line
LM is extended to 0.0 mole fraction methanol to find location of
1
B (0.73333 MB and 0.26667
toluene). We can use mass balance to find point
1
B accurately. If
1
D is pure methanol,
1
D = 50
(all methanol in feed) and
1
B = 150. Then from toluene balance 0.2 200 = 150
1
tol,B
x , which
gives
1
tol,B
x 0.266667 .
1 2
B F which is then split into I (Some of which is recycled) and
2
B
which is toluene product.

1 1
F Recycle 100 100 M B D
Since
1
D contains no toluene
1
B 0.26667 F .4 Recycle 0

1 2
.4
B 100 150 kmo h. F
0.26667
,
1 1
D M B 50.0 kmol h
For Column 2:
2
B essentially pure toluene,
2 2
B 0.266667 F 0.26667 150 40
OK Satisfies overall external M.B.

2 2 2
D Recycle F B 150 40 110

2
D 10.0, which also satisfies external M.B.

282




11-D7. a.) Proposed Split: Bottoms Essentially pure toluene
Distillate ~ .83 methanol. (See figure for Solution to 11.D7)

M MB T
F 100, z 0.5, z .1, z .4
Assume all toluene in bottoms & bottoms is pure.
B 100 .4 40, D 60

M,dist
60 x 50 ,
M,dist MB,dist
x 0.8333 & x 0.166667
b.) Proposed Split. Distillate pure M
Bottoms 0.2 MB, 0.8 T (See figure).
D 100 .5 50, B 50
Note Doubtful this will work.

283




11.D.8. New Problem for 3
rd
edition. Part a.

Cost in June 2010 $947, 000 556 397 $1, 326, 000
Part b.

F 2 x F of Example 11-2.
Since Dia F ,
11 2
Dia 2F 12 ft 2 16.97 or 17 feet.

2
1m
Diameter 17 5.182m, Tray Area 21.08m .
3.2808 ft

Tower
3
Volume 488.2m Fig. 11-1,
0 3
p
C Cost vol ~ $550 m

3 3
P,tower
C $550 m 488.2 m $268, 500
Fig. 11-2.
3 2 2
p,tray
Traycos t area ~ $750 m extrapolate , C $750 m 21.08m $15,800 tray

BM,tower
C 268, 500 2.25 1.82 1.0 1.0 $1, 093, 000
284


BM,trays
C $15,800 36 1.0 1.0 $569, 000
Total bare module cost September 2001 = $1,662,000
In June 2010, Cost $1, 662, 000 556 397 $2, 328, 000
Part c. Original feed rate 1000 lbmol h.
At
original
F , cost lbmol $1326 lbmol
At
original
2 x F , cost lbmol $1164 lbmol




11.G1. - Use Fig. 11-10b as flowsheet. Use NRTL.
Feed : 1 atm, sat'd Liq, 100 kmol/h, 0.5 MeOH, 0.4 T, 0.1 MB
Fed to Stage 30
Int. Recycle satd Liq, 100 kmol/h, 100% MB, fed to Stage 20

Block
1
B : N = 46, total condenser, partial reboiler, D = 50, L/D = 3
Dist : 0.999759 MeOH, 0.000241 tol, 1.537 E 0.8 MB
Bot : 8.02 E 0.5 MeOH, 0.266586 tol, 0.73333 MB

Block B2: N = 85, L/D = 9, B = 40, feed = 41
Dist: 0.0001094 MeOH, 0.0008684 Tol, 0.999022 MB


Bot : 2.855 E 35 MeOH, 0.99731 tol, 0.0026888 MB
Thus, this is feasible.

11.G2. a) For Fig. 11-10A.

f
Bottom 0.999186 tol
L
Col 1. N 90, N 41 , 8 D 60
0.000814 MB D



f
Dist. 0.99938 tol, 0.000605 MB,
Col 2. N 20, N 10 , L D 2, B 10 Bot .996679 MB, 0.003088 MeOH,
0.000233 toluene

If increase L/D in column 1,
f
Col 1. L D 9, N 90, N 41, Bot. 0.99941 toluene.

f
Col 2. L D 2, N 24, N 12, Dist. 0.99954 MeOH, Bot. 0.9976 MB

Which now meets specifications. Thus Figure 11-10a without recycle is feasible.

b.) For Fig. 11-10B converged N = 30, L/D = 6
Dist. Col 1. 88.7% M & 11.3% T No MB
(azeotrope)
Would not converge higher N.
Does not appear to work; thus, not feasible.

11.G.3. New Problem for 3
rd
edition. F 100, 10% Ethanol, 5.0 atm, Satd liquid feed
N = 10 includes partial reboiler, total condenser, D = 10, L D 2

285

P = 1 N
F
= 5 Q
R
= 52,948 x
D,E
= .72033 x
B,E
= .031085
N
F
= 6 Q
R
= 53,029 .74564 .028263
N
F
= 7 .76332 .026298
N
F
= 8 53,148 .77539 .024957
N
F
= 9 53,175 Q
C
= -78614.5 .78037 .024363
N
F
= 10 52896.9 cal/s .69964 .033373



P
col
= 3 atm
L D 2

N
F
= 8 Q
R
= 67995.5 .75453 0.027275
N
F
= 9 Q
R
= 68022.1 Q
c
= - 74274 0.76027 0.026637
N
F
= 10 67794 .69695 .033672


P
col
= 5 atm
L D 2

N
F
= 8 Q
R
= 76435.3 .74276 .028582
N
F
= 9

c
7646 Q 71692.8
.74840 .027955
N
F
= 10 76264 .69411 0.033988

Size optimal feed columns. Sieve tray 1 uses 18 inch tray spacing at 85% approach flooding, Fair
calculation method for flooding.

col
P 5 Max diameter tray 2 0.34867m

col
P 3 Max diameter tray 2 0.38070m

col
P 1 Max diameter tray 2 0.46429m

Part d. D1. 1.0 atm gives the best separation because the relative volatility is highest.
D2. The lowest Q
reboiler
is 1.0 atm. The effect of pressure on Q
reboiler
in this problem occurs
because the feed is always a saturated liquid at 5.0 atm. For the 5 atm column this feed remains a
saturated liquid and the feed line is vertical. At lower column pressures the feed flashes and is a two-
phase feed in the column. These feed lines have a negative slope. For the feed lines at lower pressures
the slopes of the bottom operating lines are steeper, which means lower boilup ratios, V
reboiler
/B. This
means lower Q
reboiler
at the lower pressures. Another way to think about this is the flashing feed produces
vapor and thus less vapor is required from the reboiler.
D3. The lowest absolute value of Q
condenser
is 5 atm. All columns have the same D and L/D.
Thus, V entering the condenser is the same. At higher pressures the latent heat of vaporization is lower.
Since Q
condenser
= V, the result is a lower absolute value of Q
condenser
at the higher pressures.
D4. The smallest diameter column is at 5 atm. Vapor density is highest.

Part e. Increasing pressure above 1 atm for the same purity requires more stages, but smaller column
diameter. Thus capital cost initially goes down. Above 8 atm the column must be designed for high
pressure operation, which makes it more expensive. Operating cost may go up if a higher L/D is required
to achieve the desired purity.

11.G.4. New Problem for 3
rd
edition.
a. (L/D)
min
= 1.3962 L/D = 1.5358. Obtain N (Aspen Notation) = 19 with N
feed
= 9 (on stage).
Distillate is 0.75056 ethanol and bottoms is 0.00005987 mole fraction ethanol. Q
R
= 569,172 cal/s,
286

Q
c
= - 443187 cal/s, Dia = 1.0755 m, A = 0.90843 m
2
, H = 11.5824 m, Vol = 10.52 m
3


Part b. Note that balancing the flow rates to achieve the desired separation in each column is trial and
error. The easiest was to do this is to first find a flow rate that works for the high pressure column (note
that D changes every time the flow rate is changed). This gives a value for Q
C,high pressure
= - Q
R,low pressure
.
Then design the low pressure column with this Q
R
. Check that both columns work. The correct flow rate
into the high pressure column is between 570.6 and 577.275. The results are reported below with the
latter value:
High pressure column: F = 577.275 kmol/h, Q
R
= 0.7555(569172) = 430,000 cal/s. N = 20, N
feed
= 9,
distillate is 0.75004 mole fraction ethanol and bottoms is 0.00009733 mole fraction ethanol. Q
c
= -
243,900 cal/s, L/D = 1.5618, T
cond
= 382.16 K, Dia = 0.79443 m, A = 0.49568, H = 12.192 m, Vol =
6.043 m
3
.

Low pressure column: F = 422.725 kmol/h, Q
R
= 243,900 cal/s, N = 19, N
feed
= 9, distillate is 0.75087
mole fraction ethanol and bottoms is 0.00003829 mole fraction ethanol. Q
c
= -190,627 cal/s, L/D =
1.5803, T
cond
= 351.56 K, T
reb
=373.16 K, Dia = 0.7014 m, A = 0.3864, H = 11.5824 m, Vol = 4.475 m
3
.

Part c. Energy requirement of multieffect distillation system was set at 75.55 % of the single column.
This is not optimized, but was set because it was known to work. Cooling is only in the low pressure
column of the multieffect system, and is significantly less than with only one column. The total volume
of columns is the same; however, this is misleading because volume in the single column would have
been less if it was designed at 3.0 atm. The capital cost will be higher for building two columns than one
larger one, but energy costs are less.


287
SPE 3
rd
Edition Solution Manual Chapter 12
New Problems and new solutions are listed as new immediately after the solution number. These new
problems are: 12.A5, 12.A6, 12.C2, 12.D1, 12.D3, 12D8, 12D13, 12D19, 12D21, 12D22, 12.G3.
12.A1. By raising T or dropping p can make gas desorb. The direction of transfer of solute
controls whether a column is a stripper or absorber. If operating line (on Y vs X) is
below equilibrium have a stripper.
12.A5. New Problem in 3
rd
edition. c. AspenPlus
12.A6. New Problem in 3
rd
edition. A. a. B. d. C. e. D. h. E. i
12.B1. Calculate: N or
out
y , or N for A and
out
y for B, or m, or N for A and m for B, or L/V,
or feed composition, or b, or m and b from 2 experiments, or Overall Efficiency.
12.B2. Two feeds, Sidestream, Reboiled absorber, Coupled absorber and stripper (see Figure
12-2), Interstage coolers (absorption) or heaters (strippers), Packed columns, Two
different solvents, Two different stripping gases, Add solid adsorbed to solvent (see
Chapt. 14). Cross-flow, Co-current flow, Combinations of flow patterns, etc.
12.B3. See Isom and Rogers (1994).
12.C4.
o equil actual
Eq 12 22
E N N
Eq 12 34
(10-1)
mV mV
o
mV mV
n 1 E 1 n 1 E 1
L L
E
n ( L mV) n L mV
mV
o
mV
n 1 E 1
L
E
n mV L
(10-4)
QED
Ref. Lacks, D.J., Chem. Engr. Educ., 302 (Fall 1998).
12.C5.
2
x dy
y , m
1 1 x dx
1 1 x
When
dy
x 0, , b 0
dx
When
dy 1 1
x 1, , b
dx
Apply Kremser as x 0 or x 1.0
12.C6. Graphically,
N 1 1
MIN
N 0
y y
L V
x x
where
N 1 N
y & x are in equilibrium
288


N 1 N
y mx b for linear.
For absorbers
min min
L L
m & 1
V mV

Thus as
min
L
N , 0
mV

Eq. (12-23) becomes
* N 1 1
1 0 *
min N 1 1
y y L
where y mx b
y y mV

or
N 1 1 N 1 1
N 1 min min N 0
0
y y y y L L
y b V x x V
x
m m

which agrees with graphical analysis.

12.D1. New Problem 3
rd
Edition.
tot
H 1.186
y x x 0.395 x
p 3

m 0.395 b 0



















Eq. 12-30
*
N 1 1
*
1 1
y y mV mV
n 1
L y y L
5
mV
n
L



39.5 0.004 0.0000395 39.5
n 1
L 0.0002 0.0000395 L
5
L
n
39.5

39.5 39.5
n 1 24.67
L L
L
n
39.5

*
N 1
y unknown.

1
y 0.0002
0 N 1 1 N
L x Gy Gy L x

unknown

N
x

G
unknown
1
N
*
1 0
y m x .395 0.0001 0.0000395
L
0
x .0001

L
unknown
Since
N
x unknown,
N 1
y 0.004
Eq. (12-30) or (12-31) easiest to use
289


T & E be sure L mV 1

L mV/L L/mV RHS
50 0.79 1.2658 7.58
75 0.52668 1.8987 3.90
65 0.60769 1.64557 4.67
63 0.62698 1.59494 4.895
62 0.637097 1.56962 5.015 OK


N N 1 1 0
G 100
x y y x 0.004 0.0002 0.0001 0.006229
L 62

There are alternative solution paths, but L = 39.5 is not valid, it becomes
n 1 0
n 1 0
.
Alternative: Trial-and-error McCabe-Thiele solution.

12.D2.
a) L & V constant. 97% rec. H
2
S 3% left in gas.

out
y 0.03 0.0012 0.000036
Equil.:
2
H S
tot
p
423x
y 169.2x
p 2.5

M.B. with Const. L & V:
out
0.0012 V 0.000036 V x L

out
10
V
x 0.0012 0.000036 0.001164 5.82E 6
L 2000

b) See Figure.
in out in out out in
L L
y x y x where y , x & y , x are on op. line
V V
.
c)



in
y 0.000012
y
Eq
Note that
min
L G can
be calculated from a
crude sketch.
x* 7.09E 6
min
L G
min
L 0.0012 0.000036
164.124
G 7.09 E 6

0
out
y 0.0000036
min act
L L
200 M
G G

0
x
M = 1.2186
169.2
290

d) L/G too high L too high Liquid too dilute.
Need much better solvent (e.g., MEA solution).

12.D3. New Problem 3
rd
Edition. Mass Balance:

IN IN out
Vy Lx y V Lx , Equil.,
tot
H
y x
p

Substitute equil. into M.B.

IN IN
tot tot
HVx HVx
Lx Lx L x x
p p

Solve
6
tot
6 6
IN
10.96 10 0.4 10
HVx
p 0.274 atm
L x x 100 2 10 0.4 10

b.
6 6
out out
tot
H 10.96
y x 0.4 10 16.0 10
p 0.274

Can also do graphically, with Kremser equation (trial and error) or by solving mass balance first,

6
out in out
L
y x x 16 10
V
Then
tot out out
p Hx y 0.274 atm.


12.D4. Mass bal.
in in in in out out
y V L x y V Lx

291


out
y y
in out in out
in
L L L
x x x x
V V V
12 0.0002 0.00001 0.00228
Op. Eq.,
L
y x y
V
out
in
L
x
V


in out
point y , x 0.0, 0.00001 and
out in
point y , x 0.00228, 0.0002 are on op.
line. See graph.

Can also use Kremser equation for this problem.


292



12.D5. Since equilibrium is linear in weight ratio units can do either McCabe-Thiele or
Kremser solutions. For McCabe-Thiele solution know that points
out in in out
Y , X and Y , X are on operating line. McCabe-Thiele diagram is shown in
Figure. N = 5.9 stages. HETP = 10/.59 = 1.69 ft.



Kremser: Several different forms can be used. We will illustrate with Eq. (12-26)
written in ratio units.

*
N 1 in N+1 out
Y Y 0.02, Y mX 1.5 0.06 0.09

*
1 out 1 in
Y Y 0.50, Y mX 1.5 .40 0.60

n .02 .09 .5 .6
N 5.88
n .02 .5 .09 .6

HETP h N 10 5.88 1.70 feet

12.D6. First, assume Nitrogen is an ideal gas: 1 lbmol = 359 cu ft at 0C and 1 atm

333.16
359 437
273.16
cu ft/lbmol at 60C

2
N flow rate 2500 437 5.72 lbmol/h
Water flow rate: Ignore
2
CO in water.
w
MW 18

100, 000
18
5560 lbmol/h
293

Equilibrium at 60C: H = 3410, y =
TOT
H
x 3410x
P


L 5560
972.0
V 5.72
. External mass balance is:

6
out in in out in out
L L L
y x y x x x 972. 9 10 0.00875
V V V

Can solve with either McCabe-Thiele diagram or with Kremser Eq. (12-34). The
McCabe-Thiele solution is shown in the figure. Note different scales on axes. Need 5
real stages.



Kremser:
* 6
N 1 1 1 0
y 0, y 0.00875, y mx 3410 9.2 10 0.03137 , mV L=3.508


0 0.03137
n 1 3.508 3.508
0.00875 0.03137
N 5.07
n 1 .4 3.508 1

Probably use 6 real stages.

12.D7.
Wt
G 0.828 wt frac air
kg gas
1050 869.4
h
kg air/h

mole
869.4
G 29.98
29
kmol air/h

3 in
in
in
kg NH y 0.172 0.172
Y 0.2077
1 y 1 0.172 .828 kg air


in,molar
29
Y 0.2077 0.3543
17

Inlet
3
NH 0.172 1050 180.6 kg
3
NH
2% remains in gas
3
3.612 kg NH
294


mole
out
3.612 17
Y 0.007087
869.4 29

Equilibrium data Table 12-2 -
3
NH
mole
TOT
p
p, mmHg p, mmHg
y
P 1.30 760 988


3
wt
kgNH
X
kgW


p

y mole frac
mole
y
Y
1 y

0.05 11.2 0.01134 0.011466
0.075 17.7 0.017915 0.01824
0.100 25.1 0.0254 0.02607
0.15 42.7 0.04322 0.04517
0.20 64 0.06478 0.06926
0.25 89.5 0.090587 0.09961
0.30 119 0.12044 0.13694
0.40 190 0.19231 0.2381
0.50 275 0.27834 0.3857
0.60 380 0.3846 0.6250

M.B.
in
X L
wt
mol mol
3
3
LX
G Y
kg NH
17
17
kmol NH
out
GY ,

wt
mole wt out
mole
L
Y X Y
17 G


Note: Units, although mixed, work in Mass Balance & in Operating equation. See graph.

Minimum Solvent

0.3543 0.007087
Slope
0.477 0
wt ,min
mole
L
= 0.7279=
17 G


wt ,min
L .7279 17 29.98 370.99kg W h
Actual Solvent,
wt min
kg W
L 1.5 L 556.48
h

Op Line Slope
wt
mole
L 556.48
1.092
17 G 17 29.98


295




296


12.D8. New Problem 3
rd
Edition.










Assume Water (not total liquid) flow rate is constant in both sections. Assume air flow rate (not total gas)
constant is each section. In bottom section G 0.8 mol air h. In top section
G 0.8 0.5 .95 1.275 mol air h. Keep X as kg HCl kg water (from equil. data).
Convert p to y (mole fraction) to Y (mole ratios).
kg HCl
X
kg water

p
tot
p
y
p

kg HCl y
Y
kmol air 1 y

0 0 0 0
0.0870 0.000583 3.8355 E-7 3.8355E-7
0.1905 0.016 0.00001053 0.00001053
0.316 0.43 0.0002829 0.0002830
0.47 11.8 0.007763 0.0078240.
0.563 56.4 0.037105 0.038535
0.667 233 .15329 .18104
0.786 840 .55263 1.2353

2
F 0.5 .475 air
1
F 1.0, G=.8

.05 mol HCl
0.05 mole frac HCl y 0.05263
.95 mol air

out out
y 0.002 Y 0.002004
L
L
G
G
tot
p 2 atm 760. 2 mmHg
1IN IN
.2 mol HCl
y 0.20 mole frac HCl Y =0.25
.8 mol air

1520 mmHg
out
X
IN
X 0
L

297

Y vs X equilibrium data is curved.
Using these units, we need L in kg water hr and G and G in kmol air hr , and we need to
convert the X terms from kg HCl/h to kmol HCl/h.
Top Operating Line
HCl IN HCl out
GY (L/ MW )X (L/ MW )X GY ,
IN
X 0

out
HCl
L
Y X Y
G(MW )
HCl
L
0.002004
1.275(MW )
Goes through (0, 0.002004)
Bottom Operating Line
HCl out in HCl
GY (L/ MW )X GY (L/ MW )X
IN out
HCl HCl
L L
Y X Y X
G(MW ) G(MW )

HCl HCl out
L 0.8MW 0.25 L 0.8MW X
a) Feed line saturated gas at Y 0.05263. Two operating lines intersect at feed line.









From plot
*
OUT
X 0.69 (see figure)
Top operating line:
2
out int er sec t F
HCl
L
Y X 0 Y
(G(MW ))

Bottom operating line:
2 IN
*
F OUT int er sec t F
HCl
L
Y X X Y
(G(MW ))

Solve for
2
int er sect HCl F OUT
X G(MW ) Y Y L. Then
2 2 IN
* HCl
F OUT F OUT F
HCl
G(MW ) L
Y X Y Y Y
(G(MW )) L

IN 2 2 IN 2 2
*
OUT F F F OUT MIN F F F OUT * *
OUT OUT
L G G(MW) G(MW)
X Y Y Y Y L Y Y Y Y
(G(MW)) G X X
MIN
0.8(MW) 1.275(MW)
L .25 0.05263 0.05263 0.00200
0.69 0.69


MIN HCl
L / MW 0.22883 0.09355 .3224 kg water h

Since MW
min
= 36.461, L
min
= 11.755 kg/h, L = 1.2407 L
min
= 14.584 kg/h. L/(MW)
HCl
= 0.40

*
out
X
1,IN
Y 0.25
out
0.002 Y
HCl
L (G(MW ))
Y
int er sect
X
X

F2
Y 0.05263
HCl
L/(G(MW ))
Sketch for Min L determination
298


299


b. M.B.
2
2 F IN HCl IN out HCl out
F Y GY (L/ MW )X GY (L/ MW )X

2
2 F IN out
out IN
HCl
F Y GY GY
X X
(L(MW ))


out
0.475 0.05263 0.8 0.25 1.275 0.002
X 0 0.5561 kg HCl kg water
14.584 / 36.461


Top
(L/ MW) 0.4
0.3137
G 1.275

Bottom
(L / MW) 0.4
0.5
G 0.8

Top goes through
HCl
IN out
(L/MW )
X 0, Y 0.002, Slope 0.3137
G

Arbitrary point for plotting: X .4, Y .3137 .4 0.1255
Y .1255 .002 0.1275

Bottom from
out IN
X 0.5561, Y 0.25
To intersection Top Operating and Feed Line.
Need ~ 1.6 stages. Opt. Feed for
2
F is Stage 1 (Feed 1 is at bottom.)
Check Slope bottom
.25 0
0.504
.5561 .06
0K.


300


Figure for 12.D8.
301

12.D9. Repeat 12.D2 with Kremser.
,
,

Eq. (12-22)

Graphical solution was 10.4. Pretty close!


12.D10. Use Kremser equation such as

*
out
out
*
in
out
A
A
N 1
A
A
y y
1 L mV
y y
1 L mV



*
in in
out
A A
A
N 4, m 1.414, L V .65, y 0, y m x 1.414 .02 .02828
L mV .65 1.414 .459
Equation becomes:
out
A
y .02828 .02828 .552 .01267
Overall mass balance:
in in out out
y V L x y V L x


4
out out in in
V V 1
x y y x .01267 0 .02 4.93 10
L L .65


12.D11. Any of the vapor forms of Kremser equation can be used but problem is trial and error.
For example, use Eq. (12-21) inverted for L mV 1

*
1 1
N 1 *
N 1 1
L
1
y y
mV
y y
L
1
mV
becomes,
5
1.2
1
m
.27
1.2
1
m

Set up table and try values of m.

m 1.0 1.2 1.3 1.4 1.41 1.415
RHS .1344 0/0 .233 .2658 .2691 .2706

By linear interpolation m = 1.414. Note that m = 1.2 is a trap for the unsuspecting
student since L/(mV) = 1.0 and special form of Kremser is required.

12.D12. Note this requires information in Section 13.4.
mV 169.2
y m x m 169.2, b 0, 0.846
L 200
2000
L V 200
10
*
1 0 0 in
y m x b 0, x x 0
1 out N 1
y y 0.000036, y 0.0012
0.0012 0
n 1 0.846 0.846
0.000036 0
N 10.69
n 1 0.846
302

L .796 1000 796 kg solvent/h ,
All stages
L 796
0.0316
G 25,190



in out 1,in
.204 0.025 .0012
X .256, X 0.02564, Y .001201
.796 0.975 1 .0012


x X y Y
.05 0.0526 .002 .002004
.10 .11111 .004 .00402
.15 .1765 .006 0.00604
.20 .25 .008 .00806
.25 .3333 .01 .0101
Equilibrium, y = 0.04 x


Plot weight ratios.
Op. Line:
j j 0 j 1
L L
Y X Y X
G G

Slope = - L/G = - 0.0316. Step off stages backwards (start w. stage N) since it is
different than other stages and we wouldnt be sure when to switch if stepping off
forwards. Need 4 equilibrium stages.
Note: Can also plot y vs X, since y ~ Y and G ~ V




303

12.D13. New Problem 3
rd
Edition. Strip Vinyl chloride from water at 25C and 850 mmHg.

tot
H
y x
p

1243.84x
1147.904x
850 760

Want 0.1 ppm water leaving. Entering air is pure, L 1 kmole hr.



















* *
1 out IN
y y 1147.904 x 5739.52 ppm (mole)

MIN
mix
L 5739.52 0 kmol
1171.33 G 0.00085373
G 5.0 0.1 h

b and c. Want
MIN
G 2 G 0.00170746 kmol h air
L G 585.665 (See figure for part b labeled HW5 Prob 1b). m = 1147.904.
c. Eq. 12-28
*
N
x 0

585.665 5.0 0 585.665
N n 1
n 25.0 1147.904 0.1 0 1147.904
4.78327
n 1147.904 585.665 0.672944


d.
out IN IN out
y Lx Gy Lx G
For L 1 kmol h, G 0.00170746 kmol h
out IN out IN
L
y x x y 585.665 5.0 0.1 0
F

2869.76 ppm 0.00286976 mole frac.
Probably send waste gas to incinerator. Will require additional fuel to burn.
e. All concentrations are dilute enough that L G and equilibrium are straight and operation is very
close to isothermal.




*
out
y
out
x
y

IN 0
5.0 x x
N
y

N out
x x 1 ppm
1147.904
out 1
y y
0 IN
x x
IN N 1
y 0 y
N
1
*
out
y
*
out
y
304


305


12.D14. a) 95% removal CH
4,
5% remains Constant V:
CH
4
out
Y 0.05 0.00129 0.0000645

b.

Slope
min
0.00129 0.0000645
L V 19.5429
0.00006271 0


actual
min
L V 1.4 L V 27.360; L 27.360 V 2736.0
c) Ext. bal.
4,in 4
CH CH
4 4
out in CH CH ,out
V V
x y x y
L L


4 out 4,in 4,out
CH , CH CH
V 100
x y y 0.00129 0.0000645 0.00004479
L 2736

d) Use methane values in Kremser eqn. (12-22) to find N

*
1 0
20.57 100
mV
m 3600 / 175 20.57, 0.75183; y mx b 0
L 2736



0.00129 0
n 1 0.75183 0.75183
0.0000645 0
N 6.11 stages
n 1 0.75183

e) Now use Argon values with N = 6.11 to find
Ar,out Ar,out
y & x .

Ar Ar,in N 1,Ar Ar,in Ar,0
7700
m 44.00, y y 0.00024, x x 0.0
175


*
1 0
Ar
44 100
mV
1.6082, y mx b 0
L 2736

Eq. (12-23)
7.11
Ar ,1
7.11
0.00024 y 1 1.6082 1 1.6082
0.60846
0.00024 0
1
1
1.6082


Ar,out Ar,1
y y 0.00024 0.00024 0.60846 0.00009397

Ar,out Ar,in Ar,out
V 100
x y y 0.00024 0.00009397 0.00000534
L 2736

4
CH Eq.
4 4
4 4
CH CH
CH CH
TOT
p 3600 x
y 20.5714 x
p 175

4in
CH
y
0.00129
x* 0.00006271
20.5714 20.5714

in
x

min
L V
CH
4
out
y 0.000645
4
CH
y
in
y 0.00129

4
CH
x
306

% Argon recovery in liquid
0.00000534 2736
100 60.85%
100 0.00024


12D15. Need equilibrium data. From DePriester chart:

C3 C3 C3 C3 C4 C4 C4 C4
K y x 1.23 m , K y x 0.34 m
Butane is a design problem:
C4 C4
L mV
5.882, 0.17
mV L

1.2% of the butane leaves as a gas. Thus,

*
1,C4 1,C4 C4 0,C4
y 0.006 0.012 0.0000072, y m x 0


*
N 1 1
*
1 1
y y mV mV
.0006 0
1
n .83 .17
L y y L
.0000072 0
N 2.39
L
n 5.882
n
mV

Propane is simulation:
*
N 1,C3 1,C3
C3 C3
L mV
y 0.0017, y 0, 1.626, 0.615
mV L


N 1
*
N 1 1 N 1
*
1 1 1
L
1
y y y mV
y y y 1 L mV
,
N 1
1
y
y 0.000298
5.7034


12.D16. Was 12.D19 in 2
nd
edition.
. a.) Equil.
2
2 2 2
CO
CO CO CO
TOT
H
atm
y x , H 25 C 1640
P mol frac

50 mmHg 50 760 0.06579 atm

2 2
Feed
2
7 TOT
CO CO
CO
P 1.0
x y .00035 2.134 10
H 1640

Equilibrium in column:
2
2 2 2 2
CO
CO CO CO CO
TOT
H
1640
y x x 24982 x
P 0.06579


Basis: L = 1 kmol total/h. Assume L & G constant. Input = 2.1341 10
-7
kmol CO
2
/h. 95%
removal = (.95) (2.1341 10
-7
) = 2.027395 10
-7
kmol
2
CO h in outlet gas.
5% CO
2
remains in liquid
7 7
.05 2.1341 10 0.106705 10 kmol
2
CO h

in out
x x outlet liquid mole frac
7
7
0.106705 10
0.106705 10
1 kmol h


b.)
307


c.)
5 5
min
V 1.5 V 1.5 3.803 10 5.704 10 kmol h
Conditions for Kremser eq. are satisfied.
CO
2
Mass Bal:
2,out
7 5 5
CO
2.1341 10 in 0.106705 10 out w. water 5.704 10 y

2,out
7
3
1 CO 5
2.027395 10
y y 3.5543 10
5.704 10

Eq. 12-29
* *
N N 0 0
n x x x x
N
L
n
mV


3
* * 7 N 1 1
N 0
y y 3.5543 10
x 0, x 1.4227 10
m m 24982


7
7 7
5
0.106705 10 0
n
2.1341 10 1.4227 10
N 5.3569
1.0
n
24982 5.704 10

12D.17.
*
1
L L
y .0002 1.414 .0002828,
mV 14.14

Can use variety of forms of Kremser equation, but cannot easily use forms with
*
N 1
y since
*
N 1 N N
y mx and x is unknown and hard to calculate. Try Eq. (12-21).

N 1
*
N 1 1
*
1 1
L
1
y y .0083 .0002828 mV
36.91
L y y .0005 .0002828
1
mV

Do by trial-and-error
L/mV 2 3 2.9 2.91
RHS 15 40 36.699 37.02
Slope 24982
max
Slope
L V

Since L = 1,
4
max
L
2.62968 10
V

5
min 4
1
V 3.803 10 kmol h
2.62968 10

3
max in
7
max in out
y y L 5.3314 10 0
V x x 2.1341 0.10605 10

out
x

in
y 0

y
x
out
7 3
max
y 24982 2.1341 10 5.3314 10
7
in
x 2.1341 10
308


Linearly interpolate L/mV = 2.907. Then, L = (2.907) (14.14) = 41.10 kmol/h.

12.D18. a.
total
p 11.5
y 0.00757
p 1520
,
0.00757
x 0.0127, K y x 0.596
0.0127




0
L, x 0
N
N
x
1 i 0
T
L L
y x y x
V V
slope
T
L 100
2 3
V 150

N 1
V 100 ,
F
V 50, L 100
1
x .01, y .00596

N 1 N
L L
y x y x ,
V V

N 1
y 0.0058
N
150 0.0004 50 .003 100 .0058
x 0.0067
100

0 F F N 1 N 1 N T 1
Ext. MB: Lx V y V y Lx V y
1
y 0.0004,

N 1
L 100
Slope 1.0
V 100

F
V
F
y 0.003
N 1
V
1
Bot. of Column:
309



c. Minimum L.

Pinch is at
F
y .
F F
x y 0.596 0.003 0.596 0.00503

min T min
L V Slope 0.003 0.0004 0.00503 0 0.51653

min T
L 0.51653 V 0.51653 150 77.48kmol h .

N 1
y

min
min
N 1
L
L 100 slope
V

F
y
x
1
y
min min
T
L L
slope
V 150

Eq.
N
x
y
0
x

310

12.D19*. New Problem 3
rd
Edition. Found m and L/V in Example 12-1.
L/(mV) = (L/V)/m = 133/105.6 = 1.259, (mV)/L = 0.794, y
1
* = 105.60 = 0

If use Eq. (12-22), N = {ln[(1 - .794)((100-0)/(10-0)) + .794]}/ln[1.259] = 4.5

12.D20. a) 99% removal
2
H O , 1% left, .000024 1000 .01 0.00024 moles out in L

2 out
H S
0.00024
x 0.00000024
1000

Moles
2
H Sout in gas = (.000024) (1000) (.99) = 0.02376

2 out
H S
0.02376 0.02376
y 0.00691
V 3.44

b) Equil.
2
2 2 2 2 2
H S
2 H S H S H S H S H S
tot
H
26800
H S. y x x 1729.03 x , m 1729.03
p 15.5


2
2 2 2 2 2
CO
CO CO CO CO CO
tot
H
728
y x x 46.9677 x , m 46.9677
p 15.5

Can use Kremser eq. for
2
H S design [dilute linear system]. For example, Eq. (12-28)

2
*
0 N
*
N N
H S
x x L L
n 1
mV x x mV
N
mV
n
L


2 2
H S H S
L 1000 mV
0.16813, 5.9479
mV 1729.03 3.44 L



2 2
*
N N 1 0 N
H S H S
x y m 0, x 0.000024, x 0.00000024

.000024
n 1 0.16813 0.16813
.00000024
N 2.4807
n 5.9479

c) For
2
CO .
2 2
*
N N N 1,CO CO 0
x unknown, x y m 0, x 0.000038 Kremser (12-31)
2
2
2
2
*
CO
N N
N 1 *
CO 0 N
CO
CO
mV
1
L 46.9677 3.44 x x mV
, with N 2.4807 and 0.16157
x x L 1000
mV
1
L


2
N,CO 0 2 N 1 3.4807
2
mV
1
1 0.16157
L
x x 0.000038 0.000031916 CO
1 .16157 mV
1
L
Little amount of CO

311


12.D21. New Problem 3
rd
Edition. Abs.
23.78
y x 4.756x
5

Stripper
23.78
y x 118.90x
0.2



Abs






Abs
735.302
L V 7.353
100

External M.B.
abs abs,IN abs,IN abs,out
abs,out
abs
L x V y y
x
L


abs,out
735.302 0.00001 100 0.00098 0.000079
x
735.302



abs,out strip,IN
x x 0.00013253
Kremser Eq.
7.353 mV
m 4.756, b 0, L mV 1.5460, 0.64681
4.756 L


N 1 IN 1 out
y y 0.00098, y y 0.000079


*
1 IN
y mx 4.756 .00001 0.00004756
Eq. (12-22).

*
N 1 1
*
1 1
y y mV mV
0.00098 0.00004766
n 1
n .35319 0.64681
L y y L
0.000079 .00004756
N
L n 1.5460
n
mV



abs
n 11.1216
2.40889
N 5.53
n 1.5460 0.4357


y
x

EQ
x 0.00098 4.756 0.00020605

4.756 slope equilibrium
IN
y 0.00098
out
y 0.000079
act abs,MIN
L 1.6 L 735.302 kmol h

IN
x 0.00001
MIN
L 100 4.5956 459.56 kmol h

0.00098 0.000079
Operating line Slope 4.5956
0.00020605 0.00001

MIN
Slope L G
312

Stripper







MIN
Strip
L 0.015758 0
128.6037
V 0.00013253 0.00001


MIN,Strip Strip
735.302
V L 128.6037 5.718 kmol h.
128.6037


Strip
V 1.5 5.718 8.576 kmol h.

Strip Strip IN Strip Strip IN Strip out
Strip out
Strip
V y L x x
y
V

0 735.302 0.00013253 0.00001
8.576


*
Strip,IN Strip,out N N 1
y y 0.010506, x 0 y 118.9

Kremser (12-28)

*
0 N
*
N N
x x L L
n 1
mV x x mV
L 735.302
N , 0.721106
n mV L mV 118.90 8.576


0.00013253 0
n 0.27889 0.721106
0.00001 0
N 4.54
n 1.38676


12.D22. New Problem 3
rd
Edition. K 0.22. y Kx 0.22x. Plot e.g., at x 0.006, y 0.00132
L 75, V 150, L V 0.5
External M.B.:


N 1 0 N 1
Lx Vy Lx Vy

N N 1 1
V
x y y 2 0.003 0.0004 0.0052
L

Points
0 1 N N 1
x , y 0, 0.0004 , and x , y 0.0052, 0.003 are on op. line.
Plot Op. Line. See graph (labeled 12.D.b). 2 stages more than sufficient.
EQ
y 118.90 0.00013253 0.015758
y
IN
y 0
118.90
IN out ,abs 0
x x 0.00013253 x
118.90
MIN
L
V

N out
x x 0.00001

118.90 = Slope Equilibrium
= Slope Operating line
313




314


12D23. Apparatus similar to Figure 12-2, except part of treated gas is heated and used as stripping gas.

Absorber: Work in terms of mole ratios.

i n out in
Y .15 .85 0.1765, Y 0.005 .995 0.00502, X 0.005 .995 0.00502
G 1400 .85 1190 mol carrier gas/day, L 800 .995 796mol solvent/day
Equilibrium: y .5x . Convert to mole ratios
x X y Y
0 0 0 0
.05 .0526 .025 .0256
.1 .1111 .05 .0526
.15 .1765 .075 .0811
.2 .25 .1 .1111


Plot ratios on McCabe-Thiele diagram
Absorber: Op. Line:
out in
L L L
Y X Y X , 0.669
G G G

External balance Absorber:
out
796 .00502 1190 .1765 1190 00502 796 X

out Abs
X 0.2614
Step off stages as shown in figure. Need 8 equilibrium stages.

Stripper:
in out abs
Y .00502 same as Y ,
in out abs
X 0.2614 same as X

out in abs
X 0.00502 same as X , L 796 mol solvent day, have 4 stages
Stripper equilibrium: y = 3 x. Convert to mole ratios.

x X y Y
0 0 0 0
.025 .0256 .075 .0811
.05 .0526 .15 .1765
.075 .0811 .225 .290
.1 .111 .3 .4286
.15 .1765 .45 .818


Problem is trial-and-error. Select
out
Y . Draw operating line from
out in
Y , X to
in out
Y , X .
See if need 4 stages. When need exactly 4 stages, L/G = slope.

From Figure, final result shown:
strip
G L slope 796 1.0998 723.7mol carrier gas/day.

out strip
Y ~ 0.287 mol ratio

315







316



12.E1. Convert ppm(wt) to mole fraction:
6
ppm 10 wt frac
Basis 1000 g. of steam: Feed Liquid 1000 ppm = 0.001 wt frac

mole frac
lg Nitrobenzene 0.00812 gmole 0.0001465
999 g water = 55.4507 0.9998535
Total 55.4588 mol

1000g
Avg mol. wt 18.0314
55.4588

Liquid Water F 1726 g h 95.7219 mol h

out
L F C where C S V 99 61.8 37.2g water , C 2.0648 mol
L 95.7219 2.0648 97.7867 mol h
Outlet: Basis 1000 g

6
28.1 ppm 28.1 10 wt. frac. moles Mol frac.

3 4
28.1 10 g nitrobenzene 2.2825 10
6
4.11 10

3
1000 28.1 10 g water
999.9997 18.016 55.5062
55.5064 mol
~ 1.0
V S 99 18.016 5.495 mol h .
Equil.
tot
y mx b, b 0, m H p 28.0
Kremser Eq. Several forms can be used. Use Eq. 12-28.

L 97.7867 mV
0.63555, 1.5734
mV 28.0 5.495 L
,
* N 1
N
y b
x 0
m


*
0 N
*
N N
x x L L
n 1
mV x x mV
N 5.763
mV
n
L


Eq
act
N
5.763
Effic 0.524
N 11

Ref. Hwang et al, IEC Research, 31 (7) 1992, 1753 & 1759.

12-F1. H = 59.3 (Perrys 4
th
ed., p 14-4).
TOT
H 59.3
y x 11.86x
P 5

To be absolutely correct should convert this to mole ratios, although at these low
concentrations could use mole fractions with small error.

y x
Y , X
1 y 1 x


317

x y X Y
0 0 0 0
.001 .01186 .001001 .012
.0015 .01779 .001501 .01810
.002 .02372 .002003 .0243
Change specified conditions to mole ratios.
in in in in out out out out
x 0, X 0; y .02, Y .02041; y .002; Y .002003; x .001, X .001001
See Figure for plot of operating line, equilibrium and stages. N = 3.3

Height 5 ft
HETP 1.515
N 3.3 equil stage


in out
out in
Y Y L .02041 .002003
18.4
G X X .001001 0

If use mole fractions find L/V = 18.0

12.F2.
E p
K 26.0, K 0.6, K 0.019
After one pass of mass balance obtain:
318

ethane pentane octane
x
i,1
0.032 0.005 0.963
x
i,2
0.035 0.031 0.934
x
i,3
0.033 0.162 0.805
y
i,1
0.975 0.003 0.021
y
i,2
0.962 0.019 0.019
y
i,3
0.885 0.099 0.016


For new temperature used multi-variant Newtonian convergence.

1,New 2,New 3,New
T 73.90 F, T 80.93 F, T 99.15 F.


2.G1. a)
A,out
N 4, L 570, y 0.00317
b)
A,out A
N 8, L 500, y 0.00315 while with L 490, y 0.00358
c)
A,out A
N 16, L 490, y 0.002978 while with L 480, y 0.00337
d) Have a pinch point.

12.G2. Used Peng-Robinson.
a. Total number of stages required 8
b. Feed stage location for the solvent 1
c. Feed stage location for stream A 8
d. Feed stage location for stream B 6
e. Outlet mole fractions of gas stream leaving absorber 0.9991009, 0.00024691,
0.00019067, 0.0004617
f. Outlet mole fractions of liquid leaving absorber 0.012878, 0.0310992, 0.0198928,
0.93612992
g. Outlet gas flow rate 161.6478 kmol/h
h. Outlet liquid flow rate 213.352 kmol/h
i. Highest temperature in column 19.1287 C and stage it occurs on 8

12.G.3. New Problem 3
rd
Edition. Used NRTL.
Column pressure = 1.0 atm. Feed gas flow rate = 752 kmol/h. Feed gas temperature = 100
o
C.
Liquid feed temperature = 75
o
C. Recovery of isopropyl alcohol = 0.98000.
T
1
= 302.5K, T
2
= 298.6K, T
3
= 299.9K, T
4
= 301.9K, T
5
= 303.3K.
Leaving gas: G = 802.2 kmol/h, Mole fractions: IPA = 0.02443, W = 0.033836, N
2
= 0.93721
Leaving liquid: L = 149.8 kmol/h, Mole fractions: IPA = 0.002671, W = 0.99638, N
2
= 0.000950
Column diameter = 1.5455 m.



319
SPE 3
rd
Edition Solution Manual Chapter 13
New Problems and new solutions are listed as new immediately after the solution number. These new
problems are: 13.A12, 13.A13, 13.D3, 13.D5, 13D6, 13D10, 13D22, 13D30-13D34, 13D36-13D42,
13.E2, 13.E3, 13.G1, 13.G2 . Chapters 13 and 14 from the 2
nd
edition were rearranged to place all the
extraction material into chapter 13 and the material for other separations in Chapter 14. Thus, the
numbers of many problems have changed.
13.A3. The amount of solvent should be increased. This will decrease F/S and move the mixing
point M towards S. As a result the saturated extract product
N
E will be moved down
(less solute). The difference point will be moved towards the triangular diagram. The
combined effect will be that fewer stages are required. By adjusting F/S a condition
requiring exactly two stages can be found.
13.A5. The vertical axis will be the extract phase and the hypotenuse will be the raffinate phase.
These will be connected by tie lines. Usual procedure can be used.
13.A7. Situation where E = R and point is at infinity. All operating lines are parallel.
However, this does not correspond to minimum number of stages in extraction.
13.A.11 c.
13.A12. a. C will be spread out and go into both raffinate and extract streams.
b. C will concentrate around the feed edge. If C is very dilute in the feed, can concentrate C.
Then by stopping the feed but continuing to flow solvents, solutes A and B can be
removed. Solute C can now be collected by withdrawing a stream near the feed stage.
13.B1. Specify:
o N N
A B A B A B
T, p, z , z , F, x , x , y , y plus:
1
N
N
1 N
1
N
N
B F
B F
A F
B A
F A
F B
F A
y , R, E, N
x , R, E, N
x , R, E, N
R, E, y , x
N, N , y , E
N, N , x , R
N, N , x , R, etc.
Could also not be given one of standard variables (such as solvent concentration).
13.B2. a). One can build stages which are cross-flow (e.g. see Figure 12-12) within a counter-
current cascade. This effectively increases stage efficiency. Not that upward flowing less dense
liquid will be mixed.
b.) Build chambered stages within a counter-current cascade to prevent mixing of the
dense liquid and give better cross flow on each stage.
c.) Put in baffles to prevent MIXING of both less and more dense liquids. This will be
more effective if counter-current is arranged so that flow across stages is always in same
direction (see sketch)
320


13.C.7. Start by defining and the coordinates of as:

o 1
E R ,
o 1
A o A 1 A
x E y R x ,
o 1
D o D 1 D
x E y R x
Removing from 2
nd
and 3
rd
equations we obtain

o 1
A o A 1 A o 1
x E y R x E R (13-43a)

o 1
D o D 1 D o 1
x E y R x E R (13-43b)
Assume that
o 1
E R . Next write the three independent mass balances around stages 1 to j.

j 1 j
R E ,
j 1 j
A j 1 A j A
x R x E y ,
j 1 j
D j 1 D j D
x R x E y
These equations are now in a form similar to the form of the mixing equations developed
previously. To develop the three point form of a straight line use the first equation to remove
from the other two equations, solve for
j 1 j
R E in each of these equations, and finally set the
results equal to each other. The development proceeds as follows:

Use
j j 1
E R to remove from the other mass balances.

j 1 j
j j 1 A j 1 A j A
E R x R x E y ,
j 1 j
j j 1 D j 1 D j D
E R x R x E y
Solve for
j 1 j
R E ,
j
j 1
A A
j 1
j A A
y x
R
E x x
,
j
j 1
D D
j 1
j D D
y x
R
E x x

Finally, set these equations equal to each other.

j j
j 1 j 1
A A D D
A A D D
y x y x
x x x x
rearrange to:
j j 1
j j 1
A A A A
D D D D
y x y x
x x x x

This last equation says that the slope of the line between the points
j j
D A
y , y and
D A
x , x is equal to the slope of the line between the points
j 1 j 1
D A
x , x and
D A
x , x and
thus the lines are colinear. Furthermore, the lever-arm rule is valid for this system.

13.D1. a. If we have a single column with only pure solvent then
321


N N 1
R R
y x x y
E E
and point
N N 1
x , y 0.001, 0.0
is on op. line. E 44, R 100 , Slope R E 2.273
Op line intersect equilibrium at x 0.0037 . Thus, cannot get to
o
x 0.012.
b. Now E 44 30 74 and R E 1.35.
Still want
N
x 0.001, but
N 1
44 0 30 0.004
y 0.00162
74

At x 0.001, equilibrium value of y 1.613 x 0.001613 . Alternative works, but
have pinch point and need very large number of stages.

c. This alternative (Say use 25 kg/min of 0.004 butanol)

N 1
44 0 25 .004
y 0.00145
69
which is below equilibrium point.
Now
equil
R E 100 69 1.449 m 1.613 Thus, this will work. Obtain
1
0.00145 69 0.012 100 0.001 100
y
69
0.0174
Op line closer to equilibrium require lot more stages.
If use 20 kg/min of 0.004 butanol:
N 1
20 0.004
y 0.00125
64

R E 100 64 1.5625 ,
1
0.00125 64 0.011 100
y 0.01844
64

Will also work. Becoming close to pinch at top
*
1
equil y 0.019356
If 15 kg/min,
N 1 eq
15 .004
y 0.0010, R E 100 59 1.6949 m
59


*
1 1
1.0010 59 0.011 100
y 0.1964 y 0.19356
59

Wont work.
Thus, there is a small range where option c will work, but with many stages.

13.D2.
IN F IN
R 20, E 20, x x , y 0, m 8.3333, b 0, N 2
Kremser equation
R 20
0.12
mE 8.333 20
,
*
1 0 F N+1
y mx b 8.3333 x , y 0
Eq (13-11)
*
1 1
N 1 *
N 1 1
R
1
y y
mE
y y
R
1
mE


1 F F F 3
1-0.12
y 8.3333 x 8.3333 x 7.3460 x
1- 0.12
,
1 F
y 0.9873 x
322

Mass balance,
F N 1
Rx Rx Ey ,
F F
N F
20 x 19.746 x
x 0.01269 x
20

Recovery =
N F
1 x x 0.9873, which is higher than 0.963 obtained in cross-flow.

13.D3. New Problem in 3
rd
edition.
From M.B.
out
R .013 E .001 R .007 E y

Where R = 100 and the unknowns are E and y
out
.

and Equilibrium:
out out
y 1.613 x 1.613 .007 0.01129



out
R .013 .007 100 0.006
S E 58.309kg h
y 0.001 0.01129 0.001


Alternative Solution: 1 Equilibrium Stage

















Another alternative solution:
y 1.613x
1.613
R
E

x
out
x .007
.001
out
y 0.01129

y
from graph
R 0.001 0.01129
1.715
E 0.013 0.007
R
E 58.309
1.715

.013
323


If Eq. line is straight, can Use Kremser with N=1.
Both representations are correct. Treating similar to a flash is easier.


13.D4. Since concentrations are low, use wt. fractions and total flow rates.
Equilibrium: y 0.828 x or m 0.828

0 N N 1
R 550 lb h, E 700 lb h, x 0.0097, x 0.00046, y 0.0003,
mE R 1.0538 and R mE .94893


*
1 IN
y mx b 0.828 .0097 0 0.00803
Kremser Eq. (13-11b),
*
N 1 1
*
1 1
y y mE mE
n 1
R y y R
N
R
n
mE


1 N 1 0 N
R R
y y x x
E E
550 550
.0003 0.0097 .00046 0.0075
700 700


.0077316
n .0538 1.5038
.0004716
N 33.6
n .94893


13.D5. New problem in 3
rd
edition.

Part a) Can do with Kremser eq or graphically.
y m x b, m 0.828, b 0

R 400
R 400, E 560, 0.862664
mE 0.828 560


0 N N+1
mE 1
x 0.005, x 0.0003, y 0.0001, 1.159
R .862664

Or

1
E, y
1
R, x


2
E, y
1
0
R, x
1
x
R E Slope
2 1,IN
y y
0
x x
x
y
1
y
Op. Line:
2 1 1 0
R R
y x y x
E E

Points
1 2 0 1
x , y , x , y
On Op. Line
But
1 1
x and y unknown
T & E in this configuration
Slope
R
E
known, N = 1
where
2 1,in
y y
Eq.
324

Since
R
1.0
mE
, can use equation such as 13-11b

*
1 0
y m x b 0.828 0.005 0.00414

*
N 1 1
*
1 1
y y mE mE
n 1
R y y R
N
R
n
mE


0.0001 0.00414
n 1 1.1592 1.1592
0.0034571 0.00414
N
n0.862664

where
1 N 1 0 N
R 400
y y x x 0.0001 0.005 0.0003 0.003457
E 560


1.52637
N 10.332
0.14773

Alternate solution: Eq. (12-28) becomes

*
0 N
*
N N
x x R R
L R, V E , N n 1 n mE R
mE x x mE


* N 1
N
y b 0.0001
x 0.000120773
m 0.828
,
1.52637
N 10.332
.14773

b)


13.D6. New problem in 3
rd
edition.
Part a.
N 1 0 1 N
Ey Rx Ey Rx Ext. M.B.

1 0 N 1 N
R R
y x y x
E E


1
100 100
y 0.005 0.0002 0.0005 0.003414
140 140

Part b. Kremser Eq.
*
1 0
y 0.828 x 0.828 0.005 0.00414
*
1 N 1
min 0 N
y y R 0.00414 0.0001
0.85957
E x x 0.005 0.0003

MIN
400
E R 0.85957 465.3kg h
0.85957

Equil.
Slope = 0.828
min
R E slope

0
x 0.005
N
x 0.0003

N 1
y 0.0001

y
325

Convert
* N 1
N
y 0.0002
x x L R y y V E, x 0.00016556
m 1.208


L R 100
0.5913
mV mE 1.208 140

Lots of different forms can be used.
For example
*
0 N
*
0 N
x x L L
n 1
mV x x mV
N
n mV L

Becomes
*
0 N
*
N N
x x R R
n 1
mE x x mE
N
n mE R


0.005 0.00016556
n 0.4087 0.5913
0.0005 0.00016556 1.8717
N
1 0.5254
n
0.5913
3.6
Part c.

















MIN
R 0.00604 0.0002
1.29777
E 0.005 0.0005


MIN
R 100
E 77.05
1.29777 1.29777

Maximum extract out
EQ 0
y x 0.00604.
Part d. The roles of extract and diluents are switched in the two problems, which changes the definitions
of y and x.

13.D7.
N N 1 0
N 30, R 500, y 0.0002, x 0.0111, x 0.00037
Equilibrium:
*
N N 1
y 0.828x, m 0.828, x y .828 0.00024155
0
x 0.005
MIN
R
slope
E

Eq. y 1.208x
x
N
x 0.0005

N 1
y 0.0002
y 1.208x
y
EQ
y 1.208x 0.00604

326

Since rather dilute and linear equilibrium use one of the Kremser equations.

*
o N
*
N N
x x R R
n 1
mE x x ME
N
mE
n
R
(12-28 (modified))
Where
*
0 N
*
N N
x x
83.756
x x
. Solution is trial-and-error.



E R/mE Calculated N
500 1.21
Negative-Not possible Need
R
1
mE

700 0.8626 17.01 E too high
650 .929 26.175 E too high
640 .9435 29.84 E too high
639 .945015 30.30 E too low

By linear interpolation need E ~ 639.6 kg/h. Can use other forms of the Kremser equation.

13.D8. Was 13.D10 in 2
nd
edition.
Convert Kremser
x is raffinate R L
y mx m K, b 0
y extract, V E

a)
*
N N
N 1 3 *
0 N
Use 12-31
x x 1 mV L 1 K E R
Other forms OK x x
mV KE
1 1
L R


*
N n 1
x y K 0

N 0 3 3
30.488 25
KE
1 1
100 R
x x 0.00092 0.00001376
KE
30.488 25
1
1
R
100

b) Can use External balance or Kremser to find
out 1
y y

1 N 1 0 N
Ey Ey R x x

1 0 N
R 100
y x x 0.00092 0.00001379 0.003625
E 25


13.D9. Assume very dilute, R = 1500 kg/h, E = 750 kg/h
Equil.
d d
Y K X becomes y K x
From Table 13-3.
d,oleic oleic d,linoleic d,linoleic
K m 4.14, K m 2.17
99% recovery oleic:
1,oleic
.99 .0025 1500 y 750
1,oleic
y .00495
327

Use Kremser, Eq. (13-11b).
oleic
4.14 750 m E
2.07
R 1500


*
1 oleic 0,oleic
y m x 4.14 .0025 0.01035

*
N 1 1
*
1 1
y y mE mE
n 1
R y y R
N 5.44
R
n
mE

For linoleic acid:
lin
R 1500
.9216
m E 750 2.17
,

N 1
y 0 ,
*
1 L 0,L
N 5.44, y m x 2.17 003 .00651
Can use Eq. (13-11a):
*
1 1
N 1 *
N 1 1
R
1
y y
mE
y y
R
1
mE


1
.07834
y .00651 .00651 .00124796
.40866

linoleic
1
y 0.00526
Recovery of linoleic: Rec .003 1500 .00526 750 Rec = 0.877
13.D10. New problem in 4
th
edition.


Analytical or graphical solution OK.
Stage 1
1
1 F i 1,in 1,out 1 1
Fx E y Ey R x
Equilibrium
1,out 1
y 1.02 x

1
F 1 1
Fx 1.02 E R x

F
1
1
100 0.015 F x
x 0.0099338
1.02 E R 1.02 50 100

Stage 2
1 1 2 2,in 2 2,out 2 2,out 1 2 1
R x E y E y R x , R R F 100

2,out 2,out
y 1.02 x


1 1
2,out 2 1 1
2 2
0.0099338 100
R x
x 0.006579, R R F 100
1.02 E R 151

Mix with Feed 2
3,in
R 100 70 170
328


2
2,out 2 F 2
3,in
3,in
x R x F 0.006579 100 0.005 70
x
R 170


3,in
x 0.0059286
Stage 3
3 3,in 3 3,in 3 3,out 3 3,out
R x E y E y R x

3,out 3,out
y 1.02 x

3 3,in
3,out
3 3
R x 170 0.0059286
x 0.00456
1.02 E R 1.02 50 170

Stage 4
4 4,out 4 4,in 4 4,out 4 4,out 4 3
R x E y E y R x , R R 170

4,out 4,out
y 1.02 x

4 4,out
4,out
4 4
R x 170 0.00456
x 0.003508
1.02 E R 51 170


4,out 4out
y 1.02 x 0.003578

329




330



13.D11. R R F 2501, E E 1000
Equilibrium:
D
K 1.57 . For dilute this becomes
D
m K in wt. frac. units.
Abietic Acid Recovery:
N F
x R .95 F x .95 1.0 0.5 .0475

N
.0475 .0475
x 0.0000190
2501 R
,
F
1
.05 1.0 .05
.05 F x
y 0.0000025
E 1000

Top op. Eq.:
1 0
R R
y x y x
E E

Goes through pt
0 1
R 2500
x 0, y . Slope 2.5
E 1000

Bottom Op. Eq.:
N 1 N
R R
y x y x
E E
through point
N N 1
x , y 0 , R E 2.501
Need 8 stages (see Figure).



13.D12. m 1.613, R mE 1.2399 ,
*
N 1 4 1 in
y y 0, y mx 0.00742
Eq. (13-11a)
1
4
y 0.00742 1 1.2399
0.17594
0 0.00742
1 1.2399


1
y 0.17594 0.00742 0.00742 0.006114
Overall bal.
in in out
3
10 0.0046 5 0.006114 Rx Ey Ey
x 0.00154
R 10


13.D13. a. xy . 90% recovery, 10% left 0.1 1000 0.003 0.3 kg out

,out
x 0.0003
331

O xy 95% recovery, 5% left 0.05 1000 0.005 0.25,
O,out
x 0.00025




For ortho,
max
y 0.15 0.005 0.00075

max,ortho
R 0.00075 0
0.1579
E 0.005 0.00025

For para
max
y 0.08 0.003 0.00024

max,para
R 0.00024 0
0.08888
E 0.003 0.0003

b. The p-xylene recovery controls.
min
1000
E 11, 250
0.08888

E 1.5 11250 16875 ,
R
0.0592592
E

Can use Kremser eq. (13-11b) for -xy to find N

*
N 1 1
*
1 1
y y mE mE
n 1
R y y R
N
R
n
mE


N 1
m 0.080, R E 0.0592592, y 0,
*
1 o,p
y mx 0.080 0.003 0.00024
Mass balance: 90% entering -xy leaves w. solvent.

1
0.9 1000 0.003
y 0.00016
16, 875
wt frac

R 0.0592592 R
0.74074, n 0.300106
mE 0.080 mE
,
mE .080
1.35
R 0.0592592

332


0 0.00024
n .35 1.35
n 0.30 0.00016 0.00024
N 4.012
0.300106 0.30016

Note: Can use other forms of Kremser eq if desired.
c. For o-xy check if recovery > 95%
Eq. (13-11a)
*
1 N 1
1 1
* N 1 *
1 0 N 1 1
R
1
y unknown, y 0
y y
mE

y mx 0.15 0.005 0.00075 y y
R
1
mE


R 0.0592592
0.39506, N 4.012
mE 0.15

* *
1 N 1 1 1 N 1
R
1
mE
y y y y
R
1
mE
5.012
1 0.39506
0.00075 0.00075 0.0029194
1 0.39506

External M.B.
N 1
1 N 0
Ey R x Ey R x

0 1
N
5 16875 .00029194 Rx Ey
x 7.3584 E 5
R 1000


1
0
Ey
% Recovery 100 98.53%
Rx

13.D14. (was 14.D4. in 2
nd
ed.) a)
S 10.0 2 MF
F 15.0 3 SM


Once have M, use trial-and-error to find tie through M. (final result is shown). This
gives E and R.
A w A w
y .115, y 0.04, x .23, x .73.
b) Plot raffinate,
A
R x .1 . Find tie line through this point (not trial-and-error). This gives
E. Draw Line ER. Intersection with line SF gives M.

S S MF
. Find S 85.7
F 15.0 SM
kg/h.
333


13.D15. Since dilute, use Kremser equations. Assume units are weight fractions.
a) Column 1 at 40C.
N 0
x 0.0008, N 11, , x 0.01, E 1000, , R 100
Equilibrium:
*
1 0
m 0.1022, thus y mx 0.001022. Kremser (Eq. 13-11a):

1
1 2
N 1
1
1
y 0.001022
1.022
0.93664
y 0.001022
1
1
1.022

This simplifies to:
1 N 1
y .093664y .00092628
External MB:
N 1 o 1 N
y E Rx y E Rx ,
N 1 1
y 1000 1 1000 y .08
which simplifies to:
N 1 1
1
y 1000 y .92
1000

Solve 2 eqs and 2 unknowns:
5
1,coll N+1,coll
y 0.00092693, y 0.6929 10
b) Column 2 at 25C:
N 1,col2 1,col1
y y 0.00092693 ,

5 *
1,col2 N 1,col1 0 1 0
y y .6929 10 , x 0, N 9, m 0.0328, E 1000, y mx 0

Use Kremser to solve for R. This is trial and error. For example, Using Eq. (13-11a),

*
1 1
N 1 10 *
N 1 1
R
R
1
1
0.0328 1000 y y
mE
0.007475
y y
R R
1 1
mE 32.8


R 50 60 50.5 50.35
RHS 0.007855 0.001981 0.007307 0.007467

Within error R = 50.35

N 1 0 1
N
y E R x y E .92693 0 .006929
x 0.0183
R 50.35


334

c) Could be practical if ms were larger, and have bigger shift in m. A similar scheme is used
commercially for citric acid. Not practical here since have to pump around too much
solvent. In addition, benzene is carcinogenic and would probably not be used as solvent.

13.D16. a.) R E 10 8 1.25, R mE 1.25 1.613 0.77495

0
*
1 A
y m x 1.613 0.01 0.01613. Use Eq. (13-11a),

1
7
y 0.01613 1 0.77495
0.27044
0.0002 0.01613
1 0.77495

1
y 0.01182

0
A A N 1 1
E E
x x y y 7.02498 E 4
R R

b.) Graphical check works fine (not shown)

13.D17.
j j IN j 1
j 0 j
R R R 10
y x y x , 5
E E E 2


6
x 0.0018 (See graph)
Note:
6 N,countercurrent
x x 0.000702 even though use more total solvent.

335




13.D18. (was 14.D2. in 2
nd
ed.) Plot S, F, R and E. Draw lines SF and RE. Intersection is point M.

Lever arm rule:
S MF 20.3
4.511
F 4.5 SM
S 100 4.511 451.1kg/h

Or Mass Bal. S + R = M and
A A A
S y F x M x (S .15 .5 F .21 M)

Solve simultaneously S = 483.3

336

Difference is due to accuracy in reading numbers. Lever-Arm Rule more accurate!




13.D19. Equil.
d A A
K y x 0.287 0.158 1.816
Acetone
0 0 1 1
0.005
y 0 Y 0, x 0.005 X 0.00503
0.995


N 1 N+1
0.10
x 0.10 wt frac X 0.1111
.9


D
F 1000 .9 900 kg/h water,
S
F 1371 .995 1364.1 kg/h chloroform.

D S
900
F F 0.6598
1364.1

Equil.
337

x
A
X
A
y
A
= 1.86 x
A
Y
A
0 0 0 0
0.01 0.0101 0.01816 0.01850
0.03 0.0309 0.05448 0.0576
0.05 0.0526 0.0908 0.09987
0.07 0.0753 0.1271 0.1456
0.09 0.9890 0.1634 0.1954
0.1 0.1111 0.1816 0.2219


External M.B.

D
N 1
S
F
X Y
F
D
6 1 N N
S
F
X Y or Y 0.6598 0.1111 0.6598 .00503
F


N
N
N
Y
0.06999, y 0.0655
1 Y

Results pretty close to 13.D43.
1 2
2 vs 2 w i
2 3
accuracy of graphs.

Note: The graph below should read acetone, not acetic acid as the solute.

338




13.D20. a) Batch Operation Mix together & settle. Find fraction recovered:
Operating Eq.:
0 0 F
R R
y x x , R 5, S 4, x x
S S

Which is,
F
y 1.25 x 1.25 x
Equilibrium y 8.333 x, m 8.333
Eq. (13-21) written for batch
0 iN
F
F

R S x y
1.25 x 0
x 0.1304 x

9.583
m R S

Frac. Rec 1 0.1304 0.8696
b) Continuous solvent addition:
Eq. (13-28)
t ,final
t ,final t ,feed
F t

x
S 1 1
n x x 0.8 n

m 8.333 x R


t ,final F
x x exp 0.8 8.33 0.00127
Recovery = 99.87%.

339

13.D21 (Was 14.D1 in 2
nd
ed.) a. Let A = methylcyclohexane and D = n-heptane.
Mass Balances:
1 2
F F S M or M 350

F F S M
1 2
1 A 2 A A A
F x F x S y M x ,
F F S M
1 2
1 D 2 D D D
F x F x S y M x

Then
F F S
1 2
M
1 A 2 A A
A
Fx F x S y
100 .6 50 .2 0
x 0.2
M 350


F F S
1 2
M
1 D 2 D D
D
Fx F x S y
100 4 50 .8 0
x 0.229
M 350


Plot M. Find tie line through M. (See figure.) This gives location of points E and R.
Find
DR AR AE DE
x 0.48, x 0.42, y 0.06, y 0.05.


b. Mass balances:
M E R
A A A
M E R and Mx Ey Rx
Solving simultaneously: E = 214 and R = 136 kg/h

13.D.22. New problem in 3
rd
edition. With interface at center, heavy phase flow area is
2
f s
1
A D 4 0.411
2
and
f
per s 5
1
P D D 2.630
2
















2
2 2
C
r .1
2

C 1.00326 m

r
.1
C/2


s
r D 2 0.5115
Center
.1
Chord
arc
.4115
Interface
(length = C)

340

Draw right triangle for interface below center to calculate new perimeter.

0.1 .1
sin .1955
r .5115

11.274
Then angle of arc, 180 2 157.452

Length of
3.14159 0.5115 157.452
r
arc 1.4056
180 180


f
er
P C arc length 2.4089m

Mensuration formulas are from CRC Standard Mathematical Table.

f
3
settler c er c
Re 4Q P 4 0.006 998 2.4089 0.95 10 10, 466


Interference somewhat more likely than in Example 13-5.


13.D23 (was 14.D7. in 2
nd
ed.) a) F S 500 300 M
Pyrdine
F O M
A A A
F x S y 500 .3 0 M x
M
A
x 150 800 0.1875
Plot M on line FS . By T & E find tie line through M (Use Conjugate line)

p p
y 0.223, x 0.84 ;
w w
y 0.02, x 0.84 ;
Mass balances:
1 1
R E M 800 , 0.84R 0.02E 0.43M
Solve simultaneously,
1 1
E ~ 400, R ~ 400 (Note: More accurate than pyrdine values.)
b)
1 2 2
R S 400 300 700 M

1 0 2
1 A 2 A 2 AM
R x S y 400 0.15 60 M x

2
pyr M
60
x 0.086
700


Find tie line by T & E:
pyr2 pyr2
y 0.120; x 0.053 ;
w2 w2
y 0.005, x 0.945

MB:
2 w2 2 w2 m2w
R x E y M x 0.945
2 2
R 0.005 E 700 0.48


2 2
R E M 700

Solve simultaneously:
2
E 346 and
2
R 354

341


342

13.D24 (was 14.D10. in 2
nd
ed.) a) Feed 40% MCH 55% n-heptane, F = 200. Solvent 95% aniline
& 5% n-heptane,
total
S 600. S F M 800
Lever arm rule:
S FM
3 .
F MS
Find M (Easy way is divide line FS into 4 parts)

Use tie line through M to find points E & R (T & E)

Extract:
MCH MCH
y ~ 0.045, Raffinate:x ~ 0.36wt fracs

Mass balance E + R = 800 = M and lever arm rule
E MR
.
R ME
Measure distances on figure.

Solve simultaneously: R = 124.61 kg/h, E = 800 R = 675.39

b) 2 stage cross flow. Stage 1: F = 200, = 300,
S 3 FM
F 2 MS
. Find point M. Tie line through M
gives points R
1
and E
1
.
Mass balance
1 1
500 F S M R E and lever arm rule
1 1 1
1 1 1
R E M
E M R

Find:
1 1
R 207.04 kg h, E 292.95

Note: Isotherms are very sensitive. Thus, calculation is not extremely accurate.
Stage 2: Mass balance
1 2 2 2 2
R S M 507.04 R E and lever arm
2 2 1
1 2 2
S M R
E M S

Find
2
M and from tie line through
2 2
M find R . Then can find R
2
and E
2
from mass balance
(given above) and new application of lever arm rule,
2 2
2 2
R E M
E R M

Solving simultaneously,
2 2
R 196.16 kg h. E 310.88

343




13.D25 (was 14.D9. in 2
nd
ed.) a. Draw lines from S to F and from
1 N
R to E . Intersection gives point
M (see Figure). Then from lever-arm,


S FM
1.25
F SM
S 1.25 2000 2500

b. is at intersection of lines
N N 1 0 1
E R and E R . Then step off stages as shown. Need 2 stages.


344


13.D26. (was 14.D6. in 2
nd
ed.) Guess a value for M and step off stages. Repeat until need 3 stages.
After three trials found M shown in Figure. This required 3 1/10 stages which is close enough.
Extract Composition: Acetic Acid = 10.5%, Water = 3.5%.
Raffinate Composition: Acetic Acid = 5%, Water = 93%
Solvent Flow Rate: F S F SM SF 15 57 2000 S 2000 S = 5600 kg/h
Raffinate Flow Rate:
1 0 0 1 1 1
R E E R R 5600, R 770kg/h.
Extract Flow Rate:
N 1
E F S R 2000 5600 772 6830

13.D27 (was 14.D12. in 2
nd
ed.)
E 0
0
A wE
y y 0 (Pure solvent)
Lever arm rule:
0
0
E MF
1.112
F E M
. Find M. Line RM intersects satd extract at
N
N A
E , y 0.18
Lines
N 1 0
F E & R E intersect at . Step off stages 3more than enough. Need ~ 2
M.B.
N 1
E R M 211.2 kg/h & lever arm:
N 1
1 N
E MR
2.287
R E M
(from graph).
Solve simultaneously,
1 N
R 64.25, E 146.95 kg/h
345


346

13.D28 (was 14.D14. in 2
nd
ed.) To find : 1) Plot
N N 1
E and R F
2)
j j 1 N N 1
E R E R 1500

N N 1
N A N 1 A
A
E x R x
x 0.06666
3) is on line through points
N N 1
E and R .
Plot . Or, use lever-rule.

N 1 N
N 1
N
R E
R 1.5
E

Step off three stages starting at point
N
E . This gives points

1 1 1 0
1 A D 0 A D
R x 0.275, x 0.675 and E y .13, and y 0.0 .

Mass Balance:
0 N 1 1 N
E R R E
0 1 N N 1 1
E R E R R 1500
and
0 1
E 0.13 1000 0.4 R 0.275 2500 0.2
Solving simultaneously,
1
R 655 kg/h,
0
E 2155 kg/h

13.D29 (was 14.D16. in 2
nd
ed.) a) Plot Points
N 1
F, S, E and R
Find point at intersection of lines
N 1
FE and R S
2 stages is more than enough. (see graph)

b) Draw lines
N 2 1
FS and E NOT calc. value E R .
Intersection is mixing point M
Mass balance F + S = M and Lever arm
F dist. S to M
0.786
S dist F to M

Give S F 0.786 1000 0.786 1272kg/h.
347

Alternate: Overall MB, F S Mand Diluent mass balance,

F,D S,D M,D
650 F x S y M x 0.28 M
Solve simultaneously: M 2321 and S 1321 kg/h. But this is less accurate.



13.D.30. New problem in 3
rd
edition. Equation (13-59) becomes Q
c
/A
i
< u
t
/(1 + safety factor).
Using the equals sign and solving for the safety factor Sf we have,
Sf = u
t
A
i
/ Q
c
-1 = 0.00172 (1.0)(4.0)/.006 1 = 0.1467
where A
i
= D
s
L
s
. Thus safety factor is 14.67% instead of 20%. This may still be acceptable.
13.D.31. New problem in 3
rd
edition. Soln. A. Kremser Soln.
50
R mE 0.30998 1.0
161.3


2 0
R 50, E 100, m 1.613, b 0, y 0.0, x 0.01
For example, Use 13-11.
*
1 0
y mx b 0.01613

*
1 1
N 1 *
N 1 1
R
1
y y
mE
y y
R
1
mE
becomes
1
2
y 0.01613 1 0.30998
0.7633696
0 0.01613
1 0.30998


1
y 0.01613 0.01613 .7633696 0.00381684
1 1
0.00381684
x y m 0.0023663
1.613

Soln. b.) Do mass balances and equilibrium for single stage.
348


IN F
Sy Fx Sy Fx
0 0.5 100y 50x
also y x 1.613 . Solve simultaneously and obtain identical result.
Soln. c. Do graphically as single stage system.
Soln. d. Do graphically as counter-current system, N=1. Solution is valid, but awkward.

13.D.32. New problem in 3
rd
edition. Fixed Dispersed Phase.
At feed conditions
sol feed feed tol sol feed
tol
sol
tol feed sol feed
feed feed
feed
Q Q Q Q Q Q
Q
Q Q 1 Q Q
Q Q
Q

a)
sol feed
tol
sol
feed
0.6 .006 Q Q .6
.375
Q
.006 .6 .006 1.6
1
Q


Equation 13-48 operation in ambivalent range.

0.3
.3
3
L L H
3
L H L
865 0.95 10
0.375
1 0.625 998. 0.59 10

From Example 13.5
0.3
1.10235
The
0.375
1.10235 0.6614
0.625

Either phase can be dispersed.
b)
d
1.0
0.5
2.0
, also ambivalent range

.5
1.10235 1.10235
.5
Either phase dispersed
c)
d
2.0
.6667.
3.0
According to 13-48 at border.

.6667
1.10235 2.2
.3333

water probably dispersed
d)
d
5.0
.8333
6.0
Equation (13-48), water (heavy) dispersed.

0.8333
1.10235 5.5 water dispersed.
0.16667

13.D.33. New problem in 3
rd
edition.
re s liq d c
t V Q Q 1.5min 90s


3
d c
Q Q 0.0072m s ,
3 3
liq
V 90 s 0.0072m s 0.648m
Note that there is a 1 inch air gap at top

2
liq t tank
V H 0.0254 d 4 0.648,
t tan k
H 2d

2
liq tank tank
V 2d 0.0254 d 4 0.648
Using Goal Seek
tank tank
d 0.7489 and H 1.4978


349

13.D.34. New problem in 3
rd
edition. N = 500 rpm = 8.335 rps

i tank
d 0.20 d 0.2 0.8279 0.16558 m

3 3
M w M w
Use water values for 998kg m and 0.95 10 kg m s

2
2
i L
L,estimate 3
L
0.16558 8.335 998
d N
Re 240, 064
0.95 10

Curve b in Figure 13-32 again predicts a constant
0
p
N 40
Then from Equation (13-52),
0
2 5
P M i c
P N d g where
c
g 1.0 and N 8.335.

2 5
M M
P 4.0 8.335 .16558 1.0 0.034587 (A)

M
will be fairly close to
c W
998 since
W Toluene
Q 5Q (see Equation (13-53)).
The series of messy terms for Equation (13-56a) can be calculated. Since the tank dimensions and
physical properties are the same as in Example 13-5, the only term on the RHS of Equation (13-56a) that
is different is P. Thus the result in the same as Equation B in Example 13-5,
0.3
d
0.0576 P

(B)
In addition to Equations A and B, we need to solve Equation (13-53)

M d d d c d d
1 865 998 1 (C)
Solving equations A, B and C with Goal Seek we obtain
d d d,feed
0.146 and 0.874.
Then solving Equation C,
M d d d c
1 0.146 865 0.854 998 978.6
Equation (B)
0.3
d
P 0.05076 2.876 P 33.84 W.
13.D35 (was 14.D11. in 2
nd
edition) From Eq. (12-46),
1 1 2 2
1 1 1 0 0
1 2
E K E K
B 1 , C , D R x
R R

(Eq. 6-6) For 1 < j < N
j j j 1 j 1
j j j j j j
j j 1
E K E K
A 1, B 1 , C , D Fz 0
R R

(Eq. 12-48) For Stage N
N N
N N N N N N 1 N 1
N
E K
A 1, B 1 , D F z E y
R

Example 13-4:
0
0 A D,0
R 1000, x 0.35, x 0.65, N 6 ,
N 1 A, N 1 D,N+1
E 1475, y 0, y 0
For Acetic Acid,
j j
A Aj A
K y x : Use Fig. 14-4 to estimate
A, j
K .

1 2 3
4 5 6
A A A
A A A
1 A1
1
1
0.03 0.5 0.09
K 0.3, K 0.33, K 0.43
0.1 0.15 0.21
0.12 0.14 0.16
K 0.5, K 0.5, K 0.5,
0.24 0.28 0.32
For first guess assume constant E 1475 and R 1000.
1475 0.3
E K
Then B 1 1 1.4425
R 1000
C
2 2
1
2
1 D A,0
E K 1475
0.33 0.48675
R 1000
D R x 1000 0.35 350

350

and so forth with
6 N
D D 475 0 0 . Thus matrix for acetic acid is,

13.D.36. Part a. New problem in 3
rd
edition. See figure

org Aq,0 Aq, N
* *
Aq org,1 org, N 1
Min
F C C
0.10 0.008
0.736
F C C 0.133 0.008


org,Min Aq
L
F 0.736 F 0.736 200 L h 147.2
h

b.
org
F 1.4 147.2 206.08 ,
org
Aq
F
206.08
1.0304
F 200

Operating line goes through
Aq,N org,N 1
C 0.008 and C 0.008 with slope 1.0304.

org,1
C 0.097
See Figure. 3 stages more than enough.
3
~ 2
4
stages needed.

1.4425 -0.48675
0 0
0
0
1475
1 .33
1000

1475
.43
1000

- 1
0
0
0 0
- 1
1475
1 .43
1000

1475
0.5
1000

0 0
0 - 1 0
3
1
2
4
5
6
0 0
0
- 1
1475
1 0.5
1000

1475
1 0.5
1000

1475
0.5
1000

0
1475
0.5
1000

0 0 0 0 - 1
1 2 3 4 5 6

1475
1 0.5
1000

351

Part c.
3
4
MW Zr NO 91.22 4 14.0067 3 15.994
MW water 2 1.00797 15.994 18.00994
352

Basis 1 liter 0.10 mol
3
4
Zr NO
Have
3
4
33.917g Zr NO
and 1000 g 33.917 966.083 g water
which is 966.083 18.00994 53.64 mol water
Mole frac.
3
4
.1
Zr NO 0.00186
53.64 .1

Mass frac.
3
4
33.9179
Zr NO 0.033917
1000 g

System is dilute if consider mole fraction, less so if use mass fractions. If densities are constant,
then constant flow rates is valid. Even with variable density, solving problem with mole fractions and
constant molar flow rates would be accurate. This would require converting equilibrium data to mole
fractions. Use of fractions with concentrations in mol/L is NOT correct.

13.D.37. New problem in 3
rd
edition.

Part a. HETP
large-scale
= HETP
pilot
(D
large
/D
pilot
)
0.38
= (0.24 m) (1.1 m/.05 m)
.038
= 0.78 m

f
large-scale
= f
pilot
(D
pilot
/D
large
)
0.14
= (1.4 s
-1
)(.05 m/1.1 m)
0.14
= 0.91 s
-1


Part b. HETP
large-scale
= HETP
pilot
(D
large
/D
pilot
)
0
= HETP
pilot
= 0.24 m

f
large-scale
= f
pilot
(D
pilot
/D
large
)
0
= f
pilot
= 1.4 s
-1


c. Use of the more conservative design developed for difficult systems (n
1
= 0.38, n
2
= 0.14) results in a
much higher HETP and thus a much taller column and more expensive column than use of the design
procedure for easy systems (n
1
= 0, n
2
= 0). Considerably more data is needed for a large variety of
systems to determine best design practice. If a variable speed motor is used in the large-scale system the
difference in predicted optimum frequency is not as serious because the system can tuned to find the
optimum frequency.

13.D.38. New problem in 3
rd
edition.

water
MW 18.02,
toluene
MW 92.14,
extract toluene
raffinate water
C C
m 20.8
C C

F 1.0 kmol hr , S 0.06 kmol hr.
IN
x 0.00023 ,
IN
y 0

IN out out
F x Fx Sy and
out out
y mx
out in
x Fx / F Sm
Note m m. m is equilibrium in mole fraction units. Assume extract has properties
toluene and raffinate properties of water.

3
3
3
3
1 92.14 kg toluene
kmole benzoic
865 kg tol m kmol toluene
m extract
m 20.8 122.71
kmole benzoic
1 18.02 kgW
m raffinate
998 kgW m kmol W

Units on m are
kmol benzoic kmol extract
kmol benzoic kmol raffinate

353


out
1.0 0.00023
x 0.0000275
1.0 0.06 122.71
,
out
y 122.71 0.0000275 0.00337
If use m 20.8 find
out
x 1 .00023 1 .06 20.8 0.000102, RONG! W
13.D.39. New problem in 3
rd
edition.
Feed is 0.1
4
CC , 0.9 AA. F 10 kmol h . Solvent pure. S 10 kmol h.
1 equil. stage
Lever arm:
S 10 FM
1
F 10 SM
, Alternatively
4 4
4 4
F,CC M,CC
M,CC S,CC
x x
S
1
x x F

Then
4
M,CC
x 0.05 Find Mixing Point M.
[The figure is shown at the end of problem 13D39 as the single stage mixing line.]
Phases split along the line TE to find the line through M
Rafinate:
4
CC AA
x 0.041, x 0.54 . Extract:
4
CC AA
y 0.095, y 0.07
Overall Balance: E+R+=F+S+=20
CC
4
Balance: .095E+0.041R = (0.0) S+0.1 for F=1.0
Solve simultaneously, R 16.6667, E 20 R 3.3333
NOTE: Since CC
4
mole fracs can be read more accurately, the CC
4
balance is
probably more accurate than the acetic acid balance equations.


13.D.40.

CC
4







S
1
S
2

1 2
E
1
E
2

R
2
R
1
= R
2
single stage = 16.6667
Mix with S
2
= 10 (pure)



2,AA 2 acetic
1 2 2,AA
M 0 2 M ,acetic S
1
1 2 R ,acetic M acetic M
x x x
R 16.6667 SM
10 S R M x x 0.54 x


2,AA
M
x .54 1.6667 2.6667 .3375
Find M
2
and by trial and error find a tie line though M
2
. See figure on next page.
Extract 2,
4
CC AA
y 0.046 y 0.065
Raffinate 2,
4
CC AA
x 0.018 x 0.57

2 2 1
R E R S 16.6667 10 26.6667

4
CC balance
2 2
0.018R 0.0046E 0.041 16.6667 0.0 10
Substitution
2 2
0.018 R 0.046 26.6667 R 0.68333

2 2
R 19.40 and E 7.16 kmol h.


354


355

13.D41. New problem in 3
rd
edition.
N 1
R F 10,
CC 4,N 1
x 0.1,
AA,N 1
x 0.9

0
E S 14.5,
TEA,0
y 1.0 ,
CC 4, N
y 0.091

Mixing. Use lever arm rule.

4 4
4 4
N 1,CC M,CC
M,CC S,CC
x x
14.5 S FM
1.45
10 F x y SM


4 4
4
N 1,CC S,CC
M,CC
S
x y
.1 1.45 0
F
x 0.041
S F 1 2.45


Find M. Draw
N 1
E MR line. See figure on next page.
Raffinate:
4
1,CC 1,AA
x 0.008 x .58
Passing Streams
N N 1 O 1
E R &E R intersect at .
Very close to parallel. Use parallel lines to step off stages.
Estimate # Stages = 3.
Flow rates
3 1
24.5 F S E R

4 3 1
CC balance. F .1 S 0 1.0 E .091 R 0.008

3 1
1.0 24.5 0.008
E 9.69 kmol h , R 24.5 9.69 14.81
.091 .008

Can compare to 13.G.2 Part c.
Extract 10.066 and Raffinate 14.433
Extract Mole fraction
4
TEA CC AA
y 0.841 x 0.0913 y 0.067
Raffinate Mole fraction
4
TEA CC AA
x .418 x 0.0056 x .577
Two results are reasonably close.
356


357



13.D42. a. First, plot points E
N
and R
1
on the saturated extract and saturated raffinate curves,
respectively.
Second, Find point at the intersection of lines FE
N
and R
1
S.
Third, step off equilibrium stages. Need about 3. See graph.
Part b. Easiest: use the lever-arm rule. Find mixing point M at the intersection of lines FS and E
N
R
1
.Then
0.81 1235 /
S FM
F kg h
F SM

Can also write 3 mass balances (overall, pyridine, and water) and solve for the unknown flow
rates F, E
N
and R
1
. Unfortunately, this will not be very accurate because it is difficult to read the water
values accurately.


13D.43 (was 14.D5. in 2
nd
ed.) Plot points for
0 1
F, S E , and R (on saturated raffinate line)
Use lever-arm rule to find point M.
0
E M F 1000
S 1371 FM

Line
1
R Mintersects the saturated extract curve at
N acetone
E . x 0.067 .
Lines
N 1 0
FE and R E intersect at (a second piece of paper was attached to find
accurately). Step off stages. 3 is more than sufficient. Need about 2 & 2/3 stages. This
is close to the 2 + estimated in problem 13.D19 with a McCabe-Thiele analysis.

358


359


13.E1.
m o
D D
K 0.05, K 0.15, R 20, E 200, F 1
Since
o
D
E
K 1
R
ortho goes up column and since
M
D
E
K 1
R
meta goes down.

m,N 1 o,N 1 m,0 o,0
y y 0, x x 0
Estimate: E E .52F and R R .48F

R 20 R 20.48
E 200.52 and R 20.48, 0.09974 and 0.1024
E 200.52 E 200

Recoveries:
ortho,1 ortho,1
.92 .52 1 E y or y 0.002386

ortho,N
x 0.00203

meta,N meta,N
.94 .48 1 Rx or x .02179
Plot equilibrium curves and operating lines (see Figure)
Feed cannot be 3
rd
stage since cannot get
N
m
x desired. Cannot be 5 as will be past
intersection of R E and meta op lines.
Thus feed must be 4
th
stage. Do not get match of total number of stages.
Need 8 1/3 for ortho and ~ 5 2/3 for meta.
A very slight adjustment of recovery meta will change this. (Meta is approaching a pinch
point at feed stage). 93% recovery was not enough. Therefore, need ~ 93.5% recovery
with ~ 8 stages.


360

13.E.2. New problem in 3
rd
edition. Part a. 96% recovery. 4% p-xy left in diluent ,


N,p xy
x 0.04 .004 0.00016 wt. frac.
Part b. Paraxylene:
a
y
K 0.080 m, E 20, 000, R 1000
x

* N 1
N
y
x 0.
m
Thus
Eq. (12-28) converted to extraction notation is convenient. L R V E



*
0 N
*
N N
x x R R
n 1
mE x x mE
N
mE
n
R


0 N
R 1000
x 0.004, x 0.00016, 0.625
mE 0.080 20, 000


0.004
n .375 .625
0.00016 2.3025
N 4.899
n 1 .625 0.470

Part c. ortho-xy
*
0 N
m 0.150, x 0.006, x 0,
0.150 20000
mE
3
R 1000

Eq. (12-31) Converted:
*
N N
N 1 *
0 N
x x 1 mE R
x x
mE
1
R


N 5.899
1 3
x 0.006 1.842 E - 5
1 3

Part d. Alternative Solutions are presented below for meta-xylene.
m-xy
*
0 N
m 0.050, x 0.005, x 0, N 4.899 E 20,000, R 1000, b 0

0.05 20, 000
mE R 1
1000

Must use special form. But the L mV 1 form in terms of x is not available. Thus, need to
derive, or translate or find in another source. Looking at development of Eq. (12-12).

0 N
N x x x

Solving for N,
0 N
x x
N
x

Where x is determined in same way y was determined for Eq. (12-12),

1 0
1
j j 1 0
L
y x b
y
V
x x x const x
L V L V

Alternatively,
N 1 N
N 1
N
L
y x b
y
V
x x
L V L V

361

Translating to this extraction problem,
N 1 N
L V R E, y 0, x x

0 N 0 N
N
x x x x
N
x x

And solving for x
N
,
0
N
x 0.005
x 0.0008476
N 1 5.899


Alternative Solution: Redefine terms to match Eq. 12-12 [Relating y to solvent and x to raffinate is
arbitrary. Switch these definitions.]

N 1
y meta xylene in hexane 0.005

1 0
y mxy out is unknown x is now inlet solvent 0

d
1 1
m 20, b 0
K 0.05
,
L 20, 000
L E 20, 000; 1
mV 20 1000
, V 1000

N 1 1 1 0 1
L
y y N y x b 4.899 y 20 0 0
V


362

Solve for y
1
,


N 1
1
y 0.005
y 0.0008476
5.899 5.899


This is actually
N
x

in normal notation.

Part e. Shown for normal notation.















0.00032 0
Slope Operating line 0.08333
0.004 0.00016


MIN
R
Slope 0.08333
E
,
MIN
R 1000
E 12, 000 kg h
0.08333 0.08333


13.E.3. New problem in 3
rd
edition. Part a. Plot the equilibrium data and points F and S. Straight line
from power F to point S passes through mixing point M. Since amounts of F and S are equal,
M is at the half-way point of the line. Find tie line through M by trial-and-error. This is
difficult since tie line is very sensitive.

Approximately, raffinate
AR DR
x 0.326 x 0.575
and extract
AE DE
y 0.046 y 0.058

Mass Balances: E R S E 40 R 40 E

AE A,S A,s AF
Ey Rx Sy Fx 20 0 20 .4 8
Solve simultaneously, E 18.0 kg , R 22.0 kg.

Part b. First add solvent until reach saturated raffinate curve at intersection with FS line.
Initial Raffinate
AR D
x 0.36, x 0.54

INIT AR AF INIT AS INIT
R x Fx S x 20 .4 S 0 8

init AR
R 8 x 8 0.36 22.22 kg

INIT INIT
S R F 2.22 kg
Second, use Eq. (13-27) for the continuous solvent addition batch extraction.
pxy equil slope = 0.080
EQ
y 0.080 .004 0.00032
0,pxy
x
0.004

x
N,pxy
x
0.00016

N 1
y 0
y
363


t ,final ,A
t ,feed ,A
x
t ,A
A x
dx
S
R y


t ,feed,A
x is the raffinate after solvent addition to form two phases

t ,feed,A A,initial raffinate
x x 0.36 ,
t ,final,A
x 0.292
From equilibrium find values
A
y (extract), Approximate values are:
A,t
x
A
y 1 y
0.36 0.048 20.8
0.326 0.046 21.7
0.292 0.045 22.22


0.292
A
A 0.36
0.36 0.292 dx
20.8 4 21.7 22.22 1.47
y 6


added t
S 1.47R Eq. A
In the derivation
t
R is assumed constant,
t t ,INIT
R R 22.22 kg

added
S 32.66 kg
With this approximation
added
E S . 32.66 kg
Solute mass balance

t A,INIT added A,added t A,final A,Avg
R x S y R x Ey

A,added A,INIT A,final
y 0, x 0.36, x 0.292

A,Avg
22.22 0.36 0.292
y 0.046
32.66


If we do not assume R is constant, then Eq. (13-27) is


t ,A added
t ,INITIAL,raf
x S
t ,A
added
A 0 x
d R x
dS
y

We would need to do a numerical integration with a calculation of
t ,A
R x versus
A
y . this
can be done, but is challenging.

13.G.1. New problem in 3
rd
edition.
Extract 1:
TRA carbontet Acetic acid
flow 3.90769, x 0.84986, x 0.085102, x 0.065042
Raffinate 1:
TEA carbontet Acetic Acid
flow 16.03923, x 0.41361, x 0.041332, x 0.54506
Extract 1:
TEA carbontet Acetic Acid
flow 11.63396, x 0.91426, x 0.036586, x 0.049149
Raffinate 2:
TEA carbontet Acetic Acid
flow 14.40527, x 0.41633, x =0.016472, x =0.56719 .
Entering carbon tet 0.10 10 1.0 kmoles hr
Leaving in raffinate 0.016472 14.40527 0.23728
Extracted In Out in Raffinate 0.7627
% extracted = 76.27%
364


13.G.2. New problem in 3
rd
edition.
Part a, 3 stage cross-flow. All flow rates are kmol/h
Total Flow rate TEA flow CCl
4
flow Acetic flow
Extract 1 3.961 3.366 .3371 .2576
Extract 2 11.634 10.637 .4256 .5718
Extract 3 11.052 10.419 .1554 .4777
Raffinate 3 13.353 5.580 .0819 7.693
Carbon tet remaining in raffinate 3 is 0.0819 kmol/h. Since carbon tet feed was 1.0 kmol/h, 0.9181
kmol/h was extracted. Fraction extracted = 0.9181/1.0 = 0.9181.

Part b. 3 stage counter-current with S = 10 kmol/h.
Extract 1 4.9142 3.723 .7242 .4672
Raffinate 3 15.086 6.277 .2758 8.533

Carbon tet remaining in raffinate 3 is 0.2758 kmol/h. Since carbon tet feed was 1.0 kmol/h, 0.7242
kmol/h was extracted. Fraction extracted = 0.7242/1.0 = 0.7242.

Part c. 3 stage counter-current with S set to give same fraction extracted as in part a (0.9181) and outlet
raffinate carbon tet flow rate of 0.0819 kmol/h. This is trial-and-error.
First trial: S = 20 and CCl
4
raf 3 flow rate = 0.0289
Second trial: S = 18 and CCl
4
raf 3 flow rate = 0.04045
Third trial: S = 16 and CCl
4
raf 3 flow rate = 0.0590
Fourth trial: S = 14 and CCl
4
raf 3 flow rate = 0.0908
Fifth trial: S = 14.5 and CCl
4
raf 3 flow rate = 0.08104
This is close enough.
Final Results:
Extract 1 10.066 8.469 0.9189 .6786
Raffinate 3 14.433 6.031 0.0810 8.321

363
SPE 3
rd
Edition Solution Manual Chapter 14
New Problems and new solutions are listed as new immediately after the solution number. These new
problems are:14.A3, 14.A4, 14.C5, 14.D6, 14.D9, 14.D11, 14.D15-14D17, 14.E2, 14.E3. Chapters 13
and 14 from the 2
nd
edition were rearranged to place all the extraction material into chapter 13 and the
material for other separations in Chapter 14. Thus, the numbers of many problems have changed.
14.C.5. New problem in 3
rd
edition. Part a. y y, x x, m 1, F U, S O, R U, E O
where F, U, S, O, R, E are kg
Eq. (13-27b) becomes
F IN
U U
y x x y
O O
and (13-21)
IN IN
U
x x y 1 U O
O
and y = x
Part b. Eq. (13-29b) becomes
t ,final t ,feed
O 1
n x x
U K
Where K y x at equilibrium = 1.0 in washing.
14.D1. (was 13D29 in 2
nd
ed.) a) Translate eq. (12-28),
*
0 N
*
N N
x x U U
n 1
mO x x mO
N
n mO U
Note: x in wt frac. translates to x in
3
kg m if densities are constant. Densities cancel. For
washing equilibrium is equal overflow & underflow concentrations. Thus, m = 1, b = 0
* N 1
2 4 N N 1 0 N
y b
H SO x y 0, x 1.0, x 0.09
m
U 40 mO 1
0.8 and 1.25
mO 1.0 50 U 0.8

1.0 0
n 1 0.8 0.8
0.09 0
N 4.96
n 1 0.8
b) HC Use Eq. (12-31) or (14-8)
*
N N
N 1 *
0 N
1 mO U x x
x x
1 mO U
N 0 N 1 5.96
1 mO U
1 1.25
x x 0.75 0.0674
1 1.25
1 mO U

3
kg m
HC
HC
N 1 *
N
y b 1.0 50
mO
N 4.96, 1.25, x 0
U 40 m
Alternative:
364

Note:
2 4
N N
0 0
HC H SO
x x
=0.09
x x

Thus, if one is clever and realizes change will be same for HC &
2 4
H SO
mO
since & N are identical
U
, dont need to use Kremser eqn for part b.

14.D2. (was 13.D22 in 2
nd
ed.) a. 1000 cc sand = 400 cc underflow liquid. This is about 400 g = 0.4 kg
liquid. Equil: y = x. Use nomenclature of Table 13-4.
Operating Eq.
j j 1 in out
U U
y x y x
O O

Slope
U .4
0.8.
O .5
Goes through point (y = 0, x = 0.002)
Overall bal.
in in out out
O y U x U x O y

out
O .4 0.035 .4 .002 .5 y
out
.0140 .0008
y 0.0264
.5

Need 6 2/3 Stages See Graph (Can also use Kremser eq.)

b. Mass Balance:
in
j 1 j j j j j
U x O y U x O y
Op. Eq.:
in
j j j j 1
j j
U U
y x y x
O O


U
U 0.4, O 0.2, 2 slope
O
,
out
x 0.002 (see graph)
Obtain approximately same separation, but use much more wash water.
14.D3. (was 13D23 in 2
nd
ed.)
365



Basis: 1 kg
3
CaCO solids
Feed Mole frac. can be arbitrary. Pick
0
x 0.01as basis
M.B.
iN iN out out
U x O y U x O y

out out iN iN
U U
y x x y
O O


out out
y , x at Equil (y = x) line
Point
in in in
x , y x , 0 is on op line
Slope Op line
U 3
O 2

See graph.
Find
4
0
x 0.00127
0.127
x 0.01

Recovery
4
0
x
1 1 0.127 0.873
x

Recovery is significantly better with counter-current process.
4
3 2 1
y
4

U = 3
U = 3
y
2

x
4

y
1

y
3

O 2
4

O 2
2

O 2
3
O 2
1

4
y 0
in

3
y 0
in

2
y 0
in

1
y 0
in

366



14.D4. (was 13D24 in 2
nd
ed.)
0.8 0.8 0.2 0.8, 4, O 4000
1
kg/h
367


1
F
kg liquid
1.0
kg L liquid kg liquid L liquid
U 1000 dry solids 4 1600
kg solid
h L solid h
2.5
L solid


2
F
kg .8 1.0 kg liquid
U 2000 3200
h .2 2.5 h


1 2
T F F
kg liquid
U U U 4800
h

In section 2:
T T
j j 1 0 1
U U
y x y x
O O

Slope
T
U O 4800 4000 1.2 Goes through point
0 1
y , x 0, 0.006
Intermediate feed at
F
x x 0.02
In Section 1:
1 1
F F
j j 1 N N 1
U U
y x y x
O O

Slope
1
F
U O 1600 4000 0.4 . Goes through point
N N+1
y , x
Also intersects Section 2 op. line at feed line. (Or calculate
N
y from mass balance). Equilibrium
is y = x. Step off stages (see Figure). Need 5.4 equilibrium stages. Opt. Feed is 4
th
.



14.D5. (was 13D25 in 2
nd
ed.) 0.8, 1 0.2


1 1
kg dry 1 1 2000 L
F: 1000 F 5 wt %
kg dry L solid
h h
2.25 0.2
L L under flow


2 2
kg dry 1 1 L
F : 2000 4000 F 2 wt %
h 2.5 0.2 h

368

Liquid Volumes:
2000L L liq
total 0.8 liq 1600 5 wt %
h h


3200 L liq
4000 0.8 liq 2 wt %
h


T
underflow
F : Total liqd h 4800

0 f f
kg liquid
U 4000 4800 where 1.0 kg L
h


a
N 1 N 1 0 0
kg N 0H U kg liq
O 4000 kg h , y 0, 1.2, U x 1600 0.05 3200 0.02
O h kg liq


0
x 144 4800 0.030 ,
N
x specified 0.006
Ext. MB,
N 1
O y
0 0 N N 1 1
N 1
U x U x O y

0 0
1 N 0 N
U x U U 4800
y x x x 0.030 0.006 0.0288
O O O 4000

Convert to Kremser

*
1 0 0
O V, U L, m 1, y mx x 0.030, mV L 4000 4800 0.8333
Eq. (12-30)
0 0.030
n 1 0.8333 0.83333
0.0288 0.030
N 8.83 or 9 stages
n 1.2

Use 2 feeds!

14.D.6. New problem in 3
rd
Edition. 2.5 kg wet is 1 kg dry solids-insoluble, and 1.5 kg underflow liquid.
Part a.
1 kg dry solids
10 kg total 4 kg dry insoluble solids
2.5 kg total


1.5 kg liquid
U 4 kg dry solids 6 kg liquid.
kg dry solids
, Ov 10 kg liquid.
Before 1
st
mixing: 0.05 frac BaS 6 kg liquid 0.3 kg BaS
1
st
Mix:
0.3 kg BaS
0.01875 mass frac in U & Ov.
16 kg liquid total

Settle (6 kg liquid in U) 0.01875 0.1125 kg BaS
2
nd
Mix Pure Water
0.1125 kg BaS
0.00703 mass frac in U & Ov.
16 kg liquid

Settle (6 kg liquid in U) 0.00703 0.0421875 kg BaS
3
rd
Mix Pure Water
0.0421875kg
0.00264 mass frac BaS in U and Ov.
16 kg liquid

Part b. Result is same. Can also be done graphically.
Part c. Countercurrent. Easiest solution approach is to use Kremser equation.

*
N N
N 1 *
0 N
x x 1 m Ov U
x x
1 m Ov U

369


*
0 N N 1
N 3, x 0.05, m 1, Ov 30, U 6, x y m 0

N 0 4
1 30 6
x x 0.05 0.00641 0.0003205
1 30 6

External M.B.
0 N 1 N 1
x U y Ov x U y Ov

1 0 N
y U x x Ov 6 0.05 0.0003205 30 0.009936

14.D7. (was 13D27 in 2
nd
ed.) Equilibrium:
j j
x y wt. fractions
Operating Eq.:
j j j,in j 1
U U
y x y x
O O

Basis 1000 cc wet sand.

3
vol water 1.0 g kg
U .4 1000 cm wet sand 0.4 kg
vol sand wet cc 1000 g

O = 0.2 kg. Thus, each operating line has slope
.4
2.
.2

Each op line goes through pt.
j,in j 1
y , x

0 N,in N 1,in N 2,in
x 0.035, y y y 0

N S,in N 2,in N 1 1,in N 2
x 0.002, y y , y y , y y
Start at stage N where
N
x = 0.002. Find
N
y then work backwards to stage N-2. This
gives inlets for first 3 stages so can then work forward (see Figure). Note: that stages 5 and
N-2 are not connected. 8 stages gives more than enough separation, but 7 is not enough.



370

14.D8. (was 13D28 in 2
nd
ed.) Use Kremser equation

solv solid
F F .95, y mx is equilibrium with m = 1.18, and N = 11.
Recovery is
N 0
1 x x .
Eq. (12-31) becomes
*
N N 1
x y m 0 .

solv
*
N N N solid
N 1 *
0 N 0
solv
solid
m F
1
x x x F
x x x
m F
1
F


solv N
12
solid 0
m F x 1 1.121
1.18 .95 1.121. Then 0.041
F x
1 1.121

Thus Recovery = 0.959

14.D.9. New problem in 3
rd
Edition. Assume
Solid solvent
F and F are constant despite removal of sugar from solid.

Solid F solv,IN E
F 1.0, x 0.055, y 0, m 1.18,
Eq. (13-21) becomes
Solid
F solv,IN E Solid solv
solv
F
x x y m F F
F

a.
solv Solid solv
F 3.0, F F 1 3

0.055 1
x 1.18 0.01211
3 3
,
E
y m x 1.18 0.01211 0.0143 wt frac.
b. x = 0.004. Solve for
solv
F .

E solv,IN
Solid
solv F
xm y
F
F x x


F
solv Solid
E Solv,IN
x x 0.055 0.004
F F 1.0 10.805 kg
m x y 1.18 .004 0



14.D10. (was 13D30 in 2
nd
ed.)
2 2
G H stream 100 lb h of H CONSTANT
L Slurry stream 120 lb h CONSTANT
Operating Line. Must work in weight ratios.
in out
L L
Y X X Y
G G


in
in
in
x x y
x , Y , X 0
1 x 1 y 1 x


4 4
out in
in out
2 2
in out
lb CH lb CH
30 .05
Y 0.30, Y .0527
lb H 100 lb H .95


in out out
G Y Y L X

out
100
X 0.30 0.0527 0.206
120
,
out
X .206
x 0.171
1 X 1.206

371

Operating line becomes,

out
L L 120
Y X Y where 1.20
G G 100
and goes through
in out
X , Y .
Equilibrium Curve: y = 1.2 x becomes
Y X
1.2
Y 1 X 1

1.2 X
Y
1 .2 X

x X Y
0 0 0
.05 .0526 .0038
.10 .1111 .1364
.15 .1765 .2195
.20 .2500 .3158
.25 .3333 .4286
Plot Y vs X



See Figure for Plot. Need 5 1/8 stages.
14.D.11. New problem in 3
rd
Edition. 10,000 kg h wet solids, frac. vol. liquid ,
1 frac vol dry solids.
Basis
3
1 m wet solids :
Weight liquid + weight solids
3 3
1.0 1000 kg m 1.0 1 1500 kg m
0.4

3
400 kg 900 kg 1300 kg total m wet solids.
Thus
400
1300
of weight is underflow liquid, U
400 kg
10, 000 3076.9
1300 h
.

14.D12. (was 13D32 in 2
nd
ed.)
solv solid
F F 1.36
372

Op. Eq.:
solid solid
1 0
solv solv
F F
y x y x
F F

Where y and x are
3
kg m .
E
y m x is equilibrium.

*
F 0 N F 0 N 1 N solv solid
x x , x 1 .975 x .025 x , y 0, x 0, N 5, F F 1.36
Can use any of Kremser equations such as Eq. (12-31).

solv
*
N N solid
N 1 6 *
0 N
solv
solid
m F
1
x x F 1 1.36 m
0.025
x x
1 1.36 m
m F
1-
F

Which becomes:
6
0.1582 m 1.36 m 0.975 0 Find m = 1.313
Check:
6
1 1.313 1.36
.025 0.025005
1 1.313 1.36
which is OK.
14.D13. (was 13D33 in 2
nd
ed.)
a) Use Eq. (13-21),
0 in
E in

R S x y
x , equil. y m x, y 0

m R S

E
g L in liqd

R S 10 12.5 0.8, m 1.18
g L in solid
,
F
F F
0.8 x 0
0.8
x x 0.4040 x
1.18 0.8 1.98

Frac. Rec. 1 0.404040 0.5959596
b) Eq. (13-29b)
t ,final
E t ,feed

x
S 1
n

m x R


t ,final F F
x x exp 1.25 1.18 0.228779 x , Frac Rec = 1 0.228779 = 0.7712

14.D14. (was 13D34 in 2
nd
ed.)
4 2 coal
BaSO BaS 2 CO
Equil: Soln conc in underflow = soln conc in overflow. Thus really washing
Equil : y x, m 1, b 0
in out
kg kg solution
U 350 in sol. 1.5 525 kg soln., x 0.20, x 0.00001
h kg insoluble solid

0 0 N * *
in N 1 0
U x x
kg 525
O 2075 , y 0.0, x 0, y 0.2 0.00001 0.0506, x 0.0506
h O 2075

Eq. (12-29)
* *
N N 0 0
n x x x x
n 0.00001 .2 .0506
N 6.99 or 7.0
n L mV n 525 1.0 2075

14.D.15. New problem in 3
rd
Edition. With 1000 kg/h dry solids U 1.5 1000 1500 kg h
a) Can use Kremser eq. with large N to find
Min
Ov or a sketch
373













Equilibrium is y x
Min
U
0

N 0
y 0
0
x .15

N
x 1 .99 .15 0.0015
Min
1500
Ov 1485
10101

*
1
Min 0 N
y 0 U 0.15
1.0101
Ov x x 0.15 0.0015

y
b.
Min
Ov 1.2 Ov 1782

U 1500
0.84175
Ov 1782

Kremser:
y y V Ov
x x L U

Eq. (12-28)
*
N N 0
x 0.0015, x 0, x .15, m 1, U Ov 0.84175

*
0 N
*
N N
x x U U
n 1
m Ov x x m Ov
N
m
n
U Ov


0.15 0
n 1 .84175 .84175
.0015 0 2.81
N 16.33
1 .17227
n
.84175

In theory, can use McCabe-Thiele, but it is difficult to accurately step off this large number of
stages.
c.
U 1500
Ov 2000, .75 m 1
Ov 2000


n 1 .75 100 .75
N 11.29
1
n
.75


eq
act overall
sub actual
N
11.29
N 15 E 0.753
N 15

For
E
m use N = 15 and change m
E
with same equation

E E
E
.75 .75
n 1 100
m m
N
m
n
.75

*
1
y
374

Vary m
E
until N = 15.
E
m .911
On a McCabe-Thiele diagram this is trial and error. Kremser is much easier.

14.D.16. New problem in 3
rd
Edition. Part a.
IN IN
U 2 kg, O 2 kg, x 0.06, y 0

Solution (translation of Eq. (13-21)) is

IN IN
U
x x y 1 U O 1 .06 0 2 .03
O

Part b. Want x 0.005 O is unknown,
IN N
x 0.06, y 0, U 2 Solve for O

IN IN
U U
x x x y
O O
,
IN IN
IN IN
x y x x O
O U
U x x x y


0.06 0.005
O 2 22 kg water
0.005 0

14.D.17. New problem in 3
rd
Edition. K = 1 Eq (1328) becomes
t ,final t ,feed
O
n x x
U

Part a.
t ,feed
O 2, U 2, x 0.06

1
t ,final t ,feed
O
x x exp 0.06 e 0.02207
U

Part b.
t ,feed t ,final
U 2, x 0.06, x 0.005

t ,final t ,feed
0.005
O U n x x 2 n 4.97 kg
0.06

Part c.
t ,final
x x in Part a.

normal batch
O O in Part b.

14.D18. One equilibrium stage.
N+1 S S
A A D
F 1000, x .2, S 662, y y 0

0 A,0 A,N+1
A,M
0
E y F x
x 0.12
E F
(same as Example 14-2)
Plot M. By trial and error find tie line through M (Final result shown in Figure).

1 1 1 1
A D A D
y .238, y 0; x .078, x .656
Flow rates: Diluent balance:
1
1 D D,N+1
R x F x

1
1 D,N+1 D
R F x x 1219.5

1 1
E M R 1662 1219.5 442.5
375



14.D19. This problem is essentially a repeat of Example 14-2, except using exactly 3 stages.
Clearly,
1
A
x 0.04 since now have more stages. F,
0
E and M are unchanged. Problem
is trial-and-error. Guess location of
1
R . Find
N
E and . Step off 3 stages and see if
have correct location of
N
E . The third and final trial is shown in the figure.
A1
x 0.026 and
A3
y 0.38.


14.D20. Although this is leaching, this cross-flow problem is very similar to cross-flow extraction.
We can derive

j 1 j,in
Mj
j 1 A j,in A
A
j 1 j,in
R x E y
x
R E


j 1 j,in
M R E where
Mj j
j j
j A A
j
A A
M x y
R
x y

Stage 1:
0 1,in
0 1,in A a
R 1000, E 421, x .2 y 0 ,
M1
A
x 200 1421 .1407
Find M on line
M1
0 A
SR at x (see Figure). By trial-and-error find tie line through M.
376

This gives
1 1
E and R . Find
1 1
A A 1
y .35, x .113, M 1421

1
1421 .1407 .35
R 1254.9
.113 .35

Stage 2:
M2
A
1254.9 .113 0
x 0.085
1254.9 421

2 2
A A 2
y .18, x .058, from tie line , M 1675.9

2
1675.9 0.085 .18
R 1305.0
.058 .18


Stage 3:
M3
A
1305 .058 0
x 0.044
1726
,
3
A,3 A 3
y .09, x .03, M 1726


3
1726 0.044 .09
R 1323.3
.03 .09
kg/h




14.D21. a. Basis 1 kg mix in underflow:
NaC NaC
crystals
x values 0.8 1.0 0.2 y

Since crystals are pure NaC, NaOH is in liquid only. Since 20% of the underflow is liquid,

NaOH NaOH
x 0.2 y . Generate equilibrium table.

377


Soln (y)
NaOH
x Mass frac NaOH
NaC
y
NaC
x
0 0 .270 .854
0.004 .02 .253 .8506
0.008 .04 .236 .8472
0.012 .06 .219 .8438
0.016 .08 .203 .8406
0.020 .10 .187 .8374
0.024 .12 .171 .8342
0.028 .14 .156 .8312
0.032 .16 .141 .8282
0.036 .18 .126 .8252

Feed is 45 wt% NaC crystals. x values: NaC (soln) = 0.5193, NaOH (soln) = 0.099, water 1-0.5193-
0.099 = 0.3817. Since feed is 55% liquid,

F,NaOH NaOH
x 0.55 y 0.099
,
NaOH
y 0.099 0.55 0.18

From the equilibrium data
NaC
y 0.126

F = 100, S = 20, Plot F & S and find M.

FM 20
,
100 SM
Tie line through M gives E & R.

E RM
1.119
R EM
(measured on figure)

E R 120 1.119 R R 120


R 56.63
kg/min,
E 63.37
kg/min

NaC NaOH
R : Raffinate 0.833 x , 0.026 x


NaC NaOH
E: Extract y 0.16, y 0.135


The underflow is z wt frac crystals (Pure NaC) + (1-z) wt frac solution

NaC
y 0.16 is soln in equil

Thus,
z 1.0 1 z 0.16 0.833


0.333 0.16
z 80.1% OK
0.84

was 80% solids in problem statement.

c. Same M. Plot
1
R
draw line
1
R
M to
N
E
.

2 stages more than sufficient

N 1 1 1
E R 120 1.137 R R 120


N 1
1 N
E R M 103.5
1.137
R E M 91.0


378


1
R 56.14
kg/min,
N
E 63.86
kg/min

1 1,NaC 1,NaOH
R : x 0.845, x 0.01


N NaC NaOH
E : y 0.152 y 0.147



379






380

14.E1a. This is difficult part converting data
Basis 1 lb oil-free solids


oil
y

solvent
y

z solids
1.0
x
1 z

oil
oil
y z
x
1 z

0 1.0 0.20 0.830 0
0.1 0.9 .242 0.80515 0.01948
0.2 0.8 .283 0.7794 0.044115
0.3 0.7 .339 0.74683 0.07595
0.4 0.6 0.405 0.71174 0.1153
0.5 0.5 0.489 0.67159 0.16420
0.6 0.4 0.600 0.625 0.2250
0.65 0.35 0.672 0.598086 0.26124
0.70 0.3 0.765 0.56657 0.303399
0.72 0.28 0.810 0.552486 0.3222

Note:
solids
y 0 for all streams, Z = lb solution/lb oil free solids.

Plot data on triangular diagram. See Figure 14.E1a, b, c, d, e.

b&c. F + S = M
1
= 1500


oil,F oil,S 1 oil,M
F x S y M x ,
1
oil, M
1000
x 0.252 0.168
1500


See Figure 14.E1a, b, c, d, e.

Check: Lever Arm
1
1
M S F 2
.
S 1 M F
Find tie line through M
1
.

Extract
1
E ,
oil,1 oil,1 solids,1
y 0.34; Raffinate: x 0.092 and x 0.730

Mass Balances:
1 1
1500 E R ,
1 1
252 0.34 E +0.922 R

1 1
R 1040.3, E 459.7 lb
Finish step c) Stage 2:
1 2 2
R S M 1540.3 ,
1 2 oil,M
R 0.092 M x

2
oil,M
x 0.062. Plot
2
M and find tie line through
2
M .
Extract:
oil oil,2 solids,2
2
y 0.115; Raffinate: x 0.025 and x 0.80.
MB:
2 2
1540.3 E R ,
2 2
95.7 0.115 E 0.025 R

2 2
R 904.8 lb, E 635.5 lb
d & e Same answer as b & c but R & E are flowrates.
f. See Figure 14.E1f. 3 stages is more than enough. Need ~
1
2
3
equil stages.

381

Lines
N N 1 0 1
E R and E R intersect at .










382









383


14.E.2. New problem in 3
rd
Edition. Converting data is the difficult part, but is obviously identical to
Problem 14.E.1.
Basis 1 kg oil-free solids

oil
y

solvent
y

z
solids
1.0
x
1 z

oil
oil
y z
x
1 z

0 1.0 0.20 0.830 0
0.1 0.9 .242 0.80515 0.01948
0.2 0.8 .283 0.7794 0.044115
0.3 0.7 .339 0.74683 0.07595
0.4 0.6 0.405 0.71174 0.1153
0.5 0.5 0.489 0.67159 0.16420
0.6 0.4 0.600 0.625 0.2250
0.65 0.35 0.672 0.598086 0.26124
0.70 0.3 0.765 0.56657 0.303399
0.72 0.28 0.810 0.552486 0.3222

Approximate solution, use Eq. (13-29a) Oil balance:
t ,final
c ,feed
x
t
t x
dx S
R y


S = Mass Solvent,
t
R Mass raffinate (solids + solute)
x = Mass frac. solute (oil) in raffinate
y = Mass frac. solute (oil) in raffinate in extract (solvent)

a) M is now at saturated raffinate curve.
oil,M solids,M
x 0.21, x 0.63
Mass balance F + S = M
Solids .748F + (0) (S) = 0.63M


initial
0.748
M F 1187.3 kg R
0.63

S 187.3 kg
b) Now mixing is from S to a point on raffinate curve.
From equilibrium curve in solution to 14.E.1.
384


oil
x
oil
y
oil
1 y
0.21 .54 1.852
0.1625 .498 2.0080
0.115 .40 2.50
0.0675 .28 3.57
0.02 0.1 10.0

Insoluble Solids M.B.
Initial 0.748, F = 100, Final 0.81,
final
t
R
.81
final final
t t
R 748 R 923.5 kg
Raffinate is 0.81 solids, 0.02 oil and 0.17 solvent
Solvent remaining in raffinate is 0.17 923.5 157.0 kg
Needs to be recovered by evaporation.
Do Simpsons rule in 2 parts.

1
0.21 0.115
1.852 4 2.008 2.50 .1961
6


2
0.115 0.02
0.4241
2.50 4 3.57 10
6 0.6202


added t
S 0.6202 R , but what is
t
R ?
Eq. (13-29a) assumes
t
R Const.
Use average value of
t
R .

init
t ,avg t t ,final
1
R R R 1187.3 923.5 1055.4
2

or
added t ,avg
S 0.6202 R 0.6202 1055.4 654.6

total added
S Initial addition + S 187.3 654.6 841.9
Extract
S total remain in raffinate
amt S S 841.9 157.0 684.9by solvent
Oil in extract
F,0.1 final,oil t ,final
x F x R 0.252 1000 0.02 923.5 233.5
Total wt extract 684.9 233.5 918.4

oil
y 233.5 918.4 0.254

solvent
y 0.746

14.E.3. New problem in 3
rd
Edition. Solid Matrix is insoluble. Solids = (.748) 1000 = 748 kg.
t
R not
Constant, but Solid is.

t
Solids
Solids
R
x


A
A
Solids
x
ydS d R x Solids d
x


385


final ,A Solids
A,raf ,init
Solids ,raf ,init
x x
A Solids
x
x
d x x
S
Solids y

Changes limits integration.

oil Solids oil Solids
x 0.21, x 0.63, x x 0.21 0.63 .3333

oil Solids oil Solids
x 0.115, x 0.705, x x 0.115 .705 0.163

oil Solids oil Solids
x 0.02, x 0.81, x x 0.02 0.81 0.0247
Numbers for use in Simpsons rule are from Solution 14.E.2.


1
.3333 0.163
1.852 4 2.008 2.50 0.3515
6



2
0.163 0.0247
2.50 4 3.57 10 0.6173
6
Total 0.9688


added
S Solids total integral 748 0.9688 724.6 kg

total
S initial added 187.3 724.6 911.9 kg
Extract Amount Solvent
total raf ,final
S S 911.9 157 754.9 kg
Oil in extract = 0.252 (1000) 0.02 (923.5) = 233.5
Total weight extract 754.9 233.5 988.4

wt frac solvent = 0.764, wt frac oil = 0.236





386
Chapter 15 Solution Manual
Since this is a new chapter, all problems are new.
A. Discussion Problems.
15.A1. The mole fraction water is constant but since the temperature within the vessel varies the total
molar density C
m
varies and the water concentration = C
w
= y
w
C
m
also varies. Thus, Eq. (15-
10a) incorrectly predicts molecular diffusion. Equation (15-10b) predicts no molecular
diffusion because dy
w
/dz = 0.
B. Generation of Alternatives.
15.B1. For example, one could operate with both inflow and outflow at the bottom of the tube. If flow is
controlled with a constant head tank, the height of liquid in the tube will be very close to
constant.
C. Derivations.
15.C4. Substitute in q = ( Re)/(4) into Eq. (15-35d) and obtain = [(3
2
Re)/(4
2
g)]
1/3
.
15.C5. Start with Eq. 15-52a), set v
B
=0 and solve for y
A
v
A
. Then N
A
= C
m
y
A
v
A
. Substitute in the
expression for y
A
v
A
and Eq. (15-52e) for J
A
. This gives the desired result.
15.C6. This problem is included to show that one can derive the expressions in books. There is a lot of
algebra, but the derivation works. First, can expand the derivative,
2 2
1 1 1
2
1 1
(1 2 )
[ ( ) ]
AB x x
x x B A B x
| | c + c
=
|
c c +
\ .
Then take the derivative and expand all terms. The denominator becomes
3 3
1 2 1
[ ( ) ] [ ] A A B x Bx Ax + = + and the numerator simplifies to
2 2
2
2 . A B x Multiply by x
1
. Q.E.D.
15.C7. With CMO and y as mole fraction,
*
(1 ) 0
mol A A B B A A A B
v y v y v y v y v = + = + = . Since N
A
= -N
B
,
C
A
v
A
= -C
B
v
B
and for an ideal gas C
i
= y
i
C
m
. The total molar concentration C
m
is constant.
Then, v
A
= -(1-y
A
)v
B
/y
A
(Eq. A)
In terms of mass fractions y
A
=(y
A,mass
/MW
A
)/[y
A,mass
/MW
A
+ (1 y
A,mass
)/MW
B
]. (Eq. B)
Substitute Eq. B into Eq. A and simplify.
,
,
(1 ) /
/
A mass B
A B
A mass A
y MW
v v
y MW

= (Eq. C)
Then in mass terms
*
, , mass A mass A B mass B
v y v y v = + which after substituting in Eq. C and simplifying
*
, ,
(1 ) ( / )(1 )
mass A mass A B A mass B
v y MW MW y v ( =

. (Eq. D)
If MW
A
= MW
B
,
*
mass
v = 0. We can write
v
B
= N
B
C
B
= N
B
y
B
C
m
= N
B
C
m
(1 y
A
) (Eq. E)
where the y are mole fractions. Substituting Eq. B into Eq. E and then substituting this into Eq. D,
we obtain
,
, ,
*
, ,
1
(1 ) (1 )
/ (1 ) /
A mass
A
m B A mass A mass
B B
mass
A mass A A mass B
y
MW
C N y y
MW MW
v
y MW y MW
| | | |
+
| |
\ . \ .
=
+
(Eq. F)
Since y
A,mass
varies throughout the distillation,
*
mass
v is different for each stage.
387


D. Problems.
15.D1. D
prop,water
= 0.87E-9 m
2
/s. Eq. (15-9),
, , ,0
( / )( )
A
A z AB AB A L A
dC
J D D L C C
dz
= = . If
,0 A
C = 1.2
kg/m
3
is the known value,
, A L
C can be larger or smaller than
,0 A
C . For smaller
, A L
C we have

, A L
C =1.2 (0.2E-5)(0.0001)/0.87E-9 = 0.9701
If it is larger value, then
, A L
C =1.2 +(0.2E-5)(0.0001)/0.87E-9 = 1.430

15.D2. Taking the ratio of Eq. (15-23c) at the unknown T and at T =298.16,

exp[ / ( )]
( ) (298.16)
exp[ / (298.16 )]
o
o
E TR
D T D
E R

= 1.52E-09 for T = 335.18K. Flux


9 5
, , ,0
( / )( ) (1.52 10 / 0.0001)(0.9701 1.2) 0.35 10
A
A z AB AB A L A
dC
J D D L C C
dz

= = = =

The temperature can be found with Goal Seek from a spread sheet, but one has to trick Goal Seek
into working. Multiply the desired and the calculated fluxes by 1,000,000 and have Goal Seek
match these two values.

15.D3.a. 0.181cm
2
/s, b. 0.198 cm
2
/s, c. 0.0725 cm
2
/s, d. 0.198 cm
2
/s

15.D4. a. 0.0875 cm
2
/s, b. 0.096 cm
2
/s, c. 0.175 cm
2
/s, d. 0.096 cm
2
/s.

15.D5. Use Arrhenius form in Eq. (15-23c) but for mole fraction 0.0332 instead of infinite dilution. Write
the equation for both known temperatures and divide one of these equations by the other. The
constant D
o
divides out. Take the natural log of both sides and solve for E/R. The result is
1
2 2 1
( ) 1 1
/ ln /
( )
AB
AB
D T
E R
D T T T
| | | |
=
| |
\ . \ .

The constant D
o
can be found from the known conditions at T
1


1 1
( ) / exp[ / ( )]
o AB
D D T E RT =
Or from the known conditions at T
2
. The results are: E/R = 1348.3, E = 2677.6 cal/mol, D
AB

(x=0.0332, T=300) = 1.31310
-9
m
2
/s.

15.D6. Same equations as in 15.D5. At 298.16 K for the infinite dilution value set C
sucrose
= 0. Final
results are E
o
= 4953.8 cal/mol, D
AB
(infinite dilution, T = 320K) = 0.92510
-9
m
2
/s.

15.D7. For an ideal solution the term in brackets in Eq. (15-22) is equal to 1.0. Write this equation for
two of the x
A
values with the corresponding diffusivities (e.g., x = 0.0332 with D = 1.00710
-9
m
2
/s and x = 0.7617 with D = 1.22610
-9
m
2
/s). Then have two equations with the two unknowns:
o
AB
D and
o
BA
D . Solve for the two unknowns. Results are
o
AB
D = 0.99810
-9
m
2
/s and
o
BA
D =
1.30810
-9
m
2
/s. Check results with the other two mole fractions and find that the fit is good.

15.D8. From http://www.engineeringtoolbox.com/ the density of methanol at bp is 750.5 kg/m
3
(used a
linear interpolation), which means partial molar volume = 1/(density/MW)= 0.0426 kg/m
3
.
Viscosity of water is 1.0 cp = 0.001 Pa s = 0.001 kg/(ms).
a. With
B
= 2.26, D
AB
= 1.4310
-9
m
2
/s.
b. With
B
= 2.26, D
AB
= 1.5610
-9
m
2
/s.
388


15.D9. Combining Eqs. (15-35b) (15-35d),
( )
1/ 3
2
,max,
0.5 9 /
vertical liq
v gq = Assume that the bulk is pure
water with infinite dilution of ethanol. From Perrys Chemical Engineers Handbook, 8
th
edition,
(p. 2-305) at 1.0 bar (0.1 MPa) water has
W,m,liq
= 55.212 kmol/m
3

W,liq
=994.64 kg/m
3
and

W,m,vapor
= 0.032769kmol/m
3

W,vapor
= 0.5903 kg/m
3
. The water boils at 372.76K. At this
temperature, from p. 2-432, the viscosity of liquid water in Pas is,

29 10 4
,
exp[ 52,843 3703.6 / 5.866ln (5.879 10 ) ] 2.807 10
W liq
T T T Pa s

= + + =
The viscosity of the vapor at 372.76K is (p. 2-426)

8 1.1146 5
,
(1.7096 10 ) 1.2561 10
W vapor
T Pa s

= = or kg/(ms).
Now we can calculate the vertical velocity of the liquid water for q = 7.510
-6
m
2
/s (remember to use
liquid properties).

( )
1/ 3
2
1/ 3
2
,max,
9(994.64)(9.81)(7.5 6)
0.5 9 / 0.5 0.130 /
0.0002807
vertical liq
E
v gq m s
| |
= = =
|
\ .

A check of the units show they work. The modified Reynolds number (using gas properties) is,

, ,max
5
( )
(0.10)(0.5903)(0.81 0.130)
Re 3195.6
1.256 10
tube gas liq y
d v v


' = = =


The gas phase Schmidt number is
/
gas
EW
gas
Sc
D
| |
=
|
\ .
The viscosity and density were found earlier.
The diffusivity of ethanol and water in the vapor phase at 372.76K and 1.0 bar = 0.98717 atm can be
estimated from the Chapman-Enskog theory with the parameters in Table 15-2. This value of D
EW
=
1.65810
-5
m
2
/s. Then Sc
gas
= 1.283. Since both Re' and
gas
Sc are in the range for Eq. (15-47a), the
modified Sherwood number is,

0.77 0.33 .77 .33
( )
0.0328(Re ) 0.0328(3195.6) (1.283) 17.79
p tube B lm
gas
AB tot
k d p
Sc
D p
' = = =

15.D10. From the Chapman-Enskog theory D
NH3-air
= 2.0510
-5
m
2
/s at 318.16K and p = 1.2 atm.


The concentration at z = L is C
NH3
(L) = C
NH3
(z = 0) + J
NH3
L/D
NH3-air
. Results are 0.0002483 kmol/m
3

and 0.0002117 kmol/m
3
.

15D11. D = JL/C = 4.11410
-5
m
2
/s. Set up spreadsheet to obtain this value. Since the collision integral
was entered manually, had to do several iterations. After 6 iterations T = 396.2K (see
spreadsheet, and note that collision integral does not exactly match the value of kT/
AB
.
Problem 15.D10a, 3rd ed.
MW A 28.9 MW B 17 const 1.86E-07
T 318.16 p 1.2 T^3/2 5675.033
sigma A 3.711 sigma B 2.9 sigma AB 3.3055
eos A/kB 78.6 eps B/kB 558.3 eps AB/kB 209.4812
kT/EpsAB 1.5188 Col integ 1.197 Linear interpolation table 15-2
D AB 2.05E-05
D, cm^2/s 2.05E-01
389





15.D12*. From http://www.engineeringtoolbox.com/ viscosity is 1.0 cp = 0.001 Pas = 0.001 kg/(ms).
At 298.16 K, D
AB
= 1.114 m
2
/s. Density water = 998.3 kg/m
3
, viscosity water = 0.001 kg/(ms).
Calculate = 0.000115282 m, v
y,avg
= 0.04338 m/s, Re = 19.966. This is a long residence time
with Re< 20 so there are no ripples. Sh
avg
= 3.41 and k
avg
= 3.295E-05 m/s, and 0.000168 kg/s
carbon dioxide are absorbed.

15.D13. From http://www.engineeringtoolbox.com/ viscosity is 1.0 cp = 0.001 Pas = 0.001 kg/(ms). At
298.16 K, D
AB
= 1.114 m
2
/s. Density water = 998.3 kg/m
3
, viscosity water = 0.001 kg/(ms).
Calculate = 0.0001663 m, v
y,avg
= 0.090241 m/s, Re = 59.898. This is a long residence time,
laminar flow, with no surfactant so there are ripples. Sh
avg
= 5.8 and k
avg
= 3.89E-05 m/s, and
0.000198 kg/s carbon dioxide are absorbed

15.D14. From http://www.engineeringtoolbox.com/ viscosity is 1.0 cp = 0.001 Pas = 0.001 kg/(ms). At
298.16 K, D
AB
= 1.114 m
2
/s. Density water = 998.3 kg/m
3
, viscosity water = 0.001 kg/(ms).
Calculate = 0.0007717 m, v
y,avg
= 1.9441 m/s, Re = 5989.8. This is turbulent flow with 1300 <
Re < 8300. Sc
liq
= 899.2, Sh
avg
= 255.5 and k
avg
= 0.0003689 m/s, and 0.00188 kg/s carbon
dioxide are absorbed

15.D15. From http://www.engineeringtoolbox.com/ viscosity is 1.0 cp = 0.001 Pa s = 0.001 kg/(ms). At
298.16 K, D
AB
= 1.114 m
2
/s. Density water = 998.3 kg/m
3
, viscosity water = 0.001 kg/(ms).
Calculate = 0.000115282 m, v
y,avg
= 0.04338 m/s, Re = 19.966. This is a short residence time
with Re< 20 so there are no ripples. Sh
avg
= 9.942 and k
avg
= 9.61E-05 m/s, 7.851E-09 kg/s
carbon dioxide are absorbed.

15.D16. From http://www.engineeringtoolbox.com/ viscosity is 1.0 cp = 0.001 Pas = 0.001 kg/(ms). At
298.16 K, D
AB
= 1.114 m
2
/s. Density water = 998.3 kg/m
3
, viscosity water = 0.001 kg/(ms).
Problem 15.D11, 3rd ed.
MW A 28.9 MW B 17 const 1.86E-07
T 396.1642 p 0.9 T^3/2 7885.202
sigma A 3.711 sigma B 2.9 sigma AB 3.3055
eos A/kB 78.6 eps B/kB 558.3 eps AB/kB 209.4812
kT/EpsAB 1.891168 Col integ 1.1069 Linear interpolation table 15-2
D AB 4.11E-05
D desired 4.11E-05 chkB7-B8 1.90E-10
chk x E5 1.90E-05 Goal seek to zero changing B3
Problem 15.D11, 3rd ed.
MW A 28.9 MW B 17 const 0.0000001858
T 396.164199034186 p 0.9 T^3/2 =B3*SQRT(B3)
sigma A 3.711 sigma B 2.9 sigma AB =0.5*(B4+D4)
eos A/kB 78.6 eps B/kB 558.3 eps AB/kB =SQRT(B5*D5)
kT/EpsAB =B3/F5 Col integ 1.1069 Linear interpolation
D AB =F2*F3*SQRT(1/B2+1/D2)/(D3*F4*F4*D6) in Table 15-2
D desired 0.00004114 chkB7-B8 =B7-B8
chk x E5 =100000*D8 Goal seek to zero
changing B3
390

Calculate = 0.0001663 m, v
y,avg
= 0.090241 m/s, Re = 59.898. This is a long residence time,
laminar flow, with surfactant so there are no ripples. Sh
avg
= 3.41 and k
avg
= 2.28E-05 m/s, and
0.0001165 kg/s carbon dioxide are absorbed.

15.D17. Used a spreadsheet set up to solve Example 15-6. For = 0.001 meter one obtains x
NH3
=
0.04988, y
NH3,surface
= .21593, N
water
= 0.5393, N
NH3
= 0.0228307. The concentrations are the same
as in Example 15-6, but the fluxes are 10 larger.

15.D18. Part a. For two part solution need values at x
E
= 0.25 and 0.35. The average molecular weights
are calculated as in Example 15-5, and are used to determine the average molar densities. The
Fickian diffusivities are estimated by interpolating between values given in the Table in Example
15-5. The activity coefficients are determined in the same way as in Example 15-5. Then the
Maxwell-Stefan diffusivities are found by the same method. The values are listed below

MW
avg
m
,kmol/m
3
D
EW
, m
2
/s
E
EW
D , m
2
/s
X
E
= 0.25 25.0 36.28 0.63310
-9
1.9028 1.49510
-9
X
E
= 0.35 27.8 31.62 0.62510
-9
1.5553 1.60910
-9
Write Eq. (15-61c) for both intervals. For z from x
E
= 0.2 to 0.3 we obtain (values at x
E
= 0.2 and 0.3
are in Example 15-5),
9
9
36.28(1.495 10 )[1.7083(0.3) 2.1582(0.2)]
9.3445 10
1.9028(0.25)
E
zN


A = =
From x
E
= 0.3 to 0.4 (interval is over length - z) we obtain,

9
9
31.62(1.609 10 )[1.4338(0.4) 1.7083(0.3)]
(0.00068 ) 5.7027 10
1.5553(0.35)
E
z N


A = =
Adding the two equations to remove the unknown z and then solving for N
E
and z,, we obtain
N
E
= -2.12810
-5
kmol/s and z = 0.0004223m

Part b. Since the interval z is greater than the interval z = 0.0002577m, we subdivide the interval
from x
E
= 0.2 to 0.3 into 2 parts. The values needed are given below.

MW
avg
m
,kmol/m
3
D
EW
, m
2
/s
E
EW
D , m
2
/s
X
E
= 0.225 24.3 37.625 0.65910
-9
2.01976 1.48210
-9
X
E
= 0.275 25.7 34.99 0.62410
-9
1.79959 1.53210
-9
Equation (15-61c) is now written 3 times:
9
1
9
2
9
1 2
5.5371 10
3.9846 10
( ) 5.7027 10
E
E
E
z N
z N
z z N o

A =
A =
A A =

and solved for the 3 unknowns z
1
, z
2
, and N
E
. Obtain N
E
= -2.238910
-5
kmol/s, z
1
= , 0.0002473, and
z
2
= 0.0001780m.

15.D19 (Optional, Unsteady diffusion) At the average C = 0.001 mol/L
5 2
0.5228 10 cm / s
sucrose
D

= .
Equation becomes
5
,0
1
4(0.5228 10 )
A
A
C z
erf
C
t


Numerical values of
,0
/
A A
C C are easily obtained with a spreadsheet or with the use of Table 17-7.

391

z, cm t = 1000 s t = 10000 s t = 100000 s
0 1 1 1
0.01 0.9221 0.9753 0.9922
0.05 0.6249 0.8771 0.9610
0.1 0.3281 0.7571 0.9221
0.2 0.0505 0.5362 0.8449
0.3 0.00335 0.3535 0.7692
0.4 9.16E-05 0.2161 0.6957
0.5 1.009E-06 0.1220 0.6249
0.6 0.6352 0.5574
0.7 7.61E-12 0.0304 0.4936
0.8 0.0134 0.4346
0.9 0.00538 0.3788
1.0 0.00198 0.3281
1.2 0.000206 0.2406
1.4 1.49E-05 0.1710
1.6 7.49E-07 0.1177
1.8 0.0784
2.0 6.2E-10 0.0505
3.0 0.00335
3.5 6.20E-04
3.56 4.99E-04
4.00 9.16E-05
5.0 1.01E-06

Part b.
6
1.0 10 C

= when
4
0
/ 5.0 10 C C

= , which for t = 100000 s occurs for a thickness of <3.56
cm (Goal Seek gives 3.559 cm). Thus, at this time a layer 3.56 cm or thicker appears to be
infinitely thick.
Part c.
6
1.0 10 C

= when
4
0
/ 5.0 10 C C

= , which for o =0.10 cm occurs at t = 78.938 s (done with
Goal Seek on spreadsheet).

H. Spreadsheet Problems

15H1. Let A = air, B = hydrogen, and C = ammonia. Then N
C
= -N
A
N
B
. Substitute this expression
into Eqs. (15-65a, b)

m A C B A A A
A B
AB AC AC AB AC
m B C B B A B
A B
BA BC BA BC BC
y y y y y y
N N
z D D D D D
y y y y y y
N N
z D D D D D

| | | | A
= + + +
| |
A
\ . \ .
| | | | A
= + +
| |
A
\ . \ .


Determine N
B
from the 1
st
equation and N
A
from the second.

392

m A C B A
A
AB AC AC
B
A A
AB AC
m B C A B
B
BA BC BC
A
B B
BA BC
y y y y
N
z D D D
N
y y
D D
y y y y
N
z D D D
N
y y
D D

| | A
+ + +
|
A
\ .
=
| |

|
\ .
| | A
+ + +
|
A
\ .
=
| |

|
\ .


Put these equations and the values for mole fractions at the boundaries, diffusivities,
m
, and z = into a
spread sheet. Guess a value for N
A,guess
, calculate N
B
and N
A,calc
, and use Goal Seek to make
N
A,guess
- N
A,calc
= 0 by changing the value of N
A,guess
.
Results N
air
= -6.209E-5, N
H2
= 14.026E-5, and N
NH3
= -7.817E-5 kmol/s. As expected hydrogen
diffuses in the positive direction and ammonia in the negative direction. The surprise is the
substantial negative diffusion rate of air. (Spreadsheet shown in 15.H4, but with different
numbers.)

15.H2. Used a spreadsheet set up to solve Example 15-6. For a bulk gas that is 40% air, 15 % NH
3
and
45% water obtain x
NH3
= 0.05596, y
NH3,surface
= .24225, N
water
= 0.032964, N
NH3
= 0.001955 kmol/s

15.H3a. See solution to 15.H1 for procedure and 15.H4 for example spreadsheet.
Results: N
air
= -1.87E-8, N
H2
= 2.98E-7, N
NH3
= -2.80E-7 kmol/s.
b. For ammonia D
eff
= 1.5656E-5 m
2
/s. Estimating dC/dz with the difference approximation for a very
dilute solution, N = J = -D
eff

m
y/(z)= -(1.5656E-5)(0.08928 kmol/m
3
)(.002)/(.01m) = -2.80E-
7 kmol/s. Thus, this is accurate. For hydrogen and air D
air-H2
= 3.0550E-5 m
2
/s. Then
N
air
= J
air
= (3.0550E-5)(0.08928 kmol/m
3
)(-0.001)/(.01m) = 2.73E-7 kmol/s. The same value is
obtained for hydrogen. The hydrogen value is close, but the air value is not close. Conclusion:
Use the Maxwell-Stefan approach.

15.H4. See solution to 15.H1 for procedure. Results: N
air
= -5.903E-5, N
H2
= 14.069E-5, N
NH3
= -
8.166E-5 kmol/s. Note the substantial negative diffusion of air despite the zero driving force.
The air is dragged along with the ammonia. The spreadsheet is given below (labeled as 15.D22),
first with the numbers, and then with the formulas.
393

HW 15-D22, 3rd Ed. SPE
A=air, B=H2, C = NH3
T 273.000000000 p 2.000000000
D AB 1 atm 0.000061100 D AC 1 atm 0.000019800 DBC 1 atm 0.000074800
D AB 0.000030550 D AC 0.000009900 DBC 0.000037400
NA guess -0.000059028
del z 0.010000000 0.089278949
yA z=0 0.520000000 y B z=0 0.480000000 y C z=0 0.000000000
yA z= 0.520000000 yB z= 0.000000000 yC z= 0.480000000
yA avg 0.520000000 yB avg 0.240000000 yC avg 0.240000000
yA/z 0.000000000 yB/z -4.285389534
NB 0.000140693 kmol/s
NA calc -0.000059028 kmol/s
chk NA-Nacalc 0.000000000
100000 chk 0.000000000 Goal seek to zero
NC -0.000081665 change B10





HW 15-D22, 3rd Ed. SPE
A=air, B=H2, C = NH3
T 273 p 2
D AB 1 atm 0.0000611 D AC 1 atm 0.0000198 DBC 1 atm0.0000748
D AB =B7/D3 D AC =D7/D3 DBC =F7/D3
NA guess -0.0000590279043468439
del z 0.01 =D3/(0.0820575*B3)
yA z=0 0.52 y B z=0 0.48 y C z=0 =1-B14-D14
yA z= 0.52 yB z= =B9*B11/D3 yC z= =1-B15-D15
yA avg =(B14+B15)/2 yB avg =(D14+D15)/2 yC avg =(F14+F15)/2
yA/z =-D12*(B14-B15)/B12 yB/z =-D12*(D14-D15)/B12
NB =(B17+(D16/B8+F16/D8+B16/D8)*B10)/(B16/B8-B16/D8) kmol/s
NA calc =(D17+(B16/B8+F16/F8+D16/F8)*B19)/(D16/B8-D16/F8) kmol/s
chk NA-Nacalc=B20-B10
100000 chk =100000*B21 Goal seek to zero
NC =-B19-B20 change B10
395
SPE 3
rd
Solution Manual Chapter 16
New Problems and new solutions are listed as new immediately after the solution number. These new
problems are: 16.A1, 16.A7, 16.A8, 16.C2 16C4, 16.C5, 16.D3, 16.D9, 16.D16 to 16.D22, 16.G1-
16.G3, 16.H1-16.H2. Chapter 16 was chapter 15 in the 2
nd
edition. Problems from that edition
have the same problem number, but the chapter number is now 16 (e.g., problem 15.D6 is now
16.D6).
16.C3. (was problem 15C3 in the 2
nd
edition.) Equilibrium
x
y
1 1 x
, Operating y = x,
out
in
y
OG *
y
dy
n
y y
. Substitute in for y* and let x = y (total reflux operating line)
OG 2 2
dy dy
n
y
y y y y
y
1 1 y
1 1 y
out out
in in
y y
OG
y y
1 dy
1 dy 1
n
1 y y 1 1 y 1
out in out
OG
out in in
y 1 y 1 y 1 1
n n 1 n
y y 1 y 1
which becomes Eq. (16-81).
16.C4. New problem in 3
rd
edition. With extract dispersed,
IN M,OUT
MD *
IN M,OUT
y y
E
y y
Since
M,OUT
IN MD *
M,OUT
y
y 0, E
y
where
*
M,OUT M,OUT
y m x
Mixer mass balance with
IN
y 0,
IN M,OUT M,OUT
Fx S y F x
Solving for
M,OUT
x :
IN M,OUT
M,OUT
F x S y
x
F
Then,
IN M,OUT *
M,OUT
F x S y
y m
F
M,OUT M,OUT
M,D *
M,OUT IN M,OUT
y y
E
y m F x S y
F
M,D IN
M,OUT
MD
E m x
y
mS
1 E
F
396

External MB
IN IN S,OUT S,OUT
y 0 F x S y F x

S,IN S,OUT
S,D *
S,IN S,OUT
y y
E
y y

*
S,OUT S,OUT
Equil : y m x
Substituting:
*
S,OUT IN S,OUT
S
y m x y
F



S,IN S,OUT S,IN SD SD IN
S,D S,OUT
S,IN IN S,OUT SD
y y y 1 E m E x
E y
S mS
y m x y 1 E
F F

Substitute in
MD IN
S,IN M,OUT
MD
E m x
y y
mS
1 E
F
and do some algebra to obtain,

MD SD MD
S,OUT IN
MD SD
m S
E E 1 E 1
F
y m x
m S m S
1 E 1 E
F F

The definition of the total stage efficiency is,

IN
IN
D D,S,OUT IN S,OUT S,OUT
total,D * * *
D D,S,OUT IN S,OUT S,OUT
x x y y y
E
x x y y y
IN
with y 0

Equilibrium:
*
S,OUT IN S,OUT
S
y m x y .
F

Substitute into definition.


S,OUT S,OUT IN
total,D
S,OUT
IN S,OUT
IN
y y x
E
S
y
S
m x y
m 1
F
F x


Which after substituting in
S,OUT
y and doing some algebra, becomes.

MD SD MD
total
MD MD
MD SD MD
m S
E E 1 E 1
F
E
m SE m S E m S m S
1 1 E E 1 E 1
F F F F

16.D1. (was problem 15.D1 in the 2
nd
edition) The corrected value of
G,E
H 1.41ft is given in
Example 16-2. From the results of Examples 16-1 and 16-2 and from Eq. (16-38),
397


0.15 0.15
L,E,cor L,E,Initial
2.61 2.61
H H 0.83 0.85
2.2 2.2
ft
Eq. (16-27a)
OG L G
mV
H H H where V L 8 5
L

The value of m (the slope of the equilibrium curve) varies throughout the enriching section.
From the McCabe-Thiele diagram used to prepare Figure 16-4A values of m were found from
*
D
y 0.442 y .63 to y x 0.8 y* 0.82 .

y 0.442 0.5315 0.621 0.7105 0.8
y* 0.63 0.598 0.66 .727 .822
m 0.441 0.406 0.449 .5144 .745
OG
H
2.01 1.96 2.02 2.11 2.42
y* - y 0.188 0.0665 0.039 0.0165 0.022
1
y* y
5.32 15.04 25.641 60.606 45.45
Arithmetic Average:
OG
1
H 2.01 1.96 2.02 2.11 2.42 2.10
5

Geometric Average:
1 5
OG
H 2.01 1.96 2.02 2.11 2.42 2.10
No difference!

To find
OG
n do Simpsons rule in 2 parts because of the unusual shape of
*
1 y y vs. y.

OG,1
0.179
n 5.32 4 15.04 25.641 2.718
6


OG,2
0.179
n 25.641 4 60.606 45.45 9.353
6


OG OG,1 OG,2
n n n 12.072 ,
E OG OG
h H n 25.35
This is reasonably close to the 26.1 ft estimated in Example 16-1.

16.D2. (was problem 15.D2 in the 2
nd
edition) a. The physical properties and use of Figure 10-25
calculated in Example 16-2 are unchanged. Now F = 48.

1 2
1 2
2
flood flood
F 2
20
G G 2 1.16 0.749 lb ft
F 1 48


1 4
1 4
F 1
48
Dia 1 Dia 2 4.68 5.83 ft
F 2 20

b. In Eq. (16-37)
col
2 141, 1 55, D 2 same .
Estimate
p
h ~10 ft (we know it will be less than before),
C,V
S is unchanged, terms in
denominator are unchanged.

1 3
1 3
p
G,E G,E 1 3
p
h 1 10
1
55 10
H 1 H 2 1.33 0.40
2 141 22
h 2 10
ft
398

In Eq. (16-38),
p p
1 0.045 , 2 0.07 , h 1 10 , h 2 22 .

f ,L C,L
C , and S unchanged.

0.15
0.15
p
L,E L,E
p
h 1 1
0.045 10
H 1 H 2 0.83 0.47
2 h 2 0.07 22
ft

16.D3. New problem in 3
rd
edition. In the enriching section,
ln( / ) / [( / ) 1]
OG
HETP H mV L mV L and /
OG L G
H H mV L H
With 0.827, 1.33, / ( / ) 0.63/ (5 / 8), 2.16,
L G OG
H H m L V H and HETP = 2.15 (from
Example 16-2). With the same mass transfer coefficients, but m = 0.577,
[0.577 / (5 / 8)]0.83 1.33 2.10
OG
H and 2.10ln(0.9232) / (.9232 1) 2.19 HETP ft

A 24.4% increase in both mass transfer coefficients gives 1.244(0.83) 1.033
L
H and
1.244(1.33) 1.65
G
H .
For m = 0.63, [0.63/ (5 / 8)]1.033 1.65 2.69
OG
H and
2.69ln[0.63/ (5 / 8)] / [0.63/ (5 / 8) 1] 2.68 HETP .
With the same mass transfer coefficients, but with m = 0.577, 2.61
OG
H and HETP = 2.71.

A 24.4% decrease in both mass transfer coefficients gives:
For m = 0.63, 0.63, 1.01, 1.65, 1.64
L G OG
H H H HETP
For m = 0.577, 0.63, 1.01, 1.59, 1.65
L G OG
H H H HETP .

Clearly, the variation in mass transfer coefficients results in a large range for HETP (from 1.64 to
2.69 feet for m = 0.63) while the small change in m had little effect. To be safe the larger value
of HETP = 2.69 would be used. This is a safety factor of 1.20. Note that Bolles and Fair (1982)
recommend a safety factor of up to 1.70.



16.D4. (was problem 15.D4 in the 2
nd
edition) Since we have total reboiler,
s,in
y 0.04
Feed line:
q .6
q .6, 1.5
q-1 .4
. Plot this and operating lines (see figure).
Top:
D
L L L L D 0.9
y x 1 x where 0.474
V V V 1 L D 1.9


D
L
y intercept 1 x 0.484
D

Bottom goes through
B
y x x 0.04 .
399



G S,out E,in E,out
AI A
dy
n , where y y .605, y .92
y y

For enriching section, from Eq. (16-16) draw line of slope
G
L
H L 1.3
0.474 .770
V H 0.8
to get
I I
y and x . For stripping section slope is

G
L
H L 1.3
2.568 4.17
V H .8

From figure generate following table.

y
A
y
AI
y
AI
-y
A
1/(y
Ai
-y
A
)
Stripping: .04 .13 0.09 11.111
.3225 .46 .1375 7.2727
.605 .63 .025 40.00
Enriching: .605 .62 .015 66.66
.7625 .80 .0375 26.66
.92 .95 .03 33.333

Simpsons rule,
G,S
.565
n 11.11 4 7.2727 40 7.55
6
,
strip G G,S
h H n 9.8ft.

G,E
.315
n 66.66 4 26.66 33.33 10.8
6
,
E G G
h H n 1.3 10.8 14.1 ft

16.D5. (was problem 15.D5 in the 2
nd
edition) a) Total Reflux.
out in
y 0.956, y 0.65
Eq. (16-81),
OG
.956 1 .65 .65 1
1
n n n
1 2.26 .65 1 .956 .956 1

400


OG OG
6
n 4.0257, H 1.4904
4.0257

b) Finite reflux. Plot op. line. Find y & y* (see graph). Use Simpsons rule in 2 parts.

c) Changes in L/V in equation connecting HETP and
OG
H

y y* y* y 1
y* y

0.783 0.842 0.059 16.949
0.82225 0.8795 0.05725 17.467
0.8615 .9125 0.051 19.608
.90075 .943 0.04225 23.6686
0.94 .973 0.033 30.303
OG1
0.8615 0.783
n 16.949 4 17.467 19.608 1.3924
6

OG2
OG
0.94 0.8615 1.89166
n 19.608 4 23.6686 30.303
6 n 3.2841

401


402



16.D6. (was problem 15.D6 in the 2
nd
edition) a.
OG
n 11.11 3.56 14.67

OG OG
H h n 7.47 14.67 0.509 m
b. From McCabe-Thiele diagram we find the following

y 0.016 0.066 0.116 .494 .872 .922 .972
y* .0267 0.1067 0.1815 .623 .9201 .9523 .9832
y* - y 0.017 0.0407 0.0655 .129 0.0481 .0303 .0112
1/(y* - y) 93.46 24.57 15.27 7.752 20.79 33.00 89.29

Do integration with Simpsons rule in three parts.

OG
.1 .756
n 93.46 4 24.57 15.27 15.27 7.752 4 20.79
6 6


.1
20.79 4 33.0 89.29 15.97
6

The difference is because of inaccuracies in Simpsons rule.

16.D7. (was problem 15.D7 in the 2
nd
edition) Bottoms
A
x partial reboiler 0.1 Distillate
A
x .9
From equilibrium data in 4.D7. @
A
x 0.1, equil.
in
y 0.262 y With total condenser,
out D
y x 0.9
Find average
A A
A A
y x
1 y 1 x
. From data in 4.D7.,
*
y 0.929 when x 0.9.
Bottom
0.262 0.1
3.195
0.738 0.9
,
dist
.929 0.9
1.454
.071 0.1

Geometric Avg.,
1 2
avg
3.195 1.454 2.155
Eq. (16-81),
OG
0.9 1 0.262
1 0.262 1
n n n 4.80
1 2.155 0.262 1 0.9 0.9 1


OG OG
H h n 0.42 m

16.D8. (was problem 15.D8 in the 2
nd
edition)
tot
m H P 22500 855 26.3

403


26.3 .0011
L
max 28.93
V .0011 .0001
,
L
act 26.04
V
. Basis V = 1, then L = 26.04.

out in out
L
y x x 26.04 .001 0.02604
V


*
A,out
L 26.04
x 0, 0.9895
mV 26.316

Use Colburn Eq. (16-34b),

OL
1 0.0011 0
n n 0.010488 .9895 9.51
0.010488 0.0001 0

h 0.84 9.51 7.99 m
Can check with Eq. (16-63) and get same
OL
n

16.D9. New problem in 3
rd
edition.
For example, if 1,
O ED
n we have for the perfectly mixed model,
1/ 2
1
O ED
MD
O ED
n
E
n

For the plug flow model,
1 exp( ) 1 exp( 1) 0.632
MD O ED
E n
For same value of
O ED
n the plug flow model always predicts a higher stage efficiency.

16.D10. (was problem 15.D10 in the 2
nd
edition) For both countercurrent and cocurrent
in
out
y
OG
*
y
dy
n
y y
. For both cases
* *
y 0, and y y y , then

in
out
y
in
OG
out y
y dy 0.01
n n n 4.6
y y 0.0001

Note that
OG
n is same because of irreversible reaction.
c. Flow rates enter into solution only as a check that at least the stoichiometric amount of sulfuric
acid is available to neutralize the ammonia.

16.D11. (was problem 15.D11 in the 2
nd
edition) From Eq. (16-72) with irreversible reaction,

in
out
A
OG
A
y
50.0 ppm
n n n 8.517
y 0.01 ppm
for both cocurrent and countercurrent.

16.D12. a. (was problem 15.D12 in the 2
nd
edition)
tot
m H p 2.7 1.1 2.4545, y mx

min
L .013 .00004
2.4453
G .0053
(see Figure)

in out
out
min
y y L
15 L G 36.679, x 0.0003533
G L G

Use Eq. (16-57),
* *
in in out out out
y 0.013, y m x 0.0008672, y 0.00004, y 0
404


* *
in out
y y 0.0121328, y y .00004

OG
.01296 .0121328
n n 6.1246
0.0120928 .00004
, h 4.59 ft


b. Cocurrent. Operating and equilibrium lines shown in figure. Lowest
out
y is at intersection point
= 0.00081. For
out
y 0.00085 ,

in out *
out Aout out
y y
.013 .00085
x 0.00033125, y mx 0.000813
L V 36.679


m 2.4545
0.0669184
L V 36.679
. Use Eq. (16-20),

OG
1 0.013 0.000813
n 1.0669184 0.0669184 5.496
1.0669184 0.00085 0.000813

h 1.98 ft

16.D13. (was problem 15.D13 in 2
nd
edition) a. Same conditions as Example 16-3. Assume same
OL
H .
If operation is possible, find
OG out
n & y .
Dilute Use Eq. 16-70. m = 30.36 (Example 16-3)

*
out out
y m x b 30.36 0.001 0.03036

mV 30.36
2.024
L 15
,
*
in out
OG *
out out
y y 1 mV mV
n n 1
mV L y y L
1
L

Mass bal.
in in out out
Lx Vy Lx Vy

out in in out
L L
y x y x 15 0 0.03082 15 0.001 0.01582
V V


OG
1 0.03082 0.03036 mV
n n 3.024
3.024 0.01582 0.03036 L

Not possible, term inside brackets is negative.
b) Same conditions as Example 16-3 except
out
x 0.002 . Assume same
OL
H . If operation is
possible, then
405


*
out out
y m x 30.36 .002 0.06072

out
y 15 0 0.03082 15 0.002 0.00082

OG
1 0.03082 0.06072
n n 3.024 2.024
3.024 0.00082 0.06072
still not possible
c) Same conditions, except
out
L V 40 and x 0.0003

*
out
y 30.36 0.0003 0.0091, mV L 30.36 40 0.759

out
y 15 0 0.03082 40 0.0003 0.01882

OG
1 0.03082 0.0091
n n 1.759 0.759
1.759 0.01882 0.0091
0.656

16.D14. (was problem 15.D14 in the 2
nd
edition) a. Use equilibrium data shown in Figure 4-16,
x 0.8, m 0.415; x 0.16, m 1.5
Mixed:
MV y active
E 1 exp K aA h V [Eq. (16-77)] where
c
V A 30 and b 2 12 .
Then:
y MV
active
V
K a n 1 E
h A


y
30
x 0.8, K a 1 2 n 1 .77 330.62
2 12
where 1 2 0.8

y
30
x 0.16, K a n 1 .69 263.5
.8 2 12

Eq. (16-6a),
y x y
1 m 1
K a k a k a


x y
1 0.415 1
x 0.8,
330.62 k a k a
,
x y
1 1.5 1
x 0.16,
263.5 k a k a

Solving simultaneously,
x
k a 1408.19 and
y
k a 366.317
b. x 0.01, m 6.06. Then from Eq. (16-6a),
y
K a 142.18 &
MV
E 0.468 from Eq. 16-77.

16-D15. (was problem 15.D1 in the 2
nd
edition) Assume feed to Example 16-4 is satd liquid, z = 0.5, &
separation complete
D B
x ~1, x ~ 0 .
F f
2.5 .5
x z 0.5, y 0.7143
1 1.5 .5


D
min
L 1 0.7143
x 1, 0.5714
V 1 0.5
,
B
max
L 0.7143 0
x 0, 1.4286
V 0.5 0

At
intersection op lines D
L L
x .5, y x 1 x .8 .5 .2 1.0 0.6
V V


actual
L 0.6 0
1.2
V 0.5 0

Calculate at x 0.1, 0.3, 0.7 in example , 0.9
406

x Sect L/V
2
m Eq. 15 80
1 1 x

L
mV

0.1 Strip 1.2
2
2.5
1.8904
1 .15

0.6348

0.3 Strip 1.2
2
2.5
1.1891
1 .45

1.0092
0.7 Enrich .8
2
2.5
0.5949
1 1.05

1.3447
0.9 Enrich .8
2
2.5
0.453
1 1.35

1.767

mV
E const. 0.97. Calculate E
pt
. Solve Eq. (16-78)
MV
pt
E L
E n 1
mV L mV

and then
y
K a from Eq. (16-76a),
y pt
active
V
K a n 1 E
A h

x

pt
E
y
K a
0.1
0.971
0.6348 n 1 0.5891
.6348

133.42
0.3
0.971
1.0092 n 1 0.6802
1.0092

171.02
0.7

0.971
1.3447 n 1 0.731
1.3447

196.9

0.9

0.971
1.767 n 1 0.7733
1.767

222.65


16.D16. New problem in 3
rd
edition. Using Simpsons rule the new value for A
1
= 7.18, the new value
for A
2
= 10.85 and the new total area = 18.03. Then the calculated height of the enriching section
(0.4054 m)(18.03) = 7.31 m compared to the previous result of 7.95 m. This is an error of 8.1 %.
Thus, a rather small error in mole fractions becomes a larger error.

16.D17. New problem in 3
rd
edition. Relative errors in
G
k a 24.4%.
Same relative errors in
G
H and
L
H .

In the enrichment section the slope of the mass transfer line is


G
L
0.4054 24.4% H L 5
Slope
V H 8 0.253 24.4%


407

a) If
L
H correct but
G
H varies by 24.4%, range of slope is from 1.242 to 0.761.
Value at
G
H 0.4054 is slope = 1.0015. At top:



If equilibrium line straight from azeotrope to point x = 0.7472, y = 0.7815, then can fit
this portion of equilibrium as,

y 0.7668x 0.2085

MT line y sx b. Since y x 0.8, b 0.8 1 s

If s 1.242, b 1.7936

If s 1.0015, b 1.6012

If s 0.761, b 1.4088

All calculations at
A
y 0.8.
I
y at intersection equilibrium, y 0.7668x 0.2085 and
mass transfer line,
y b
y sx b x
s


408

Substitute in for x,
I
I
y b
y 0.7668 0.2085
s

Solve for
I
y

I
0.7668b
0.2085
s
y
.7668
1
s





s 1.242, b 1.7936:


I
y
0.81356


I
y y
0.013565

I
1
y y
73.72

s = 1.0015, b = 1.6012:
0.812426

0.01246 80.48*
s = 0.761, b = 1.4038:
0.80842

0.008419 118.78

*83.3 in Example 16.1 since numbers rounded first. Amount of error depends on
distance between equilibrium and operating lines. Less error if closer, but more impact
on
AI I
1 y y .

Assume same relative errors:

L
H no error,
G
AI A
80.48 73.72
1
H higher, error 8.4%
y y 80.48


L
H no error,
G
AI A
1 118.78 80.48
H lower, error 47.6%
y y 80.48

Assume error in H is same every point. Thus enriching area can be a lot different than
calculated. But if
G
H is down by 24.4%, area is up by 47.6% so there is some
cancellation of error.

b. If
L
H &
G
H both vary could have s 1.634 and b 2.1072 . Thus,
I
y 0.81493,
I
y y 0.493,
I
1
y y
= 66.97

Area
66.97
19.6 16.31.
80.48

Since
GE G,E G,E
H 1.24(0.4054) 0.503 m, z H n 8.20 mwhich is

a 3.2 % increase.


16.D18. New problem in 3
rd
edition.
a. Equilibrium is y = mx. Value of m is unknown, but C
extract
= m
conc units
C
raffinate
with

3
_ 3
_ /
20.8
_ /
conc units
kmol Benzoic m extract
m
kmol Benzoic m raffinate
. We need m in
409


/
/
mole fraction units
mol Benzoic mol extract
m
mol Benzoic mol raffinate
. The resulting conversion is,
_ _ _
( )( )
raffinate
extract
mole fraction units conc units
extract raffinate
MW
m m
MW

Since the system is dilute, extract properties are essentially the same as pure solvent and
raffinate properties are essentially the same as diluent.

m = (20.8)(92.14/865)(1000/18) = 123.1

Note that y = x
D
and E
My
= E
MD
.

E
My
= (y
in
-y
out
)/(y
in
-y
out
*) where y
out
* = m x
out
= (123.1)(1.99E-06) = .000245

E
My
= (0-.000230)/(0-.000245) = 0.939

b. From Eq. (16-76b), n
O-ED
= E
My
/(1-E
My
) = 0.939/(1 0.939) = 15.393

O ED mixer
O ED
D D D
K a V
n
Q MW
,
2 2 3
/ 4 (0.75) 0.75 / 4 0.331
mixer
V h D m
2 2 3
/ ( )
(15.393)(.0012)(865) / [(0.331)(92.14)]
0.524 kmol / s m mol frac disp m dispersed / m total volume
O ED O ED D D mixer D
K a n Q V MW

Note that
O ED
K a is larger than in Example 16-5 because the residence time
mixer
res
D C d
V
t 37.45s
(Q Q )
is shorter than the 361 s in Example 16-5. Thus, this problem
requires much more vigorous mixing.

c. Differential Model
MD OED
E 1 exp n


OED MD
exp n 1 E 0.061


OED OED
n n 0.061 2.797 n 2.797


2 2 3
/ ( )
(2.7969)(.0012)(865) / [(0.331)(92.14)]
0.0952 kmol / s m mol frac disp m dispersed / m total volume
O ED O ED D D mixer D
K a n Q V MW


d. Use of mixed models: If use
O ED
K a 0.524 from mixed staged model, then
O ED mixer
O ED
D D D
K a V
n 15.393
Q MW
and with differential model
410

MD OED
E 1 exp n 1 exp( 15.393) .9999998 . Obviously, mixing models gives
wrong results.

16.D19. New problem in 3
rd
edition.
a.
in out
in
D D
MD,Conc *
D D,out
C C
E
C C
with
*
D,out CD C,out
C m C


D,in
C 0.000 ,
D,out
C 0.00023,
C,out
C 0.00536


*
D,out CD C,out C,out
C m C 0.0481 C 0.0002578


MD
.00023
E .892
.0002578

b.
MD,Conc
O ED
MD,Conc O ED
O ED MD,Conc
E
n 0.892
E n 8.26
1 n 1 E 0.108


2 3 2 3
/ ( )
(8.26)(.0012)(865) / [(0.331)(92.14)]
0.2811 kmol / s m kmol/m m dispersed / m total volume
O ED O ED D D mixer D
K a n Q V MW


c. Differential Model:

MD,Conc OED
E 1 exp n
OED MD,Conc
exp n 1 E 0.108


OED OED
n n .108 2.2256 n 2.2256


2 3 2 3
/ ( )
(2.2256)(.0012)(865) / [(0.331)(92.14)]
0.0757 kmol / s m kmol/m m dispersed / m total volume
O ED O ED D D mixer D
K a n Q V MW


d.
MD,Conc OED
E 1 exp n 1 exp 8.26 0.99974

16.D.20. New Problem in 3
rd
Edition.
a. Eq. (16-92). Terms:
d
0.167,
-9
benzoic-water
2.2 10 D
C
C A C
4 3 2 . 6 8
D

(from Example 16-6)

2
2
i
0.2070 16.6667 d
5.8632
g 9.807

Note N in rps
411


i
p
d 0.2070
820.13
d 0.0002524


p
tank
d
0.0002524
0.00030487
d 0.8279


D p
d g 865 0.0002524 9.807
96.447
0.0222


9
5 12 2
C
2.2 10
k 0.00001237 432.68 820.13
0.0002524


1 2 5 4
1 2
1
x 0.00030487 96.447 0.014801
0.167

Significantly higher. But Note:
d
0.06, so correlation may not be valid. Same
With same
d
k 0.001905.
LD D CD C
1 1 1
K k m k


LD
1 1 1
1930.2
1 K 0.001905
0.014801
20.8


1
LD LD
K 0.0005181 & K a 2.0586 s

OED
K a 865 2.0568 92.14 19.13

MD
19.31
E 0.951,
20.31
which agrees with guess for residence time.
b. Resistance (dispersed) =
1
524.9
0.001905

Resistance (continuous)
1
1404.5
0.000712

Sum of resistances 1929.4
LD
Gives K 1 1929.4 0.0005181
% resistance from dispersed 524.9 1929.4 100 27.2%
This is a significant contribution because of significantly higher predicted
continuous phase mass transfer coefficient.
c. If
D
k is ignored ,
LD C CD
K k m 0.014801 20.8 0.000712
Note that this is significantly too high.
d. To be safe, use lower estimate of
MD
E . Note that
d
is really too large for use of
Eq. (16-92).

16.D.21. New problem in 3
rd
edition. In settler velocity approaches zero
(
t
u 0.00172 in Example 13-5, but
p
d 0.000204 not 0.000150 assumed). Thus
t
u ~ 0.00495 m s and Eq. (16-88a) is reasonable approximation.

C C benzoic-water
Sh k d 2.0 D where in settler d 0.0002524
412


9 5
C
k 2.0 2.2 10 0.0002524 1.7433 10
Using
5 7
LD C CD
K ~ k m 1.7433 10 1 20.8 8.381 10 m s
If the interface in the settler is at the centerline, then the volume of aqueous phase is
2
S S
1
D 4 L .
2
From Example 13-5,
S
D 1.023m and
S S
L 4D . Then Volume
aqueous
3
phase 1.682 m . The water residence time is
3 3
1.682 m 0.006 m s 280.3s.

Entering total velocity is
3
0.0072 m s and
d
0.167 leaving mixer. A drop that starts at the bottom of the water phase travels a
vertical distance of
S
D 2 0.5115 m to reach the interface. This requires
0.5115 m 0.00495m s 103.2s.
Assume equal distribution of drops. Everything (half the drops) above interface are
collected very quickly. All drops collected in 103.2 280.3 0.368 fraction of settler.
If average over this fraction is
d,IN
0.5 , then
D,avg
0.5 0.167 0.368 0.031.
d
p
6 0.031 6
a 737
d 0.0002524
and
LD
settler
K a ~ 0.000618

OED D LD D
settler settler
K a K a MW 865 0.000618 92.14 0.0058


settler D
OED,settler
V Q 0.0058 3.364 0.0012 0.0058
n 1.7314.
865 92.14 865 92.14

Since settler is not well mixed,
settler,D
E 1 exp 1.734 0.823
This is high because of long residence time.

16.D.22. New problem in 3
rd
edition.
a)
M,OUT S,OUT
SD *
M,OUT S,OUT
y y
E 0.82,
y y

IN M,OUT M,OUT
MD * *
IN M,OUT M,OUT
y y y
E 0.794
y y y


*
M,OUT raf ,OUT
y mx .
From Example 16.5 in mole fraction units m 123.1.
Mixer mass balance with
IN
y 0 (Example 16.5) is
F M,OUT C,OUT
Fx Sy Fx
In Example 13.5 feed is
3
0.006m s and solvent is
3
0.0012m s


3 3
S 0.0012m s 865kg m 92.14kg kmol 0.01127kmol s

3 3
F 0.006m s 998kg m 18.02kg kmol 0.3323kmol s

M,OUT feed OUT OUT
0.01127
x x S F y 0.00026 y
0.3323

Then substitute into
MD
E

M,OUT
M,OUT
y
0.794
123.1 0.00026 0.03390 y

413


Solving, we obtain
M,OUT
y 0.00589.


M,OUT M,OUT MD
x y m E 0.00589 123.1 0.794 6.027E 5.
For settler,
*
S,OUT S,OUT
y m x
Settler M.B. is,
M,OUT M,OUT S,OUT S,OUT
Sy Fx Sy Fx

S,OUT M,OUT S,OUT M,OUT
S
x x y y
F


S,OUT S,OUT
x 6.027E 5 0.03390 y 0.00589
Substitute into equation for
SD
E


S,OUT
S,OUT
0.00589 y
0.82
0.00589 123.1 6.027E 5 0.00019967 0.03390y

Solving, we obtain
S,OUT
y 0.006174.
From Mass Balance,


S,OUT
x 6.027E 5 0.03390 0.006174 0.00589 5.0653E 5

*
S,OUT S,OUT
y m x 123.1 5.0653E 5 0.006235

S,OUT
Total,D *
S,OUT
y
0.006174
E 0.99015
y 0.006235

b) From 16.C4
0.01127
mS
123.1 4.17495
F 0.3323

Total,D
E
0.794 0.82 1 0.794 4.17495 1
1 4.1794 0.794 1 4.17495 0.82 4.17495 0.794 0.82 1 0.794 4.17495 1
.99003


16.G1 and 16.G2. New problems in Chapter 16. Aspen Plus runs showed that N =13 (Aspen Plus
notation) with feed on N
feed
= 11 (optimum location) gave ethanol mole fractions of x
D
= 0.7990
and x
B
= 0.020298. These values are within the specified tolerances. The stripping section starts
with the vapor leaving the reboiler (y
in,strip
= y
reb
= 0.18709) and ends at the intersection of the two
operating lines. This last value can be determined by calculating the points on the operating lines
(x
n
, y
n+1
). For example n = 1, x
1
and y
2
are on the enriching section operating line. When the
slope changes from 0.61 in the top to 2.07 in the bottom, the intersection point has been passed.
This occurs for y
out,strip
= y
11
= 0.44631.

The mole fractions of ethanol in the liquid and vapor leaving each stage (Aspen stage notation is
used) are:

414

Stage x y Equilibrium: Calculation m
1 0.79904 0.81824
2 0.77157 0.79904 The equilibrium parameter m is the average slope of the
3 0.74561 0.78189 equilibrium curve from x (calculated at y) to x
I
.
4 0.71971 0.76569 At the reboiler y = 0.18709 for x = 0.020289, and y* =
5 0.69245 0.74957 0.44631 for x
oper
=0.10033. Then y
avg
= 0.3167.
6 0.66203 0.73263 The slope can be determined by taking the chord from
7 0.62575 0.71375 x = 0.04 (y*= 0.29209) to x = .05 (y*= 0.33018).
8 0.57882 0.69129 m = (0.33018 0.29209)/0.01 = 3.809.
9 0.51135 0.66233 The equilibrium values are from Analysis in Aspen Plus.
10 0.40050 0.62090 At y
out,strip
= 0.44631, x
oper
= 0.22553, y*= 0.55339, and
11 0.22553 0.55339 y
avg
= 0.49985. m = (0.50405 0.49482)/(0.15 0.14)
12 0.10033 0.44631 or m = 0.923.
13 0.020289 0.18709

To use Simpsons rule for the first integral in Eq. (16-22a) we also need an average m for the y
and y* values calculated at the average between y
in,strip
and y
out,stip
, which is y = 0.3167. At this y,
x
op
= 0.1659 (determined from the stripping section operating line) and y*=0.5174. The average
between y and y* = 0.41705. m = (0.42921 0.41012)/(0.09 0.08) = 1.909. The second integral
in Eq. (16-22a) is the usual estimation from Simpsons rule of the n
OG
integral,

,
,
/ ( * )
A out
A in
y
G A A A
y
H dy y y = (0.2835/6)(0.44631 0.18709)[1/(0.44631 0.18709) + 4/(0.5174 - 0.3167)
+ 1/(0.55339 0.44631)] = 0.4057 m

The first integral in Eq. (16-22a) can also be estimated from Simpsons rule,


,
,
( / ( / )) / ( * )
A out
A in
y
L A A A
y
H L V mdy y y = (0.1067/2.032){[(0.44631 0.18709)/6]
[3.8577(3.809) + 4(4.983)(1.909) + 9.3388(0.923)]} = 0.1393

Then h = .4057 + 0.1393 = 0.545 m, which is somewhat more than the 0.507 m calculated in
Example 16-1. Note that the Aspen Plus calculation does not assume CMO whereas the
calculation in Example 16-1 did, but the calculation here required an assumption of how to
calculate m. Thus, it is difficult to say which is more accurate.

16.G2. New problem in 3
rd
edition.
Enriching section y
in,enrich
= y
out,strip
= 0.44631, and y
out,enrich
= x
D
=x
1
= 0.79904. At y
in,enrich
=
y
out,strip
= 0.44631, the calculation of m is the same as done previously for y
out,strip
: x
oper
=
0.22553, y*= 0.55339, and y
avg
= 0.49985; m = (0.50405 0.49482)/(0.15 0.14) = 0.923.
At y
out,enrich
= x
D
=x
1
= 0.79904, y* = 0.81824, and y
avg
= .80864; m = (0.81180 0.80481)/0.01 =
0.699.

In Example 16-1 the enriching section integration was done with Simpsons rule in two parts.
From y
in,enrich
to y = 0.725 and from y = 0.725 to y
out,enrich
. For the calculation here we will use y =
0.71375, which is in the table of data from Aspen Plus, as a convenient break mole fraction to do
the integration in two parts. For y = 0.71375, x
oper
= 0.66203, y*= 0.73263, and to find m, y
avg
=
0.72319. m = (0.72623 0.72101)/(0.01) = 0.522.
415


The average y from y
in,enrich
= 0.44631 to y = 0.71375 is 0.58003, x
oper
= 0.44423, y* = 0.63659,
and to find m, y
avg
= 0.60831. m = (0.61020 0.6067)/(0.01) = 0.350.

For the 2
nd
integral in Eq. (16-22a) integrated from y
in,enrich
= 0.44631 to y = 0.71375 we obtain,

,
,
/ ( * )
A out
A in
y
G A A A
y
H dy y y = (0.4054/6)[.71375 - .44631][1/(.55339-.44631) + 4/(.63659 -.58003) +
1/(.73263-.71375)] = 2.404 m

Same integral integrated from y = 0.71375 to y
out,enrich
= 0.79904. The average y from y = 0.71375
to y
out,enrich
= 0.79904 is 0.75640, x
oper
= 0.73068, y* = 0.77246, and to find m, y
avg
= 0.76443. m
= (0.76587 0.75985)/(0.01) = 0.602.

=(0.4054/6)[.79904 - .71375][ 1/(.73263-.71375) + 4/(.77246 -.75640) + 1/(.81824 - .79904)] = 2.041

Total for integral 2 = 4.445 m

For first integral from y
in,enrich
= 0.44631 to y = 0.71375 we obtain,

,
,
( / ( / )) / ( * )
A out
A in
y
L A A A
y
H L V mdy y y = (0.253/0.625)[(0.71375 0.44631)/6][(0.923)(9.388) +
4(0.350)(62.2665) + (0.522)(52.966)] = 1.102

Same integral integrated from y = 0.71375 to y
out,enrich
= 0.79904,


,
,
( / ( / )) / ( * )
A out
A in
y
L A A A
y
H L V mdy y y = (0.253/0.625)[(0.79904 - 0.71375)/6][(0.522)(52.966) +
4(.602)(18.2815) + (.699)(52.0833)] = 0.870

Total for integral 1 = 1.972 m

Total height of enriching section = 4.445 + 1.972 = 6.417 m
Total height of packing = 0.545 (from 16.G1) + 6.417 = 6.962 m.
This is less than the total of 8.457 m calculated in Example 16-1. However, it does agree
reasonably well with the number of stages (11) in the column since,
11 HETP (estimated = 0.655 m in Example 16-2) = 7.208.

Both this result and Example 16-1 require calculating a small difference and then taking the
inverse of this number. This type of calculation can cause very significant errors. The graphical
calculation was based on accurate experimental measurements of the equilibrium data, and this
data is probably more accurate than the NRTL correlation used in the computer. On the other
hand, calculation errors are probably larger in the graphical than in the computer calculation.

Both calculations depend significantly on the accuracy of the mass transfer data (H
L
and H
G
),
which can easily have errors greater than 20%, which can cause even larger errors in calculation
of NTU and HTU or in HETP (see problem 16.D16) for the staged calculation.
416


16.G3. New problem in 3
rd
edition.
Part b. Equilibrium stage optimum. (L/D)
min
= 5.1, L/D = 6.12, N = 33, N
F
= 14, 1 section, y
C4,dist

= 0.008556, x
C3,bot
= 0.005706, Dia = 1.804 m on plate 32, minimum diameter = 1.565m on tray
14.

Part c. VPLUG optimum. N = 33, N
F
= 14, y
C4,dist
= 0.007953, x
C3,bot
= 0.005302, Dia = 1.812 m
on plate 32, DC backup/tray spacing = 0.391, weir loading = 0.021m
2
/s. Note that this separation
is better than the equilibrium result.

Part d. MIXED optimum. N = 49, N
F
= 22, y
C4,dist
= 0.007651, x
C3,bot
= 0.00510, Dia = 1.808 m on
plate 48, DC backup/tray spacing = 0.390, weir loading = 0.021m
2
/s. Note that this separation
require significantly more stages than both the equilibrium-stage and the plug flow results.

16.H1 and 16H.2. New problems in 3
rd
edition. Fit for the ethanol-water VLE was done in Appendix B of
Chapter 2. The spread sheet for both problems is given below with a y value chosen in the
enriching section (ignore the stripping section operating line values). Overall mass balances to
find D and B were done with Eqs. (3-3) and (3-1). L and V were determined at the total
condenser and flows in the stripping section were determined at the feed stage with the calculated
value of q. L bar = L + q F. The intersection point of the two operating lines was determined
from Eq. (4-38). y
reb
is the start of the stripping section and is the y value in equilibrium (y
eq
)
with y = x = x
B
. In the stripping section the value of 1/(y
eq
y) was determined at the start of the
stripping section (y
reb
), at the end of the stripping section (y
intersection
), and at the average of these
two values. Then Simpsons rule was used to calculate n
OG,strip
= 1.57787 from Eq. (16-24b). To
determine H
OG
from Eq. (16-27a) an average slope m of the equilibrium curve is required. The
slope of the chord from the equilibrium y at the intersection point of the two operating lines to the
reboiler vapor that is in equilibrium with x
B
,
int sec int sec
( ( ) ) / ( )
strip eq er tion reboiler er tion B
m y x y x x
= 1.95, and H
OG,strip
= 1.265. The resulting height of the stripping section, 1.998 feet, is somewhat
more than the 1.66 feet determined in Example 16-1.

In the enriching section a similar procedure was used except the integration to find n
OG,enrich
was
done in two parts. The average slope of the equilibrium curve was determined from,
1 int sec int sec
( ( )) / ( )
enrich equilibrium er tion D er tion
m y y x x x = 0.4558 with
1 D
y x . The resulting
value of H
OG,enrich
= 1.935. The resulting height of the enriching section, 29.35 feet, is somewhat
more than the 26.1 feet determined in Example 16-1 and more than the 25.35 feet determined in
problem 16.D1.

Note: in both 16.H1 and 16.H2 the average slope of the equilibrium curve m has to be calculated
with fairly large chords, not by taking the derivative of the 6
th
order polynomial fit to the VLE.
The reason is that the fit oscillates around the experimental data points and the slopes will
fluctuate greatly.

417



418
SPE 3
rd
Edition Solution Manual Chapter 17.
New Problems and new solutions are listed as new immediately after the solution number. These new
problems are: 17A2, 17A7-17A9, 17B2, 17C4,17D3, 17D8, 17D10-17D13, 17D15b-h, 17D16-
17D18, 17E1, 17H2-17H7. Chapter 17 was chapter 16 in the 2
nd
edition. Problems from that
edition have the same problem number, but the chapter number is now 17 (e.g., problem 16.D6 is
now 17.D6). Figures in the solutions to these old problems still have the designation of chapter
16.
17.A2. New problem in 3
rd
edition. Change the value of thetatot in the spreadsheet until the area matches
the desired value. This can be done as a manual trial and error or a loop can be added to the spreadsheet.
17.A7. New problem in 3
rd
edition. Part a. Increase the stirrer speed.
If no gel, increasing stirring increases mass transfer coefficient k which reduces M and hence x
w
is
reduced. This reduces x
p
increasing retention R.
Part b. Decrease the stirrer speed. This reduces k, increases M and x
w
. When x
w
> x
gel
, a gel forms and
R increases (probably to l.0) .
17.A8. New problem in 3
rd
edition. Since there is a gel the retention of the low molecular weight
compound also increases.
17.A9. New problem in 3
rd
edition. Do not invest. Osmotic pressure can often be ignored in UF because
with large molecules with high molecular weight the mole fraction is always low even if the weight
fraction is high. With low mole fraction the osmotic pressure is low. If there is a concentrated salt with a
low molecular weight the mole fraction will be high and the osmotic pressure cannot be ignored.
17.B1. Look at Suk, D.E. and Matsuura, T. (2006) Membrane-based hybrid processes: a review, Sep.
Sci. Technol. Vol. 41, pp.595626 for additional processes.
17.B.2. New problem in 3
rd
edition. One possible approach is as follows: Increase stirring to increase the
mass transfer coefficient and reduce the wall concentration to prevent gel formation. Then use a permeate
in series cascade with recycle of the retentate from the second module in series back to the feed of the
first module. The low molecular weight product is the permeate from the second module. The
intermediate molecular weight polypeptide product is the retentate from the first module.
17.D1.
10
CO2 2
cc STP cm
P 15 10
cm s cm Hg
6
m
t 1 10 m
r H
p p 12 atm 12 76.0 912.0 cm Hg
4
10
CH
P 0.48 10
P L
p p 0.2 atm 15.2 cm Hg
2 4
AB CO CH
P P 31.25
2 4
CO CH p r
1.0, p p 0.016666
a) Generate RT curve from Eq. 17-6a.
p
p P
r P P
r
AB AB P P
p
y 1 1 y 1
p y 1.5042 0.5042 y
y
1 y 31.25-30.25 y
419

RT Curve
P
y
r
y
P IN
Op. Eq., F F 0.32
0 0
0.1 0.00515
0.20 0.01114

0.30

0.01830



0.40 0.02721 Plot two arbitrary points:
0.5 0.03882
0.6 0.05504
0.7 0.08000
0.8 0.12492
0.9 0.2349
1.0 1.0 Answer (from graph)

2
2
CO
CO r r P P
ms
P
J p y p y
t
(17-2b)
2
10
CO 4 2 2
15 10 cc STP cm cc STP
J 912 cm Hg 0.0276 15.2 cm Hg 0.402 0.0002859
1 10 cm cm s cm Hg cm s
2
2
2
P
P
IN
2
CO 2
P P,CO 6 2
CO
F gmole 1 hr gmol
F 1000 0.088888
F h 3600 s s
cc STP L 1.0 gmole
J 0.0002859 1.2764 E 8 gmole s cm
cm s 1000 cc 22.4 LSTP
F y
Area 2.80 10 cm
J


P
P in
in
F
F F 0.32 kgmole/hr
F
,
out in P
F F F 1 0.32 0.68 kgmole/hr.
b.
in,1
F 1 kgmole/hr

P1 P,part a P2
1 1
F F 0.32 0.16 F
2 2
,
1 2 1
out in P in,2
F F F 1 0.16 0.84 F
Stage 1.
1 1
P in
F F 0.16 1.00 0.16 ,
in,1
y 0.15
RT curve is unchanged!
Op. Line: Slope
1 .16 0.84
5.25
0.16 0.16

Find arbitrary points to plot line:
If
out ,1 p
0.15
y 0, y 0.9375
0.16
(off graph).
P IN IN
P out
P IN P IN
1 F F y
y y
F F F F
P out IN 2
y y y 0.15 CO mole frac
1 0.32
Slope 2.125
0.32
out P
y 0, y .15 .32 0.46875
out P
y 0.10, y 2.125 .10 0.46875 0.25625
P out 2
y 0.402, y 0.0276 CO conc.
420

If
out ,1 p
y 0.04, y 5.25 0.04 0.9375 0.7275
If
out ,1 p
y 0.08, y 5.25 0.08 0.9375 0.5175
Answer (from graph):
P,1 out,1 in,2
y 0.625, y 0.0595 y
Stage 2
P2 in2
in2 P2 in2
F y 0.16 0.0595
0.1905, 0.3123
F 0.84 F F 0.1905

Slope
1 0.1905
4.2500
0.1905
. Plot curve
Answer:
2
P2 out
y 0.250, y 0.015 (see graph)
Stage 1:
2
10 3
CO 4 2
15 10 cm STP
J 912 0.0595 15.2 0.625 0.0006715
1 10 cm s


2 2
CO CO 2
1L 1 mol
J J 2.9976 E 8
1000 cc 22.4 cm s


1
P
mol 1 h
F 0.16 1000 0.04444 mol s
h 3600s


2,1
P1 2
1
CO
F 0.625
Area 92.67m
J

Stage 2:
2,2
10 3
CO 4 2
15 10 cm STP
J 912 0.015 13.2 0.250 0.0001482
1 10 cm s


2,2 2
CO CO 3 2
1 L 1 mol
J J 6.6161 E 8
1000 cm 22.4 cm s


P2
l h
F 0.1905 840 mol h 0.04445 mol s
3600 s


2,2
P2 P2 2 2
2
CO
F y
Area 1, 680, 000 cm 168 m
J

It is interesting to compare parts a and b.


2
2
2
out,CO 4
P,CO
2
Part a: 1 stage Area 280m
y 0.0276 or 97.24% CH
y 0.402
Part b: 2 stage Total Area 260.67m

out 4
P1 P2
y 0.015 or 98.5% CH
y 0.625 y 0.250

421


17.D2. a.
P in
in
P out
P in P in
1 F F
y
y y
F F F F
,
P out in 2 P in
y y y 0.2 CO , F F .3
Slope
.7
2.333
.3
, When
out P
0.2
y 0, y 0.6667
0.3

422

When
in
P out
P in
y 0.2
y 0, y 0.286
1 F F .7

RT curve is same as in Problem 17.D1.
Draw op line. From graph:
2 2
P,CO r out ,CO
y 0.53, y y 0.06
b.
2
A r r P P
CO
ms
P p y p y
J
t



10 3
CO2 4 2
15 10 cm STP cm 76 cm Hg
J 60 atm 0.06 3.3 0.53 atm
1 10 cm cm s cm Hg atm


2
3
P
CO P in 2
in
F cm STP mol
J 0.002148 , F F 0.6 .
cm s F s


3
2
3
2
0.6 mol s 0.53
1000 cm STP
A 3, 254, 000 cm
1 mol cm STP L STP
0.002148
22.4 L STP cm s

Or
2
325.4 m . Very sensitive to
P r
y & y values.
Can also calculate
2 4
P
CO CH
F
J J J . A
J

Check:
in P out
F F F ,
out out
2 0.6 F F 1.4

in in P P out out
F x F y F y , 0.4 2 0.2 0.6 0.53 1.4 0.016 .402 , OK

423


17.D3.
New problem in 3
rd
edition Since no concentration polarization
w r out
x x x ,
solv
solv r p r p
ms
K
J p p a x x
t

Solve for
solv solv
r p r p solv
solv ms ms
J K
p p a x x , 33.29, J 415.4
K t t


p
p out
out
x
R 1 x 1 R x , R 0.9804
x

and M.B.
F
p F
x 1
x , 0.22, x 0.0077
Then,
F
out
x 0.0077 0.22
x 0.0098175
0.78 1
1 0.9804
1 R
0.22


p out
x 1 R x 0.0001924
Then
r
415.4
p 1.1 15.446 0.0098175 0.0001924
33.29
13.72 atm.
17.D4. Partially new problem in 3rd edition.
424

a.
3
5
4.625 / 997000 g/m
exp ( / ) exp 1.069
6.94 10
w
solv solv
r
x
M J k
x


1
P r P
P r P r P
x Mx x
x p p Mx x a


1.069 0.054 3.6 4 1 3.6 4
3.6 4
75 2 1.069 0.054 3.6 4 59.895
E E
E
E
1
2.29
atm

4.625
73 1.0689 .054 3.6 4 59.895
solv solv
ms
r p r P
K J
t E
p p a Mx x

2
g
0.0665
m s atm

Since
A solv
solv A A solv
ms ms
K K
K K , K K and
t t


2
A
1 2
ms
g
0.0665
K g
m s atm
0.029
t 2.29 atm m s

c. Write Eq. (17-37a) for old and new situations Divide new by old. Obtain

0.75
new
new old
old
k k Everything else divides out. Since rpm ,

.75
new old
2000
k k 0.000117m/ s
1000

17.D5. a.
solv solv
ms r P
K J
t p p
3
5 2
2
1.5 10 g 1
1.47 10 g cm s atm
cm s 102 atm

RT eq.,
P out
R 1 x x , Op eq.,
P in in
P out
P in P in
1 F F x
x x
F F F F

Solve simultaneously & obtain Eq. (17-26), which with M = 1 is

in
P
1 R x
0.003 0.05
x 0.000272
1 R 1 0.997 0.45
,
P
out
x
x 0.091
1 R


P r P
P r P r P
1 x x x
x p p x x a


1
1 0.000272 0.091 0.000272
3.44 atm
0.000272 102 0.091 0.000272 59.895

b. Plot the RT curve and operating line

P P r P
r
P
x x a 1 p p 1
x
M 1 a 1 x
,
in
P r
1
x
x x
See graph. Intersection occurs at
r P
x 0.0585, x 0.000752

solv
solv r P r P
ms
K
J p p a M x x
t

425



426

5
-4
solv 2 2
1.47 10 g g
J 78 atm 59.895 atm 3 0.0585-7.52 10 0.000991
cm s atm cm s
P P
solv
F 1 x
A
J
,
P in
2 2
F F 5 kg s 3.33 kg s
3 3


4
6 2 2
4 2
3.33 kg s 1 7.52 10 1000 g kg
A 3.36 10 cm 336 m
9.91 10 g cm s

17.D6. Eq. (17-45) can be written as
solv solv g r
J / k n x n x
Plot
solvent r
J vs n x . Slope = k and intercept =
g
k n x
solv solv
cm
J /
min

r
wt frac
x
dextran


r
n x
0.052 0.012 4.423
0.037 0.03 3.507
0.026 0.06 2.813
0.0134 0.135 2.00
See Figure. Plot /
solv solv solv
J J versus ln x
r

g g
Intercept 0.0185
n x 1.159, which is x 0.314
Slope 0.01596
.

17.D7. Stirred cell data:
P
c g L ,
solv solv
2
ms
K J 69.25 g
23.1
t p 3.0 m s bar


2
/ 23.1/ (997 / ) 0.0232
solv solv solv
L
J J g L
m s bar

427


2
A solv P
J ~ J c 0.0232 0.30 0.00696 g (m s) . Also,
A c out c solv
J M c 1 R J
With
A
c c
out solv
J 0.00696
M 1, 1 R 0.03
c J 10 (0.0232)

c
R 0.97
Spiral Wound:
A sol v P c out c solv
J J c M c 1 R J
Solving for
P
c
out c
c 1.0
M 4.1660
8 0.030 c 1 R


2 solv
solv
ms
K
J p 23.1 3.5 80.8g / (m s)
t

Since osmotic pressure is ignored,
c
M does not effect solvent flux in UF.

17.D8. New problem in 3
rd
edition From 17-6b,
2
He
He
p
p He H p
r
r
P
P
y 1 1 y 1
P
y
1 y


2
10
He H He H2 10
23.7 10
P P 0.2610
90.8 10
,
p r
p p 0.2


He
r
.739 0.2 .975 1
0.021397
y 0.025 .07656
.261 .739 0.25 .279475

Check
H 2 H
2 2
H
2
2 2 H
2
p
P H He P
r
r
H He H He P
P
y 1 1 y 1
P
y
1 y
0.975 3.8314 1 .2 .025 1
.9234
3.8314 2.8314 .975


He H
2
r r
y 1 y 0.07656 OK.
Then
He out
He
F r He
P r ,He
y y
y y
0.05 0.07656
0.5152
0.025 0.07656

b. Use solution in Eqs. (17-9) to (17-10e)
p
r
p F,He
r
F,He
p
.75
a 1 .2 .261 1 2.3648
1 p .25
p y
1 .05 1
b 1 .739 .2 3 1.4874
p 1 1 1 .25 .25
c y (1 ) .261 .05 .25 0.0522


2
P
b b 4ac 1.4874 2.212359 .49377 1.4874 1.645
y
2a 2 2.3648 4.7296

Must use minus sign to have positive
p
y .
He
P
.15763
y 0.00333
4.7296

428


He
F,He
r,He P
y
.05
y y 3 .00333 0.10001
1 1 .25

c. Solve RT eq. (17-6b) for
p
y :
p p 2
r r p p p
r r
p p
y y 1 y 1 y 1 1 y
p p


p p 2
p r
r r
b
a
p p
0 1 y 1 y 1 1
p p
c
p r
y y

p
r
p
a 1 .739 .2 0.1478
p


p
r
r
p
b 1 1 y 1 .739 .2 1 .06 .739 .80786
p


r
c y .06 .261 0.01566

2
p
b b 4ac .80786 .65264
y 0.004842
2a .2956

Use + sign for positive
p
y ,
He He
He He
F r
P r
y y
0.05 0.06
0.1813
y y 0.004842 0.06

d. For Part A
3
IN
F 100m STP / h.
He,IN
y 0.05,
2 IN
H
y .95
2 He
rH r
y .9234, y 0.07656

2 2
p,H p,H
y 0.025, y 0.975

.5152
Use 17-5a written as
2
H
2
H
P P r r p p
P A
F y p y p y
tm s


3
P IN
F F 51.52 m STP h


3
3 3
P 3
1000000 cm
51.52m STP cm STP
1 h
F 14311.11
h m 3600 s s


H
2
2 2 2
3
4
P P
3
10
H r r,H p P,H
2
cm STP
14311.11 .975 1.0 10 cm
F y tm s
s
A
cm STP cm
P P y P y
90.8 10 380 .9234 76 .975 cm Hg
cm s cm Hg


2 2
A 555,186 cm 55.52 m

17.D9. a. Plot the data on a semilog plot in the form of
2
/ / ( )
solv solv solv
J J L m h

429



r
solv
g
x
J k n .
x

g r
k n x k n x . Intercept
g
k n x
From graph, slope k 18.3 and
2
L m
k 18.3 5.08
m h s

When
2 2
r solv solv solv g
x 0, J / =J 82.9 L m h 23.0g / (m s) k n x

2
g g
23.0 g m s
n x 4.53 x 92.8%
5.08 m/ s

The value of x
g
is very sensitive.

b. There is only one point further out on the n axis. Any error in point is greatly amplified in the
least squares regression. Hence, another point in this region would be most useful. The higher
the concentration, the better.

17.D10. a) New problem in 3
rd
edition Solve RT eq. and op. eq. simultaneously
2700
3.375
800
,
p
r
p
0.3 .5
a 1 2.375 1.6116
1 p 0.7 2


p
N
r
p
y 1 .5 .3 .25 1
b 1 2.375 3.8884
p 1 1 1 2 .7 .7 .7


in
c y 1 3.375 .25 .7 1.20536

2
p
3.8884 3.8884 4 1.6116 1.20536
y ,
2 1.6116
use minus sign to have y
p
between
0 and 1..
430


p
3.8884 15.1196 4 1.6116 1.205
y 0.365295
2 1.6116

From op. eq.:
IN
r p
y .3 .25
y y .365295 0.2006
1 1 .7 .7

b) Since
A
p p,A r r p p
ms
P A A

F y p y p y
t


p p,A ms
A r r p p

F y t
A
P A p y p y
. Since
IN p IN

F 1 mol s, F F 0.3 mol s

4
3
10
2 3 3
mol
0.3 0.365295 1.2 10 cm
s
A
cm STP cm 1.0 L STP
1
2700 10
L STP cm s cm Hg 10 cm STP
22.4
mol
76 cm Hg
2.0 .2006 .5 .365295 atm
atm


4 2
A 6.569 10 cm

c)
p p,A ms
p IN
A A r r p 2

F y t

A , F F 0.4 mol s
P p y p y

4
3
10
2 3 3
mol
0.4 0.325 1.2 10 cm
s
A
1
cm STP cm
1.0 L STP 76 cm Hg
22.4L STP
2700 10 2.0 0.175 0.5 0.325 atm
cm s cm Hg mol 10 cm STP atm


4 2
A 9.0823 10 cm
17.D11. New problem in 3
rd
edition.
0 IN
p out
x 0.001
x 0 since R 1.0, x 0.0016667
1 .6

Gelling occurs at a solvent flux of
2
5200 L/(m )
solv
J day which is
2 2
5200 997 60.0
86400
solv solv solv
L g day g
J J
m day L s m s

Then
2 3
gel out solv solv 5
60.00g / (m s) 997000g / m
x x exp (J / ) k 0.0016667 exp 0.01334
2.89 10 m / s


17.D12. New problem in 3
rd
edition
p r
p / p 1.0 4.5 .
He
He H2
H2
P
0.261.
P
Use Eq. (17-6b),

431


2
p,He He H p
r
r ,He
p
p
y 1 1 y 1
p
y
1 y
1
0.1 .261 1 1 .1 1
4.5
0.25446
.261 .261 1 .1

Eq. (17-7c)
He HE
He He
F r
p r
y y
.2 .254
0.353
y y .1 .254


17.D13. New problem in 3
rd
edition
in
.035 NaCl x , cut .55, perfectly mixed
Rejection
n
p
1 R x
x
1 R
(17-27)
a) If
p
x 0.00050 & 0.55, what value R required. Find R (including concentration
polarization effect). From Eq (17-27),
p p IN IN
x x R x Rx which gives

IN p IN p
R x x x x
IN p
IN p
x x
0.035 0.0005
R 0.9935
x x 0.035 .55 .0005

b) If
p
x 0.0016, 0.55,
IN p
IN p
x x
0.035 0.001
R 0.9869
x x 0.035 .55 .001

c) If R (inherent rejection coefficient with M = 1) for part b is R 0.992, what was value of
M that gave R 0.9869

CaseB
CaseB CaseA
CaseA
M
R 1 1 R .
M
Let A be highly stirred
A
A
M 1
R R
.

CaseA CaseB
CaseB
CaseA
M 1 R 1.0 1 0.9869
M 1.6375
1 R 1 0.992


17.D14. RT curve:
wB w w
w
wB w w
x 43 x
y
1 1 x 1 42 x

wB
43(mole frac). Since
w,IN
x 0.10, only need RT curve below 0.10. Create table and plot
w
x
w
y
0.10 0.8269
0.08 0.78299
0.05 0.6935
0.03 0.571
0.01 0.3028
0.0025 0.0973
0.001 0.0413


B
cal kcal 74.12 g
141.6 10.5 kcal mol
g 1000 cal mol

Feed 0.9 10.5 0.1 9.72 10.42 kcal mol
432


B
P
1
C 0.625 74.12 0.046
1000
,
W
P
1
C 1.0 18.016 0.018
1000


P,in
C 0.9 0.046 0.1 0.018 0.0435 kcal mol C

, ,
0.1(18.016) 0.9(74.12) 68.51
feed W F W B F B
MW x MW x MW
a) Assume
P
y 0.5 to calculate
p
0.5 10.5 0.5 9.72 10.11
and
PL,in
in out in out
P
C
T T where T T 30 then,
0.0435
30 0.129.
10.11
This is a
molar ratio. Slope Op line
1
6.75 , and op line goes through point (mole fractions)
in
out P
x 0.10
x 0, y 0.775.
0.129
Plot operating line. From graph,
P out
y 0.57, x 0.031
(mole fraction water). This value of y
p
is reasonably close to our assumption.


/ ( ) / ( ) ( / )
p F p p F F p F
F F F MW F MW MW MW
433


, , p p W W p B B
MW y MW y MW = 0.57(18.016) + 0.43 (74.12) = 42.13
.129(42.13/ 68.51) 0.0791 in (lb/h)/(lb/h).

2 2
Area Permeate Rate Flux ( 0.0791 100 lb h) 0.2lb h ft 39.53 ft
b) Cut
in,w
0.08 x 0.10
Slope
1 cut 0.92
11.5.
cut 0.08
Find
P
y 0.68 from graph.
Then
P
0.32 10.5 0.68 9.72 9.97

in out P o
PL,in
0.08
T T 30 9.97 kcal mol 48.3 C
C 0.0435kcal / (mol C)


c)
out P
x 0.05, x 0.6935 (From RT table or graph).

in out
P in
P out
x x 0.10 0.05
F F 0.0777
y x 0.6935 0.05


P
0.3065 10.5 0.6935 9.72 9.959
434


in out
PL,in
0.0777
T T 30 C 9.959 47.8 C
C 0.0435

17.D15. Parts b to h are new in 3
rd
edition RT curve:
P r
x 1 R M x 0 .
Mass balance perfectly mixed
in r,in P P out r,out
F x F x F x
Since
in
P r,out r,in
out
F
x 0, x x
F


out in P P
in in in
F F F F
1 0.8
F F F
. Then
in
out
F 1
F 0.8


r,out
1
x 0.10 0.125
0.8
,
out in
F 0.8F 80 kg h
Alternate graphical solution gives same result.


b. Area = F
p
/ J
solv
= (20kg/h)(1 L)/0.997 kg)(24h/day)/ (2500 L/m
2
day) = 0.193 m
2
.

c. Gel formation occurs when x
w
= 0.5 and x
w
= M x
out
= 0.125 M. M = 0.5/0.125 = 4.0

d. Gel formation occurs when x
w
= 0.5 and x
w
= M x
out
= M x
F
/ (1 ) = 1.2 (0.1)/(1 )
Then 1 = 0.12/0.5 = 0.24 and = 0.76.

e. Gel formation occurs when x
w
= 0.5 and x
w
= M x
out
= 1.2 x
F
/ (1 ) = 1.2 x
F
/ (1 0.2).
Obtain x
F
= 0.333.
x
r,out
= x
F
/ (1 ) = 0.3333/0.8 = 0.416

f. We have x
r,out
= x
F
/(1 ) = 0.20/0.75 = 0.26667. M = x
gel
/ x
r,out
= 0.5/0.26667 = 1.875.
First occurs when J
solv
= 2500 = k ln (M). Obtain k = 3977 L/(m
2
day) = 4.60310
-5
m/s.

g. M = 1.875 and k = 3977. Since we change the pressures, J changes which will change M. However
with constant stirring k is constant. First, assume no gel and calculate J and M.

r p
solv solv
ms
p p
J K
t
,
2
solv solv
2
ms r p
K J 2500L (m kg) L
2083.33
t p p 2.2 1.0 bar m day bar



Then, without gel,
2
solv
J 2083.33 3.4 1.0 5000 L m day

From Eq. (17-34) M = exp (J
solv
/ k) = 3.516.

6.5
r,in
x 0.10
0.125
P
x
P r
x x

RT curve
- 4
Op. line
P in r ,in
P r ,out
P in P in
1 F F x
x x
F F F F

0.8
Slope 4
0.2

r,in
r,out P
P in
x
0.10
When x 0, x 0.5
F F 0.2

r r,out
x x
435

0.2. Then,
F
w
3.516 0.2
Mx
x 0.878,
1 .8
gel forms.
With a gel, previous work is incorrect. Set R = 1.0, x
p
= 0,
F
r
0.2
x
x 0.25,
1 .8

And from Eq. (17-45), J
solv
= k ln (x
gel
/x
r
) = 3977 ln (0.5/0.25) = 2756.6 L/(m
2
day)
Note: The same answer is obtained in parts g and h if convert to J
solv
and use k in m/s.
h. Gel forms since it did previously,
F r
0.2
k 3977, 0.26, x 0.2, x 0.27027
.74


gel 2
solv 2
r r
x
.5 L 0.5
J k n k n 3977 n 2446.6 L (m d ay)
x x m d ay 0.27027

17.D16. a) New problem in 3
rd
edition
0
B B p,B r,B
Case B, R 0.976, M 1.0, p 1.1, p 12.06

C p,C r,C
Case C, R .939, p 1.1, p 15.2
For Experiment C.
r p
C C
C B
B r p
B
p p
1 R 1.0 .061 14.1
M M 3.27
1 R .024 10.96 p p

b)
IN
out
C
x 0.0093 0.26
x 0.01230
1 .74
1 R 1 .939
.26


p C out
x 1 R x 0.0007505

solv
solv r p C out p
ms
K
J p p a M x x
t

1.387 152 1.1 15.446[ 3.27 .01230 0.0007505]
2
18.71 g m s


17.D17. New Problem in 3
rd
edition.
a) log a log P b , P 0.0001, 42, P 10000, 1.9
log 42 a log 0.0001 b 1.6232 4a b
log 1.9 a log 10, 000 b 0.27875 4a b
1.9020 2b
b 0.95100 , a .16805
Equation: log 0.16805 logP 0.951

b) If 8, log 0.90309

log .951
log P
0.16805 0.16805


2
O
log P 0.28509 P 1.93 Barrers


436

17.D.18. New problem in 3
rd
edition. Ideal Gas: Vol% = Mole %
2
10
N2 CO He He
P 250, P 2700, P 300, P 550 10

3
P IN
F F 0.4 m s.

3
2
3 6 3 6 3
IN P 3
10 cm
F 1.0 m s 10 cm s , F .4 10 cm s
m

Part A
r p
76 cm Hg
p 2.5 atm 190 cm Hg p 76 cm Hg
atm

Part B
r p
p 76 cm Hg p 0.4 76 30.4 cm Hg

ms
.00254 cm
t 0.8 mil 0.002032 cm
mil

Eq. (17-11d)
i ms r
m,i
P i ms p
P t p
K
F A P t p
Need to guess value of
P
F A or of
P
y ,
Since
2
CO has highest permeability,
2
CO will be concentrated; thus, guess
2 2,IN
p,CO CO
y y 0.40
Then
2
2 2
IN,CO
r,CO p,CO ,guess
y
y y
1 1
where 0.4 and
2
IN,CO
y 0.4.
Then,
2 2 2 2
P guess CO ms p,CO r r.CO p p,CO
F A P t y p y p y
Use
P
F A in Eq. (17-11d) to calculate all
mi
K
Then check
IN,i
r,i
mi
y
y 1.0?
1 K 1

Put in Spread Sheet. Can use Goal Seek to force
r,i
y 1.0 as change
2
P,CO
y .

Results: a.
2 2 2
P,N P,CO P,He P,H
y .15037, y .54446, y .3351, y .27154, .999885

2 2 2
r,N r,CO r,He r,H
y .3164, y .3037, y .06099, y .3189 1.0000766
b. Same answers for mole fractions since
r p
p p is same.


306 2008 HW 8 Problem 2


yF N2 0.25 yF CO2 0.4
yF
He 0.05 yF H2 0.3
F,cm3/s 1000000 tm, cm 0.002032 theta 0.4

pr, cm Hg 190 pp, cm Hg 76

P N2 0.000000025 P CO2 0.00000027

P He 0.00000003 P H2 0.000000055

F p 400000


change yp to get sum=1

yp CO2 0.544455884 yr CO2 0.303696077

Fp/A 0.003983763 A, cm 2 100407575.7


A, m 2 10040.75757

K N2 0.475237299 y r N2 0.316417678

K CO2 1.792765611 yr CO2 0.303696077

437

K He 0.549397235 yr He 0.060993544

K H2 0.851323363 yr H2 0.318969314


Goal seek Sum 1.000076613

yp N2 0.150373483

yp CO2 0.544455884

yp He 0.033509684

yp H2 0.271546029

sum 0.99988508


17.E1. New problem in 3
rd
edition For dilute systems
solution solvent total
J J ,
IN p out
F F F , Basis:
IN
F 1.0
Transfer Eq. (17-7c)
p
F out
IN p out
F
x x 0.022 0.056
0.6106
F x x 0.00032 0.056


p
out
x
0.00032
R 1 1 0.9943
x 0.056
,
s u c rose solvent p solvent solution p
Permeate
J J x J x

permate p
0.997 0.4 x 0.997 0.4 0.00032 0.99713 kg L

Initial assumption is OK.

2 2
/ (3.923 / ) / (997 / ) 0.003935 / ( )
solv solv solv
J J g m s g L L m s

2 2
sucrose
J (3.923g / m s)(0.00032) 0.00126g / (m s)
b) Eq. (17-27)
1
p r p
water
water solute
sucrose
p r p r p
x Mx x
K
K
x p p Mx x a

p r solv r p
x 0.00032, x 0.056, 0.997, p 60 atm, p 1.1 atm, M 1, a 59.896

w s
1 .00032 0.056 0.00032
g water
3.131
g sucrose 0.00032 60.0 1.1 0.056 0.00032 59.895

From Eq. (17-16c),
water solv
ms r p r p
K J
t p p a Mx x


2
3.923
0.0706 g m s atm
60 1.1 59.895 1.0 .056 0.00032
water
ms
K
t

Eq. (17-18)
2
0.00126 g sucrose
0.0226
0.056 0.00032 m s wt frac x
sucrose sucrose
ms r p
K J
t M x

c. Solution 1. R 1 M 1 R 1 2.1 1 0.9943 0.98803
Then RT equation is
p out out
x 1 R x 0.01197 x
Operating Equation is (17-23)
F
p out out
x 1 .39 0.022
x x x
.61 0.61


p out
x 0.6393 x 0.036066
Solve RT & operating equation simultaneously.
out
x 0.05537 ,
p
x 0.000663
438

Check
p
out
x
R 1 0.98803
x
0K
x
water
water r p r p
ms
K
J p p a M x
t

=
2
0.0706 60 1.1 59.895[ 2.1 0.05537 0.000663] 3.67g/(m s)
/ (1 )
sucrose water p p
J J x x


2
3.67 0.000663 / (1 .000663) 0.00243 g (m s)
sucrose
J
Alternatively,
sucrose
sucrose r p 2
ms
K g
J Mx x 0.0226[ 2.1 (0.05537) 0.000663] 0.00261
t m s

6.9% different
Solution 2. RT Eq. (17-21),
p p
r
p
x x 3.140 59.895 0.997 1 3.140 .997 60 1.1 1
x
2.1 1 3.140 59.895 .997 1 x

Simplifies to,
p p
r
p
x 186.5 x 185.391
x
2.1 391.66 x

Linearize
r p
x @x 0.0003 0.02509 . Note x
p
= 0.0003 is an arbitrary point.
Slope =
p
r
x
0.0003
0.01196
x 0.02509

Then linear form of RT equation is
p r
x 0.01196 x or
p out
x 0.01196 x
Solve simultaneouly with Operating Equation
F
p out out
x 1
x x 0.6393 x 0.036066

out
x 0.05538 ,
p
x 0.0006623. Very close to value obtained with retention analysis.

17.E2. (was 16.D11 in 2
nd
ed.) Eq. (17-59b):
PL,in
P
in out
in P
C
F
T T
F
,
in out
T T 85 25 60
Stage 1. Assume
P
y ~ .05 water, 0.95 ethanol

P w E
0.95 0.05 2290.3 kJ kg (See Example 17-9)
For Feed
PL,in PL,w PL,E
C 0.1 C 55 C 0.9 C

kJ kJ
0.1 4.1915 0.9 2.7595 2.903
kg K kg K

Where average temperature from 25 to 85 is 55C and
P
C values are from Perrys 7
th
, pp. 2-
306 and pp. 2-237.


P1
P1
in
F 2.903
60 0.0760 and F 0.0760 100 7.60 kg hr
F 2290.3
.
439

Op. line intersects
P out IN
y x x 0.10 (water wt. frac.)

1 1 0.760
Slope 12.16
0.0760

Op. Eq. is,
P out out
0.1
y 12.16 x 12.16 x 1.32
0.0760

If
P out
y 1, x 0.32 12.16 0.0263
Plot Op. Line on Figure 16-17a and find intersection:

P1 out1 2,in
y 0.66, x x 0.055 (water values)

out1 in2
F F 100 7.60 92.40 kg h


Stage 1 Trial 2. Since y
p
y
p, assumed
, do a second trial.

P
y 0.66 water, 0.34 E,
P w E
0.66 0.34 0.66 2359 0.34 985 1892 kJ kg

P1
1in
2.903 60
F
0.0921
F 1892
1
P1 out
F 0.0921 100 9.21 kg h, F 100 9.21 91.79 kg h
Slope
1 0.0921
9.86
0.0921
,
P out out
0.1
y 9.86 x 9.86 x 1.086
0.0921

If
P out
0.086
y 1, x 0.00872
9.86

Plot operating line and determine (from graph)
1 1 2
P out in
y 0.64, x x 0.05 water. This
value of y
p
is close to the assumed value of 0.66. Can proceed to stage 2.

Stage 2: Estimate
P
y 0.50 (water),
P w E
0.50 0.50 1672
For
in2 PL,in
x 0.050, C 0.05 4.1915 0.95 2.903 2.967

2
2
2
P
2 in
in
F
2.967
60 0.1065, F .1065 90.66 9.64 kg h
F 1672


2
out
F 90.66 9.64 8.10 kg hr , Slope
1 0.1065
8.40
0.1065


P out out
0.050
y 8.40 x 8.40 x 0.4421
0.1065

If
out P
x 0, y 0.4421 . Draw op. line. Intersection gives
P
y 0.34
For
2
out
x use MB.
2 2 2 2
2
in in P P
out 2,
out
F x F y 90.66 0.050 9.64 0.34
x
F 81.00


2 2
out ,w out, ,ETOH
x 0.0155 or x 0.9845.
This is a close as we can get graphically.
Mixed Permeate:
p,mix
9.21 0.64 9.64 0.34
y 0.487
9.21 9.64
wt frac water
440


P
2
kg 1000 g
F
h kg
Area
J g h m
, J from Fig. 16-17b based on
out
x
Stage 1
2
J 0.8333 g h m ,
2
1
9.34 1000 g h
A 11, 208 m
0.8333

Stage 2
2
J 0.208 g h m ,
2
2
9.64 1000
A 46, 346 m
0.208

Other flow patterns will reduce area. Area is large because of low flux caused by low ethanol
permeation rate.

17.F1. RT eqn.,
x x
y
1 1 x
,
Benz
16.6 .3
x 0.3, 16.6, y 0.87676
1 15.6 .3


Benz
18.3 .2
x 0.2, 18.3, y 0.8266
1 17.3 .2


Benz
6.66 .1
x 0.1, 6.66, y 0.4253
1 5.66 .1
. Plot RT equation.
Operating equation Slope
1 .9
9
.1
. Plot on graph. Find
Benz
P out ,Benz
y ~ 0.844, x ~ 0.238
441




benz benz
P P benz P iP
y 1 y 0.844 94.27 1 0.844 164 105.15 cal g

benxw Lbenz L,iP
PL,in P P benz,in P
C x C 1 x C 0.3 0.423 .7 0.73 0.6379 cal g C
442


P
in out
PL,in

T T
C
0.1 105.15
50 C 66.48 C
0.6379


17.H1. (was 16.G1 in 2
nd
edition)
This is set up for Area being the unknown and cut being known. Problem 17.H1
Fr,in 10000.000000 yin,A 0.2500 cut=Fp/Fin 0.2500
tmem,cm 0.002540 pr,cm Hg 300.0000 pp,cm Hg 30.0000
yin,B 0.5500 P,A 0.0000000200
Fptot 2500.000000 yin,C 0.2000 P,B 0.0000000050
Fr,out 7500.000000 P,C 0.0000000025

Guess values of A or equivalently Fp/A until sum y,r and sum u,p are = 1.00
Fp/A 0.0007059 (this is final result)
KA 2.507328 KB 0.7720 KC 0.4015
sum x eq
y,r,A 0.181576 y,p,A 0.455271198
y,r,B 0.583244 y,p,B 0.450268647
y,r,C 0.235190 y,p,C 0.094429331 Area, cm2 3541578.1
sum y,r 1.000010 sum y,p 9.999692E-01

These results agree very well with Geankoplis results.

17.H.2. New problem in 3
rd
edition Part a)

2
p r,out
y 0.5243, y 0.0610, A 3, 200,152 cm
b)
2
p,avg r,out
y 0.6193, y 0.0203, A 2, 636,196 cm

17.H3. New problem in 3
rd
edition

Counter current. Shows final guess for theta.
Fin,
cm3/s 100000 yin 0.209 thetatot 0.714 PB/tms 0.00175
PA/tms 0.003905 pr, cm Hg 114 pp, cm Hg 76
M 15 N 100 yroutguess 0.2 erroracc 0.0000001
df 0.9

j=N-i+1 100 99 98 97 96 95 94
Fr 28600 29314 30028 30742 31456 32170 32884
yp 0.173174301 0.1738615 0.174547 0.1752323 0.175916 0.1765981 0.17727918
yr 0.144051968 0.1447613 0.14547 0.1461768 0.146883 0.147588 0.14829195
Area 9710.750234 9664.4075 9618.879 9574.1438 9530.183 9486.978 9444.50995
Fp 714 1428 2142 2856 3570 4284 4998
Fp/Fr,j-1 0.024356963 0.0475556 0.069677 0.0907935 0.110973 0.1302761 0.14875885
yp 0.235015561
Areatot 824015.8215
yincalc 0.208999973
Fincalc 100000
Massbal 9.09495E-13
yrout 0.144051968

443




17.H4. New problem in 3
rd
edition The spread sheet equations are shown below for part b. Part a agreed
with problem 17.D14. Part b answers: y
p,W
= 0.412, = 0.2122, x
out,W
= 0.0160, = 0.158,
Area = 79.0 ft
2
. Note that if the starting guess for y
p,W
is too high, Goal Seek will converge
on an answer with y
p,W
> 1, which is obviously not physically possible.



17.H5. This problem is very similar to Example 17.7. It is easiest to solve on a spreadsheet, which is
shown below. The results are shown in the spreadsheet.
New problem in 3
rd
edition

444



445

17.H6. New problem in 3
rd
edition The spreadsheet is similar to that for problem 17.H5 and is shown
below,

17.H.7. The same spread sheet that was used in problems 17.H5 is used.
446




445
SPE 3
rd
Edition Solution Manual Chapter 18.
New Problems and new solutions are listed as new immediately after the solution number. These new
problems are: 18.A3, 18.A16, 18.B4, 18.C4, 18.C14, 18.D3, 18.D8, 18D9, 18D14, 18D15,
18D18, 18D21, 18D24, 18D25, 18.D29, 18.D30, 18.F1, 18.H1-18.H2. Chapter 18 was chapter
17 in the 2
nd
edition. Most problems from that edition have the same problem number, but the
chapter number is now 18 (e.g., problem 17.D6 is now 18.D6).
18.A1. 1c; 2 b; 3a
18.A.2. 1c; 2a; 3b
18.A.3. New problem in 3
rd
edition. One barrier is lack of knowledge. Most chemical engineers are not
trained in use of adsorption, chromatography, and ion exchange. Thus, they do not think of these
processes as a potential solution. A second barrier is the simulation tools are not as developed and widely
available as the simulation tools for equilibrium staged separations such as distillation.
18.A4. Regeneration is too difficult.
18.A5. In the SMB the mass transfer zone between the two solutes stays inside the cascade. In a
chromatograph the MTZ exits the column and must either be completely separated,
which requires a significant amount of desorbent, or recycled appropriately.
18.A7. d
18.A.8. New problem in 3
rd
edition. The LUB approach assumes constant pattern behavior. Linear
systems do not have constant pattern behavior.
18.A9. d
18.A10. e
18.B.4. New problem in 3
rd
edition. There are obviously many possibilities. One is to develop sorption
processes that use an energy separation processes (e.g., pressure or temperature) to produce purge or
desorbent from the feed so that a separate purge or desorbent does not have to be added.
18.C1.
T e e P1 e cry P2
1 1 f
available e e P4 cry e cry P2 di col.
V 1 1 f 1 f K V
P cry clay P1 cry cry P2 P1 cry f cry f P2
1 f 1 f 1 1 f f
B e P e f
1 (same as 18-3b)
18.C4. New problem in 3
rd
edition.
Amount in mobile phase =
e
(Vol. Col. Segment)
e c T
c z a x c
Amount in pores = 0 (no pores)
Amount exchanged
c RT DE
z A c y K
No
e
1 term because
RT
c is equivalent/L
Obtain,
e c T int er
ion
e c T c RT DE
z A x c v
u
z A c x z A c y K
Simplify to,
int er
ion
RT
DE
e T
v
u
c y
1 K
c x
(18-44)
446

18.C7.
A A
1 2
AF
eff A
inter
C z u t 1
1 erf
C 2
4E t u
v

Sketch of break through:


At 5% point,
A st
1 2
st A
L u t
1.164
t u
4E
v
and at 95%,
A final
1 2
A
final
L u t
1.164
u
4E t
v

Or
1 2
1 2 A
st A st
u
2.328 E t L u t
v

let
1 2
1 2 2 1 2 A
1 st A 1 1 1 2
u
x t , u x 2.328 E x L 0.
v


t
1 2 1 2
2 A
A
A 1 2
1
A
E u
E u
2.328 2.328 4Lu
v v
x
2u

Let
1 2
2 fin
x t , then
1 2 1 2 2
A A
A 1 2
2
A
E u 2.328 E u
2.328 4L u
v v
x
2u


By definition,
2 2
MTZ final st 2 1
t t t x x
Use sign for both (has to be to have positive times).
final
t

erf (a) .9 a 1.164
95%
w
t
erf (a) 0.90 a 1.164
st
t
5%
1
t
447

2 2 2
1 2 1 2 1 2 1 2
A A A 2 2 A A
A 2 A 1 2 1 2
2.328 E u 4L u 2.328 E u 2.328 E u 2.328 E u
4u x 2 4L u
v v v v

2 2 2
1 2 1 2 1 2 1 2
A A 2 2 A A
A 1 A A 1 2 1 2
2.328 E u 2.328 E u 2.328E u 2.328E u
4u x 2 4L u 4L u
v v v v


2
1 2 1 2
A A
A 1 2
2 2
MTZ 2 1 2
A
2.328 E u 2.328E u
4 4L u
v v
t x x
4u

If
2
2.328 E
4L very reasonable since E is usually small ,
v


1 2
MTZ 1 2
A
2 2.328 E
t L
v u



18C9. New problem in 3
rd
edition. In Out = Accumulation
For Figure 18-7B,
inter c T,after i,after
In t v A C x

inter c T,before i,before
Out t v A C x

c i,after i,before RT c i,after T,after i,before T,before
Accumulation LA y y C LA x C x C
Note that
RT
C is constant. After dividing both sides by
c
t A , mass balance is

int er i ,after T,after i , before T, before
RT i ,after i , before i ,after T,after i , before T, before
v x C x C
L L
C y y x C x C
t t

For Figure 18-7B with a total ion wave,

total ion int er
L
u v
t

The first and third terms in the mass balance cancel each other. Thus,

RT i,after i,before
L
C y y 0
t

Which requires,
i,after i,before
y y


18.C10.
448



A
B 2 2B port A 2 2B port
B
C
C v M u C v M u
C
(1)

A 3 3A port A 2 F 3A port
C v M u C v v M u (2)
Subtract eq. (2) from (1),
A 2B
A F 3A port
B
C M
C v M u
C


F
port
3A 2B
B A
v
u
M M
C C

Then
D 4 1
v v v where
4 B4 B 4B port B
v u C M u C

1 A1 A 1A port A
v u C M u C
Thus
4B port 1A port
4B 1A f
D
3A 2B
B A B A
B A
M u M u
M M v
v
M M
C C C C
C C


D 3A 4B 1A 4B
B A B A F
v D M M M M
C C C C F v

If all
i
B A B A
D 1 1 1 1
M 1.0, 1.0
C C C C F




18.D1. Rearrange:
A
A
A MAX MAX A
p 1 1
p
q q q K
. Plot
A A A
p q vs. p

1
L
1 2 A,prod A,i A,i i A 1A port
2 3 F B B,i i B2 2B port
3 4 B,prod A3 3A port
4 1 D B4 4B port
v v v u C L u M u
v v v u C V u M u
v v v u M u
v v v u M u

A
F
B
1
2
3
4
449

296 K 480 K
p/q p p/q p
135.863 275.788 1786.943 637.7598
278.679 1137.645 1709.129 1296.2036
478.666 2413.145 1974.657 2378.6716
696.073 3757.6116 2309.538 3709.3486
939.619 5239.9722 2778.150 5329.6030
1116.143 6274.1772 3011.134 6246.5981
1189.735 6687.8589 3122.979 6687.8589


At 296 K At 480 K
Intercept
max A
1
80
q K
Intercept
max A
1
1380
q K

Slope
max
max
1
0.163636 , q 6.1125
q

max
max
1
Slope 0.260606 , q 3.8372
q

max
A
max A
1 q
0.163636
K 0.00204545
1 q K 80

max
A
max A
1 q 0.260606
K 0.00018884
1 q K 1380

450

18.D2. a = 22 liter soln/kg ads =
L soln 1 kg L
22 0.022
kg ads 1000 g g ads

b = 375 liter soln/g mole anthracene = 375
L 1 mol L
2.104
mol 178.22 g g ads


c
max A,c A
A A
q K C
q
1 K C
, thus,
A,C
L
K b 2.104
g anth


max
A,C
a 0.022 g anth
q 0.10456
K 2.104 g ads.


18.D.3. New problem in 3
rd
edition.
Part a
ij
inter
j
e p
e
p d s
e e
v
u
1 1
1
1 K K
,
inter e super
v v 40

j
j
40
u
1 0.69 1782K
,
AN
40
u 5.671 cm min
1 0.69 1782 0.00301


Time AN = L/u
AN
= 25/5.671 = 4.408 min
b.
DN S,DN
40
u 5.474, time L u 4.576
1 0.69 1782 0.00316

c.
AN DN
u u u 2 5.567
From (18-83)
1 2
S,AN S,AN
4Ru
N 104.33 N 10885
u u

HETP L N 0.002297 cm
From (18-81),
2 2
width at half height 5.54 peak max N
d. To find width in time units, peak max is in time units = retention
S,AN
time L u 4.40864 min , width 0.09946 min

t
0.425 width 1/2 height 0.042271 min

18.D4.
super
int er
e
v
10.0 cm
v 23.26
0.43 min

a)
int er
s
e p e p d
s
x
e e f
v
u
1 1 1 K
1 K T
(18-15c)


s
23.26
u 0 0.6027 cm min
0.57 .48 1.0 0.57 0.52
2100
1 17.46
0.43 0.43 684


br
t 200 cm 0.6027cm min 331.8 min
451
b) Assume wall heat capacity is small: Assume wall heat capacity is small.
s
f
int er
th
e p s p
e
p
e e F p
v
u
1 1 C
1
1
C
th
23.26 cm
u 5.911
.57 .52 2000 min 2100
1.636
.43 684 1841
th,br
t 200 cm 5.911 cm min 33.84 min
c)
x
K 1.23 g g @80 C,
s
23.26
u 80 5.4868 cm min
1.636 2.603

br
t 200 cm 5.4868 36.45 min , see figure.
2 3
in c
cm 0.0011 g tol
M.B. 10.0 A cm 331.8 min 0.684g cm
min g fluid
c conc
out 10.0 A 0.684 33.84 0.0011 36.45 33.84 C
Simplifying:
conc
331.8 33.84 .0011 2.611 C
conc
297.96 .0011
C 0.1255 wt frac.
2.611

Alternate: Eq. (18-24)
th s
th s
1 1
1 1
u C 80 u 0
0.6027 5.911
113.92
1 1 1 1
C 0
u 5.4868 5.911 u 80
C 80 0.0011 113.92 0.1253 wt frac. A very considerable amount of concentration occurs.
452


18.D5.
super int er super e
v 20 cm min v v 20 0.4 50 cm min
For step input w. unfavorable isotherm, get a diffuse wave.
Langmuir formula:
int er
s
e p
e
d p s 2
e e
v
u
1 1
1 a
1 K
1 bc

But now b 0.46 0

s
2
2
50 50
u
0.93067 .6 .46 1.124
.6 kg 1.2
1.81
1 1.01 .54
1 0.46 c .4 .4 liter
1 0.46 c


c,g/l u
s
, cm/min t
out
= L/u
s
, min
0 18.2437 2.741 min
0.25 16.676 2.998
0.50 14.794 3.3797
0.75 12.565 3.979
1.00 9.997 5.002
1.25 7.1813 6.9625
1.50 4.3499 11.4944


0.0011
z
0.0011
80, C = 0


C = 0
u
sol
(80C)
36.45 min
C
conc

C
out

36.45
0.0011
u
th

min
331.8
33.84
0.0011
33.84
0
time, min
0.1255
t
453



18.D6. a)
f
f s w
f p int er
th
e p
e
p f p p s p
e e e c
C v
u
1 1
1 W
1 C C C
A

If wall effects are negligible,

th
0.684 2240 30
u 12.61 cm min
.57 .57
1 .5 .684 2240 .5 920 1.80
.43 .43


b)
thermal,br
t 50 cm 12.61 cm min 3.965 min

int er
s
e p
e
p d xy
e e
v 30
u 300K 3.0964 cm min
.57 .57
1 1
1
1 .5 1.0 .5 12.109
K K
.43 .43

s xy
u 350K 6.5298 K 4.423 in same eqn.
c)
br
t 300K 50 3.0964 min. Exits at
F
c 0.010 .
454
At t = 20, start hot,
br,hot
t 50 12.61 20 23.965 min
Feed is concentrated.
s th s th
C 350
1 1 1 1 1 1 1 1
3.2989
u 300 u u 350 u 3.0964 12.61 6.5298 12.61 C 300
C 350 0.010 3.2989 0.032989 g L. This continues until breakthrough at
s
t 20 L u 350 K 20 50 6.5298 27.6572 minutes
18.D7.
int er super e
v v 15 0.434 34.56 cm min
a) At 4C:
int er
s
e p
e
p d s
e e
v
u
1 1
1
1 K K 4
(A)
s
34.56
u 4 C 0.3715 cm min
0.566 0.57 1.0 0.566 0.43
1 1820 0.08943
0.434 0.434
r s
t L u 60 cm 0.3715 cm min 161.49 min . Concentration out is zero from t = 0 to t =
161.49 minutes. Then exits at
F
C 0.01.
18.D6.
27.6572
23.965 16.1478
0
t
g/L
0.010
0.032989
455

b)
s
f f
int er
th
e p p s e p pw
e e p f e c p f
v
u
1 1 C 1 WC
1.
C A C


th
34.56
u 17.293 cm min
0.566 0.43
0.25 1820
1.743
0.434 1.00 1000
,

br,th th
t L u 60.0 17.293 3.4696 min +1200

Eq. A but with K(60)

s
34.56
u 60 C 0.720258 cm min
0.566 0.43
1.743 1820 0.045305
0.434


br,conc 60 s
t L u 60 60.0 0.720258 83.3035 min +1200


high F
th th s s
1 1 1 1
c 60 c
u u u 4 u 60


3
high
c 0.01 2.6918 0.05783 1.38839 0.05783 0.019796 kmol m

18.D.8. New problem in 3
rd
edition. Example 18-3:
inter,F
v 18.60 and
inter,purge
y 25.58 cm min.

s inter,F,
u v 0 C 0.3799 cm min ,
s inter,purge,
25.58
u v 0 C 0.3799 0.5225 cm min
18.60


th inter,purge,
u v 6.466 cm min ,
s inter,purge,
u v 80 C 4.343 cm min

s inter,F,
18.60
u v 80 C 4.343 3.158 cm min
25.58

If
purge
t hot purge time and
F
t is cold feed time, with

purge hot wave breakthrough
t t
inter,purge
v 18.56

min (from Example 18-3) then breakthrough
equation is
s Inter,F, F s Inter,purge, thermal,breakthro ugh
u v 0 C t u v 0 C t 120 cm
u
th

C = 0
83.3
C
F

3.4696
C = 0
0
Elution time:
C
high

60
0
z
60
60
C = 0
60
456


F
120 0.5225 18.56
t 290.35
0.3799
min
The next feed input at 290.35 + 18.56 = 308.91 min. This starts a cold thermal wave at
Inter,F th Inter,F
v , u v 4.701 cm min which breaks through in another 25.53 min for total time
to cold breakthrough of 308.91 + 25.53 = 334.44 min.
The solute is hot, first at
Inter,purge s Inter,purge
v u 80 , v 4.343 cm min and then

s inter,F
18.60
u 80 ,v 4.343 3.158 cm min
25.58
after 18.56 minutes. Next solute step is

s inter,purge, s,F inter,F
u v 80 18.56 u 80 , v t 120
t 120 4.343 18.56 3.158 12.47min
Exit Time Solute
F purge
t t t 290.35 18.56 12.47 321.38min.
Since Exit Time Solute 334.44

breakthrough cold wave, the solute is at 80C the
entire time.

Solute exits from 290.35+18.56=308.91 min to 321.38 minutes = 12.47 minutes & it exits at
superficial velocity of 8.0 cm min.

Mass Balance
All solute in = Solute out

F super c IN out super c out,AVG
t v A c t v A c

F
out ,AVG IN
out
t 290.35
c c 0.0009 wt frac 0.02096 wt frac.
t 12.47

This is same as peak concentration in Example 18-3, but greater than
out ,avg
x 0.00748. To
have same concentrations need to recycle the material exiting at feed concentration in counter-
flow system. NOTE: Counter flow system has advantages of not contaminating the product end
of the column and typically has less spreading of the zone.

18.D.9. New problem in 3
rd
edition. a.
e inter Super
v v 0.05 m s.
inter
v 0.5 0.43 0.1163 m s

e inter
s,feed,M
e e p d e p s A,p
v
u 0.01712 m s
1 K 1 1 RTK


M
from Eq. (18-27) is same as Example 18-4,
M
0.2128
Pressurization Step
Feed end (for pressurization) 0.75m(Measured from closed product end)

0.2128
after
4.0 atm
z 0.75 m 0.5584 m
1.0 atm

which is 0.75 0.5584 0.1916 m from feed and

after
0.2128 1
M
4.0
y 0.003 0.001007
1.0

Feed Step

feed
s
u 0.01712 m s 7 sec 0.11984 m 0.1916 m for pressurization step
= 0.3114 m. Does not breakthrough in first cycle. From 0 to 0.11984 m, concentration is
F
y .
457
Blowdown.
Measuring from closed top,
before
z 0.75 0.3114 0.4386 m
0.2128
after
1.0
z 0.4386 0.5890 m
4.0
The far end of the feed wave does not get removed from the bed. The close wave
0.11984 or 0.75 0.11984 0.6302 m from closed end has
0.2128
after
1.0
z 0.6302 0.8463
4.0
m, so it all exits. The mole fraction of this portion is
0.2128 1.0
after feed
1.0
y y 0.003 2.9781 0.00893
4.0
Part of the feed that was pressurized also exits during blowdown.
This
0.2128
after before before before
1.0
z 0.75 z 1.3431 z z 0.5584 m
4.0
from closed
(product) end. This is 0.75 0.5584 0.196 mfrom feed end. This gas entered at an unknown
pressure between
L
p 1.0 and
H
p 4.0. Can calculate this pressure from Eq. (18-28c)
1
1 0.2128
after
before after
before
z 0.5584
p p 4.0 1.00003 atm
z 0.75
0.2128 1
after,press
4.0
y 0.003 0.001007
1.00003
After Pressurization Step.
This gas is depressurized to 1.0 & exits column
0.2128 1
after,BD
1
y 0.001007 0.00300
4
or essentially the feed composition.
Exit from Col
Part b. Want
after
z blowdown 0.75, then
after
before after
before
p
z z
p
0.2128
before
1.0
z 0.75 0.5584
4.0

from closed end, which is 0.75 0.5584 0.1916 m from feed end. Want the feed to end at this
point. During constant pressure feed step, feed travels
s,feed F F
u t 0.01712 t . Then for
pressurization step
after
z (from feed end)
F
0.1916 0.1712 t . From closed end this is
y
0.008933
0
time
.0030
458


after
F after before
before
0.2128
F F
p
0.5584 0.1712 t z z
p
4
0.5584 0.1712 t 0.75 0.5584 or t 0.
1

Thus, need a purge step if have feed step at constant pressure for complete cleanout.
18.D10. a)
1
0.2128
after A before
0.4
pt.10 : z 0.4, 0.2128, p 3.0 1.05128 atm
0.5


0.2128 1
after
3.0
y 0.002 0.000876
1.015128

Travels,
0.4 m
25.126s 1.0s for blow-down 26.126s
0.01592 m s

b) Start with Arbitrary point at t = 1 sec (end repress)
after
z 0.48(.02 from feed end)

1
.2128
before
.48
p 3.0 2.4763 atm
.5
,
0.2128 1
after
3.0
y 0.002 0.00172
2.4763

Dist. Traveled @ t = 30s: 0.02 + 0.01592 29s = 0.48168 m
For blow-down: distance from closed end = 0.01832 cm

.2128
after
0.5
z 0.01832 0.026824
3.0
,
.2128 1.0
after ,BD
0.5
y 0.00172 0.007048
3.0

Purge:
M,purge
u 0.01751 m s . Exits bottom column during purge (point 11)
(distance traveled)/u
purge

0.5-0.026824
31 s + 58.023s
0.01751


18.D11. If repressurize with product, bed remains clean.
Feed step is same as to point 3 (at 0.462 m from feed end) on Figure 18-13.
Blowdown then pt. 4 (0.056 m from top) and purge exits at pt. 8 (56.36s)
Product gas is cleaner (y = 0), but there is lower productivity less feed per cycle.
See Figure.


459



18.D12. a) The clean bed receiving feed has a shockwave for Langmuir isotherm.

2
3 2 2
super c c
v 320 cm min A , where A r 4 cm 50.2654 cm

super inter super e
v 6.366 cm min, v v 6.366 10.434 14.669 cm min

int er
sh
e p e p d
s
e e
v
u
1 1 1 K
q
1
c


after before
before before
after before
q q q
where c 0, q 0
c c c


3
after after
0.190 50
c 50 mol m , q 1.1446 mol kg
1 0.146 50



sh
14.669
u 0.5843 cm min
0.566 0.57 1.0 0.566 0.43 1.1446
1 1820
0.434 0.434 50


br sh
t L u 50 cm 0.5843 cm min 85.579 min
Outlet concentration is zero until
br
t then becomes 50.
Concentrated solution eluted by dilute soln. Gives diffuse wave for Langmuir isotherm.

s diffuse
e p
e
p d p 2
e e
v
u u
1 1
1
a
1 K
1 bc

s
2
14.669
u
0.566 0.57 1.0 0.566 0.43 1820
0.19
1
0.434 0.434
1 0.146 c
2
14.669
193.92
1.74336
1 0.146c

Create Table.
y = 0.0082
y = 0 y = 0
BD
3
8
4
460


s s
s s
s
s
s
s
c 50, u 3.218, t L u 50 3.218 15.537 min
c 0, u 0.07497 cm min , t L u 666.93 min
c 40, u 2.491, t 20.071 min
c 30, u 1.737, t 28.779 min
c 15, u 0.7052, t 70.898
c 5, u 0.2205, t 226.80



18.D13. A de xtran, B fructose

1
A 1 A 1 port
C v u M u (1)

B 2 B2 2 port
C v u M u (2)

3
A 3 A 3 port
C v u M u (3)

B 4 B4 4 port
C v u M u (4)

3
F,super 3
2 3 F F,super F 2 2
e
v
Vol Feed 1000 cm min
v v v , v , v 1.9894 cm min
D 40
4 4

Solve eqs. (2) and (3) simultaneously,
B F
port
B
2 3
A
C v
u
C
M M
C



A
e
A
e
1 1
C 0.7435
1 .6
1 0.23 1 K
.4
,
B
e
B
e
1 1
C 0.4914
1 .6
1 .69 1 K
.4


port sw
0.4914 1.9894 cm min
u 3.03175 cm min L t
0.4914
0.99 1.01
0.7435


sw
port
L 60
t 19.791 min
u 3.03175


2
3 1
1 port 1,super 1 e
A
M 0.97 D
v u 3.03175 3.955 cm min : V v 1988.176 cm min
C 0.7435 4


3 2
2 port 2,super
B
M 0.99
v u 3.03175 6.1079 cm min; V 3070.15cm min
C 0.4914


3 port 3
3 3,super
A
M u 1.01 3.03175
v 4.1184; V 2070.14 cm min
C 0.7435


4 port 3
4 4,super
B
M u 1.03 3.03175
v 6.3547; V 3194.19 cm min
C 0.4914

Recycle flow
3
1,super
V 1988.176 cm min
461



3
4,super 1,super
D V V 3194.19 1988.176 1206.0 cm min ,
D
1.2060
F

Check:
4 1
B
port B A D 4 1 4 1
B
F F B A F
2 3
A
M M
C
u C C V V V M M D
C
V F V C C v
M M
C


1.03 .97
.4914
D .4914 .7435
1.2060
.4914
F
.99 1.01
.7435
, OK
Raffinate Product
3
2 1
V V 3070.15 1988.18 1081.97 cm min
Extract Product
3
4 3
V V 3194.19 2070.14 1124.05 cm min

18.D.14. New problem in 3
rd
edition. From Eq. (18-40c)
K H K Li H Li
K K K
Andersons data:
K H
K 2.9 1.3 2.2308
DeChows data:
K H
K 2.63 1.26 2.0873
For the shockwave Eq. (18-46) holds for K
+

Since resin is initially in H
+
form,
K,before K,before T
x C C 0 and
K,before R,K RT
y C C 0.

K,after K,after T
x C C 1.0

K,after R,K,after RT
y C C 1.0
a)
inter
sh,K
K,after K,before
RT
DE,K
e T K,after K,before
v
u
y y
C 1
1 K
C x x


sh,K sh
sh
25 0.42
L
u 1.115 cm min, t 44.84 min
1 2.2 1 0 u
1 1.0
0.42 0.1 1 0

Same for both sets of data since
K H
K does not enter into equation when initial and feed
contain only one ion.
b)
t Sh,K sh
C 1.0, u 9.542 cm min, t 5.24 min
c)
t K,before K,after
C 1.0, x 0.2, x 0.85.
K
y values depend on equilibrium parameter.

KH K
K
KH K
K x
y
1 K 1 x

Andersons Data:
K,before
2.2308 0.2
y 0.3580
1 2.2308 1 0.2


K,after
2.2308 0.85
y 0.9267
1 2.2308 1 0.85

462


sh
25 0.42
u 10.662
1 2.2 0.9267 0.3580
1 1.0
0.42 1.0 0.85 0.2
,
sh
K
t L u 4.69 min


DeChows data:

K,before
2.0873 0.2
y 0.3148
1 2.0873 1 0.2


K,after
2.0873 0.85
y 0.9220
1 2.0873 1 0.85


sh
25 0.42
u 10.100
1 2.2 0.9220 0.3148
1 1.0
0.42 1.0 0.85 0.2
,
sh
K
t L u 4.95 min
% difference
4.69 4.95
100 5.55%
4.69

d) There is a difference if either initial or feed contains both ions. System with higher
K H
K had higher shock velocity.
18.D15. New problem in 3
rd
edition. Part a.
sh,i
i ,after i ,before
RT
DE
e T i ,after i ,before
v
u
y y
c 1
1 K
c x x

For both Na &
i,before i,before
i,after i,after
x y 0
K ,
x y 1.0

Thus same
sh
u ,
sh
25 0.42
u 5.186 cm min
1 2.2
1
0.42 0.5


center sh
t L u 50 5.186 9.64 min

Part b.
i ,sh
i ,after i ,before
RT
DE
e T i ,after i ,before
v
u
y y
c 1
1 K
c x x

For both Na
+
and K
+
: x
before
= 0.4 and x
after
= 0.9. For Na
+


,
(2.0 / 1.3)(0.4)
0.506
1 ( 1) 1 [(2.0 / 1.3) 1](0.4)
Na H Na
Na before
Na H Na
K x
y
K x


,
(2.0 / 1.3)(0.9)
0.933
1 ( 1) 1 [(2.0 / 1.3) 1](0.9)
Na H Na
Na after
Na H Na
K x
y
K x


463


sh, Na
Na ,after Na ,before
RT
DE
e T Na ,after Na ,before
v (25 / 0.42)
u 5.98
1(2.2)(1.0) 0.933 0.506
y y
c 1
1
1 K
(0.42)(0.5) 0.9 0.4
c x x


,
/ 50 / 5.98 8.36min.
Na sh Na
t L u

For K
+
we obtain,


,
(2.9 / 1.3)(0.4)
0.598
1 ( 1) 1 [(2.9 / 1.3) 1](0.4)
K H K
K before
K H K
K x
y
K x


,
(2.9 / 1.3)(0.9)
0.953
1 ( 1) 1 [(2.9 / 1.3) 1](0.9)
K H K
K after
K H K
K x
y
K x


sh,K
K,after K,before
RT
DE
e T K,after K,before
v (25 / 0.42)
u 7.054
1(2.2)(1.0) 0.953 0.598
y y
c 1
1
1 K
(0.42)(0.5) 0.9 0.4
c x x



,
/ 50 / 7.054 7.09min.
K sh K
t L u

Part c.
RT
DE
e T
v
u
c 1 dy
1 K
c dx


Na Li H Li Na Na H
2 2
Na
Na H Na Na Li H Li Na
K K dy K
dx
1 K 1 x 1 K K 1 x



Na
2
Na
2.0 1.3 dy
0.955,
dx
1 2.0 1.3 1 .5

Na
25 0.42
u 5.409
1 2.2
1 0.955
.42 0.5


Na shNA
t L u 9.244 min

K Li H Li K K H
2 2
K
K H K K Li H Li K
K K dy K
dx
1 K 1 x 1 K K 1 x


2
2.9 1.3
dy
0.855,
dx
1 2.9 1.3 1 .5

K
25 0.42
u 5.979
1 2.2
1 0.855
.42 0.5

Part d.
Na
x 0
K
x 0

Na 0
Na
Na H
Na
x
dy
K 2.0 1.3 1.538,
dx

Na
u 3.477,
Na
t 14.38 min

K 0
K
K H
K
x
dy
K 2.9 1.3 2.231,
dx

K
u 2.442,
K
t 20.47 min
Part e.
Na K
x .9 x .9
464


Na
2
Na
dy 1.538
0.698,
dx
1 0.538 .9
Na
u 7.159,
Na
t 6.984 min

K
2
K
dy 2.231
0.502,
dx
1 1.231 .9

K
u 9.5075,
K
t 5.259 min
Part f. The velocities and hence the derivatives are equal. Thus,


Na Li H Li K Li H Li Na K
2 2
Na K
Na Li H Li Na K Li H Li K
K K K K dy dy
dx dx
1 K K 1 x 1 K K 1 x

With x
Na
= x
K
. The result from a spreadsheet is x = 0.35056



18.D16.
int er super e
v v 15 0.40 37.5 cm s

3
f 3
MW p 28.9 g mol 50 kPa
1.0 kg
0.5832 kg m
m kPa RT 1000 g
0.008314 298 K
mol K


q kg toluene kg carbon
is in
c kg toluene kg air
. Then, shockwave velocity is

int er
sh
e p e p d
s
e e f
v
u
1 1 1 K
q
1
y


sh
3
2 1
3
2 1
37.5 cm s
u
0.6 0.65 1.0 0.6 0.35 q q 1500 kg m
1
0.4 0.4 0.5832 kg m y y


sh
2 1
2 1
37.5 cm s
u
q q
1.975 1350.308
y y

For
sh,1 1 1 2 2
2000 .0005
u : y 0, q 0, y 0.0005, q 0.47619
1 2200 0.0005


sh,1
37.5
u 0.00002916 cm s 0.104976 cm h
0.47619
1.975 1350.308
0.0005


sh2 1 1
u : y 0.0005, q 0.47619
2 2
2000 0.0015
y 0.0015, q 0.69767
1 2200 0.0015


sh2
37.5
u 0.00012539 cm s 0.451393 cm h
0.69767 0.47619
1.975 1350.308
0.0015 0.0005

At
min sh1 min sh2
L : u t L u t 10 h where t is in hours.
465

Solve for
sh2
sh2 sh1
u 10 0.451393 10
t 13.03 h
u u 0.451393 0.104976


min sh1
cm
L u t 0.104976 13.03 h 1.368 cm
h

Thus, for any column of partial length we will see a single shockwave exit the column.
18.D17.
super
int er
e
v
21.0
v 52.5 cm s
0.4


3
v
pV n RT
MW n MW p 28.9 50
0.5383 kg m
V RT 1000 g kg 0.008314 298

Since
init F
C C , Get 2 diffuse waves

int er
s 2
e p
e s
p d
e e f
v q 2000
u where
y 1 1 1 2200y
1 q
1 K
y


s
52.5 52.5 cm s
u
q 0.6 0.35
.6 1500 q
1.975+1350.23
1 0.65 1.0
y
0.4 0.4 0.583 y

y
q
y

s
u cm s
s s
t L u 25 u
0.0010 195.31 0.0001991 125,581s = 34.8835 h
0.00075 284.799 0.0001365 183,117.6s = 50.866 h
0.00050 453.515 0.00008573 291,596.6s = 80.999 h
2
nd
wave (0.00050) - add 20 hours 100.999 h
0.00025 832.466 0.00004671 535,250.5 = 148.681 + 20 = 168.681 h
0.00 2000 0.00001914 1285937.96 = 357.205 + 20 = 377.205 h


s
u y 0.0005
0
t
z
2
0.00025 0.00075
s
u y 0.005

s
u y 0
s
u y 0.001
466







18.D.18.
. New problem in 3
rd
edition.
Part a.
S,G
u 11.12 S cm min is calculated in Example 18-9.


S,F
20
u 8.416
0.61 1.0 0.88
1 0
0.39
,
S,G S,F
u u u 2 9.771
From Eq. (18-93),
2
S,G S,F
N 4Ru u u 229.465
From Eq. (18-78a)
eff
eff
2N E 1
N L v E L
2 v



2
2 229.465 5.0 cm min
L 114.73 cm
20 cm min

Part b.
G S,G
t L u 114.73 11.25 10.20 min

F S,F
t L u 114.73 8.416 13.63 min
Part c. Eq. (18-80a),
1/ 2
t
S
L 1
u N
,
1/ 2
t ,G
1
10.20 min 0.673 min
229.465


1/ 2
t ,F
1
13.63 min 0.900 min
229.465


18.D19.
AgK Ag Li K Li
K K K 8.5 2.9 2.93,
Ag
Ag
Ag
2.93 x
y
1 1.93 x

a) Ion wave:
total con int er super e
u v v 3.0 0.4 7.5 cm min
Breakthrough of ion wave, 50 cm 7.5 cm min 6.667 min
377.2
100.99
34.88
50.866
0.001
168.88
t
0.00025
0.00075
0.0005
c
80.999
0


467

b) Shock wave,
int er
sh
Ag after Ag before
RT
E
e T Ag after Ag before
v
u
y y
C 1
1 K
C x x

before:
Ag Ag
x y 0 . after:
Ag Ag
x y 1.0 .

sh
7.5 cm min
u 1.4516 cm min
2.0 1.0
1 1.0
1
0.4 1.2 1.0
,
sh
sh
L 50 cm
t 34.44 min
u 1.4516 cm min

c) Diffuse wave:
int er int er
s
RT Ag-K
RT
E
E 2
e T
e T
Ag K Ag
v v
u
C K 1 dy
C 1
1 K
1 K
C dx
C
1 K 1 x


s,Ag
2 2
Ag Ag
7.5 cm min 7.5
u
1 2.0 2.93 12.208
1 1.0 1
0.4 1.2
1 1.93 x 1 1.93 x


Ag s out
s
2
7.5 cm L
x 1.0, u 3.0965 , t 16.147 min
12.208
min u
1
2.93


Ag s out
7.5 50
x 0.5, u 1.8021 , t 27.745 min
12.208
1.8021
1
3.86


Ag s out
s
7.5 L 50
x 0, u 0.5678 cm min , t 88.0555 min
13.208 u 0.5678

From spreadsheet:
x
Ag
u_dif,Ag t_dif,Ag
1 3.097163211 16.14380534
0.9 2.852615804 17.5277722
0.8 2.598940969 19.23860549
0.7 2.337763116 21.38796684
0.6 2.071284133 24.13961427
0.5 1.802351071 27.74154314
0.4 1.53450084 32.58388572
0.3 1.27196632 39.3091764
0.2 1.019637556 49.0370324
0.1 0.782936124 63.86217015
0 0.567638906 88.08416667

18.D20. Was 18D23 in 2
nd
edition. Table 18-5,
Ca Li
CaK 2 2
K Li
K 5.2
K 0.6183
K 2.9

a.)
RT T Ca
c 2.0 eq L, c 0.02 eq L, x 0.8 at t 0 .

CaK RT
T
0.6183 2.0 K C
61.83 shockwave
C 0.02
.
Column:
super inter
L 75 cm, v 20 cm min, v 20 .4 50 cm min
468


p e E
0, 0.4, K 1.0
Feed:
int er
sh
RT E
e T
v
u
C K y
1
C x
, before:
Ca Ca
x 0, y 0 , after:
Ca
x 0.8
from Eq. (18-43)
Ca
y 0.971965

sh
50
u 0.16407 cm min
1 2.0 0.971965
1 1.0
0.4 .02 0.8
,
br
sh
75
t 457.1 min
u

b.) Regenerate: at 500 min Ion wave at
int er
v 50 cm min takes
75
1.5
50
min.
New
Ca RT
T
K C
1.2366
C

Ca
y unchanged. Use Eq. (18-43) with new value
Ca RT T
K C C .
Ca
x 0.9689677 . Obtain diffuse wave.

3
Ca Ca imter Ca
diffuse 3
Ca RT E
Ca
Ca Ca
T e Ca
1.2336 1 y 1 x v dy
u where
dy C K
dx
1 x 1 y
1
C dx

(Wankat, 1990, Eq. (9-25b)).
At
Ca
Ca Ca
Ca
dy 50
x 0, 1.2366, u 6.96088 cm min
1.0 2.0
dx
1 1.2366
0.4 1.0


out
75 cm
t 10.7745 min
6.96088
(slow wave)
At
Ca Ca
x 0.96897, and y 0.971965

3
Ca
3
Ca
1.2336 0.028035 1.96897 dy
0.908386
dx
0.03103 1.971965


Ca
50
u 9.022 cm min
2.0 1.0
1 0.908386
1.0 0.4
,
out
75
t 8.312898 min
9.022
(fast wave)
At
Ca Ca
x 0.5, y 0.534927

3
Ca
3
Ca
1.2336 0.465073 1.5 dy
0.97013
dx
.5 1.534927


Ca
50
u 8.546 cm min
2.0 1.0
1 0.97013
1.0 0.4
,
out
75
t 8.776 min
8.546
. (in-between)

18.D.21. New problem in 3
rd
edition.

F
port
3 2
y Tol
v
u
M M
C C
,
Tol
e p
e
p d Tol
e e
1
C 0.132234
1 1
1
1 K K 300


469


2175.2696 300
Tol
K 300 K 0.0061 e 8.5972,
2115.1052 300
xy
K 300 K 0.0105 e 12.1092

xy
1
C 0.10017
1.63627 0.68930 12.1092


F
port F
v
u 0.6479 v 0.6479 cm min
.95 1.05
.10007 .132234
,
port SW
L u t 64.79 cm

2 2 port xy
0.95 .6479
v M u C 6.1447 cm min
0.10017
,
3 2 F
v v v 5.14476

1 1 port Tol
0.6479
v M u C .95 4.6547
0.132234


4 4 port xy
1.05 0.6479
v M u C 6.7914
0.10017


Tol prod 2 1
v v v 6.1447 4.6547 1.4900

xy,prod 4 3
v v v 6.7914 5.14476 1.6466

D 4 1
v v v 6.7914 4.6547 2.1367, D F 2.1367
Check:
Out tol prod xy,prod
v v v 3.1366 ,
total in F D
v v v 3.1367 OK.

18.D22.
2
p 2
eff D s
M,c
d 1
E E u K
6 k
, where
p
m,c m,c p
d
1
6 k k a



3
super
super 2
v
20 cm min
v 6.366 cm min , v 15.915 cm min
cm


s
e
e
v 15.915
u 7.821 cm min
1
2.035
1 0 K


2
2
2
eff
1
cm
7.821 0.6 0.69
min
E 0.15 8.063 cm min
5.52 min 0.4

Eq. (17-69)
s
1 2
F
eff s
int er
z u t C 1
X 1 erf
C 2
4 E u t
v
for step up
Argument of erf,
1 2
200 7.821 t
a
15.849 t

470

Step down:
s s
1 2
eff s
int er
L u t 8u 1
X L, t 8 1 erf
2
4 E u t 8
v

Argument of
1 2
323.04 7.821 t
erf , a
15.849 t 126.792

Total Solution X X
If
1 2
s
L 123.03
t 25.573 min a 0, a 7.358
u
279.6


1 1
X and X 1 1.000 0
2 2
,
F
c c 0.50 or c 25.0 (for smaller t, can ignore X )
If
s
s
L 8u
t 33.575, a 0, X 1 2
u
,
1 2
63.96
a 2.773, erf .998
532.13


1
X 1.998 0.999
2
For higher t, X = 1.0,
F
c 1
0.999 0.499 c 24.975
c 2
.
Peak at
1 2
31.2765
25.575 4 29.575, a 1.44473, erf a 0.95847
468.663


1
X 1.95847 0.979236
2


1 2
31.2835
a 1.69189, erf a 0.983186
341.89
,
1
X 1 .983186 0.0084
2


F
c
0.979235 0.0084 0.970835
c
, c 50 0.970835 48.54
See also Problem 18.G1.


18.D23. Was 18D24 in 2
nd
edition.
out initial initial F1 F1
C C C C X z, t 17.5 C X z, t 28


18.D.24. New problem in 3
rd
edition.
F
port
3 2
xy Tol
v
u
M M
C C
,
Tol
e p
e
p d Tol
e e
1
C (300K) 0.132234
1 1
1
1 K K 300



2175.2696 300
Tol
K 300 K 0.0061 e 8.5972,
2115.1052 300
xy
K 300 K 0.0105 e 12.1092



xy
1
C (300K) 0.10017
1.63627 0.68930 12.1092

471


F
port F
v
u 1.4812 v 1.4812 cm min
.90 1.10
.10007 .132234
,
port SW
L u t 148.12 cm

2 2 port xy
0.90 1.4812
v M u C (300K) 13.3084 cm min
0.10017
,
3 2 F
v v v 12.3084

1 1 port Tol
1.4812
v M u C (300K) .90 10.0812
0.132234



Tol prod 2 1
v v v 13.3084 10.0812 3.2272


4 4 port xy
1.10 1.4812
v M u C (300) 16.2655
0.10017


xy,prod 4 3
v v v 16.2655 12.3084 3.9571

D 4 1
v v v 16.2655 10.0812 6.1843, D F 6.1843
Check:
Out tol prod xy,prod
v v v 7.1843,
total in F D
v v v 7.1843 OK.

18.D.25. New problem in 3
rd
edition. Zones 2 & 3 are same as in 18.D.24 since at 300 K

port F
u 0.6479 v 0.6479 cm min ,
port SW
L u t 64.79 cm

2
v 6.1447 ,
3
v 5.14476

1 1 port Tol
v M u C 273 K and
4 4 port xy
v M u C 350 K

1 4
M 0.5 and M 2.0 (reciprocal values).

Tol
K 273K 0.0061 exp 2175.2695 273 17.612

xy
K 350K 0.0105 exp 2115.1052 350 4.423

Tol
1
C 273K 0.07259
1.63627 0.68930 17.612


xy
1
C 350K 0.2135
1.63627 0.68930 4.423


1
v 0.5 0.6479 0.07259 4.4627,
4
v 2.0 0.6479 0.2135 6.0707

Tol prod 2 1
v v v 1.6820 ,
xy prod 4 3
v v v 0.9260,
D 4 1
v v v 1.608
D/ F 1.608

18.D26. a)
2
s
sA s,B
4Ru
N , R 1.5
u u
,
A
s
s
A
v 1.0
u 0.15385
6.5 1
1 K


D
s
1.0
u 0.147059
1 5.8
, u 0.15045
472

Need
2
4 1.5 0.15045
N 17689
0.0067873
, L 0.05 N 884.45 cm.
b)
R,A
A
L 884.45
t 5748.88 95.813 min
u 0.15385


1 2 1 2
t ,A
A
L 1 1
95.813 min 0.7204 min
u N 17689


2 2
R A
2 2
A,max t
t t t 95.813 C
exp exp
C 2
2 0.7204
p

t,min 90 92 94 95 95.813 96 97
A A,max
C C
7.27E-15 8.3E-7 0.0421 0.52898 1.00 0.9669 0.2573

18.D27.
A
A A F
F
A F A F
C
0.33 X L, t 1.0 0.33 X L, t 0.4t
C
1 .55 X L, t .8t 0.55 0 X L, t t


18.D28. a)
p center
25.0 cm
u L t 0.706cm min
35.4 min
,
MTZ,lab p MTZ
L u t 0.706 2.8 1.9774 cm
b) Large-Scale system
2
p, LS eff MTZ, LS
MTZ,lab
d t
t
D
2
p,lab eff
d D
2
2
1.0
69.44
0.12

Independent of velocity
MTZ,LS
t 69.44 2.8 194.44 min

super
e
p,LS
p,lab
v
u
u
LS
super
e
v
lab
12 4
9 3
lab

p,Ls
4
u 0.706 0.941 cm min
3


MTZ,l arge scale p MTZ
L u t 0.941 cm min 194.44 min 183.03 cm
For frac. bed use = 0.80 & symmetrical pattern,

MTZ
0.5 183.03 0.5 L
L 457.6 cm 4.576 m
1 Frac bed use 1 .8
,
MTZ
br center
t
t t
2


center p
457.6
t L u 486.27 min
0.941
,
br
194.44
t 486.27 389.05 min
2
.
This is length of feed time if column is completely regenerated.

18D.29.
Ca Li
CaK 2 2
K Li
K 5.2
K 0.6183
K 2.9

a.)
RT T Ca
c 2.5 eq L, c 0.03 eq L, x 0.7 at t 0 .
473


CaK RT
T
0.6183 2.5 K C
51.525 shockwave
C 0.03
.
Column:
super inter
L 90 cm, v 25 cm min, v 25 / .39 64.10 cm min

p e E
0, 0.39, K 1.0
Feed:
int er
sh
RT E
e T
v
u
C K y
1
C x
, before:
Ca Ca
x 0, y 0 , after:
Ca
x 0.7
from Equilibrium,
Ca Ca K RT Ca
2 2
Ca T Ca
y K C x
400.75
(1 y ) C (1 x )


Solve this for unknown y value. I used a spreadsheet.

0.95128
Ca
y

sh
64.1
u 0.22000 cm min
1 2.5 0.95128
1 1.0
0.39 .03 0.7
,
br
sh
90
t 409.10 min
u

b.) Regenerate: Ion wave at
int er
v 35.0 / 0.39 89.74 cm min takes
90
1.003
89.74
min.
New
CaK RT
T
K C (0.6183)(2.5)
1.4057
C 1.1

Ca
y 0.95128 unchanged. Use equilibrium with
old y and with new value
Ca RT T
K C C
,

Ca Ca K RT Ca
2 2
Ca T Ca
y K C x
400.75
(1 y ) C (1 x )
to
find
Ca
x 0.94251. Obtain diffuse wave.

3
Ca Ca imter Ca
diffuse 3
Ca RT E
Ca
Ca Ca
T e Ca
1.4057 1 y 1 x v dy
u where
dy C K
dx
1 x 1 y
1
C dx

(Wankat, 1990, Eq. (9-25b)).
At
Ca
Ca Ca
Ca
dy 89.74
x 0, 1.4057, u 9.7631 cm min
1.0 2.5
dx
1 1.4057
0.39 1.1

As an alternative can do numerical calculation of derivative. At x = 0, y = 0. x = 0.001, y = 0.001404 and
/ (0.001404 0) / (0.001 0) 1.404 y x , which is reasonably close.


out
90 cm
t 9.22 min
9.7631
(slow wave)
At
Ca Ca
x 0.94251, and y 0.95128

3
Ca
3
Ca
1.4057 0.04872 1.94251 dy
0.85169
dx
0.05749 1.95128

474


Ca
89.74
u 15.049 cm min
2.5 1.0
1 0.85169
1.1 0.39
,
out
90
t 5.981min
15.049
(fast wave)
From equilibrium, at the arbitrary value
Ca Ca
x 0.5, y 0.55544

3
Ca
3
Ca
1.4057 1 0.55544 1.5 dy
0.95282
dx
.5 1.55544


Ca
89.74
u 13.695 cm min
2.5 1.0
1 0.95282
1.1 0.39
,
out
90
t 6.572 min
13.695
.
This is in-between the other two waves.
c. To not have a diffuse wave must have
CaK RT
T T
K C (0.6183)(2.5)
1.0
C C

This requires C
T
> 1.546.

18.D30. New Problem in 3
rd
edition.
K H K Li H Li
K K K
DeChows data:
K H
K 2.63 1.26 2.0873
a.) This will be a shock wave since K
+
is more concentrated in the feed to the column than it
is initially and K
K-H
> 1.


inter
sh,K
K,after K,before
RT
E,K
e T K,after K,before
v
u
y y
C 1
1 K
C x x



t K,before K,after
C 1.0, x 0.2, x 0.85.
K
y values depend on equilibrium parameter.

KH K
K
KH K
K x
y
1 K 1 x




K,before
2.0873 0.2
y 0.3148
1 2.0873 1 0.2


K,after
2.0873 0.85
y 0.9220
1 2.0873 1 0.85


sh
25 0.42
u 10.100
1 2.2 0.9220 0.3148
1 1.0
0.42 1.0 0.85 0.2
cm/min,
sh
K
t L u 49.5 min

All three times are the same for the shock wave.

475


b.) This will be a diffuse wave since K
+
is less concentrated in the feed to the column than it
is initially and K
K-H
> 1.

inter
diffuse,K
RT K K
E,K
e T K K
v 25 / 0.42
u
C dy dy 1 2.2(1)
1 K 1
C dx (0.42)(1.0) dx

At x
K
= 0.15,
2 2
2.0873
1.543
(1 ( 1) ) [1 (1.0873)(0.15)]
K KH
K KH K
dy K
dx K x


diffuse,K
K K
K K
25 / 0.42 59.524 59.524
u 6.554cm/ min
dy dy 2.2(1)
1 5.238(1.543)
1 1 5.238
(0.42)(1.0) dx dx

Thus, at x
K
= 0.15, t
K
= L/u
diffuse,K
= 500/6.554 = 76.29 min. Then at x
K
= 0.5 we obtain

2 2
2.0873
0.876
(1 ( 1) ) [1 (1.0873)(0.5)]
K KH
K KH K
dy K
dx K x
diffuse,K
K
K
59.524 59.524
u 10.65cm/ min
dy
1 5.238(0.876)
1 5.238
dx

Thus, at x
K
= 0.5, t
K
= L/u
diffuse,K
= 500/10.65 = 46.94 min. Then at x
K
= 0.8 we obtain

2
0.5970
(1 ( 1) )
K KH
K KH K
dy K
dx K x

diffuse,K
K
K
59.524 59.524
u 14.42cm/ min
dy
1 5.238(0.597)
1 5.238
dx

t
K
= L/u
diffuse,K
= 500/14.42 = 34.67 min.

18.D31. New problem in 3
rd
edition. a.
Super int er e
v 10 v 10 .4 25, 0.4, L 30.0
476
RT T K Na
2.9
c 2.4, c 1.10, K 1.45
2.0

int er
sh,K
K,after K,before
RT
DE
e T K,after K,before
v 25
u
1 2.4 1 0
y y
c 1
1 1.0
1 K
.4 1.1 1 0
c x x
sh,K center,expected
u 3.783 cm min, t 7.75 min. ,
center,measured
t 7.31 min
7.31 7.75
% error 100 6.00%
7.31

b.
sh,expt center
u L t 30 7.31 4.10 cm min.
c.
MTZ sh MTZ
L u t 4.10 7.57 7.06 2.093 cm
Frac. bed use (symmetric wave)
MTZ
1 0.5 L L 0.965
d.
Lab
l arg e scale Lab
Lab
2 2
p Super eff p eff
l arge scale
MTZ MTZ MTZ,Lab 2 2
p eff p Super eff
Lab
d v 16 d 200
L L L
d 100 d v
D D
D D
With same beads assume no change in
eff
. D
MTZ,l arge scale
L 16 2 2.093 cm 66.98 cm
MTZ
frac bed use 1 0.5 L L 1 0.5 66.98 200 0.833
Breakthrough start time
center MTZ sh MTZ sh
t 0.5 t L u 0.5 L u
inter,large scale
sh sh,lab,exp tl
inter,lab scale
v
u u 8.2
v
Breakthrough start time [200 0.5(66.98)] / 8.20 20.31min
18.F1. New problem in 3
rd
edition. Constraints:
2
max
w L 860 ft T 500 C 932 F
3
0
wLh 2580 ft , h=2580/860=3ft., p 6 atm 88.14 psia 73.14 psig
Seider etal. (2004), Eq. (16.61)
End
View
Weight vessel
i s i s s
Seider etal,
D t L 0.8 D t
Eq.16 59
h
w
477


2
d 0 0
P exp 0.60608 0.91615 n p 0.0015655 n p 96.66 psig
Wall thickness
(Eq. 16.60)
d i i
p i
d
P D 96.66 D
t 3.7057E 3 D
2SE 1.2 P 2 13,100 1 1.2 96.66

where S 13.100 psi p. 529 with SA-387B steel, E = 1.0

3 3
s
490 lbm ft 0.284 lbm in p. 529 (Seider et al, 2004)
Relate w to h.:
1
0.5h 1.5 3
cos cos 3 D
r r D


90

1
0.5w w w 3
cos cos 90 cos
r D D D

1
860
Weight D 3.7057E 3 D 0.8D 3.7057 E 3 D 490
Dcos 90 cos 3 D

D in ft.
1
(In Spreadsheet A cos cos
In Spreadsheet, angle is in radius 90 2
D Weight Width L
3.5 33655 1.80 477.0
3.7 31358 2.17 397.1
3.8 30735.9 2.33 368.7
3.9 30325. 2.49 345.1
4 30070.31 2.65 325
4.1 29933 2.7947 307.7
4.2 29889 2.939 292.6
4.3 29918 3.08 279.2
6 35103 5.196 165
8 44837 7.42 115.96
10 56197 9.54 90.15
12 68940 11.62 74.02
14 83144 13.67 62.88
Goal seek L = 60
D = 14.64 ft Weight = 88052.75 Width = 14.333 L = 60
s
t 0.054266 ft
From Seider et al, p. 527:
p m v PL
C F C C in mid 2000 (MS = 1103)
(Eq. 16.53) horizontal
2
v
C exp 8.717 0.2330 n w 0.04333 n w
(Eq. 16.55)
0.20294
pL
C 1580 D 2724
From p. 531,
m
F 1.2 for low-alloy steel,
v
C 118, 323
p
C 144, 711 in mid 2000
Installed Cost:
p m
Calc C with F 1.0 in 2000
p
C 118323 2724 121047
Bare module factor,
Bm
F 3.05 for horizontal

Bm p m
C C F 1.0, 2000 3.05 1.0 1.0 1.2 1 $393, 400

Absorbent: p. 553
3
p
C $60 ft ,
p
C 60 2580 $154, 800

478



18.G1. Was 17G1 in 2
nd
edition. Figures are labeled 17G1.




479






480







481





18.G2. Was 17G2 in 2
nd
edition.
a.) With QDS with 50 nodes find
center
t 4.52 min

MTZ
t 6.0 3.13 2.87 min

18.G3. Was 17G3 in 2
nd
edition.

a) D F 1.0. D 141.55 E R F.
Find
A B
C 0.343 and C 0.219
Eq. (17.31a)
F
port
3A 2B
B A
v
u
M M
C C


3
F 2 2
e c
F 141.55 cm min
v 4.5057 cm min
D 4 0.4 10 4


port
4.5057
u 2.7295 cm min
1 1
0.219 0.343


sw port
t L u 50 2.7295 18.32 min

1
A int er A 1 port
C v u M u 2.7295 cm min

1,int er
2.7295
v 7.9577 cm min
0.343


1,super 1,int er
v 0.4 v 3.18308 cm min
Recycle Rate
2
3
3.18308 10 4 250 cm min
Obtained raffinate = 96.6% and extract = 94.3%.
b) One approach is to keep a symmetric cycle. Then D = 283.1 and
E F
E R 212.325
2

Flow optimizer can be used to give
sw
t ~ 9.1and Recycle rate ~ 500. Depending on
values obtain raffinate and extract > 97%.

18.G4. Was 17G4 in 2
nd
edition. Figure below is labeled 17G4.

18G5. Was 17G5 in 2
nd
edition. Figure below is labeled 17G5.


482




483



484
18.G6. Was 17G6 in 2
nd
edition.
a.
p
1
m,a
k 1.5 min , L 25.0 cm
485
super
20.2 ml min
v 6.366 cm min
2.0 m 2 4
Eq. (18-66)
-1
6.366 cm min
25.0 cm < 4.5 19.1
1.5 min

Satisfied, but close. Thus some bypasses but most undergoes equilibration.
18.G7. Was 17G7 in 2
nd
edition. Figure is labeled 17.G7.
486
18H1. New problem in 3
rd
edition. Spreadsheets with numbers and formulas shown.
487
488
18.H.2. New problem in 3
rd
edition. The spreadsheets are shown on the next pages. They are based on
the previous, but includes both a step up and a step down. Because of the quirk in Excel not
allowing negative arguments, it was set up with multiple solution paths. The correct solution
occurs when there are numbers.
Time, min 15 20 22.5 25 25.5726 27.5 30 33.575 35 37.5 40
C 0 .0134 1.798 24.96 25.0 42.32 48.52 24.97 11.13 1.114 .040
489
490

Potrebbero piacerti anche